*NURSING > EXAM > Rutgers University Psych and Pharm Final Question And Answers( Download To Score An A) (All)

Rutgers University Psych and Pharm Final Question And Answers( Download To Score An A)

Document Content and Description Below

NU 425 PHARM PSYCH FINAL STUDY GUIDE/NOTES Pharm Kee study guide 23 1. A patient’s wandering and hostility levels have increased per family reports. What should concern the nurse in this patient... who is taking memantine (Namenda) 10 mg/d? - The patient is not taking enough of the medication 2. What statement by the patient indicates an understanding of how to relieve some side effects associated with the use of benzotropine mesylate (Cogentin)? “I can suck on hard candy or chew sugarless gum to prevent dry mouth” Pharm Kee 27 1. The nurse is caring for a patient who has begun taking chlorpromazine HCl (Thorazine) 75 mg BID to treat schizophrenia. A family member tells the nurse that the patient’s agitation, hallucinations, and delusional symptoms have improved, but the patient continues to withdraw from social interaction and won’t bathe unless reminded to do so. The nurse will tell the family member that a. all symptoms will eventually resolve over time with this medication. b. the patient may need an increased dose of the antipsychotic medication. c. these results may indicate that the patient does not have schizophrenia. d. they should consider discussing adding another medication. ANS: D 2. The nurse is assessing a young adult patient with schizophrenia who recently began taking fluphenazine (Prolixin). The patient is exhibiting spasms of facial muscles along with grimacing, and the nurse notes upward eye movements. The nurse suspects which side effect? a. Acute dystonia b. Akathisia c. Pseudoparkinsonism d. Tardive dyskinesia ANS: A 3. The nurse is preparing to administer loxapine (Loxitane) 50 mg to a patient who has schizophrenia. The patient has been taking this medication twice daily for 15 months. The nurse notes smacking lip movements and involuntary movements of all extremities. Which action by the nurse is correct? a. Administer the medication as ordered to treat these symptoms of psychosis. b. Hold the dose and notify the provider of these medication adverse effects. c. Request an order for an anticholinergic medication such as benztropine (Cogentin). d. Suggest that the provider increase the dose to 125 mg twice daily. ANS: B 4. A patient who takes loxapine (Loxitane) to treat schizophrenia is noted to be restless and fidgety and is pacing around the room. The nurse caring for this patient will perform which action? a. Contact the provider to discuss changing to benztropine (Cogentin). b. Notify the provider of these symptoms and request an order for lorazepam (Ativan). c. Question the patient about adherence to the drug regimen. d. Recognize these signs of a serious adverse drug reaction and notify the provider. ANS: B 5. A patient arrives in the emergency department with dehydration. The patient reports taking fluphenazine (Prolixin) to treat schizophrenia. The nurse notes rigid muscles and an altered mental status. The patient has a temperature of 103.6° F, a heart rate of 98 beats per minute, and a blood pressure of 90/58 mm Hg. The nurse will anticipate administering which medication? a. Dantrolene (Dantrium) b. Haloperidol (Haldol) c. Propranolol (Inderal) d. Tetrabenazine (Xenazine) ANS: A 6. The parent of a young adult who has schizophrenia is concerned that the patient spits out pills that are given. The nurse will suggest contacting the patient’s provider to discuss which intervention? a. Changing to a liquid form of the mediation b. Providing a home health nurse to supervise medication administration c. Teaching the patient the importance of taking the medication d. Weekly intramuscular injections of the medication ANS: A 7. The nurse is preparing to administer intramuscular haloperidol (Haldol) to a patient who has schizophrenia. What action will the nurse perform? a. Massage the site after injecting the medication to ensure complete absorption. b. Teach the patient to return every week to receive medication doses. c. Use a small bore needle when injecting the medication. d. Use the Z-track method and inject the medication into deep muscle tissue. ANS: D 8. The nurse is teaching a patient who will be discharged home on a typical antipsychotic medication to treat schizophrenia. Which statement by the patient indicates a need for further teaching? a. “I should not drink alcohol while taking this medication.” b. “I should use a heating pad to treat muscle spasms while taking this medication.” c. “I should use sunscreen while taking this medication.” d. “I will need frequent blood tests while taking this medication.” ANS: B 9. A patient who is about to begin taking the atypical antipsychotic medication clozapine (Clozaril) is concerned about side effects. What information will the nurse include when teaching the patient about this medication? a. “You are more likely to experience dry mouth, constipation, and urinary retention.” b. “You may experience weight gain, drowsiness, and headaches.” c. “You will not experience extrapyramidal side effects with this medication.” d. “You will not need frequent lab work while taking this medication.” ANS: B 10. A family member of a patient who has been taking fluphenazine (Prolixin) for 3 months calls to report that the patient is exhibiting agitation and restlessness. The nurse learns that the patient’s delusional thinking and hallucinations have stopped since taking the medication. The nurse will perform which action? a. Reassure the family member that tolerance to these side effects will subside over time. b. Remind the family member that complete drug effects may not occur for several more weeks. c. Suggest that the family member contact the provider to discuss an order for a benzodiazepine. d. Tell the family member to withhold the medication and notify the patient’s provider. ANS: C 11. A patient has been taking risperidone (Risperdal) for 2 weeks. The patient reports drowsiness and headache. What will the nurse do? a. Counsel the patient to request changing to aripiprazole (Abilify). b. Reassure the patient that these are common side effects of the medication. c. Suggest that the patient have serum glucose testing. d. Suggest that these may be signs of agranulocytosis. ANS: B 12. The nurse is performing a medication history on a patient who reports taking lorazepam (Ativan) for the past 6 months to treat an anxiety disorder. The patient states that the medication is not working as well as previously. The nurse will a. contact the provider to discuss changing to another benzodiazepine. b. notify the provider and discuss increasing the dose of lorazepam. c. suspect worsening of the anxiety disorder. d. understand that the patient has developed tolerance to this drug. ANS: D 13. A patient who is taking chlorpromazine calls the clinic to report having reddish-brown urine. What action will the nurse take? a. Notify the provider and request orders for creatinine clearance and BUN levels. b. Reassure the patient that this is a harmless side effect of this medication. c. Tell the patient to come to the clinic for a urinalysis. d. Tell the patient to discard any drug on hand and request a new prescription. ANS: B 14. A patient has begun taking buspirone hydrochloride (BuSpar) 7.5 mg twice daily to treat acute anxiety and calls 1 week later to report little change in symptoms. What will the nurse tell the patient? a. “Therapeutic effects may not be evident for another week.” b. “The provider may need to increase the dose to 15 mg twice daily.” c. “Notify the provider and request an order for another anxiolytic.” d. “Stop taking the drug and notify the provider that it doesn’t work.” ANS: A 15. A patient who is taking fluphenazine (Prolixin) to treat psychosis is experiencing symptoms of acute dystonia. While performing a medication history, the nurse learns that the patient takes herbal medications. Which herbal supplement would be of concern? a. Ginkgo b. Ginseng c. Kava kava d. St. John’s wort ANS: C 16. A patient is brought to the emergency department with decreased respirations and somnolence. The nurse notes a heart rate of 60 beats per minute and a blood pressure of 80/58 mm Hg. The patient is known to take alprazolam (Xanax) to treat anxiety. Which medication will the nurse anticipate the provider to order? a. Benztropine (Cogentin) b. Flumazenil (Romazicon) c. Lorazepam (Ativan) d. Propranolol (Inderal) ANS: B 17. A patient arrives in the emergency department complaining of difficulty breathing, dizziness, sweating, and heart palpitations. The patient reports having had similar episodes previously. The nurse will expect the provider to order which medication? a. Flumazenil (Romazicon) b. Haloperidol (Haldol) c. Lorazepam (Ativan) d. Propranolol (Inderal) ANS: C 18. The nurse is teaching a patient about taking an anxiolytic agent to treat grief-related anxiety. Which statement by the patient indicates understanding of the teaching? a. “I may have wine with dinner to help with relaxation.” b. “I may need to take this medication for up to a year.” c. “I may stop taking the medication when my symptoms go away.” d. “I should try psychotherapy or a support group in addition to the medication.” ANS: D BOOK QUESTIONS 1. The nurse suspects that a patient who is experiencing facial grimacing, involuntary upward eye movement, and muscle spasms of the tongue and face may have which condition? -Acute dystonia 2. A patient ask the nurse to explain how antipsychotic drugs work to make him feel better. The nurse understands that antipsychotics act in which way? -Blocking actions of epinephrine. 3. Ant antipsychotic agent, fluphenazine (Prolixin), is ordered for a patient with psychosis. The nurse understands that this agent can lead to extrapyramidal symptoms that may be treated with which medication? -benztropine (Cogentin) 4. An atypical antipsychotic is prescribed for a patient with psychosis. The nurse understands that this category of medications includes which drugs? (Select all that apply) -clozapine (Clozaril), olanzapine (Zyprexa), aripiprazole (Abilify) 5. A patient is prescribed lorazepam (Ativan). What does the nurse know to be true regarding lorazepam? -It may cause anterograde amnesia and sleep-related behaviors. 6. A patient is receiving aripiprazole (Abilify) Which nursing intervention(s) will the nurse include in the patient’s care plan? (Select all that apply) -Remain with the patient until the medication is swallowed, monitor vital signs to detect orthostatic hypotension, assess the patient for evidence of neuroleptic malignant syndrome. 7. A patient appears to have had an overdose of phenothiazines. The nurse anticipates that which interventions(s) may be used? (Select all that apply) - Gastric lavage, adequate hydration, maintaining an airway, activated charcoal administration. PHARM STUDY GUIDE QUESTIONS 1. Neuroleptic drugs are useful in the management of which type of illness? -Psychotic disorders 2. The patient has been started on an antipsychotic medication for treatment for her schizophrenia. She asks the nurse when she will start to feel better. What is the nurse’s best response? - “Responses vary, but it may be about 6 weeks.” 3. The patient has been started on chlorpromazine HCL (Thorazine) or treatment of intractable pain. What information will the nurse include in the patients education about this class of medication? - “Change positions slowly from sitting to standing to prevent orthostatic hypotension.” 4. Typical and traditional antipsychotic may cause extrapyramidal symptoms (EPS) or pseudoparkinsonism. Which symptom is considered an extrapyramidal symptom? - Shuffling gait. 5. What agent would the nurse expect to give to decrease EPS? - Benztropine (Cogentin) 6. Phenothiazines are grouped into 3 categories based on their side effects. In which group is fluphenazine (Prolixin)? - Piperazine 7. The patient has been prescribed fluphenazine (Prolixin) for treatment for schizophrenia. What information should the nurse include in the patient teaching for this medication? (select all that apply) -Notify your healthcare provider if you have dizziness, headache or nausea; This medication must be taken every day; You should not drink alcohol when taking this medication. 8. The 72 yr old patient has recently been diagnosed with schizophrenia. This healthcare provider has prescribed fluphenazine (Prolixin) 20 mg/day. In reviewing his medications prior to discharge from the hospital, the nurse notes the dose, What should concern the nurse about the amount of medication is prescribed? - The patients dose should be 25% to 50% less than the adult dose. 9. A patient presents to the emergency department with an overdose of chlorpromazine HCL (Thorazine). What is the priority action by the nurse? -Maintain the airway. 10. The 58 yr old patient presents to the emergency department. He is highly a danger to self and others. The healthcare provider has ordered haloperidol (Haldol) 5 mg IM. What should the nurse know about this medication when giving it as an antipsychotic? - It has a sedative effect on agitated, combative patients. 11. What is the drug category for atypical antipsychotics? - Serotonin/dopamine antagonists. 12. The atypical antipsychotics have a weak affinity for the D2 receptors. Consequently, what happens to the occurrence of EPS when taking these medications? -Fewers EPS 13. Atypical antipsychotics have a stronger affinity for which type of receptors that block serotonin receptors? - D4 14. The patient has just been prescribed risperidone (Risperdal). What side effects should the nurse include in the health teaching about this medication? -Hyperglycemia 15. The medication alprazolam belongs to which anxiolytic drug group? - Benzodiazepines 16. The patients has been started on IM extended release olanzapine due to noncompliance with the oral medication regimen. What does the nurse know is/are benefits(s) of this medication? (select all that apply) - Only required to be given every 2-4 weeks; few to no extrapyramidal symptoms; does not cause agranulocytosis. 17. Which patient(s) should not be taking fluphenazine (Prolixin)? (Select all that apply) -35 yr old with hepatitis C; 47 yr old with subcortical brain damage; 53 yr old with blood dyscrasias. 18. Lorazepam is an anxiolytic drug; however, it may be prescribed for other purposes. For which other conditions(s) might be prescribed? (select all that apply) - Alcohol withdrawal, anxiety associated with depression, preoperative medication, status epilepticus. Pharm Kee 23 1. An older patient exhibits a shuffling gait, lack of facial expression, and tremors at rest. The nurse will expect the provider to order which medication for this patient? a. Carbidopa-levodopa (Sinemet) b. Donepezil (Aricept) c. Rivastigmine (Exelon) d. Tacrine (Cognex) ANS: A 2. A nursing student asks the nurse to differentiate the pathology of Alzheimer’s disease from that of Parkinson’s disease. Which description is correct? a. Alzheimer’s disease involves a possible excess of acetylcholine and neuritic plaques. b. Alzheimer’s disease is caused by decreased amounts of dopamine and degeneration of cholinergic neurons. c. Parkinson’s disease is characterized by an imbalance of dopamine and acetylcholine. d. Parkinson’s disease involves increased dopamine production and decreased acetylcholine. ANS: C 3. The spouse of a patient newly diagnosed with mild, unilateral symptoms of Parkinson’s disease (PD) asks the nurse what, besides medication, can be done to manage the disease. The nurse will a. counsel the spouse that parkinsonism is a normal part of the aging process in some people. b. recommend exercise, nutritional counseling, and group support to help manage the disease. c. tell the spouse that the disease will not progress if mild symptoms are treated early. d. tell the spouse that medication therapy can be curative if drugs are begun in time. ANS: B 4. A patient who has Parkinson’s disease is being treated with the anticholinergic medication benztropine (Cogentin). The nurse will tell the patient that this drug will have which effect? a. Helping the patient to walk faster b. Improving mental function c. Minimizing symptoms of bradykinesia d. Reducing some of the tremors ANS: D 5. The nurse is preparing to administer a first dose of benztropine (Cogentin) to a patient diagnosed with parkinsonism. The nurse would notify the patient’s provider if the patient had a history of which condition? a. Asthma b. Glaucoma c. Hypertension d. Irritable bowel disease ANS: B 6. The nurse is caring for a patient who is receiving trihexyphenidyl (Artane) to treat parkinsonism. The patient reports having a dry mouth, and the nurse notes a urine output of 300 mL in the past 8 hours. Which action will the nurse perform? a. Encourage increased oral fluids. b. Obtain an order for intravenous fluids. c. Report the urine output to the provider. d. Request an order for renal function tests. ANS: C 7. A nursing student asks the nurse why patients who have parkinsonism receive a combination of carbidopa and levodopa. The nurse will explain that the combination product a. allows larger doses of levodopa to be given without causing increased adverse reactions. b. causes more levodopa to be converted to dopamine before crossing the blood-brain barrier. c. eliminates almost all drug side effects of both levodopa and carbidopa. d. reduces peripheral side effects by inhibiting decarboxylase in the peripheral nervous system. ANS: D 8. Which antiviral medication improves symptoms of Parkinson’s disease in some patients? a. Acyclovir (Zovirax) b. Amantadine HCl (Symmetrel) c. Interferon (INF) d. Zanamivir (Relenza) ANS: B 9. A patient who has parkinsonism has been taking carbidopa-levodopa and has shown improvement in symptoms but develops dystonic movements, nausea, and vomiting. Which medication will the nurse expect the provider to order for this patient to replace carbidopa-levodopa? a. Amantadine HCl (Symmetrel) b. Benztropine (Cogentin) c. Bromocriptine mesylate (Parlodel) d. Tacrine (Cognex) ANS: C 10. A patient who has parkinsonism will begin taking selegiline HCl (Eldepryl) to treat symptoms. What information will the nurse include when teaching this patient about this drug? a. “Avoid consuming foods that are high in tyramine.” b. “This drug will prevent the need to take levodopa.” c. “You may have red wine with dinner on occasion.” d. “You will not have serious drug interactions with this drug.” ANS: A 11. A patient who has parkinsonism will begin taking carbidopa-levodopa. What information will the nurse include when teaching this patient about this medication? a. “Call your health care provider immediately if your urine or perspiration turn a dark color.” b. “Rise slowly from your bed or your chair to avoid dizziness and falls.” c. “Take the drug with foods high in protein to improve drug delivery.” d. “Discontinue the drug if you experience insomnia.” ANS: B 12. The nurse is teaching a patient who has Parkinson’s disease about the side effects of carbidopa-levodopa. Which statement by the patient indicates a need for further teaching? a. “I may experience urinary retention, dry mouth, and constipation.” b. “I may feel dizzy at first, but this side effect will go away with time.” c. “I should report nightmares and mental disturbances to my provider.” d. “I should take the drug with food to increase absorption.” ANS: D 13. A patient is taking entacapone (Comtan) along with carbidopa-levodopa to treat parkinsonism. The nurse notes that the patient’s urine is orange in color. The nurse will a. notify the provider of possible drug toxicity. b. reassure the patent that this is a harmless side effect. c. request an order for liver function tests. d. request an order for a urinalysis. ANS: B 14. The nurse is caring for an 80-year-old patient who has Alzheimer’s disease who will begin taking rivastigmine (Exelon). What will the nurse include in the plan of care for this patient? a. Administer the drug once daily. b. Assist the patient to stand and walk. c. Give the drug with food to increase absorption. d. Use nonsteroidal anti-inflammatory drugs (NSAIDs) instead of acetaminophen for pain. ANS: B 15. The nurse is providing teaching for the family of a patient who has been newly diagnosed with Alzheimer’s disease (AD). Which statement by the family member indicates understanding of the teaching? a. “Alzheimer’s disease is a chronic, progressive condition.” b. “Alzheimer’s disease affects memory but not personality.” c. “The onset of Alzheimer’s disease is usually between 65 and 75 years.” d. “With proper treatment, symptoms of this disease can be arrested.” ANS: A 16. The nurse is teaching a family member about an elderly parent’s new prescription for tacrine (Cognex) to treat Alzheimer’s disease (AD). The family member asks what to expect from this drug. The nurse will respond that the patient will a. demonstrate improved ambulation. b. have reversal of all symptoms. c. have decreased deterioration of cognition. d. show improved communication ability. ANS: C 17. The nurse is helping to develop a plan of care for a patient who has advanced Alzheimer’s disease. The patient will be taking a new medication. Which is a realistic goal for this patient? a. Demonstrate improved cognitive function. b. Exhibit improved ability to provide self-care. c. Receive appropriate assistance for care needs. d. Show improved memory for recent events. ANS: C BOOK QUESTIONS 1. Which of the following assessment findings could the nurse see in a patient with parkinsonism? (Select all that apply) -Muscle rigidity, involuntary tremors, bradykinesia. 2. A patient is receiving carbidopa-levodopa for parkinsonism. What should the nurse know about this drug? - Carbidopa-levodopa may lead to excessive salivation. 3. The nurse has initiated teaching for a family member of a patient with Alzheimer’s disease. The nurse realizes more teaching is needed if the family member makes which statement? - “It may take several medication to cure the disease. 4. A patient is taking rivastigmine (Exelon). The nurse should teach the patient and family which information about rivastigmine? -That GI distress is a common side effect. 5. Which is a nursing intervention of a patient taking carbidopa-levodopa for parkinsonism? -Inform the patient that perspiration may be dark and stain clothing. 6. What would the nurse teach a patient who is taking anticholinergic therapy for parkinsonism (Select all that apply) - avoid alcohol, cigarettes and caffeine; relieve dry mouth with hard candy or ice chips; use sunglasses to reduce photophobia; receive routine eye examinations. 7. A patient is taking rivastigmine (Exelon) to improve cognitive function. What should the nurse teach the patient/family member to do? (Select all that apply) -Rise slowly to avoid dizziness; remove obstacles from pathways to avoid injury; closely follow the drug dosing schedule. CHAPTER 1 1. Which outcome, focused on recovery, would be expected in the plan of care for a patient living in the community with serious and persistent mental illness? Within 3 months, the patient will: a. deny suicidal ideation. b. report a sense of well-being. c. take medications as prescribed. d. attend clinic appointments on time. ANS: B Recovery emphasizes managing symptoms, reducing psychosocial disability, and improving role performance. The goal of recovery is to empower the individual with mental illness to achieve a sense of meaning and satisfaction in life and to function at the highest possible level of wellness. The incorrect options focus on the classic medical model rather than recovery. 2. A patient is hospitalized for depression and suicidal ideation after their spouse asks for a divorce. Select the nurse's most caring comment. a. "Let's discuss some means of coping other than suicide when you have these feelings." b. "I understand why you're so depressed. When I got divorced, I was devastated too." c. "You should forget about your marriage and move on with your life." d. "How did you get so depressed that hospitalization was necessary?" ANS: A The nurse's communication should evidence caring and a commitment to work with the patient. This commitment lets the patient know the nurse will help. Probing and advice are not helpful or therapeutic interventions. 3. In the shift-change report, an off-going nurse criticizes a patient who wears heavy makeup. Which comment by the nurse who receives the report best demonstrates advocacy? a. "This is a psychiatric hospital. Craziness is what we are all about." b. "Let's all show acceptance of this patient by wearing lots of makeup too." c. "Your comments are inconsiderate and inappropriate. Keep the report objective." d. "Our patients need our help to learn behaviors that will help them get along in society." ANS: D Accepting patients' needs for self-expression and seeking to teach skills that will contribute to their well-being demonstrate respect and are important parts of advocacy. The on-coming nurse needs to take action to ensure that others are not prejudiced against the patient. Humor can be appropriate within the privacy of a shift report but not at the expense of respect for patients. Judging the off-going nurse in a critical way will create conflict. Nurses must show compassion for each other. 4. A nurse assesses a newly admitted patient with depression. Which statement is an example of "attending"? a. "We all have stress in life. Being in a psychiatric hospital isn't the end of the world." b. "Tell me why you felt you had to be hospitalized to receive treatment for your depression." c. "You will feel better after we get some antidepressant medication started for you." d. "I'd like to sit with you a while so you may feel more comfortable talking with me." ANS: D Attending is a technique that demonstrates the nurse's commitment to the relationship and reduces feelings of isolation. This technique shows respect for the patient and demonstrates caring. Generalizations, probing, and false reassurances are nontherapeutic. 5. A patient shows the nurse an article from the Internet about a health problem. Which characteristic of the web site's address most alerts the nurse that the site may have biased and prejudiced information? a. Address ends in ".org." b. Address ends in ".com." c. Address ends in ".gov." d. Address ends in ".net." ANS: B Financial influences on a site are a clue that the information may be biased. ".com" at the end of the address indicates that the site is a commercial one. ".gov" indicates that the site is maintained by a government entity. ".org" indicates that the site is nonproprietary; the site may or may not have reliable information, but it does not profit from its activities. ".net" can have multiple meanings. 6. A nurse says, "When I was in school I learned to call upset patients by name to get their attention, but I read a descriptive research study that says that this approach doesn't work. I'm going stop calling patients by name." Which statement is the best appraisal of this nurse's comment? a. One descriptive research study rarely provides enough evidence to change practice. b. Staff nurses apply new research findings only with the help from clinical nurse specialists. c. New research findings should be incorporated into clinical algorithms before using them in practice. d. The nurse misinterpreted the results of the study. Classic tenets of practice do not change. ANS: A Descriptive research findings provide evidence for practice but must be viewed in relation to other studies before practice changes. One study is not enough. Descriptive studies are low on the hierarchy of evidence. Clinical algorithms use flow charts to manage problems and do not specify one response to a clinical problem. Classic tenets of practice should change as research findings provide evidence for change. 7. Two nursing students discuss career plans after graduation. One student wants to enter psychiatric nursing. The other student asks, "Why would you want to be a psychiatric nurse? All they do is talk. You'll lose your skills." Select the best response by the student interested in psychiatric nursing. a. "Psychiatric nurses practice in safer environments than other specialties. Nurse-to-patient ratios must be better because of the nature of patients' problems." b. "Psychiatric nurses use complex communication skills, as well as critical thinking, to solve multidimensional problems. I'm challenged by those situations." c. "I think I'll be good in the mental health field. I do not like clinical rotations in school, so I don't want to continue them after I graduate." d. "Psychiatric nurses don't have to deal with as much pain and suffering as medical surgical nurses. That appeals to me." ANS: B The practice of psychiatric nursing requires a different set of skills than medical surgical nursing, although substantial overlap does exist. Psychiatric nurses must be able to help patients with medical and mental health problems, reflecting the holistic perspective these nurses must have. Nurse-patient ratios and workloads in psychiatric settings have increased, similar to other specialties. Psychiatric nursing involves clinical practice, not simply documentation. Psychosocial pain is real and can cause as much suffering as physical pain. 8. Which research evidence would most influence a group of nurses to change their practice? a. Expert committee report of recommendations for practice b. Systematic review of randomized controlled trials c. Nonexperimental descriptive study d. Critical pathway ANS: B Research findings are graded using a hierarchy of evidence. A systematic review of randomized controlled trials is Level A and provides the strongest evidence for changing practice. Expert committee recommendations and descriptive studies lend less powerful and influential evidence. A critical pathway is not evidence; it incorporates research findings after they have been analyzed. 9. A bill introduced in Congress would reduce funding for the care of people with mental illnesses. A group of nurses write letters to their elected representatives in opposition to the legislation. Which role have the nurses fulfilled? a. Advocacy b. Attending c. Recovery d. Evidence-based practice ANS: A An advocate defends or asserts another's cause, particularly when the other person lacks the ability to do that for him- or herself. Examples of individual advocacy include helping patients understand their rights or make decisions. On a community scale, advocacy includes political activity, public speaking, and publication in the interest of improving the individuals with mental illness; the letter-writing campaign advocates for that cause on behalf of patients who are unable to articulate their own needs. 10. An informal group of patients discuss their perceptions of nursing care. Which comment best indicates a patient's perception that his or her nurse is caring? a. "My nurse always asks me which type of juice I want to help me swallow my medication." b. "My nurse explained my treatment plan to me and asked for my ideas about how to make it better." c. "My nurse told me that if I take all the medicines the doctor prescribes I will get discharged soon." d. "My nurse spends time listening to me talk about my problems. That helps me feel like I'm not alone." ANS: D Caring evidences empathic understanding, as well as competency. It helps change pain and suffering into a shared experience, creating a human connection that alleviates feelings of isolation. The remaining options give examples of statements that demonstrate advocacy or giving advice. 11. A patient who immigrated to the United States from Honduras was diagnosed with schizophrenia. The patient took an antipsychotic medication for 3 weeks but showed no improvement. Which resource should the treatment team consult for information on more effective medications for this patient? a. Clinical algorithm b. Clinical pathway c. Clinical practice guideline d. International Statistical Classification of Diseases and Related Health Problems (ICD) ANS: A A clinical algorithm is a guideline that describes diagnostic and/or treatment approaches drawn from large databases of information. These guidelines help the treatment team make decisions cognizant of an individual patient's needs, such as ethnic origin, age, or gender. A clinical pathway is a map of interventions and treatments related to a specific disorder. Clinical practice guidelines summarize best practices about specific health problems. The ICD classifies diseases. 12. Which historical nursing leader helped focus practice to recognize the importance of science in psychiatric nursing? a. Abraham Maslow b. Hildegard Peplau c. Kris Martinsen d. Harriet Bailey ANS: B Although all these leaders included science as an important component of practice, Hildegard Peplau most influenced its development in psychiatric nursing. Maslow was not a nurse, but his theories influence how nurses prioritize problems and care. Bailey wrote a textbook in the 1930s on psychiatric nursing interventions. Kris Martinsen emphasized the importance of caring in nursing practice. 13. A nurse consistently strives to demonstrate caring behaviors during interactions with patients. Which reaction by a patient indicates this nurse is effective? A patient reports feeling: a. distrustful l of others. b. connected with others. c. uneasy about the future. d. discouraged with efforts to improve. ANS: B A patient is likely to respond to caring with a sense of connectedness with others. The absence of caring can make patients feel distrustful, disconnected, uneasy, and discouraged. 1. An experienced nurse says to a new graduate, "When you've practiced as long as I have, you'll instantly know how to take care of psychotic patients." What is the new graduate's best analysis of this comment? Select all that apply. a. The experienced nurse may have lost sight of patients' individuality, which may compromise the integrity of practice. b. New research findings must be continually integrated into a nurse's practice to provide the most effective care. c. Experience provides mental health nurses with the tools and skills needed for effective professional practice. d. Experienced psychiatric nurses have learned the best ways to care for psychotic patients through trial and error. e. Effective psychiatric nurses should be continually guided by an intuitive sense of patients' needs. ANS: A, B Evidence-based practice involves using research findings to provide the most effective nursing care. Evidence is continually emerging; therefore, nurses cannot rely solely on experience. The effective nurse also maintains respect for each patient as an individual. Overgeneralization compromises that perspective. Intuition and trial and error are unsystematic approaches to care. 2. Which patient statements identify qualities of nursing practice with high therapeutic value? (Select all that apply.) "The nurse: a. talks in language I can understand." b. helps me keep track of my medications." c. is willing to go to social activities with me." d. lets me do whatever I choose without interfering." e. looks at me as a whole person with different needs." ANS: A, B, E Each correct answer demonstrates caring is an example of appropriate nursing foci: communicating at a level understandable to the patient, using holistic principles to guide care, and providing medication supervision. The incorrect options suggest a laissez-faire attitude on the part of the nurse, when the nurse should instead provide thoughtful feedback and help patients test alternative solutions or violate boundaries. BOOK QUESTIONS 1. In which scenario is it most urgent for the nurse to act as a patient advocate? A. An Adult cries and experiences anxiety after a near- miss MVA on the way to work. B. A homeless adult diagnosed with schizophrenia lives in a community expecting a category 5 hurricane C. A 14-year-old girl’s grades decline because she consistently focuses on her appearance and social networking D. A parent allows the prescription to lapse for 1 day for their 8 year old child’s medications for ADHD. Answer: B 2. The nurse interacts with a veteran of WWII. The veteran says, “Veterans modern wars whine and complain all the time. Back when I was in the service, you kept your feelings to your self.” Select the nurse’s best response. A. “American society in the 1940s expected WWII soldiers to be strong.” B. “WWII was fought in traditional way but the enemy is more difficult to identify in todays wars.” C. “We now have a better understanding of how trauma affects people and the importance of research based, compassionate care.” D. “Intermittent explosive disorder (IEDs), which were not used during WWII, produce traumatic brain injuries that must be treated.” Answer: C 3. A patient reports to the primary care provider about sleeplessness, constant fatigue, and sadness. In our current health care climate, what is most likely the treatment approach that would be offered to this patient? A. Group therapy B. Individual psychotherapy C. Complimentary Therapy D. Psychopharmacological Therapy Answer: D 4. The nurse prepares outcomes to the pan of care for an adult diagnosed with mental illness. Which strategy recognizes the current focus of treatment services for this population? A. The pt diagnoses are confirmed using advances neuroimaging techniques B. The nurse confers with the tx team to verify the pts significant disability C. The nurse prioritizes problems accordance to Maslow’s hierarchy of needs D. The pt and family participates actively in establishing priorities and selecting interventions. Answer: D 5. Which scenario best demonstrates empathetic Caring? A. A nurse provides comfort to a colleague after an error of medication administration B. A nurse works a fourth extra shift in 1 week to maintain adequate unit staffing. C. A nurse identifies a violation of confidentiality and makes a report to an agency privacy officer D. A nurse conscientiously reads current literature to stay aware of new EBP. Answer: A CHAPTER 2 1. A nurse explains multiaxial diagnoses to a psychiatric technician. Which information is accurate? a. It is a template for treatment planning. b. Nursing and medical diagnoses are included. c. Assessments of several aspects of functioning are included. d. It incorporates the framework of a specific biopsychosocial theory. ANS: C The use of five axes requires an assessment beyond the diagnosis of a mental disorder and includes relevant medical conditions, psychosocial and environmental problems, and global assessment of functioning. The Diagnostic and Statistical Manual of Mental Disorders (fourth edition, text revision) (DSM-IV-TR) is not a template for treatment planning and does not use a specific biopsychosocial theory. Nursing diagnoses are not included in multiaxial diagnoses. 2. Which documentation of diagnosis would a nurse expect for a person with mental illness? a. I Rheumatoid arthritis II 100 III Posttraumatic stress disorder IV Arrested for shoplifting 2 months earlier V None b. I Mental retardation II Histrionic personality disorder III 75 IV Hypertension V Home destroyed by tornado last year c. I Schizophrenia, paranoid II Death of spouse last year III 40 IV None V Alcohol abuse d. I Generalized anxiety disorder II Avoidant personality disorder III Fibromyalgia IV Declared bankruptcy 6 months ago V 60 ANS: D The option beginning with a diagnosis of generalized anxiety disorder places a clinical disorder on Axis I, a personality disorder on Axis II, a medical problem on Axis III, a psychosocial problem on Axis IV, and global assessment of functioning (GAF) on Axis V. The other options misplace and incorrectly order the clinical data. 3. A patient is admitted to the psychiatric hospital for assessment and evaluation. Which assessment finding best indicates that the patient has a mental illness? The patient: a. describes coping and relaxation strategies used when feeling anxious. b. describes mood as consistently sad, discouraged, and hopeless. c. can perform tasks attempted within the limits of own abilities. d. reports occasional problems with insomnia. ANS: B A patient who reports having a consistently negative mood is describing a mood alteration. The incorrect options describe mentally healthy behaviors and common problems that do not indicate mental illness. 4. The goal for a patient is to increase resiliency. Which outcome should a nurse add to the plan of care? Within 3 days, the patient will: a. describe feelings associated with loss and stress. b. meet own needs without considering the rights of others. c. identify healthy coping behaviors in response to stressful events. d. allow others to assume responsibility for major areas of own life. ANS: C The patient's ability to identify healthy coping behaviors indicates adaptive, healthy behavior and demonstrates an increased ability to recover from severe stress. Describing feelings associated with loss and stress does not move the patient toward adaptation. The remaining options are maladaptive behaviors. 5. A nurse at a behavioral health clinic sees an unfamiliar psychiatric diagnosis on a patient's insurance form. Which resource should the nurse consult to discern the criteria used to establish this diagnosis? a. DSM-IV-TR b. Nursing Diagnosis Manual c. A psychiatric nursing textbook d. A behavioral health reference manual ANS: A The DSM-IV-TR gives the criteria used to diagnose each mental disorder. The Nursing Diagnosis Manual focuses on nursing diagnoses. A psychiatric nursing textbook or behavioral health reference manual may not contain diagnostic criteria. 6. A nurse must assess several new patients at a community mental health center. Conclusions concerning current functioning should be made on the basis of: a. the degree of conformity of the individual to society's norms. b. the degree to which an individual is logical and rational. c. a continuum from mentally healthy to unhealthy. d. the rate of intellectual and emotional growth. ANS: C Because mental health and mental illness are relative concepts, assessment of functioning is made by using a continuum. Mental health is not based on conformity; some mentally healthy individuals do not conform to society's norms. Most individuals occasionally display illogical or irrational thinking. The rate of intellectual and emotional growth is not the most useful criterion to assess mental health or mental illness. 7. A 40-year-old adult living with parents states, "I'm happy but I don't socialize much. My work is routine. When new things come up, my boss explains them a few times to make sure I understand. At home, my parents make decisions for me, and I go along with them." A nurse should identify interventions to improve this patient's: a. self-concept. b. overall happiness. c. appraisal of reality. d. control over behavior. ANS: A The patient feels the need for multiple explanations of new tasks at work and, despite being 40 years of age, allows both parents to make all decisions. These behaviors indicate a poorly developed self-concept. 8. A patient tells a nurse, "I have psychiatric problems and am in and out of hospitals all the time. Not one of my friends or relatives has these problems." Select the nurse's best response. a. "Comparing yourself with others has no real advantages." b. "Why do you blame yourself for having a psychiatric illness?" c. "Mental illness affects 50% of the adult population in any given year." d. "It sounds like you are concerned that others don't experience the same challenges as you." ANS: D Mental illness affects many people at various times in their lives. No class, culture, or creed is immune to the challenges of mental illness. The correct response also demonstrates the use of reflection, a therapeutic communication technique. It is not true that mental illness affects 50% of the population in any given year. Asking patients if they blame themselves is an example of probing. 9. A critical care nurse asks a psychiatric nurse about the difference between a DSM-IV-TR diagnosis and a nursing diagnosis. Select the psychiatric nurse's best response. a. "No functional difference exists between the two diagnoses. Both serve to identify a human deviance." b. "The DSM-IV-TR diagnosis disregards culture, whereas the nursing diagnosis includes cultural variables." c. "The DSM-IV-TR diagnosis profiles present distress or disability, whereas a nursing diagnosis considers past and present responses to actual mental health problems." d. "The DSM-IV-TR diagnosis influences the medical treatment; the nursing diagnosis offers a framework to identify interventions for problems a patient has or may experience." ANS: D The medical diagnosis is concerned with the patient's disease state, causes, and cures, whereas the nursing diagnosis focuses on the patient's response to stress and possible caring interventions. Both the DSM-IV-TR and a nursing diagnosis consider culture. The DSM-IV-TR is multiaxial. Nursing diagnoses also consider potential problems. 10. A nurse finds a new patient uncommunicative about recent life events. The nurse suspects marital and economic problems. The social worker's assessment is not available. The most effective action the nurse can take is to: a. ask the patient who shares a room with him or her. b. consult Axis IV of the DSM-IV-TR in the medical record. c. focus questions on the topics of marital and economic issues. d. delay discussion of these topics until the social worker's assessment is available. ANS: B The physician's admission note identifies psychosocial and environmental problems on Axis IV pertinent to the patient's situation, providing another source of information for the nurse. Asking the patient who shares a room with him or her violates patient privacy rights. Persistent questioning may cause the patient to withdraw. Delaying the discussion until the social worker's assessment is available is not an effective solution. 11. A newly admitted patient is profoundly depressed, mute, and motionless. The patient has refused to bathe and eat for a week. Which score would be expected on the patient's global assessment of functioning? a. 100 b. 80 c. 50 d. 10 ANS: D The patient is unable to maintain personal hygiene, oral intake, or verbal communication. The patient is dangerous to self because of the potential for starvation. A GAF score of 100 indicates high-level functioning. A score of 80 or 50 suggests higher functional abilities than the patient presently displays. 12. The spouse of a patient with schizophrenia says, "I don't understand why childhood experiences have anything to do with this disabling illness." Select the nurse's response that will best help the spouse understand this condition. a. "Psychological stress is actually at the root of most mental disorders." b. "We now know that all mental illnesses are the result of genetic factors." c. "It must be frustrating for you that your spouse is sick so much of the time." d. "Although this disorder more likely has a biological rather than psychological origin, the support and involvement of caregivers is very important." ANS: D Many of the most prevalent and disabling mental disorders have been found to have strong biological influences. Helping the spouse understand the importance of his or her role as a caregiver is also important. Empathy is important but does not increase the spouse's level of knowledge about the cause of the patient's condition. Not all mental illnesses are the result of genetic factors. Psychologic stress is not at the root of most mental disorders. 13. Which belief by a nurse supports the highest degree of patient advocacy during a multidisciplinary patient care planning session? a. All mental illnesses are culturally determined. b. Schizophrenia and bipolar disorder are cross-cultural disorders. c. Symptoms of mental disorders are constant from culture to culture. d. Some symptoms of mental disorders reflect a person's cultural patterns. ANS: D A nurse who understands that a patient's symptoms are influenced by culture will be able to advocate for the patient to a greater degree than a nurse who believes that culture is of little relevance. All mental illnesses are not culturally determined. Schizophrenia and bipolar disorder are cross-cultural disorders, but this understanding has little relevance to patient advocacy. Symptoms of mental disorders change from culture to culture. 14. A patient's history shows intense and unstable relationships with others. The patient initially idealizes an individual and then devalues the person when the patient's needs are not met. Which aspect of mental health is a problem? a. Effectiveness in work b. Communication skills c. Productive activities d. Fulfilling relationships ANS: D The information provided centers on relationships with others, which are described as intense and unstable. The relationships of mentally healthy individuals are stable, satisfying, and socially integrated. Data are not present to describe work effectiveness, communication skills, or activities. 15. In the majority culture of the United States, which individual is at greatest risk to be incorrectly labeled mentally ill? a. Person who is usually pessimistic but strives to meet personal goals b. Wealthy person who gives $20 bills to needy individuals in the community c. Person with an optimistic viewpoint about life and getting his or her own needs met d. Person who attends a charismatic church and describes hearing God's voice ANS: D Hearing voices is generally associated with mental illness; however, in charismatic religious groups, hearing the voice of God or a prophet is a desirable event. In this situation, cultural norms vary, making it more difficult to make an accurate DSM-IV-TR diagnosis. The individuals described in the other options are less likely to be labeled as mentally ill. 16. A psychiatric nurse addresses Axis I of the DSM-IV-TR as the focus of care but also considers the presence of other long-term, nonmedical disorders that may affect treatment. To which axis should the nurse refer for this information? a. II b. III c. IV d. V ANS: A Axis II refers to personality disorders and mental retardation. Together, Axis I and Axis II constitute the classification of abnormal behavior diagnosed in the individual. Axis III indicates any relevant general medical conditions. Axis IV reports psychosocial and environmental problems that may affect the diagnosis, treatment, and prognosis. Axis V is the GAF score. 17. A mentally ill person's current global assessment of functioning (GAF) score is 10. Select the nurse's highest priority related to this patient's care. a. Safety b. Hygiene c. Nutrition d. Socialization ANS: A This low GAF score indicates a consistent risk for self-harm exists; therefore the nurse's highest priority is safety. 18. A participant at a community education conference asks, "What is the most prevalent mental disorder in the United States?" Select the nurse's best response. a. "Why do you ask?" b. "Schizophrenia" c. "Affective disorders" d. "Anxiety disorders" ANS: D The prevalence for schizophrenia is 1.1% per year. The prevalence of all affective disorders (e.g., depression, dysthymia, bipolar) is 9.5%. The prevalence of anxiety disorders is 13.3%. 19. A nurse wants to find a description of diagnostic criteria for a person with schizophrenia. Which resource should the nurse consult? a. U.S. Department of Health and Human Services b. Journal of the American Psychiatric Association c. North American Nursing Diagnosis Association (NANDA) International d. DSM-IV-TR ANS: D The DSM-IV-TR identifies diagnostic criteria for psychiatric diagnoses. The other sources have useful information but are not the best resources for finding a description of the diagnostic criteria for a psychiatric disorder. 1. A patient asks a nurse, "The pamphlet I read about depression says psychosocial factors influence depression. What does that mean?" Which examples could the nurse cite to support the information? Select all that apply. a. Having a hostile family b. Having an over- or underinvolved family c. Having two first-degree relatives with bipolar disorder d. Experiencing the sudden death of a parent or loved one e. Feeling strong guilt over having an abortion when one's religion forbids it f. Experiencing symptom remission when treated with an antidepressant medication ANS: A, B, D, E Family influence is considered a psychosocial factor affecting a patient's mental health. A hostile under- or overinvolved family is critical of the patient and contributes to low self-esteem. Religious influences are considered psychosocial in nature. Life experiences, especially crises and losses, are considered psychosocial influences on mental health. Having two first-degree relatives with bipolar disorder would be considered a factor that influences the individual's risk for mental disorder, but it is a genetic, not psychosocial, factor. Treatment with a biological agent such as an antidepressant medication is an example of a biological influence. 2. A patient in the emergency department reports, "I hear voices saying someone is stalking me. They want to kill me because I found the cure for cancer. I will stab anyone that threatens me." Which aspects of mental health have the greatest immediate concern to a nurse? Select all that apply. a. Happiness b. Appraisal of reality c. Control over behavior d. Effectiveness in work e. Healthy self-concept ANS: B, C, E The aspects of mental health of greatest concern are the patient's appraisal of and control over behavior. The patient's appraisal of reality is inaccurate, and auditory hallucinations are evident, as well as delusions of persecution and grandeur. In addition, the patient's control over behavior is tenuous, as evidenced by the plan to "stab" anyone who seems threatening. A healthy self-concept is lacking. Data are not present to suggest that the other aspects of mental health (happiness and effectiveness in work) are of immediate concern. 3. Which statements most clearly reflect the stigma of mental illness? Select all that apply. a. "Many mental illnesses are hereditary." b. "Mental illness can be evidence of a brain disorder." c. "People claim mental illness so they can get disability checks." d. "If people with mental illness went to church, they would be fine." e. "Mental illness is a result of the breakdown of the American family." ANS: C, D, E Stigma is represented by judgmental remarks that discount the reality and validity of mental illness. Many mental illnesses are genetically transmitted. Neuroimaging can show changes associated with some mental illnesses. BOOK QUESTIONS: 1. A mentally ill gunman opens fire in a crowded in a crowed movie theater, killing 6 people and injuring others. Which comment about this event by a member of the community most clearly shows the stigma of mental illness? A. “Gun control laws are inadequate in our country.” B. “All there people with mental illness are violent and should be locked up.” C. “ Its frightening to feel that it is not safe to go to the movie theater.” D. “These events happen because American families no longer go to church together.” Answer: B 2. The nurse presents a class about mental health and mental illness to a group of 4th graders. One student asks, “why do people get mentally ill? Select the nurses best response. A. “there are many reasons why mental illness occur.” B. “the cause of mental illness is complicated and very hard to understand.” C. “sometimes a person’s brain does not work correctly because something bad happens or they inherit a brain problem.” D. “Most mental illnesses result in genetically transmitted abnormalities in cerebral structure; however, some are a consequence of traumatic life experience.” Answer: C 3. An adult experienced a Spinal cord injury resulting in the quadriplegia 3 years ago and now lives permanently in a skilled care facility. Which Comment by this patient best demonstrates resiliency? Answer: “Being Paralyzed taken things from me but it hasn’t kept me from being mentally involved in life.” 4. A nursing assistant says to the nurse, “ the schizophrenic in room 226 has been rambling all day.” When considering the nurses responsibility to manage the ancillary staff, which response should the nurse provide? Answer: “Its more respectful to refer to the patient by name that by diagnosis. 5. Which scenario meets the criteria for “normal” behavior? Answer: a 43 year old man cries privately 1 month after the death of his wife. CHAPTER 3 1. A 26-month-old child displays negative behaviors. The parent says, "My child refuses toilet training and shouts, 'No!' when given direction. What do you think is wrong?" Select the nurse's best reply. a. "This is normal for your child's age. The child is striving for independence." b. "The child needs firmer control. Punish the child for disobedience and say, 'No.'" c. "There may be developmental problems. Most children are toilet trained by age 2 years." d. "Some undesirable attitudes are developing. A child psychologist can help you develop a remedial plan." ANS: A These negative behaviors are typical of a child around the age of 2 years whose developmental task is to develop autonomy. The remaining options indicate the child's behavior is abnormal. 2. A 26-month-old child displays negative behavior, refuses toilet training, and often shouts, "No!" when given directions. Using Freud's stages of psychosexual development, a nurse would assess the child's behavior is based on which stage? a. Oral b. Anal c. Phallic d. Genital ANS: B In Freud's stages of psychosexual development, the anal stage occurs from age 1 to 3 years and has, as its focus, toilet training and learning to delay immediate gratification. The oral stage occurs between birth and 1 year, the phallic stage occurs between 3 and 5 years, and the genital stage occurs between 13 and 20 years. 3. A 26-month-old child displays negative behavior, refuses toilet training, and often shouts, "No!" when given direction. The nurse's counseling with the parent should be based on the premise that the child is engaged in which of Erikson's psychosocial crises? a. Trust versus Mistrust b. Initiative versus Guilt c. Industry versus Inferiority d. Autonomy versus Shame and Doubt ANS: D The crisis of Autonomy versus Shame and Doubt is related to the developmental task of gaining control of self and environment, as exemplified by toilet training. This psychosocial crisis occurs during the period of early childhood. Trust versus Mistrust is the crisis of the infant, Initiative versus Guilt is the crisis of the preschool and early school-aged child, and Industry versus Inferiority is the crisis of the 6- to 12-year-old child. 4. A 4-year-old child grabs toys from siblings, saying, "I want that toy now!" The siblings cry, and the child's parent becomes upset with the behavior. Using the Freudian theory, a nurse can interpret the child's behavior as a product of impulses originating in the: a. id. b. ego. c. superego. d. preconscious. ANS: A The id operates on the pleasure principle, seeking immediate gratification of impulses. The ego acts as a mediator of behavior and weighs the consequences of the action, perhaps determining that taking the toy is not worth the parent's wrath. The superego would oppose the impulsive behavior as "not nice." The preconscious is a level of awareness. 5. The parent of a 4-year-old rewards and praises the child for helping a younger sibling, being polite, and using good manners. A nurse supports the use of praise because according to the Freudian theory, these qualities will likely be internalized and become part of the child's: a. id. b. ego. c. superego. d. preconscious. ANS: C In the Freudian theory, the superego contains the "thou shalts" or moral standards internalized from interactions with significant others. Praise fosters internalization of desirable behaviors. The id is the center of basic instinctual drives, and the ego is the mediator. The ego is the problem-solving and reality-testing portion of the personality that negotiates solutions with the outside world. The preconscious is a level of awareness from which material can be easily retrieved with conscious effort. 6. A nurse supports parental praise of a child who is behaving in a helpful way. When the individual behaves with politeness and helpfulness in adulthood, which feeling will most likely result? a. Guilt b. Anxiety c. Loneliness d. Self-esteem ANS: D The individual will be living up to the ego ideal, which will result in positive feelings about self. The other options are incorrect; each represents a negative feeling. 7. A patient comments, "I never know the right answer" and "My opinion is not important." Using Erikson's theory, which psychosocial crisis did the patient have difficulty resolving? a. Initiative versus Guilt b. Trust versus Mistrust c. Autonomy versus Shame and Doubt d. Generativity versus Self-Absorption ANS: C These statements show severe self-doubt, indicating that the crisis of gaining control over the environment is not being successfully met. Unsuccessful resolution of the crisis of Initiative versus Guilt results in feelings of guilt. Unsuccessful resolution of the crisis of Trust versus Mistrust results in poor interpersonal relationships and suspicion of others. Unsuccessful resolution of the crisis of Generativity versus Self-Absorption results in self-absorption that limits the ability to grow as a person. 8. Which patient statement would lead a nurse to suspect that the developmental task of infancy was not successfully completed? a. "I have very warm and close friendships." b. "I'm afraid to let anyone really get to know me." c. "I am always right, so don't bother saying more." d. "I'm ashamed that I didn't do it correctly in the first place." ANS: B According to Erikson, the developmental task of infancy is the development of trust. The patient's statement that he or she is afraid of becoming acquainted with others clearly shows a lack of ability to trust other people. Having warm and close friendships suggests the developmental task of infancy was successfully completed. The third option suggests rigidity rather than mistrust. The fourth option suggests failure to resolve the crisis of Initiative versus Guilt. 9. A nurse assesses that a patient is suspicious and frequently manipulates others. Using the Freudian theory, these traits are related to which psychosexual stage? a. Oral b. Anal c. Phallic d. Genital ANS: A According to Freud, each of the behaviors mentioned develops as the result of attitudes formed during the oral stage, when an infant first learns to relate to the environment. Anal stage traits include stinginess, stubbornness, orderliness, or their opposites. Phallic stage traits include flirtatiousness, pride, vanity, difficulty with authority figures, and difficulties with sexual identity. Genital stage traits include the ability to form satisfying sexual and emotional relationships with members of the opposite sex, emancipation from parents, and a strong sense of personal identity. 10. An adult expresses the wish to be taken care of and often behaves in a helpless fashion. This adult has needs related to which of Freud's stages of psychosexual development? a. Latency b. Phallic c. Anal d. Oral ANS: D According to Freud, fixation at the oral stage sometimes produces dependent infantile behaviors in adults. Latency fixations often result in a difficulty identifying with others and developing social skills, resulting in a sense of inadequacy and inferiority. Phallic fixations result in having difficulty with authority figures and poor sexual identity. Anal fixation sometimes results in retentiveness, rigidity, messiness, destructiveness, and cruelty. 11. A nurse listens to a group of recent retirees. One says, "I volunteer with Meals on Wheels, coach teen sports, and do church visitation." Another laughs and says, "I'm too busy taking care of myself to volunteer. I don't have time to help others." These comments contrast which developmental tasks? a. Trust versus Mistrust b. Industry versus Inferiority c. Intimacy versus Isolation d. Generativity versus Self-Absorption ANS: D Both retirees are in middle adulthood, when the developmental crisis to be resolved is Generativity versus Self-Absorption. One exemplifies generativity; the other embodies self-absorption. The developmental crisis of Trust versus Mistrust would show a contrast between relating to others in a trusting fashion and being suspicious and lacking trust. Failure to negotiate the developmental crisis of Industry versus Inferiority would result in a sense of inferiority or difficulty learning and working as opposed to the ability to work competently. Behaviors that would be contrasted in the crisis of Intimacy versus Isolation would be emotional isolation and the ability to love and commit to oneself. 12. Cognitive therapy was provided for a patient who frequently said, "I'm stupid." Which statement by the patient indicates the therapy was effective? a. "I'm disappointed in my lack of ability." b. "I always fail when I try new things." c. "Things always go wrong for me." d. "Sometimes I do stupid things." ANS: D "I'm stupid" is an irrational thought. A more rational thought is, "Sometimes I do stupid things." The latter thinking promotes emotional self-control. The incorrect options reflect irrational thinking. 13. A student nurse tells the instructor, "I don't need to interact with my patients. I learn what I need to know by observation." The instructor can best interpret the nursing implications of Sullivan's theory to the student by responding: a. "nurses cannot be isolated. We must interact to provide patients with opportunities to practice interpersonal skills." b. "observing patient interactions can help you formulate priority nursing diagnoses and appropriate interventions." c. "I wonder how accurate your assessment of the patient's needs can be if you do not interact with the patient." d. "noting patient behavioral changes is important because these signify changes in personality." ANS: A Sullivan believed that the nurse's role includes educating patients and assisting them in developing effective interpersonal relationships. Mutuality, respect for the patient, unconditional acceptance, and empathy are cornerstones of Sullivan's theory. The nurse who does not interact with the patient cannot demonstrate these cornerstones. Observations provide only objective data. Priority nursing diagnoses usually cannot be accurately established without subjective data from the patient. The third response pertains to Maslow's theory. The fourth response pertains to behavioral theory. 14. A psychiatric technician says, "Little of what takes place on the behavioral health unit seems to be theory based." A nurse educates the technician by identifying which common use of Sullivan's theory? a. Structure of the therapeutic milieu of most behavioral health units b. Frequent use of restraint and seclusion as behavior management tools c. Assessment tools based on age-appropriate versus arrested behaviors d. Method nurses use to determine the best sequence for nursing actions ANS: A The structure of the therapeutic environment has, as its foci, an accepting atmosphere and provision of opportunities for practicing interpersonal skills. Both constructs are directly attributable to Sullivan's theory of interpersonal relationships. Sullivan's interpersonal theory did not specifically consider the use of restraint or seclusion. Assessment based on the developmental level is more the result of Erikson's theories. Sequencing nursing actions based on the priority of patient needs is related to Maslow's hierarchy of needs. 15. A nurse uses Maslow's hierarchy of needs to plan care for a psychotic patient. Which problem will receive priority? The patient: a. refuses to eat or bathe. b. reports feelings of alienation from family. c. is reluctant to participate in unit social activities. d. need to be taught about medication action and side effects. ANS: A The need for food and hygiene is physiological and therefore takes priority over psychological or meta-needs in care planning. 16. Operant conditioning will be used to encourage speech in a child who is nearly mute. Which technique would a nurse include in the treatment plan? a. Ignore the child for using silence. b. Have the child observe others talking. c. Give the child a small treat for speaking. d. Teach the child relaxation techniques, then coax speech. ANS: C Operant conditioning involves giving positive reinforcement for a desired behavior. Treats are rewards to reinforce speech. Ignoring the child will not change the behavior. Having the child observe others describes modeling. Teaching relaxation techniques and then coaxing speech is an example of systematic desensitization. 17. The parent of a child who has schizophrenia tearfully asks a nurse, "What could I have done differently to prevent this illness?" Select the nurse's most caring response. a. "Although schizophrenia is caused by impaired family relationships, try not to feel guilty. No one can predict how a child will respond to parental guidance." b. "Most of the damage is done, but there is still hope. Changing your parenting style can help your child learn to cope more effectively with the environment." c. "Schizophrenia is a biological illness with similarities to diabetes and heart disease. You are not to blame for your child's illness." d. "Most mental illnesses result from genetic inheritance. Your genes are more at fault than your parenting." ANS: C Patients and families need reassurance that the major mental disorders are biological in origin and are not the "fault" of parents. Knowing the biological nature of the disorder relieves feelings of guilt over being responsible for the illness. The incorrect responses are neither wholly accurate nor reassuring; they fall short of being reassuring and place the burden of having faulty genes on the shoulders of the parents. 18. A nurse uses Peplau's interpersonal therapy while working with an anxious, withdrawn patient. Interventions should focus on: a. changing the patient's perceptions about self b. improving the patient's interactional skills c. using medications to relieve anxiety d. reinforcing specific behaviors ANS: B The nurse-patient relationship is structured to provide a model for adaptive interpersonal relationships that can be generalized to others. Changing the patient's perceptions about his- or herself would be appropriate for cognitive therapy. Reinforcing specific behaviors would be used in behavioral therapy. Using medications would be the focus of biological therapy. 19. A patient underwent psychotherapy weekly for 3 years. The therapist used free association, dream analysis, and facilitated transference to help the patient understand unconscious processes and foster personality changes. Which type of therapy was used? a. Short-term dynamic psychotherapy b. Transactional analysis c. Cognitive therapy d. Psychoanalysis ANS: D The therapy described is traditional psychoanalysis. Short-term dynamic psychotherapy would last less than 1 year. Neither transactional analysis nor cognitive therapy makes use of the techniques described. 20. An advanced practice nurse determines a group of patients would benefit from therapy in which peers and interdisciplinary staff all have a voice in determining the level of the patients' privileges. The nurse would arrange for: a. milieu therapy b. cognitive therapy c. short-term dynamic therapy d. systematic desensitization ANS: A Milieu therapy is based on the idea that all members of the environment contribute to the planning and functioning of the setting. The other therapies are all individual therapies that do not fit the description. 21. A nurse psychotherapist works with an anxious, dependent patient. The therapeutic strategy most consistent with the framework of psychoanalytic psychotherapy is: a. emphasizing medication compliance b. identifying the patient's strengths and assets c. offering psychoeducational materials and groups d. focusing on feelings developed by the patient toward the nurse ANS: D Positive or negative feelings of the patient toward the nurse or therapist represent transference. Transference is a psychoanalytic concept that can be used to explore previously unresolved conflicts. Emphasizing medication compliance is more related to biological therapy. Identifying patient strengths and assets would be consistent with supportive psychotherapy. The use of psychoeducational materials is a common "homework" assignment used in cognitive therapy. 22. A person tells a nurse, "I was the only survivor in a small plane crash, but three business associates died. I got anxious and depressed and saw a counselor three times a week for a month. We talked about my feelings related to being a survivor, and now I'm fine, back to my old self." Which type of therapy was used? a. Milieu therapy b. Psychoanalysis c. Behavior modification d. Interpersonal therapy ANS: D Interpersonal therapy returns the patient to the former level of functioning by helping the patient come to terms with the loss of friends and guilt over being a survivor. Milieu therapy refers to environmental therapy. Psychoanalysis calls for a long period of exploration of unconscious material. Behavior modification focuses on changing a behavior rather than helping the patient understand what is going on in his or her life. 23. A cognitive strategy a nurse could use to assist a very dependent patient would be to help the patient: a. reveal dream content. b. take prescribed medications. c. examine thoughts about being independent. d. role model ways to ask for help from others. ANS: C Cognitive theory suggests that one's thought processes are the basis of emotions and behavior. Changing faulty learning makes the development of new adaptive behaviors possible. Revealing dream content would be used in psychoanalytical therapy. Taking prescribed medications is an intervention associated with biological therapy. A dependent patient needs to develop independence. 24. A single parent is experiencing feelings of inadequacy related to work and family since one teenaged child ran away several weeks ago. The parent seeks the help of a therapist specializing in cognitive therapy. The psychotherapist who uses cognitive therapy will treat the patient by: a. discussing ego states b. focusing on unconscious mental processes c. negatively reinforcing an undesirable behavior d. helping the patient identify and change faulty thinking ANS: D Cognitive therapy emphasizes the importance of changing erroneous ways people think about themselves. Once faulty thinking changes, the individual's behavior changes. Focusing on unconscious mental processes is a psychoanalytic approach. Negatively reinforcing undesirable behaviors is behavior modification, and discussing ego states relates to transactional analysis. 25. A person received an invitation to be in the wedding of a friend who lives across the country. The individual is afraid of flying. What type of therapy should the nurse recommend? a. Psychoanalysis b. Milieu therapy c. Systematic desensitization d. Short-term dynamic therapy ANS: C Systematic desensitization is a type of therapy aimed at extinguishing a specific behavior, such as the fear of flying. Psychoanalysis and short-term dynamic therapy are aimed at uncovering conflicts. Milieu therapy involves environmental factors. 1. A basic level registered nurse works with patients in a community setting. Which groups should this nurse expect to lead? Select all that apply. a. Symptom management b. Medication education c. Family therapy d. Psychotherapy e. Self-care ANS: A, B, E Symptom management, medication education, and self-care groups represent psychoeducation, a service provided by the basic level registered nurse. Advanced practice registered nurses provide family therapy and psychotherapy. 2. A patient states, "I'm starting cognitive behavioral therapy. What can I expect from the sessions?" Which responses by the nurse are appropriate? Select all that apply. a. "The therapist will be active and questioning." b. "You may be given homework assignments." c. "The therapist will ask you to describe your dreams." d. "The therapist will help you look at ideas and beliefs you have about yourself." e. "The goal is to increase your subjectivity about thoughts that govern your behavior." ANS: A, B, D Cognitive therapists are active rather than passive during therapy sessions because they help patients to reality test their thinking. Homework assignments are given and completed outside the therapy sessions. Homework is usually discussed at the next therapy session. The goals of cognitive therapy are to assist the patient to identify inaccurate cognitions, to reality test their thinking, and to formulate new, accurate cognitions. Dream describing applies to psychoanalysis, not cognitive behavioral therapy. The desired outcome of cognitive therapy is to assist patients in increasing their objectivity, not subjectivity, about the cognitions that influence behavior. BOOK QUESTIONS: 1. A Nurse plans a group meeting for adult pts in a therapeutic Milieu. Which topic should the nurse include? Answer: The importance of hand washing 2. Considering Maslows pyramid, which comment indicates individual is motivated by the highest level of need? Answer: “Ill help my community by volunteering at a thrift shop that raises money for the poor.” 3. Which patient is likely to achieve maximum benefit from CBT? Answer: Adult experiencing feeling of failure after losing the fourth job in 2 years. 4. An adult plans to attend an upcoming tenth high school reunion this person says to the nurse “I am embarrassed to go. I will not look as good as my classmates. I haven’t been successful in my career.” Which comment by the nurse addresses this cognitive distortion? Answer: “Do you think you are the only person who has aged and faced difficulties in life?” 5. A distraught 8 year old girl tells the nurse, “I had a horrible nightmare and was so scared. I tried to get in bed with my parents but they said ‘NO.’ I think I could have gone back to sleep if I had been with them.” Which family dynamic is likely the basis of the child’s comment? Answer: Boundaries in the family are rigid. CHAPTER 4 1. A patient asks a nurse, "What are neurotransmitters? My doctor says mine are out of balance." The best reply would be: a. "You must feel relieved to know that your problem has a physical basis." b. "Neurotransmitters are chemicals that pass messages between brain cells." c. "It is a high-level concept to explain. You should ask the doctor to tell you more." d. "Neurotransmitters are substances we eat daily that influence memory and mood." ANS: B Stating that neurotransmitters are chemicals that pass messages between brain cells gives the most accurate information. Neurotransmitters are messengers in the central nervous system. They are released from the axon terminal, diffuse across the synapse, and attach to specialized receptors on the postsynaptic neuron. The incorrect responses do not answer the patient's question, are demeaning, and provide untrue and misleading information. 2. The parent of an adolescent with schizophrenia asks a nurse, "My child's doctor ordered a positron-emission tomography (PET) scan. What is that?" Select the nurse's best reply. a. "PET uses a magnetic field and gamma waves to identify problems areas in the brain. Does your teenager have any metal implants?" b. "It's a special type of x-ray image that shows structures of the brain and whether a brain injury has ever occurred." c. "PET is a scan that passes an electrical current through the brain and shows brain wave activity. PET can help diagnose seizures." d. "PET is a special scan that shows blood flow and activity in the brain." ANS: D The parent is seeking information about PET scans. It is important to use terms the parent can understand. The correct option is the only reply that provides factual information relevant to PET scans. The incorrect responses describe magnetic resonance imaging (MRI), computed tomographic (CT) scans, and electroencephalography (EEG). 3. A patient has dementia. The health care provider wants to make a differential diagnosis between Alzheimer disease and multiple infarctions. Which diagnostic procedure should a nurse expect to prepare the patient for first? a. Computed tomography (CT) scan b. Positron emission tomography (PET) scan c. Functional magnetic resonance imaging (fMRI) d. Single-photon-emission computed tomography (SPECT) scan ANS: A A CT scan shows the presence or absence of structural changes, including cortical atrophy, ventricular enlargement, and areas of infarction—information that will be helpful to the health care provider. The other tests focus on brain activity and are more expensive; they may be ordered later. 4. A patient has delusions and hallucinations. Before beginning treatment with a psychotropic medication, the health care provider wants to rule out the presence of a brain tumor. For which test will a nurse need to prepare the patient? a. Cerebral arteriogram b. Computed tomography (CT) scan or magnetic resonance imaging (MRI) c. Positron emission tomography (PET) or single photon emission computed tomography (SPECT) d. Functional magnetic resonance imaging (fMRI) ANS: B A CT scan and an MRI visualize neoplasms and other structural abnormalities. A PET scan, SPECT scan, and fMRI, which give information about brain function, are not indicated. An arteriogram would not be appropriate. 5. The nurse wants to assess for disturbances in circadian rhythms in a patient admitted for depression. Which question best implements this assessment? a. "Do you ever see or hear things that others do not?" b. "Do you have problems with short-term memory?" c. "What are your worst and best times of day?" d. "How would you describe your thinking?" ANS: C Mood changes throughout the day are related to circadian rhythms. Questions about sleep pattern would also be relevant to circadian rhythms. The question about seeing or hearing things is relevant to the assessment for illusions and hallucinations. The question about thinking is relevant to the assessment of thought processes. The other question is relevant to assessment of memory. 6. A nurse administers a medication that potentiates the action of gamma-aminobutyric acid (GABA). Which finding would be expected? a. Reduced anxiety b. Improved memory c. More organized thinking d. Fewer sensory perceptual alterations ANS: A Increased levels of GABA reduce anxiety, thus any potentiation of GABA action should result in anxiety reduction. Memory enhancement is associated with acetylcholine and substance P. Thought disorganization is associated with dopamine. GABA is not associated with sensory perceptual alterations. 7. On the basis of current knowledge of neurotransmitter effects, a nurse anticipates that the treatment plan for a patient with memory difficulties may include medications designed to: a. inhibit GABA production. b. increase dopamine sensitivity. c. decrease dopamine at receptor sites. d. prevent destruction of acetylcholine. ANS: D Increased acetylcholine plays a role in learning and memory. Preventing the destruction of acetylcholine by acetylcholinesterase results in higher levels of acetylcholine with the potential for improved memory. GABA is known to affect anxiety level rather than memory. Increased dopamine causes symptoms associated with schizophrenia or mania rather than improves memory. Decreasing dopamine at receptor sites is associated with Parkinson disease rather than improving memory. 8. A patient has disorganized thinking associated with schizophrenia. Neuroimaging would most likely show dysfunction in which part of the brain? a. Brainstem b. Cerebellum c. Temporal lobe d. Prefrontal cortex ANS: D The prefrontal cortex is responsible for intellectual functioning. The temporal lobe is responsible for the sensation of hearing. The cerebellum regulates skeletal muscle coordination and equilibrium. The brainstem regulates internal organs. 9. A nurse should assess a patient taking a medication with anticholinergic properties for inhibited function of the: a. parasympathetic nervous system b. sympathetic nervous system c. reticular activating system d. medulla oblongata ANS: A Acetylcholine is the neurotransmitter found in high concentration in the parasympathetic nervous system. When acetylcholine action is inhibited by anticholinergic drugs, parasympathetic symptoms such as blurred vision, dry mouth, constipation, and urinary retention appear. The functions of the sympathetic nervous system, the reticular activating system, and the medulla oblongata are not affected by anticholinergic medications. 10. The therapeutic action of monoamine oxidase inhibitors (MAOIs) blocks neurotransmitter reuptake, causing: a. increased concentration of neurotransmitters in the synaptic gap b. decreased concentration of neurotransmitters in serum c. destruction of receptor sites d. limbic system stimulation ANS: A If the reuptake of a substance is inhibited, then it accumulates in the synaptic gap and its concentration increases, permitting the ease of the transmission of impulses across the synaptic gap. Normal transmission of impulses across synaptic gaps is consistent with a normal rather than a depressed mood. The other options are not associated with blocking neurotransmitter reuptake. 11. A patient taking medication for mental illness develops restlessness and an uncontrollable need to be in motion. A nurse can correctly analyze that these symptoms are related to which drug action? a. Anticholinergic effects b. Dopamine-blocking effects c. Endocrine-stimulating effects d. Ability to stimulate spinal nerves ANS: B Medications that block dopamine often produce disturbances of movement such as akathisia because dopamine affects neurons involved in both the thought processes and movement regulation. Anticholinergic effects include dry mouth, blurred vision, urinary retention, and constipation. Akathisia is not caused by endocrine stimulation or spinal nerve stimulation. 12. A patient has anxiety, increased heart rate, and fear. The nurse would suspect the presence of a high concentration of which neurotransmitter? a. GABA b. Histamine c. Acetylcholine d. Norepinephrine ANS: D Norepinephrine is the neurotransmitter associated with sympathetic nervous system stimulation, preparing the individual for "fight or flight" response. GABA is a mediator of anxiety level. A high concentration of histamine is associated with an inflammatory response. A high concentration of acetylcholine is associated with parasympathetic nervous system stimulation. 13. A patient has symptoms of acute anxiety related to the death of a parent in an automobile accident 2 hours earlier. The nurse should prepare to teach the patient about a medication from which group? a. Tricyclic antidepressants b. Atypical antipsychotics c. Anticonvulsants d. Benzodiazepines ANS: D Benzodiazepines provide anxiety relief. Tricyclic antidepressants are used to treat symptoms of depression. Anticonvulsants are used to treat bipolar disorder or seizures. Antipsychotic drugs are used to treat psychosis. 14. A patient is hospitalized for major depression. Of the medications listed, a nurse can expect to provide the patient with teaching about: a. chlordiazepoxide (Librium) b. fluoxetine (Prozac) c. clozapine (Clozaril) d. tacrine (Cognex) ANS: B Fluoxetine is a selective serotonin reuptake inhibitor (SSRI), an antidepressant that blocks the reuptake of serotonin with few anticholinergic and sedating side effects; clozapine (Clozaril) is an antipsychotic medication; chlordiazepoxide (Librium) is an anxiolytic drug; and tacrine (Cognex) is used to treat Alzheimer disease. 15. A patient hospitalized with a mood disorder has aggression, agitation, talkativeness, and irritability. A nurse begins the care plan based on the expectation that the health care provider is most likely to prescribe a medication classified as a(n): a. anticholinergic b. mood stabilizer c. psychostimulant d. antidepressant ANS: B The symptoms describe a manic attack. Mania is effectively treated by the antimanic drug lithium and selected anticonvulsants such as carbamazepine, valproic acid, and lamotrigine. No drugs from the other classifications listed are effective in the treatment of mania. 16. A drug causes muscarinic-receptor blockade. A nurse will assess the patient for: a. dry mouth b. gynecomastia c. pseudoparkinsonism d. orthostatic hypotension ANS: A Muscarinic-receptor blockade includes atropine-like side effects such as dry mouth, blurred vision, and constipation. Gynecomastia is associated with decreased prolactin levels. Movement defects are associated with dopamine blockade. Orthostatic hypotension is associated with alpha1-receptor antagonism. 17. A patient begins therapy with a phenothiazine medication. What teaching should a nurse provide related to the drug's strong dopaminergic effect? a. Chew sugarless gum. b. Increase dietary fiber. c. Arise slowly from bed. d. Report muscle stiffness. ANS: D Phenothiazines block dopamine receptors in both the limbic system and basal ganglia. Dystonia is likely to occur early in the course of treatment and is often heralded by sensations of muscle stiffness. Early intervention with an antiparkinsonian medication can increase the patient's comfort and prevent dystonic reactions. 18. A nurse can anticipate anticholinergic side effects will be likely when a patient is taking: a. lithium (Lithobid) b. buspirone (BuSpar) c. risperidone (Risperdal) d. fluphenazine (Prolixin) ANS: D Fluphenazine, a first-generation antipsychotic medication, exerts muscarinic blockade, resulting in dry mouth, blurred vision, constipation, and urinary retention. Lithium therapy is more often associated with fluid balance problems, including polydipsia, polyuria, and edema. Risperidone therapy is more often associated with movement disorders, orthostatic hypotension, and sedation. Buspirone is associated with anxiety reduction without major side effects. 19. Priority teaching for a patient taking clozapine (Clozaril) should include which instruction? a. Report sore throat and fever immediately. b. Avoid foods high in polyunsaturated fat. c. Use water-based lotions for rashes. d. Avoid unprotected sex. ANS: A Clozapine therapy may produce agranulocytosis; therefore signs of infection should be immediately reported to the health care provider. In addition, the patient should have white blood cell levels measured weekly. The other options are not relevant to clozapine administration. 20. A nurse cares for patients taking various medications, including buspirone (BuSpar), haloperidol (Haldol), trazodone (Desyrel), and phenelzine (Nardil). The nurse will order a special diet for the patient taking: a. buspirone b. haloperidol c. trazodone d. phenelzine ANS: D Patients taking phenelzine, an MAOI, must be on a tyramine-free diet to prevent hypertensive crisis. 21. A nurse caring for a patient taking a serotonin reuptake inhibitor (SSRI) will develop outcome criteria related to: a. mood improvement b. logical thought processes c. reduced levels of motor activity d. decreased extrapyramidal symptoms ANS: A SSRIs affect mood, relieving depression in many patients. SSRIs do not act to reduce thought disorders. SSRIs reduce depression but have little effect on motor hyperactivity. SSRIs do not produce extrapyramidal symptoms. 22. A patient's spouse, who is a chemist, asks a nurse how serotonin reuptake inhibitors (SSRIs) lift depression. The nurse should explain that SSRIs: a. destroy increased amounts of neurotransmitters. b. make more serotonin available at the synaptic gap. c. increase production of acetylcholine and dopamine. d. block muscarinic and alpha1-norepinephrine receptors. ANS: A Depression is thought to be related to the lowered availability of the neurotransmitter serotonin. SSRIs act by blocking the reuptake of serotonin, leaving a higher concentration available at the synaptic cleft. They actually prevent the destruction of serotonin, have no effect on acetylcholine and dopamine production, and do not block muscarinic or alpha1-norepinephrine receptors. 23. A patient has taken many conventional antipsychotic drugs over years. The health care provider, who is concerned about early signs of tardive dyskinesia, prescribes risperidone (Risperdal). A nurse planning care for this patient understands that atypical antipsychotics: a. are less costly. b. have higher potency. c. are more readily available. d. produce fewer motor side effects. ANS: D Atypical antipsychotic drugs often exert their action on the limbic system rather than the basal ganglia. The limbic system is not involved in motor disturbances. Atypical antipsychotic medications are not more readily available. They are not considered to be of higher potency; rather, they have different modes of action. Atypical antipsychotic drugs tend to be more expensive. 24. The laboratory report for a patient taking clozapine (Clozaril) shows a white blood cell count of 3000 mm3 and a granulocyte count of 1500 mm3. The nurse should: a. report the laboratory results to the health care provider. b. give the next dose as prescribed. c. administer aspirin and force fluids. d. repeat the laboratory tests. ANS: A These laboratory values indicate the possibility of agranulocytosis, a serious side effect of clozapine therapy. These results must be immediately reported to the health care provider. The drug should be withheld because the health care provider will discontinue it. The health care provider may repeat the laboratory test, but, in the meantime, the drug should be withheld. Giving aspirin and forcing fluids are measures that are less important than stopping the administration of the drug. 25. A nurse administering psychotropic medications should be prepared to intervene when giving a drug that blocks the attachment of norepinephrine to alpha1 receptors because the patient may experience: a. increased psychotic symptoms b. severe appetite disturbance c. orthostatic hypotension d. hypertensive crisis ANS: C Sympathetic-mediated vasoconstriction is essential for maintaining normal blood pressure in the upright position. Blockage of alpha1 receptors leads to vasodilation and orthostatic hypotension. Orthostatic hypotension may cause fainting and falls. Patients should be taught ways of minimizing this phenomenon. 1. A nurse prepares to administer an antipsychotic medication to a patient with schizophrenia. Additional monitoring of the medication's effects and side effects will be most important if the patient is also diagnosed with which health problem? Select all that apply. a. Parkinson disease b. Graves disease c. Osteoarthritis d. Epilepsy e. Diabetes ANS: A, D, E Antipsychotic medications may produce weight gain, which complicates the care of a patient with diabetes or lowers the seizure threshold or both, which complicates the care of a patient with epilepsy. Parkinson disease involves changes in transmission of dopamine and acetylcholine; therefore these drugs also complicate the care of a patient with the disorder. Osteoarthritis and Graves disease should have no synergistic effect with this medication. 2. The spouse of a patient with schizophrenia asks, "Which neurotransmitters are more active when a person has schizophrenia?" The nurse should state, "The current thinking is that the thought disturbances are related to increased activity of: (Select all that apply.) a. GABA." b. substance P." c. histamine." d. dopamine." e. norepinephrine." ANS: D, E Dopamine plays a role in the integration of thoughts and emotions, and excess dopamine is implicated in the thought disturbances of schizophrenia. Increased activity of norepinephrine also occurs. Substance P is most related to the pain experience. Histamine decrease is associated with depression. Increased GABA is associated with anxiety reduction. 3. An individual is experiencing problems associated with memory. Which cerebral structures are most likely to be involved in this deficit? Select all that apply. a. Prefrontal cortex b. Occipital lobe c. Temporal lobe d. Parietal lobe e. Basal ganglia ANS: A, C, D The prefrontal cortex, parietal, and temporal lobes of the cerebrum play a key role in the storage and processing of memories. The occipital lobe is predominantly involved with vision. The basal ganglia influence the integration of physical movement, as well as some thoughts and emotions. BOOK QUESTIONS: 1. A patient is diagnosed with an abscess in the cerebellum. Which nursing diagnosis has priority for the plan of care? Answer: Risk for falls R/T loss of balance and equilibrium 2. A patient begins a new prescription for risperidone (Risperdal). Which intervention should the nurse include in the plan of care? Answer: Assess sitting, standing, and lying blood pressure daily 3. Systematic measurement of body weight, BMI, waist circumference, and glucose levels would be most important for a patient beginning a new prescription of which medication? Answer: Olanzapine (Zyprexa) 4. A pt tells the community mental health nurse. “I told my healthcare provider I was having trouble sleeping and he prescribed trazadone 50mg every night. I read on the interest of that drug is an antidepressant, but im not depressed. What should I do?” Which response by the nurse is correct? Answer: “in low doses Trazadone helps relieve insomnia. Higher doses are needed for antidepressants effect to occur.” 5. Which pt would the nurse expect to have most difficulty with problem solving and decision making? Answer: A 52 year old diagnosed with schizophrenia at age 21; has taken monthly injections of Haldol decanoate for 12 years. CHAPTER 5 1. Planning for patients with mental illness is facilitated by understanding that inpatient hospitalization is generally reserved for patients who: a. present a clear danger to self or others. b. are noncompliant with medications at home. c. have no support systems in the community. d. develop new symptoms during the course of an illness. ANS: A Hospitalization is justified when the patient is a danger to self or others, has dangerously decompensated, or needs intensive medical treatment. The incorrect options do not necessarily describe patients who require inpatient treatment. 2. A patient is hospitalized for a reaction to a psychotropic medication and then is closely monitored for 24 hours. During a predischarge visit, the case manager learns the patient received a notice of eviction on the day of admission. The most appropriate intervention for the case manager is to: a. cancel the patient's discharge from the hospital. b. contact the landlord who evicted the patient to further discuss the situation. c. arrange a temporary place for the patient to stay until new housing can be arranged. d. document that the adverse medication reaction was feigned because the patient had nowhere to live. ANS: C The case manager should intervene by arranging temporary shelter for the patient until suitable housing can be found. This is part of the coordination and delivery of services that falls under the case manager role. The other options are not viable alternatives. 3. A multidisciplinary health care team meets 12 hours after an adolescent is hospitalized after a suicide attempt. Members of the team report their assessments. What outcome can be expected from this meeting? a. A treatment plan will be determined. b. The health care provider will order neuroimaging studies. c. The team will request a court-appointed advocate for the patient. d. Assessment of the patient's need for placement outside the home will be undertaken. ANS: A Treatment plans are formulated early in the course of treatment to streamline the treatment process and reduce costs. It is too early to determine the need for alternative postdischarge living arrangements. Neuroimaging is not indicated for this scenario. 4. The relapse of a patient with schizophrenia is related to medication noncompliance. The patient is hospitalized for 5 days, medication is restarted, and the patient's thoughts are now more organized. The patient's family members are upset and say, "It's too soon about the patient being scheduled for discharge. Hospitalization is needed for at least a month." The nurse should: a. call the psychiatrist to come explain the discharge rationale. b. explain that health insurance will not pay for a longer stay for the patient. c. call security to handle the disturbance and escort the family off the unit. d. explain that the patient will continue to improve if medication is taken regularly. ANS: D Patients no longer stay in the hospital until every vestige of a symptom disappears. The nurse must assume responsibility to advocate for the patient's right to the least restrictive setting as soon as the symptoms are under control and for the right of citizens to control health care costs. The health care provider will use the same rationale. Shifting blame will not change the discharge. Calling security is unnecessary. The nurse can handle this matter. 5. A nurse assesses an inpatient psychiatric unit, noting that exits are free from obstruction, no one is smoking, the janitor's closet is locked, and all sharp objects are being used under staff supervision. These observations relate to: a. management of milieu safety b. coordinating care of patients c. management of the interpersonal climate d. use of therapeutic intervention strategies ANS: A Members of the nursing staff are responsible for all aspects of milieu management. The observations mentioned in this question directly relate to the safety of the unit. The other options, although part of the nurse's concerns, are unrelated to the observations cited. 6. The following patients are seen in the emergency department. The psychiatric unit has one bed available. Which patient should the admitting officer recommend for admission to the hospital? The patient who: a. experiencing dry mouth and tremor related to side effects of haloperidol (Haldol). b. experiencing anxiety and a sad mood after a separation from a spouse of 10 years. c. who self-inflicted a superficial cut on the forearm after a family argument. d. who is a single parent and hears voices saying, "Smother your infant." ANS: D Admission to the hospital would be justified by the risk of patient danger to self or others. The other patients have issues that can be handled with less restrictive alternatives than hospitalization. 7. A student nurse prepares to administer oral medications to a patient with major depression, but the patient refuses the medication. The student nurse should: a. tell the patient, "I'll get an unsatisfactory grade if I don't give you the medication." b. tell the patient, "Refusing your medication is not permitted. You are required to take it." c. explore the patient's concerns about the medication, and report to the staff nurse. d. document the patient's refusal of the medication without further comment. ANS: C The patient has the right to refuse medication in most cases. The patient's reason for refusing should be ascertained, and the refusal should be reported to a unit nurse. Sometimes refusals are based on unpleasant side effects that can be ameliorated. Threats and manipulation are inappropriate. Medication refusal should be reported to permit appropriate intervention. 8. A nurse surveys the medical records for violations of patients' rights. Which finding signals a violation? a. No treatment plan is present in record. b. Patient belongings are searched at admission. c. Physical restraint is used to prevent harm to self. d. Patient is placed on one-to-one continuous observation. ANS: A The patient has the right to have a treatment plan. Inspecting a patient's belongings is a safety measure. Patients have the right to a safe environment, including the right to be protected against impulses to harm self that occur as a result of a mental disorder. 9. Which principle takes priority for the psychiatric inpatient staff when addressing behavioral crises? a. Resolve behavioral crises with the least restrictive intervention possible. b. Rights of the majority of patients supersede the rights of individual patients. c. Swift intervention is justified to maintain the integrity of the therapeutic milieu. d. Allow patients the opportunities to regain control without intervention if the safety of other patients is not compromised. ANS: A The rule of using the least restrictive treatment or intervention possible to achieve the desired outcome is the patient's legal right. Planned interventions are nearly always preferable. Intervention may be necessary when the patient threatens harm to self. 10. To provide comprehensive care to patients, which competency is more important for a nurse who works in a community mental health center than a psychiatric nurse who works in an inpatient unit? a. Problem-solving skills b. Calm external manner c. Ability to cross service systems d. Knowledge of psychopharmacology ANS: C A community mental health nurse must be able to work with schools, corrections facilities, shelters, health care providers, and employers. The mental health nurse working in an inpatient unit needs only to be able to work within the single setting. Problem-solving skills are needed by all nurses. Nurses in both settings must have knowledge of psychopharmacology. 11. A suspicious and socially isolated patient lives alone, eats one meal a day at a nearby shelter, and spends the remaining daily food allowance on cigarettes. Select the community psychiatric nurse's best initial action. a. Report the situation to the manager of the shelter. b. Tell the patient, "You must stop smoking to save money." c. Assess the patient's weight; determine the foods and amounts eaten. d. Seek hospitalization for the patient while a new plan is being formulated. ANS: C Assessment of biopsychosocial needs and general ability to live in the community is called for before any action is taken. Both nutritional status and income adequacy are critical assessment parameters. A patient may be able to maintain adequate nutrition while eating only one meal a day. Nurses assess before taking action. Hospitalization may not be necessary. 12. A patient with schizophrenia has been stable in the community. Today, the spouse reports the patient is delusional and explosive. The patient says, "I'm willing to take my medicine, but I forgot to get my prescription refilled." Which outcome should the nurse add to the plan of care? a. Nurse will obtain prescription refills every 90 days and deliver them to the patient. b. Patient's spouse will mark dates for prescription refills on the family calendar. c. Patient will report to the hospital for medication follow-up every week. d. Patient will call the nurse weekly to discuss medication-related issues. ANS: B The nurse should use the patient's support system to meet patient needs whenever possible. Delivery of medication by the nurse should be unnecessary for the nurse to do if the patient or a significant other can be responsible. The patient may not need more intensive follow-up as long as he or she continues to take the medications as prescribed. No patient issues except failure to obtain medication refills were identified. 13. A community mental health nurse has worked for 6 months to establish a relationship with a delusional, suspicious patient. The patient recently lost employment and stopped taking medications because of inadequate money. The patient says, "Only a traitor would make me go to the hospital." Which solution is best? a. Arrange a bed in a local homeless shelter with nightly onsite supervision. b. Negotiate a way to provide medication so the patient can remain at home. c. Hospitalize the patient until the symptoms have stabilized. d. Seek inpatient hospitalization for up to 1 week. ANS: B Hospitalization may damage the nurse-patient relationship even if it provides an opportunity for rapid stabilization. If medication can be obtained and restarted, the patient can possibly be stabilized in the home setting, even if it takes a little longer. A homeless shelter is inappropriate and unnecessary. Hospitalization may be necessary later, but a less restrictive solution should be tried first because the patient is not dangerous. 14. A community psychiatric nurse facilitates medication compliance for a patient by having the health care provider prescribe depot medications by injection every 3 weeks at the clinic. For this plan to be successful, which factor will be of critical importance? a. Attitude of significant others toward the patient b. Nutritional services in the patient's neighborhood c. Level of trust between the patient and the nurse d. Availability of transportation to the clinic ANS: D The ability of the patient to get to the clinic is of paramount importance to the success of the plan. The depot medication relieves the patient of the necessity to take medication daily, but if he or she does not receive the injection at 3-week intervals, noncompliance will again be the issue. Attitude toward the patient, trusting relationships, and nutrition are important but not fundamental to this particular problem. 15. Which assessment finding for a patient in the community requires priority intervention by the nurse? The patient: a. receives Social Security disability income plus a small check from a trust fund. b. lives in an apartment with two patients who attend day hospital programs. c. has a sibling who is interested and active in care planning. d. purchases and uses marijuana on a frequent basis. ANS: D Patients who regularly buy illegal substances often become medication noncompliant. Medication noncompliance, along with the disorganizing influence of illegal drugs on cellular brain function, promotes relapse. The remaining options do not suggest problems. 16. A patient tells the nurse at the clinic, "I haven't been taking my antidepressant medication as directed. I leave out the midday dose. I have lunch with friends and don't want them to ask me about the pills." Select the nurse's most appropriate intervention. a. Investigate the possibility of once-daily dosing of the antidepressant. b. Suggest to the patient to take the medication when no one is watching. c. Explain how taking each dose of medication on time relates to health maintenance. d. Add the nursing diagnosis—Ineffective therapeutic regimen management, related to lack of knowledge—to the plan of care. ANS: A Investigating the possibility of once-daily dosing of the antidepressant has the highest potential for helping the patient achieve compliance. Many antidepressants can be administered by once-daily dosing, a plan that increases compliance. Explaining how taking each dose of medication on time relates to health maintenance is reasonable but would not achieve the goal; it does not address the issue of stigma. The self-conscious patient would not be comfortable doing this. A better etiologic statement would be related to social stigma. The question asks for an intervention, not analysis. 17. A community psychiatric nurse assesses that a patient with a mood disorder is more depressed than on the previous visit a month ago; however, the patient says, "I feel the same." Which intervention supports the nurse's assessment while preserving the patient's autonomy? a. Arrange for a short hospitalization. b. Schedule weekly clinic appointments. c. Refer the patient to the crisis intervention clinic. d. Call the family and ask them to observe the patient closely. ANS: B Scheduling clinic appointments at shorter intervals will give the opportunity for more frequent assessment of symptoms and allow the nurse to use early intervention. If the patient does not admit to having a crisis or problem, this referral would be useless. The remaining options may produce unreliable information, violate the patient's privacy, and waste scarce resources. 18. A patient hurriedly tells the community mental health nurse, "Everything's a disaster! I can't concentrate. My disability check didn't come. My roommate moved out, and I can't afford the rent. My therapist is moving away. I feel like I'm coming apart." Which nursing diagnosis applies? a. Decisional conflict, related to challenges to personal values b. Spiritual distress, related to ethical implications of treatment regimen c. Anxiety, related to changes perceived as threatening to psychological equilibrium d. Impaired environmental interpretation syndrome, related to solving multiple problems affecting security needs ANS: C Subjective and objective data obtained by the nurse suggest the patient is experiencing anxiety caused by multiple threats to security needs. Data are not present to suggest Decisional conflict, ethical conflicts around treatment causing Spiritual distress, or Impaired environmental interpretation syndrome. 19. Which patient would a nurse refer to partial hospitalization? An individual who: a. spent yesterday in the 24-hour supervised crisis care center and continues to be actively suicidal. b. because of agoraphobia and panic episodes needs psychoeducation for relaxation therapy c. has a therapeutic lithium level and reports regularly for blood tests and clinic follow-up. d. states, "I'm not sure I can avoid using alcohol when my spouse goes to work every morning." ANS: D This patient could profit from the structure and supervision provided by spending the day at the partial hospitalization program. During the evening, at night, and on weekends, the spouse could assume supervision responsibilities. The patient who is actively suicidal needs inpatient hospitalization. The patient in need of psychoeducation can be referred to home care. The patient who reports regularly for blood tests and clinical follow-up can continue on the same plan. 20. Which employer's health plan is required to include parity provisions related to mental illnesses? a. Employer with more than 50 employees b. Cancer thrift shop staffed by volunteers c. Daycare center that employs 7 teachers d. Church that employs 15 people ANS: A Under federal parity laws, companies with more than 50 employees may not limit annual or lifetime mental health benefits unless they also limit benefits for physical illnesses. 1. A nurse can best address factors of critical importance to successful community treatment for persons with mental illness by including assessments related to which of the following? Select all that apply. a. housing adequacy and stability b. income adequacy and stability c. family and other support systems d. early psychosocial development e. substance abuse history and current use ANS: A, B, C, E Early psychosocial developmental history is less relevant to successful outcomes in the community than the assessments listed in the other options. If a patient is homeless or fears homelessness, focusing on other treatment issues is impossible. Sufficient income for basic needs and medication is necessary. Adequate support is a requisite to community placement. Substance abuse undermines medication effectiveness and interferes with community adjustment. 2. A community member asks a nurse, "People with mental illnesses used to go to a state hospital. Why has that changed?" Select the nurse's accurate responses. Select all that apply. a. "Science has made significant improvements in drugs for mental illness, so now many people may live in their communities." b. "A better selection of less restrictive settings is now available in communities to care for individuals with mental illness." c. "National rates of mental illness have declined significantly. The need for state institutions is actually no longer present." d. "Most psychiatric institutions were closed because of serious violations of patients' rights and unsafe conditions." e. "Federal legislation and payment for treatment of mental illness have shifted the focus to community rather than institutional settings." ANS: A, B, E The community is a less restrictive alternative than hospitals for the treatment of people with mental illness. Funding for treatment of mental illness remains largely inadequate but now focuses on community rather than institutional care. Antipsychotic medications improve more symptoms of mental illness; hence, management of psychiatric disorders has improved. Rates of mental illness have increased, not decreased. Hospitals were closed because funding shifted to the community. Conditions in institutions have improved. END OF BOOK QUESTIONS: 1. A pt diagnosed with MDD tells the community health nurse, “I usually spend all day watching television. If there is nothing good to watch, I just sleep or think about my problems.” What is the nurses best action? Answer: Refer the pt for counseling with recreational therapist. 2. The nurse admits a pt experiencing hallucinations and delusional thinking to an inpatient mental health unit. The plan of care will require which service occurs first? Answer: Medical Assessment 3. A nurse working in an acute care unit for adolescents diagnosed with mental illness says, “Our pts have so much energy. We need some physical activities for them.” In recognition of needs for safety and exercise, which activity could the treatment team approve? Answer: Line dancing to popular music 4. As election day nears, mental health nurse studies the position statements of various candidates for federal offices. Which candidate’s commentary would the nurse interpret as supportive of services for persons diagnosed with mental illness? Answer: “Full parity insurance coverage for mental illness.” 5. An experienced nurse in a major medical center requests a transfer from a general medical unit to an acute care psychiatric unit. Which organizational feature would best support this nurse’s successful transition? Answer: Co- assignment with a knowledgeable psychiatric nurse for an extended orientation. CHAPTER 6 1. A psychiatric nurse best implements the ethical principle of autonomy when he or she: a. intervenes when a self-mutilating patient attempts to harm self. b. stays with a patient who is demonstrating a high level of anxiety. c. suggests that two patients who are fighting be restricted to the unit. d. explores alternative solutions with a patient, who then makes a choice. ANS: D Autonomy is the right to self-determination, that is, to make one's own decisions. When the nurse explores alternatives with the patient, the patient is better equipped to make an informed, autonomous decision. Staying with a highly anxious patient or intervening with a self-mutilating patient demonstrates beneficence and fidelity. Suggesting that two fighting patients be restricted to the unit demonstrates the principles of fidelity and justice. 2. Which action by a psychiatric nurse best supports patients' rights to be treated with dignity and respect? a. Consistently addressing each patient by title and surname. b. Strongly encouraging a patient to participate in the unit milieu. c. Discussing a patient's condition with another health care provider in the elevator. d. Informing a treatment team that a patient is too drowsy to participate in care planning. ANS: A A simple way of showing respect is to address the patient by title and surname rather than assuming that the patient would wish to be called by his or her first name. Discussing a patient's condition with a health care provider in the elevator violates confidentiality. Informing a treatment team that the patient is too drowsy to participate in care planning violates patient autonomy. Encouraging a patient to participate in the unit milieu exemplifies beneficence and fidelity. 3. Two hospitalized patients fight when they are in the same room. During a team meeting, a nurse asserts that safety is of paramount importance and therefore the treatment plans should call for both patients to be secluded to prevent them from injuring each other. This assertion: a. reveals that the nurse values the principle of justice. b. reinforces the autonomy of the two patients. c. violates the civil rights of the two patients. d. represents the intentional tort of battery. ANS: C Patients have a right to treatment in the least restrictive setting. Less restrictive measures should be tried first. Unnecessary seclusion may result in a charge of false imprisonment. Seclusion removes the patient's autonomy. The principle by which the nurse is motivated is beneficence, not justice. The tort represented is false imprisonment, not battery. 4. In a team meeting a nurse says, "I'm concerned whether we are behaving ethically by using restraint to prevent one patient from self-mutilation while the care plan for another patient who has also self-mutilated calls for one-on-one supervision." Which ethical principle most clearly applies to this situation? a. Beneficence b. Autonomy c. Fidelity d. Justice ANS: D The nurse is concerned about justice, that is, the fair treatment with the least restrictive methods for both patients. Beneficence means promoting the good of others. Autonomy is the right to make one's own decisions. Fidelity is the observance of loyalty and commitment to the patient. 5. Which scenario is an example of a tort? a. The primary nurse does not complete the plan of care for a patient within 24 hours of the patient's admission. b. An advanced practice nurse recommends that a patient who is dangerous to self and others be voluntarily hospitalized. c. A patient's admission status is changed from involuntary to voluntary after the patient's hallucinations subside. d. A nurse gives an as-needed dose of an antipsychotic drug to a patient to prevent violent acting-out because a unit is short staffed. ANS: D A tort is a civil wrong against a person that violates his or her rights. Giving unnecessary medication for the convenience of staff members controls behavior in a manner similar to secluding a patient; thus false imprisonment is a possible charge. The other options do not exemplify torts. 6. A nurse's neighbor asks, "Why aren't people with mental illness kept in state institutions anymore?" What is the nurse's best response? a. "Many people are still in psychiatric institutions. Inpatient care is needed because many people who are mentally ill are violent." b. "Less restrictive settings are now available to care for individuals with mental illness." c. "Our nation has fewer persons with mental illness; therefore fewer hospital beds are needed." d. "Psychiatric institutions are no longer popular as a consequence of negative stories in the press." ANS: B The community is a less restrictive alternative than hospitals for the treatment of people with mental illness. The remaining options are incorrect and part of the stigma of mental illness. 7. Which nursing intervention demonstrates false imprisonment? a. A confused and combative patient says, "I'm getting out of here and no one can stop me." The nurse restrains this patient without a health care provider's order and then promptly obtains an order. b. A patient has been irritating, seeking the attention of nurses most of the day. Now a nurse escorts the patient down the hall saying, "Stay in your room or you'll be put in seclusion." c. An involuntarily hospitalized patient with suicidal ideation runs out of the psychiatric unit. A nurse rushes after the patient and convinces the patient to return to the unit. d. An involuntarily hospitalized patient with suicidal ideation attempts to leave the unit. A nurse calls the security team and uses established protocols to prevent the patient from leaving. ANS: B False imprisonment involves holding a competent person against his or her will. Actual force is not a requirement of false imprisonment. The individual needs only to be placed in fear of imprisonment by someone who has the ability to carry out the threat. The patient in one distracter is not competent, and the nurse is acting beneficently. The patients in the other distracters have been admitted as involuntary patients and should not be allowed to leave without permission of the treatment team. 8. A patient should be considered for involuntary commitment for psychiatric care when he or she: a. is noncompliant with the treatment regimen. b. sold sells and distributes illegal drugs. c. threatens to harm self and others. d. fraudulently files for bankruptcy. ANS: C Involuntary commitment protects patients who are dangerous to themselves or others and cannot care for their own basic needs. Involuntary commitment also protects other individuals in society. The behaviors described in the other options are not sufficient to require involuntary hospitalization. 9. A nurse at the mental health center prepares to administer a scheduled injection of haloperidol decanoate (Haldol depot injection) to a patient with schizophrenia. As the nurse swabs the site, the patient shouts, "Stop, stop! I don't want to take that medicine anymore. I hate the side effects." Select the nurse's best initial action. a. Stop the medication administration procedure and say to the patient, "Tell me more about the side effects you've been having." b. Say to the patient, "Since I've already drawn the medication in the syringe, I'm required to give it, but let's talk to the doctor about skipping next month's dose." c. Proceed with the injection but explain to the patient that other medications are available that may help reduce the unpleasant side effects. d. Notify other staff members to report to the room for a show of force and proceed with the injection, using restraint if necessary. ANS: A Patients with mental illness retain their civil rights unless clear, cogent, and convincing evidence of dangerousness exists. The patient in this situation presents no evidence of being dangerous. The nurse, an as advocate and educator, should seek more information about the patient's decision and should not force the medication. 10. Several nurses are concerned that agency policies related to restraint and seclusion practices are inadequate. Which statement about the relationship of substandard institutional policies and individual nursing practice should guide nursing practice? a. The policies do not absolve an individual nurse of the responsibility to practice according to the professional standards of nursing care. b. Agency policies are the legal standard by which a professional nurse must act and therefore override other standards of care. c. In an institution with substandard policies, the nurse has a responsibility to inform the supervisor and leave the premises. d. Interpretation of policies by the judicial system is rendered on an individual basis and therefore cannot be predicted. ANS: A Nurses are professionally bound to uphold the American Nurses Association (ANA) standards of practice, regardless of lesser standards established by a health care agency or state. Conversely, if the agency standards are higher than the ANA standards of practice, the agency standards must be upheld. The courts may seek to establish the standard of care through the use of expert witnesses when the issue is clouded. 11. A newly admitted patient who is acutely psychotic is a private patient of the senior psychiatrist. To whom does the psychiatric nurse who is assigned to this patient owe the duty of care? a. Health care provider b. Profession c. Hospital d. Patient ANS: D Although the nurse is accountable to the health care provider, the agency, the patient, and the profession, the duty of care is owed to the patient. 12. An example of a breach of a patient's right to privacy occurs when a nurse: a. asks a family to share information about a patient's prehospitalization behavior. b. discusses the patient's history with other staff members during care planning. c. documents the patient's daily behaviors during hospitalization. d. releases information to the patient's employer without consent. ANS: D The release of information without patient authorization violates the patient's right to privacy. The other options are acceptable nursing practices. 13. An adolescent hospitalized after a violent physical outburst tells the nurse, "I'm going to kill my father, but you can't tell anyone." Select the nurse's best response. a. "You're right. Federal law requires me to keep that information private." b. "Those kinds of thoughts will make your hospitalization longer." c. "You really should share this thought with your psychiatrist." d. "I am obligated to share information with the treatment team." ANS: D Breach of nurse-patient confidentiality does not pose a legal dilemma for nurses in these circumstances because a team approach to the delivery of psychiatric care presumes communication of patient information to other staff members to develop treatment plans and outcome criteria. The patient should know that the team may have to warn the father of the risk for harm. 14. A voluntarily hospitalized patient tells the nurse, "Get me the forms for discharge against medical advice so I can leave now." What is the nurse's best initial response? a. "I can't give you those forms without your health care provider's knowledge." b. "I will get them for you, but let's talk about your decision to leave treatment." c. "Since you signed your consent for treatment, you may leave if you desire." d. "I'll get the forms for you right now and bring them to your room." ANS: B A patient who has been voluntarily admitted as a psychiatric inpatient has the right to demand and obtain release in most states. However, as a patient advocate, the nurse is responsible for weighing factors related to the patient's wishes and best interests. By asking for information, the nurse may be able to help the patient reconsider the decision. The statement that discharge forms cannot be given without the health care provider's knowledge is not true. Facilitating discharge without consent is not in the patient's best interest before exploring the reason for the request. 15. The family of a patient whose insurance will not pay for continuing hospitalization considers transferring the patient to a public psychiatric hospital. The family expresses concern that the patient will "never get any treatment." Which reply by the nurse would be most helpful? a. "Under the law, treatment must be provided. Hospitalization without treatment violates patients' rights." b. "That's a justifiable concern because the right to treatment extends only to the provision of food, shelter, and safety." c. "Much will depend on other patients because the right to treatment for a psychotic patient takes precedence over the right to treatment of a patient who is stable." d. "All patients in public hospitals have the right to choose both a primary therapist and a primary nurse." ANS: A The right to medical and psychiatric treatment was conferred on all patients hospitalized in public mental hospitals with the enactment of the federal Hospitalization of Mentally Ill Act in 1964. Stating that the concern is justifiable supports the family's erroneous belief. The provisions mentioned in the third and fourth options are not part of this or any other statute governing psychiatric care. 16. Which individual with a mental illness may need emergency or involuntary hospitalization for mental illness? The individual who: a. resumes using heroin while still taking methadone. b. reports hearing angels playing harps during thunderstorms. c. throws a heavy plate at a waiter at the direction of command hallucinations. d. does not show up for an outpatient appointment with the mental health nurse. ANS: C Throwing a heavy plate is likely to harm the waiter and is evidence of being dangerous to others. This behavior meets the criteria for emergency or involuntary hospitalization for mental illness. The behaviors in the other options evidence mental illness but not dangerousness. 17. A patient being treated in an alcohol rehabilitation unit reveals to the nurse, "I feel terrible guilt for sexually abusing my 6-year-old child before I was admitted." Based on state and federal law, the best action for the nurse to take is to: a. anonymously report the abuse by telephone to the local child abuse hotline. b. reply, "I'm glad you feel comfortable talking to me about it." c. respect nurse-patient relationship of confidentiality. d. file a written report on the agency letterhead. ANS: A Laws regarding reporting child abuse discovered by a professional during a suspected abuser's alcohol or drug treatment differ by state. Federal law supersedes state law and prohibits disclosure without a court order except in instances in which the report can be made anonymously or without identifying the abuser as a patient in an alcohol or drug treatment facility. Anonymously reporting the abuse by telephone to the local child abuse hotline meets federal criteria. Respecting nurse-patient confidentiality and replying, "I'm glad you feel comfortable talking to me about it" do not accomplish reporting. Filing a written report on agency letterhead violates federal law. 18. The spouse of a patient who has delusions asks the nurse, "Are there any circumstances under which the treatment team is justified in violating the patient's right to confidentiality?" The nurse must reply that confidentiality may be breached: a. under no circumstances. b. at the discretion of the psychiatrist. c. when questions are asked by law enforcement. d. if the patient threatens the life of another person. ANS: D The duty to warn a person whose life has been threatened by a patient under psychiatric treatment overrides the patient's right to confidentiality. The right to confidentiality is not suspended at the discretion of the therapist or for legal investigations. 19. A nurse cares for an older adult patient admitted for the treatment of depression. The health care provider prescribes an antidepressant medication, but the dose is more than the usual adult dose. The nurse should: a. implement the order. b. consult a drug reference. c. give the usual geriatric dosage. d. hold the medication and consult the health care provider. ANS: D The dose of an antidepressant medication for older adult patients is often less than the usual adult dose. The nurse should withhold the medication and consult the health care provider who wrote the order. The nurse's duty is to intervene and protect the patient. Consulting a drug reference is unnecessary because the nurse already knows the dose is excessive. Implementing the order is negligent. Giving the usual geriatric dose would be wrong; a nurse without prescriptive privileges cannot change the dose. 20. A patient with paranoid schizophrenia believes evil spirits are being summoned by a local minister and verbally threatens to bomb a local church. The psychiatrist notifies the minister. The psychiatrist has: a. released information without proper authorization. b. demonstrated the duty to warn and protect. c. violated the patient's confidentiality. d. avoided charges of malpractice. ANS: B The duty of a health care professional is to warn or notify an intended victim after a threat of harm has been made. Informing a potential victim of a threat is a legal responsibility of the health care professional and not considered a violation of confidentiality. 21. After leaving work, a staff nurse realizes that documentation of the administration of a medication to a patient was omitted. This off-duty nurse telephones the unit and tells the nurse, "Please document the administration of the medication I forgot to do. My password is alpha1." The nurse should: a. fulfill the request. b. refer the matter to the charge nurse to resolve. c. access the record and document the information. d. report the request to the patient's health care provider. ANS: B At most hospitals, termination is a possible penalty for unauthorized entry into a patient record. Referring the matter to the charge nurse will allow the observance of hospital policy while ensuring that documentation occurs. Making an exception and fulfilling the request places the on-duty staff nurse in jeopardy. Reporting the request to the patient's health care provider would be unnecessary. Accessing the record and documenting the information would be unnecessary when the charge nurse can resolve the problem. 22. A patient with mental illness asks a psychiatric technician, "What's the matter with me?" The technician replies, "Your wing nuts need tightening." The patient looks bewildered and wanders off. The nurse who overheard the exchange should take action based on: a. violation of the patient's right to be treated with dignity and respect. b. the nurse's obligation to report caregiver negligence. c. preventing defamation of the patient's character. d. supervisory liability. ANS: A Patients have the right to be treated with dignity and respect. Patients should never be made the butt of jokes about their illness. Patient emotional abuse has been demonstrated, not negligence. The technician's response was not clearly defamation. Patient abuse, not supervisory liability, is the issue. 23. Which documentation of a patient's behavior best demonstrates a nurse's observations? a. Isolates self from others. Frequently fell asleep during group. Vital signs stable. b. Calmer and more cooperative. Participated actively in group. No evidence of psychotic thinking. c. Appeared to hallucinate. Patient frequently increased volume on television, causing conflict with others d. Wears four layers of clothing. States, "I need protection from dangerous bacteria trying to penetrate my skin." ANS: D The documentation states specific observations of the patient's appearance and the exact statements made. The other options are vague or subjective statements and can be interpreted in different ways. 1. A nurse volunteers for a committee that must revise the hospital policies and procedures for suicide precautions. Which resources would provide the best guidance? Select all that apply. a. Diagnostic and Statistical Manual of Mental Disorders (fourth edition, text revision) (DSM-IV-TR) b. State's nurse practice act c. State and federal regulations that govern hospitals d. Summary of common practices of several local hospitals e. American Nurses Association Scope and Standards of Practice for Psychiatric-Mental Health Nursing Practice ANS: C, E Regulations regarding hospitals provide information about the minimal standard. The American Nurses Association (ANA) national standards focus on elevating practice by setting high standards for nursing practice. The DSM-IV-TR and the state's nurse practice act would not provide relevant information. A summary of common practices of several local hospitals cannot be guaranteed to be helpful because the customs may or may not comply with laws or best practices. 2. In which situations does a nurse have a duty to intervene and report? Select all that apply. a. A peer is unable to write behavioral outcomes. b. A health care provider consults the Physicians' Desk Reference. c. A peer tries to provide patient care in an alcohol-impaired state. d. A team member has violated the boundaries of a vulnerable patient. e. A patient refuses a medication prescribed by a licensed health care provider. ANS: C, D Both instances jeopardize patient safety. The nurse must practice within the Code of Ethics for Nurses. A peer being unable to write behavioral outcomes is a concern but can be informally resolved. A health care provider consulting the Physicians' Desk Reference is acceptable practice. 3. Which situations qualify as abandonment on the part of a nurse? (Select all that apply.) The nurse: a. allows a patient with acute mania to refuse hospitalization without taking further action. b. terminates employment without referring a seriously mentally ill for aftercare. c. calls police to bring a suicidal patient to the hospital after a suicide attempt. d. refers a patient with persistent paranoid schizophrenia to community treatment. e. asks another nurse to provide a patient's care because of concerns about countertransference. ANS: A, B Abandonment arises when a nurse does not place a patient safely in the hands of another health professional before discontinuing treatment. Calling the police to bring a suicidal patient to the hospital after a suicide attempt and referring a patient with schizophrenia to community treatment both provide for patient safety. Asking another nurse to provide a patient's care because of concerns about countertransference demonstrates self-awareness. BOOK DISCUSSION QUESTIONS: 1. A nurses sibling happily says, “I want to introduce you to my fiancé. We are getting married in 6 months.” The nurse has encountered the fiancé in the clinical setting and is aware of the fiancé’s diagnoses of schizophrenia. What is the nurses best response? Answer: Say to the sibling and the fiancé, “ I hope you will be very happy together.” 2. A pt has been disruptive to the therapeutic Milieu for 2 days. A Certified nurses assistant says to the nurse, “ we need to seclude this patient because this behavior is upsetting everyone on the unit.” Considering pts’ rights, the nurse should respond, Answer: “Thank you for that suggestion. I will discuss it with the healthcare provider.” 3. A day shift nurse contacts a nurse scheduled for the night shift at home and says, “Our unit is full and there are 8 pts waiting for a bed.” The night shift nurse replies, “thank you for telling me. I am calling in sick.” Which type of problems is evident by the night shift nurse’s reply? Answer: Ethical problem of fidelity. 4. In a staff meeting at an inpatient mental health facility for persons, the administrator announces that the psychiatric technicians will now be supervised by the milieu director rather than the nurses. What’s the nurses best action? Answer: Advise the administrator of regulations in the state nurse practice act regarding supervision of UAP. 5. A colleague tells the nurse, “I have not been able to sleep for the past 3 days. I feel like a robot.” What is the nurses best action? Answer: Confer with the supervisor about the nurses ability to safely deliver care. Chapter 7 psych 1. A new staff nurse completes orientation to the psychiatric unit. This nurse will expect to ask an advanced practice nurse to perform which action for patients? c. Prescribe psychotropic medications. 2. A newly admitted patient with major depression has lost 20 pounds over the past month and has suicidal ideation. The patient has taken an antidepressant medication for 1 week without remission of symptoms. Select the priority nursing diagnosis. c. Risk for suicide 3. A patient with major depression has lost 20 pounds in one month has chronic low self-esteem and a plan for suicide. The patient has taken an antidepressant medication for 1 week. Which nursing intervention is most directly related to this outcome: “Patient will refrain from gestures and attempts to harm self”? a. Implement suicide precautions. 4. A patient’s nursing diagnosis is Insomnia. The desired outcome is: “Patient will sleep for a minimum of 5 hours nightly by October 31.” On November 1, a review of the sleep data shows the patient sleeps an average of 4 hours nightly and takes a 2-hour afternoon nap. Which evaluation should be documented? d. Never demonstrated 5. A patient’s nursing diagnosis is Insomnia. The desired outcome is: “Patient will sleep for a minimum of 5 hours nightly by October 31.” On November 1, a review of the sleep data shows the patient sleeps an average of 4 hours nightly and takes a 2-hour afternoon nap. What is the nurse’s next action? d. Revise the target date for outcome attainment and examine interventions. 6. A patient begins a new program to assist with building social skills. In which part of the plan of care should a nurse record the item “Encourage patient to attend one psychoeducational group daily”? d. Implementation 7. Before assessing a new patient, a nurse is told by another health care worker, “I know that patient. No matter how hard we work, there isn’t much improvement by the time of discharge.” The nurse’s responsibility is to: b. assess the patient based on data collected from all sources. 8. A nurse works with a patient to establish outcomes. The nurse believes that one outcome suggested by the patient is not in the patient’s best interest. What is the nurse’s best action? c. Explore with the patient possible consequences of the outcome. 9. A patient states, “I’m not worth anything. I have negative thoughts about myself. I feel anxious and shaky all the time. Sometimes I feel so sad that I want to go to sleep and never wake up.” Which nursing intervention should have the highest priority? d. Suicide precautions 10. Select the best outcome for a patient with the nursing diagnosis: Impaired social interaction, related to sociocultural dissonance as evidenced by stating, “Although I’d like to, I don’t join in because I don’t speak the language very well.” The patient will: d. select and participate in one group activity d. select and participate in one group activity per day. 11. Nursing behaviors associated with the implementation phase of the nursing process are concerned with: d. carrying out interventions and coordinating care. 12. Which statement made by a patient during an initial assessment interview should serve as the priority focus for the plan of care? d. “I hear evil voices that tell me to do bad things.” 13. Which entry in the medical record best meets the requirement for problem-oriented charting? b. “S: States, ‘I feel like I’m ready to blow up.’ O: Pacing hall, mumbling to self. A: Auditory hallucinations. P: Offer haloperidol (Haldol) 2 mg . I: (Haldol) 2 mg at 0900. E: Returned to lounge at 0930 and quietly watched TV.” 14. A nurse assesses an older adult patient brought to the emergency department by a family member. The patient was wandering outside, saying, “I can’t find my way home.” The patient is confused and unable to answer questions. Select the nurse’s best action. a. Document the patient’s mental status. Obtain other assessment data from the family member. 15. A nurse asks a patient, “If you had fever and vomiting for 3 days, what would you do?” Which aspect of the mental status examination is the nurse assessing? b. Cognition 16. An adolescent asks a nurse conducting an assessment interview, “Why should I tell you anything? You’ll just tell my parents whatever you find out.” Select the nurse’s best reply. c. “What you say about feelings is private, but some things, like suicidal thinking, must be reported to the treatment team.” 17. A nurse assessing a new patient asks, “What is meant by the saying, ‘You can’t judge a book by its cover’?” Which aspect of cognition is the nurse assessing? d. Abstraction 18. When a nurse assesses an older adult patient, answers seem vague or unrelated to the questions. The patient also leans forward and frowns, listening intently to the nurse. An appropriate question for the nurse to ask would be: a. “Are you having difficulty hearing when I speak?” 19. At what point in an assessment interview would a nurse ask, “How does your faith help you in stressful situations?” During the assessment of: d. coping strategies. 20. When a new patient is hospitalized, a nurse takes the patient on a tour, explains the rules of the unit, and discusses the daily schedule. The nurse is engaged in: c. milieu management. 21. After formulating the nursing diagnoses for a new patient, what is the next action a nurse should take? b. Determine the goals and outcome criteria. 22. Select the most appropriate label to complete this nursing diagnosis: ___________, related to feelings of shyness and poorly developed social skills as evidenced by watching television alone at home every evening. d. Social isolation 23. The acronym QSEN refers to: b. Quality and Safety Education for Nurses. 24. A nurse documents: “Patient is mute, despite repeated efforts to elicit speech. Makes no eye contact. Is inattentive to staff. Gazes off to the side or looks upward rather than at the speaker.” Which nursing diagnosis should be considered? d. Impaired verbal communication MULTIPLE RESPONSE 1. A nurse assesses a patient who reluctantly participates in activities, answers questions with minimal responses, and rarely makes eye contact. What information should be included when documenting the assessment? Select all that apply. b. Patient’s subjective responses d. Description of the patient’s behavior during the interview 2. A nurse performing an assessment interview for a patient with a substance use disorder decides to use a standardized rating scale. Which scales are appropriate? Select all that apply. a. Addiction Severity Index (ASI) b. Brief Drug Abuse Screen Test (B-DAST) e. Recovery Attitude and Treatment Evaluator (RAATE) 3. What information is conveyed by nursing diagnoses? Select all that apply. c. Unmet patient needs currently present d. Supporting data that validate the diagnoses e. Probable causes that will be targets for nursing interventions 4. A patient is very suspicious and states, “The FBI has me under surveillance.” Which strategies should a nurse use when gathering initial assessment data about this patient? Select all that apply. b. Ask the patient, “Tell me about the problem as you see it.” c. Seek information about when the problem began. e. Reassure the patient, “You are safe here.” Text Book questions 1. A nurse assesses a new patient whose chief concern is “daily crying spells.” which comment from the patient would prompt the nurse to suspect a medical reason is causing the problem rather than depression? C. Years ago I had thyroid problems but they cleared up and I stopped the medicine 2. A 55-year-old lives 100 miles from her parents and mother-in-law. In the past year, her father had back surgery, her mother broke her hip, and her mother-in-law had a cardiac event. Which nursing diagnosis is most applicable to the 55-year-old? D. Risk for caregiver role strain related to responsibilities for care of aging parents 3. A patient asks the psychiatric mental health registered nurse, Im having so much anxiety. I think hypnosis would help me. Will you do that for me? When determining a response, which factor should the nurse consider? B. State regulations regarding scope of practice 4. The nurse plans care for a newly hospitalized patient experiencing panic level anxiety after an automobile accident. The patient has no physical injuries. When selecting goals from the nursing outcomes classification the nurse will? C. Individualize outcomes based on the patient's needs 5. On an inpatient unit, one patient assaults another patient resulting in a small laceration. Considering the patients' right to confidentiality, how will the nurse effectively document this event? B. Document in each patient's medical record the events and actions taken, using initials of other patients involved Chapter 8 psych 1. A patient says to the nurse, “I dreamed I was stoned. When I woke up, I felt emotionally drained, as though I hadn’t rested well.” Which comment would be appropriate if the nurse seeks clarification? “Can you give me an example of what you mean by ‘stoned’?” 2. A patient with paranoid schizophrenia tells the nurse, “The CIA is monitoring us through the fluorescent lights in this room. Be careful what you say.” Which response by the nurse would be most therapeutic? “It sounds like you’re concerned about your privacy.” 3. The patient says, “My marriage is just great. My spouse and I usually agree on everything.” The nurse observes the patient’s foot moving continuously as the patient twirls a shirt button. What conclusion can the nurse draw? The patient’s communication is: mixed. 4. A nurse interacts with a newly hospitalized patient. Select the nurse’s comment that applies the communication technique of “offering self.” “I’d like to sit with you for a while to help you get comfortable talking to me.” 5. Which technique will best communicate to a patient that the nurse is interested in listening? a. Restate a feeling or thought the patient has expressed. 6. A patient discloses several concerns and associated feelings. If the nurse wants to seek clarification, which comment would be appropriate? “Am I correct in understanding that…?” 7. A patient tells the nurse, “I don’t think I’ll ever get out of here.” Select the nurse’s most therapeutic response. “You don’t think you’re making progress?” 8. Documentation in a patient’s chart shows, “Throughout a 5-minute interaction, patient fidgeted and tapped left foot, periodically covered face with hands, and looked under chair while stating, ‘I enjoy spending time with you.’” Which analysis is most accurate? Patient’s verbal and nonverbal messages are incongruent. 9. While talking with a patient with severe depression, a nurse notices the patient is unable to maintain eye contact. The patient’s chin lowers to the chest while the patient looks at the floor. Which aspect of communication has the nurse assessed? Nonverbal communication 10. During the first interview with a parent whose child died in a car accident, the nurse feels empathic and reaches out to take the patient’s hand. Select the correct analysis of the nurse’s behavior. The gesture is premature. The patient’s cultural and individual interpretation of touch is unknown 11. A Mexican-American patient puts a picture of the Virgin Mary on the bedside table. Under which section of the assessment should the nurse document this behavior? Culture 12. An African-American patient says to a Caucasian nurse, “There’s no sense talking. You wouldn’t understand because you live in a white world.” The nurse’s best action would be to: say, “Please give an example of something you think I wouldn’t understand.” 13. A Filipino-American patient had this nursing diagnosis: Situational low self-esteem, related to poor social skills as evidenced by lack of eye contact. Interventions were used to raise the patient’s self-esteem; however, after 3 weeks, the patient’s eye contact did not improve. What is the most accurate analysis of this scenario? The nurse should have assessed the patient’s culture before making this diagnosis and plan. 14. When a female Mexican-American patient and a female nurse sit together, the patient often holds the nurse’s hand. The patient also links arm and arm with the nurse when they walk. The nurse is uncomfortable with this behavior and thinks the patient is homosexual. Which alternative is a more accurate assessment? The patient is accustomed to touch during conversations, as are members of many Hispanic subcultures. 15. A Puerto Rican–American patient uses dramatic body language when describing emotional discomfort. Which analysis most likely explains the patient’s behavior? The patient: Belongs to a culture in which dramatic body language is the norm. 16. During an interview, a patient attempts to shift the focus from self to the nurse by asking personal questions. The nurse should respond by saying: “The time we spend together is to discuss your concerns.” 17. Which principle should guide the nurse in determining the extent of silence to use during patient interview sessions? Silence provides meaningful moments for reflection. 18. A patient is having difficulty making a decision. The nurse has mixed feelings about whether to provide advice. Which principle usually applies? Giving advice: Is rarely helpful. 19. The relationship between a nurse and patient as it relates to status and power is best described by which term? Complementary 20. A patient with acute depression states, “God is punishing me for my past sins.” What is the nurse’s best response? “You sound very upset about this.” MULTIPLE RESPONSE 1. A patient cries as the nurse explores the patient’s relationship with a deceased parent. The patient says, “I shouldn’t be crying like this. It happened a long time ago.” Which responses by the nurse will facilitate communication? Select all that apply. b. “I can see that you feel sad about this situation.” c. “The loss of your parent is very painful for you.” d. “Crying is a way of expressing the hurt you’re experiencing.” 2. Which benefits are most associated with the use of telehealth? Select all that apply. a. Cost savings for patients b. Maximization of care management c. Access to services for patients in rural areas Text Book Questions 1.An adult experiencing a recent exacerbation of ulcerative colitis tells the nurse, I had an accident while I was at the grocery store. It was so embarrassing. Select the nurse's therapeutic response? A. Most grocery stores have public restrooms available 2. A nurse counsels a widow whose husband died 5 years ago. The widow says, If I'd done more, he would still be alive. Select the nurse's therapeutic response D. Your husband was 82 years old with severe chronic obstructive pulmonary disease 3. A patient has been out of work 3 weeks with a major illness and anticipates another month of recovery. The patient tells the nurse, Im trying to keep up with my work email from home. They hired a new person in my department but the person has no experience. Select the nurse's therapeutic response A. It sounds like you're saying you are worried about your job security 4. In which nurse patient interaction would it be appropriate for the nurse to consider using touch? C. Welcoming a person of hispanic heritage to a new group session 5. A nurse prepares a patient in a rural community for an initial telehealth visit with the health care provider. Select the nurse's priority action A. Ensure that the patient's rights to privacy are respected Chapter 9 psych 1. A nurse assesses a confused older adult. The nurse experiences sadness and reflects, “The patient is like one of my grandparents…so helpless.” What feelings does the nurse describe? Countertransference 2. Which statement shows a nurse has empathy for a patient who made a suicide attempt? a. “You must have been very upset when you tried to hurt yourself.” 3. After several therapeutic encounters with a patient who recently attempted suicide, which occurrence should cause the nurse to consider the possibility of countertransference? c. The nurse feels oddly happy when the patient’s mood begins to lift. 4. A patient says, “Please don’t share information about me with the other people.” How should the nurse respond? a. “I won’t share information with your family or friends without your permission, but I will share information about you with other staff members.” 5. A nurse is talking with a patient, and 5 minutes remain in the session. The patient has been silent for most of the session. Another patient comes to the door of the room, interrupts, and says to the nurse, “I really need to talk to you.” The nurse should: d. tell the patient who has interrupted, “This session is 5 more minutes; then I will talk with you.” 6. Termination of a therapeutic nurse-patient relationship with a patient has been successful when the nurse: discusses with the patient changes that have happened during the relationship and evaluates the outcomes. 7. What is the desirable outcome for the orientation stage of a nurse-patient relationship? The patient will demonstrate behaviors that indicate: rapport and trust with the nurse. 8. During which phase of the nurse-patient relationship can the nurse anticipate that identified patient issues will be explored and resolved? Working 9. At what point in the nurse-patient relationship should a nurse plan to first address termination? a. In the orientation phase 10. A nurse should introduce the matter of a contract during the first session with a new patient because contracts: spell out the participation and responsibilities of each party. 11. As a nurse escorts a patient being discharged after treatment for major depression, the patient gives the nurse a necklace with a heart pendant and says, “Thank you for helping mend my broken heart.” Which is the nurse’s best response? c. “I’m glad I could help you, but I can’t accept the gift. My reward is seeing you with a renewed sense of hope.” 12. Which remark by a patient indicates passage from the orientation phase to the working phase of a nurse-patient relationship?d. “I want to find a way to deal with my anger without becoming violent.” 13. A nurse explains to the family of a patient who is mentally ill how the nurse-patient relationship differs from social relationships. Which is the best explanation? a. “The focus is on the patient. Problems are discussed by the nurse and patient, but solutions are implemented by the patient.” 14. A nurse wants to demonstrate genuineness with a patient diagnosed with schizophrenia. The nurse should:b. use congruent communication strategies. 15. A nurse caring for a withdrawn, suspicious patient recognizes the development of feelings of anger toward the patient. The nurse should: d. discuss the anger with a clinician during a supervisory session. 16. A nurse wants to enhance the growth of a patient by showing positive regard. The action consistent with this wish is: making rounds daily. 17. A patient says, “I’ve done a lot of cheating and manipulating in my relationships.” Select a nonjudgmental response by the nurse. a. “How do you feel about that?” 18. A patient says, “People should be allowed to commit suicide without interference from others.” A nurse replies, “You’re wrong. Nothing is bad enough to justify death.” What is the best analysis of this interchange? d. Differing values are reflected in the two statements. 19. Which issues should a nurse address during the first interview with a patient with a psychiatric disorder? c. Relationship parameters, the contract, confidentiality, and termination 20. During the first interview, a nurse notices that the patient does not make eye contact. The nurse can correctly analyze that:d. more information is needed to draw a conclusion 21. Which behavior shows that a nurse values autonomy? The nurse: d. discusses available alternatives and helps the patient weigh the consequences. 22. As a nurse discharges a patient, the patient gives the nurse a card of appreciation made in an arts and crafts group. What is the nurse’s best action? a. Recognize the effectiveness of the relationship and patient’s thoughtfulness. Accept the card. 23. A patient says, “I’m still on restriction, but I want to attend some off-unit activities. Would you ask the doctor to change my privileges?” What is the nurse’s best response? c. “That’s a good topic for you to take up with your doctor.” 24. A community mental health nurse has worked with a patient for 3 years but is moving out of the city and terminates the relationship. A new nurse who begins work with this patient will: a. begin at the orientation phase. 25. As a patient with mental illness is being discharged from a facility, a nurse invites the patient to the annual staff picnic. What is the best analysis of this scenario? b. The nurse’s action blurs the boundaries of the therapeutic relationship. 26. A nurse says, “I’m the only one who truly understands this patient. Other staff members are too critical.” The nurse’s statement indicates: a. boundary blurring. MULTIPLE RESPONSE 1. Which descriptors exemplify consistency regarding nurse-patient relationships? Select all that apply. a. Having the same nurse care for a patient on a daily basis c. Providing a schedule of daily activities to a patient d. Setting a time for regular sessions with a patient 2. A nurse ends a relationship with a patient. Which actions by the nurse should be included in the termination phase? Select all that apply. a. Focus dialogs with the patient on problems that may occur in the future. b. Help the patient express feelings about the relationship with the nurse. 3. A new psychiatric nurse has a parent with bipolar disorder. This nurse angrily recalls embarrassing events concerning the parent’s behavior in the community. Select the best ways for this nurse to cope with these feelings. Select all that apply. a. Seek ways to use the understanding gained from childhood to help patients cope with their own illnesses. e. Recognize that the feelings may add sensitivity to the nurse’s practice, but supervision is important. 4. A new nurse tells a mentor, “I want to convey to my patients that I am interested in them and that I want to listen to what they have to say.” Which behaviors are helpful in meeting the nurse’s goal? Select all that apply. b. Introducing self to a patient and identifying own role. c. Using facial expressions that convey interest and encouragement. d. Assuming an open body posture and sometimes mirror imaging. Text book questions 1. Which comment by the nurse would be appropriate to begin a new nurse patient relationship? D. Would you like to tell me about yourself? 2. A neighbor telephones the nurse daily, giving lengthy details about multiple somatic complaints and relationship problems. Which limit setting strategy should the nurse employ? B. Say to the neighbor, I can talk to you for 15 minutes twice a week 3. A patient has been oppositional, demanding, and resistant to working on goals. A mental health nurse tells the nursing supervisor, we finally had a serious talk. I let that patient know it's time to get right with god and stop this behavior. Recognizing the nurse's actions were not acceptable, select the supervisor's responding action D. Work with the nurse to prepare and analyze a process recording of the interaction 4. A nurse participating in a community health fair interviews an adult who has had no interaction with a health care professional for more than 10 years. The adult says, I like to keep to myself. Crowds make me nervous. Which action should the nurse employ? C. Ask the adult, how do you feel about the quality of your life? 5. A group of nurses privately discuss patients under their care. Which nurse's comment indicates the need for clinical supervision regarding countertransference? C. My patient tries to tell me what to do all the time. I got a divorce because my spouse used to do that. Chapter 10 psych 1. Which scenario best demonstrates an example of eustress? An individual: b. prepares to take a 1 week vacation to a tropical island with a group of close friends. 2. A patient with liver failure has been on the transplant waiting list for 8 months. The patient says to the nurse, “Why is it taking so long to have the surgery? Maybe I’m meant to die for all the bad things I’ve done.” The nurse should document the patient’s comment in which section of the assessment? b. Spiritual 3. A person with a fear of heights drives across a high bridge. Which structure will stimulate a response from the autonomic nervous system? c. Hypothalamus 4. A person with a fear of heights drives across a high bridge. Which division of the autonomic nervous system is stimulated in response to this experience? c. Sympathetic nervous system 5. A patient is brought to the emergency department after a motorcycle accident. The patient is alert, responsive, and diagnosed with a broken leg. The patient’s vital signs are temperature (T), 98.6° F; pulse (P), 72 beats per minute (bpm); and respirations (R), 16 breaths per minute. After being informed that surgery is required for the broken leg, which vital sign readings would be expected? d. T, 98.6°; P, 84; R, 22 6. As part of the stress response, the HPA axis is stimulated. Which structures make up this system? b. Hypothalamus, pituitary gland, and adrenal glands 7. Cortisol is released in response to a patient’s prolonged stress. Which initial effect would the nurse expect to result from the increased cortisol level? b. Focused and alert mental status 8. A soldier returns to the United States from active duty in a combat zone in Afghanistan. The soldier is diagnosed with posttraumatic stress disorder (PTSD). The nurse’s highest priority is to screen this soldier for which problem? a. Major depression 9. A soldier returns to the United States from active duty in a combat zone in Afghanistan. The soldier is diagnosed with post-traumatic stress disorder (PTSD). Which comment by the soldier requires the nurse’s immediate attention? b. “I saw my best friend get killed by a roadside bomb. It should have been me instead.” 10. A soldier returned home from active duty in a combat zone in Afghanistan and was diagnosed with post-traumatic stress disorder (PTSD). The soldier says, “If there’s a loud noise at night, I get under my bed because I think we’re getting bombed.” What type of experience has the soldier described? b. Flashback 11. A soldier returned 3 months ago from Afghanistan and was diagnosed with post-traumatic stress disorder (PTSD). Which social event would most likely be disturbing for this soldier? d. Fireworks display on July 4th 12. A soldier served in combat zones in Iraq in 2010 and was deployed to Afghanistan in 2011. When is it most important for the nurse to screen for signs and symptoms of post-traumatic stress disorder (PTSD)? d. Screening should be ongoing 13. A nurse assesses soldiers in a combat zone in Afghanistan. When is it most important for the nurse to screen for signs and symptoms of traumatic brain injury (TBI)? a. After a fall, vehicle crash, or exposure to a blast 14. A soldier in a combat zone tells the nurse, “I saw a child get blown up over a year ago, but I keep seeing bits of flesh everywhere. I see something red and the visions race back to my mind.” Which phenomenon associated with post-traumatic stress disorder (PTSD) is this soldier describing? a. Reexperiencing 15. A soldier who served in a combat zone returned to the United States. The soldier’s spouse complains to the nurse, “We had planned to start a family, but now he won’t talk about it. He won’t even look at children.” The spouse is describing which symptom associated with post-traumatic stress disorder (PTSD)? c. Avoidance 16. A soldier returned home last year after deployment to a war zone. The soldier’s spouse complains, “We were going to start a family but now he won’t talk about it. He will not look at children. I wonder if we’re going to make it as a couple.” Select the nurse’s best response. d. “Posttraumatic stress disorder often strains relationships. Here are some community resources for help and support.” 17. A nurse talks with the caregiver of a combat veteran with severe traumatic brain injuries. The caregiver says, “I don’t know how much longer I can do it. My whole life is consumed with taking care of my partner.” Select the nurse’s best response. a. “How are you taking care of yourself?” 18. A professor’s 4-year-old child has a body temperature of 101.6° F, diarrhea, and complains of stomach pain. The professor is scheduled to teach three classes today. Which nursing diagnosis best applies to this scenario? a. Decisional conflict MULTIPLE RESPONSE 1. An individual says to the nurse, “I feel so stressed out lately. I think the stress is affecting my body also.” Which somatic complaints are most likely to accompany this feeling? Select all that apply. a. Headache b. Neck pain c. Insomnia d. Anorexia 2. Which experiences are most likely to precipitate post-traumatic stress disorder (PTSD)? Select all that apply. c. An adolescent is kidnapped and held for 2 years in the home of a sexual predator. d. A passenger is in a bus that overturns on a sharp curve in the road, tumbling down an embankment. e. An adult is trapped for 3 hours at an angle in an elevator after a portion of the supporting cable breaks. 3. A nurse assesses the health status of soldiers returning from Afghanistan. Screening will be a priority for signs and symptoms of which health problems? Select all that apply. c. Traumatic brain injury e. Posttraumatic stress disorder 4. A professor’s 4-year-old child has a temperature of 101.6° F, diarrhea, and complains of stomach pain. The professor is scheduled to teach three classes today. Which actions by the professor demonstrate effective parenting? Select all that apply. a. Telephoning a grandparent to stay with the child at home for the day. b. Telephoning a colleague to teach his classes and stays home with the sick child. Text book questions 1.A mature, professional couple plans a large wedding in a city 100 miles from their home. Which response is most likely to be associated with this experience? Eustress 2. A college student has been experiencing significant stress associated with academic demands. Last month, the student began attending yoga sessions three times a week. Which outcome indicates this activity has been successful? The student reports improved feelings of well being 3. An adult required a heart transplant 5 years ago. Multiple medical complications followed, resulting in persistant irritability, depression, and insomnia. The adult’s spouse says, “Ive walked on eggshells for five years, never knowing when something else might go wrong.” What is the nurses priority intervention regarding the spouse? Refer the spouse to the primary care provider for health assessment. 4. A veteran of the war in Afghanistan tells the nurse, “Everyday something happens that makes me feel like Im still there. My family has grown impatient with me. They say its time for me to move on from that time in my life but I cant.” What is the nurses priority? Assess the veteran for suicide risk 5. An individual lives in a community adjacent to a military base. Loud jets fly overhead multiple times daily. The person tells the nurse, “theyre so loud I cant hear myself think.” what is the nurses best first action? Teach relaxation and stress reduction techniques to the individual. Chapter 11 psych 1. A nurse wishes to teach alternative coping strategies to a patient who is experiencing severe anxiety. The nurse will first need to: a. Lower the patient's current anxiety level. b. Verify the patient's learning style. c. Create outcomes and a teaching plan. d. Assess how the patient uses defense mechanisms. ANS: A A patient experiencing severe anxiety has a significantly narrowed perceptual field and difficulty attending to events in the environment. A patient experiencing severe anxiety will not learn readily. Determining preferred modes of learning, devising outcomes, and constructing teaching plans are relevant to the task but are not the priority measure. The nurse has already assessed the patient's anxiety level. Using defense mechanisms does not apply. 2. A patient approaches the nurse and impatiently blurts out, "You've got to help me! Something terrible is happening. My heart is pounding." The nurse responds, "It's almost time for visiting hours. Let's get your hair combed." Which approach has the nurse used? a. Distracting technique to lower anxiety b. Bringing up an irrelevant topic c. Responding to physical needs d. Addressing false cognitions ANS: B The nurse has closed off patient-centered communication. The introduction of an irrelevant topic makes the nurse feel better. The nurse is uncomfortable dealing with the patient's severe anxiety. 3. A patient who is experiencing moderate anxiety says, "I feel undone." An appropriate response for the nurse would be: a. "Why do you suppose you are feeling anxious?" b. "What would you like me to do to help you?" c. "I'm not sure I understand. Give me an example." d. "You must get your feelings under control before we can continue." ANS: C Increased anxiety results in scattered thoughts and an inability to articulate clearly. Clarification helps the patient identify his or her thoughts and feelings. Asking the patient why he or she feels anxious is nontherapeutic, and the patient will not likely have an answer. The patient may be unable to determine what he or she would like the nurse to do to help. Telling the patient to get his or her feelings under control is a directive the patient is probably unable to accomplish. 4. A patient with a high level of motor activity runs from chair to chair and cries, "They're coming! They're coming!" The patient does not follow the staff's directions or respond to verbal interventions. The initial nursing intervention of highest priority is to: a. provide for patient safety. b. increase environmental stimuli. c. respect the patient's personal space. d. encourage the clarification of feelings. ANS: A Safety is of highest priority; the patient who is experiencing panic is at high risk for self-injury related to an increase in non-goal-directed motor activity, distorted perceptions, and disordered thoughts. The goal should be to decrease the environmental stimuli. Respecting the patient's personal space is a lower priority than safety. The clarification of feelings cannot take place until the level of anxiety is lowered. 5. A patient with a high level of motor activity runs from chair to chair and cries, "They're coming! They're coming!" The patient is unable to follow staff direction or respond to verbal interventions. Which nursing diagnosis has the highest priority? a. Risk for injury b. Self-care deficit c. Disturbed energy field d. Disturbed thought processes ANS: A A patient who is experiencing panic-level anxiety is at high risk for injury, related to an increase in non-goal-directed motor activity, distorted perceptions, and disordered thoughts. Existing data do not support the nursing diagnoses of Self-care deficit or Disturbed energy field. This patient has disturbed thought processes, but the risk for injury has a higher priority. 6. A supervisor assigns a worker a new project. The worker initially agrees but feels resentful. The next day when asked about the project, the worker says, "I've been working on other things." When asked 4 hours later, the worker says, "Someone else was using the copier, so I couldn't finish it." The worker's behavior demonstrates: a. acting out. b. projection. c. rationalization. d. passive aggression. ANS: D A passive-aggressive person deals with emotional conflict by indirectly expressing aggression toward others. Compliance on the surface masks covert resistance. Resistance is expressed through procrastination, inefficiency, and stubbornness in response to assigned tasks. 7. A patient is undergoing diagnostic tests. The patient says, "Nothing is wrong with me except a stubborn chest cold." The spouse reports that the patient smokes, coughs daily, has lost 15 pounds, and is easily fatigued. Which defense mechanism is the patient using? a. Displacement b. Regression c. Projection d. Denial ANS: D Denial is an unconscious blocking of threatening or painful information or feelings. Regression involves using behaviors appropriate at an earlier stage of psychosexual development. Displacement shifts feelings to a more neutral person or object. Projection attributes one's own unacceptable thoughts or feelings to another. 8. A patient with a mass in the left upper lobe of the lung is scheduled for a biopsy. The patient has difficulty understanding the nurse's comments and asks, "What do you mean? What are they going to do?" Assessment findings include a tremulous voice, respirations at 28 breaths per minute, and a pulse rate at 110 beats per minute. What is the patient's level of anxiety? a. Mild b. Moderate c. Severe d. Panic ANS: B Moderate anxiety causes the individual to grasp less information and reduces his or her problem-solving ability to a less-than-optimal level. Mild anxiety heightens attention and enhances problem-solving abilities. Severe anxiety causes great reduction in the perceptual field. Panic-level anxiety results in disorganized behavior. 9. A patient who is preparing for surgery has moderate anxiety and is unable to understand preoperative information. Which nursing intervention is appropriate? a. Reassure the patient that all nurses are skilled in providing postoperative care. b. Describe the procedure again in a calm manner using simple language. c. Tell the patient that the staff is prepared to promote recovery. d. Encourage the patient to express feelings to his or her family. ANS: B Providing information in a calm, simple manner helps the patient grasp the important facts. Introducing extraneous topics as described in the remaining options will further scatter the patient's attention. 10. A nurse encourages an anxious patient to talk about feelings and concerns. What is the rationale for this intervention? a. Offering hope allays and defuses the patient's anxiety. b. Concerns stated aloud become less overwhelming and help problem solving to begin. c. Anxiety is reduced by focusing on and validating what is occurring in the environment. d. Encouraging patients to explore alternatives increases the sense of control and lessens anxiety. ANS: B All principles listed are valid, but the only rationale directly related to the intervention of assisting the patient to talk about feelings and concerns is the one that states that concerns spoken aloud become less overwhelming and help problem solving to begin. 11. Which assessment question would be most appropriate to ask a patient who has possible generalized anxiety disorder? a. "Have you been a victim of a crime or seen someone badly injured or killed?" b. "Do you feel especially uncomfortable in social situations involving people?" c. "Do you repeatedly do certain things over and over again?" d. "Do you find it difficult to control your worrying?" ANS: D Patients with generalized anxiety disorder frequently engage in excessive worrying. They are less likely to engage in ritualistic behavior, fear social situations, or have been involved in a highly traumatic event. 12. A patient in the emergency department exhibits disorganized behavior and incoherence after a friend suggested a homosexual encounter. In which room should the nurse place the patient? a. Interview room furnished with a desk and two chairs b. Small, empty storage room with no windows or furniture c. Room with an examining table, instrument cabinets, desk, and chair d. Nurse's office, furnished with chairs, files, magazines, and bookcases ANS: A Individuals who are experiencing a severe-to-panic level of anxiety require a safe environment that is quiet, nonstimulating, structured, and simple. A room with a desk and two chairs provides simplicity, few objects with which the patient could cause self-harm, and a small floor space around which the patient can move. A small, empty storage room without windows or furniture would be like a jail cell. The nurse's office or a room with an examining table and instrument cabinets may be overstimulating and unsafe. 13. A person has minor physical injuries after an automobile accident. The person is unable to focus and says, "I feel like something awful is going to happen." This person has nausea, dizziness, tachycardia, and hyperventilation. What is this person's level of anxiety? a. Mild b. Moderate c. Severe d. Panic ANS: C The person whose anxiety is severe is unable to solve problems and may have a poor grasp of what is happening in the environment. Somatic symptoms such as those described are usually present. The individual with mild anxiety is only mildly uncomfortable and may even find his or her performance enhanced. The individual with moderate anxiety grasps less information about a situation and has some difficulty with problem solving. The individual in a panic level of anxiety demonstrates significantly disturbed behavior and may lose touch with reality. 14. Two staff nurses applied for a charge nurse position. After the promotion was announced, the nurse who was not promoted said, "The nurse manager had a headache the day I was interviewed." Which defense mechanism is evident? a. Introjection b. Conversion c. Projection d. Splitting ANS: C Projection is the hallmark of blaming, scapegoating, thinking prejudicially, and stigmatizing others. Conversion involves the unconscious transformation of anxiety into a physical symptom. Introjection involves intense, unconscious identification with another person. Splitting is the inability to integrate the positive and negative qualities of oneself or others into a cohesive image. 15. A patient tells a nurse, "My new friend is the most perfect person one could imagine—kind, considerate, and good looking. I can't find a single flaw." This patient is demonstrating: a. denial. b. projection. c. idealization. d. compensation. ANS: C Idealization is an unconscious process that occurs when an individual attributes exaggerated positive qualities to another. Denial is an unconscious process that calls for the nurse to ignore the existence of the situation. Projection operates unconsciously and results in blaming behavior. Compensation results in the nurse unconsciously attempting to make up for a perceived weakness by emphasizing a strong point. 16. A patient experiences an episode of severe anxiety. Of these medications in the patient's medical record, which is most appropriate to administer as an as-needed (prn) anxiolytic medication? a. buspirone (BuSpar) b. lorazepam (Ativan) c. amitriptyline (Elavil) d. desipramine (Norpramin) ANS: B Lorazepam is a benzodiazepine medication used to treat anxiety; it may be administered as needed. Buspirone is long acting and not useful as an as-needed drug. Amitriptyline and desipramine are tricyclic antidepressants and considered second- or third-line agents. 17. Two staff nurses applied for promotion to nurse manager. The nurse not promoted initially had feelings of loss but then became supportive of the new manager by helping make the transition smooth and encouraging others. Which term best describes the nurse's response? a. Altruism b. Sublimation c. Suppression d. Passive aggression ANS: A Altruism is the mechanism by which an individual deals with emotional conflict by meeting the needs of others and vicariously receiving gratification from the responses of others. The nurse's reaction is conscious, not unconscious. No evidence of aggression is exhibited, and no evidence of conscious denial of the situation exists. Passive aggression occurs when an individual deals with emotional conflict by indirectly and unassertively expressing aggression toward others. 18. A person who feels unattractive repeatedly says, "Although I'm not beautiful, I am smart." This is an example of: a. Repression b. Devaluation c. Identification d. Compensation ANS: D Compensation is an unconscious process that allows an individual to make up for deficits in one area by excelling in another area to raise self-esteem. Repression unconsciously puts an idea, event, or feeling out of awareness. Identification is an unconscious mechanism calling for an imitation of the mannerisms or behaviors of another. Devaluation occurs when the individual attributes negative qualities to self or to others. 19. A person who is speaking about a rival for a significant other's affection says in a gushy, syrupy voice, "What a lovely person. That's someone I simply adore." The individual is demonstrating: a. Reaction formation b. Repression c. Projection d. Denial ANS: A Reaction formation is an unconscious mechanism that keeps unacceptable feelings out of awareness by using the opposite behavior. Instead of expressing hatred for the other person, the individual gives praise. Denial operates unconsciously to allow an anxiety-producing idea, feeling, or situation to be ignored. Projection involves unconsciously disowning an unacceptable idea, feeling, or behavior by attributing it to another. Repression involves unconsciously placing an idea, feeling, or event out of awareness. 20. An individual experiences sexual dysfunction and blames it on a partner by calling the person unattractive and unromantic. Which defense mechanism is evident? a. Rationalization b. Compensation c. Introjection d. Regression ANS: A Rationalization involves unconsciously making excuses for one's behavior, inadequacies, or feelings. Regression involves the unconscious use of a behavior from an earlier stage of emotional development. Compensation involves making up for deficits in one area by excelling in another area. Introjection is an unconscious, intense identification with another person. 21. A student says, "Before taking a test, I feel a heightened sense of awareness and restlessness." The nurse can correctly assess the student's experience as: a. Culturally influenced b. Displacement c. Trait anxiety d. Mild anxiety ANS: D Mild anxiety is rarely obstructive to the task at hand. It may be helpful to the patient because it promotes study and increases awareness of the nuances of questions. The incorrect responses have different symptoms. 22. A student says, "Before taking a test, I feel a heightened sense of awareness and restlessness." The nursing intervention most suitable for assisting the student is to: a. Explain that the symptoms are the result of mild anxiety, and discuss the helpful aspects. b. Advise the student to discuss this experience with a health care provider. c. Encourage the student to begin antioxidant vitamin supplements. d. Listen without comment. ANS: A Teaching about the symptoms of anxiety, their relation to precipitating stressors, and, in this case, the positive effects of anxiety serves to reassure the patient. Advising the patient to discuss the experience with a health care provider implies that the patient has a serious problem. Listening without comment will do no harm but deprives the patient of health teaching. Antioxidant vitamin supplements are not useful in this scenario. 23. If a cruel and abusive person rationalizes this behavior, which comment is most characteristic of this person? a. "I don't know why it happens." b. "I have poor impulse control." c. "That person shouldn't have provoked me." d. "I'm really a coward who is afraid of being hurt." ANS: C Rationalization consists of justifying one's unacceptable behavior by developing explanations that satisfy the teller and attempt to satisfy the listener. The abuser is suggesting that the abuse is not his or her fault; it would not have occurred except for the provocation by the other person. 24. A patient with severe anxiety suddenly begins running and shouting, "I'm going to explode!" The nurse should: a. Ask, "I'm not sure what you mean. Give me an example." b. Chase after the patient, and give instructions to stop running. c. Capture the patient in a basket-hold to increase feelings of control. d. Assemble several staff members and state, "We will help you regain control." ANS: D The safety needs of the patient and other patients are a priority. The patient is less likely to cause self-harm or hurt others when several staff members take responsibility for providing limits. The explanation given to the patient should be simple and neutral. Simply being told that others can help provide the control that has been lost may be sufficient to help the patient regain control. Running after the patient will increase the patient's anxiety. More than one staff member is needed to provide physical limits if they become necessary. Asking the patient to give an example is futile; a patient in panic processes information poorly. 25. A person who has been unable to leave home for more than a week because of severe anxiety says, "I know it does not make sense, but I just can't bring myself to leave my apartment alone." Which nursing intervention is appropriate? a. Teach the person to use positive self-talk. b. Assist the person to apply for disability benefits. c. Ask the person to explain why the fear is so disabling. d. Advise the person to accept the situation and use a companion. ANS: A This intervention, a form of cognitive restructuring, replaces negative thoughts such as "I can't leave my apartment" with positive thoughts such as "I can control my anxiety." This technique helps the patient gain mastery over the symptoms. The other options reinforce the sick role. 26. Which comment by a person who is experiencing severe anxiety indicates the possibility of obsessive-compulsive disorder? a. "I check where my car keys are eight times." b. "My legs often feel weak and spastic." c. "I'm embarrassed to go out in public." d. "I keep reliving the car accident." ANS: A Recurring doubt (obsessive thinking) and the need to check (compulsive behavior) suggest obsessive-compulsive disorder. The repetitive behavior is designed to decrease anxiety but fails and must be repeated. The statement, "My legs feel weak most of the time," is more in keeping with a somatoform disorder. Being embarrassed to go out in public is associated with an avoidant personality disorder. Reliving a traumatic event is associated with posttraumatic stress disorder. 27. When alprazolam (Xanax) is prescribed for acute anxiety, health teaching should include instructions to: a. Report drowsiness. b. Eat a tyramine-free diet. c. Avoid alcoholic beverages. d. Adjust dose and frequency based on anxiety level. ANS: C Drinking alcohol or taking other anxiolytic medications along with the prescribed benzodiazepine should be avoided because depressant effects of both drugs will be potentiated. Tyramine-free diets are necessary only with monoamine oxidase inhibitors (MAOIs). Drowsiness is an expected effect and needs to be reported only if it is excessive. Patients should be taught not to deviate from the prescribed dose and schedule for administration. 28. Which statement is mostly likely to be made by a patient with agoraphobia? a. "Being afraid to go out seems ridiculous, but I can't go out the door." b. "I'm sure I'll get over not wanting to leave home soon. It takes time." c. "When I have a good incentive to go out, I can do it." d. "My family says they like it now that I stay home." ANS: A Individuals who are agoraphobic generally acknowledge that the behavior is not constructive and that they do not really like it. Patients state they are unable to change the behavior. Patients with agoraphobia are not optimistic about change. Most families are dissatisfied when family members refuse to leave the house. 29. A patient has the nursing diagnosis: Anxiety, related to __________, as evidenced by an inability to control compulsive cleaning. Which phrase correctly completes the etiologic portion of the diagnosis? a. Ensuring the health of household members b. Attempting to avoid interactions with others c. Having persistent thoughts about bacteria, germs, and dirt d. Needing approval for cleanliness from friends and family ANS: C Many compulsive rituals accompany obsessive thoughts. The patient uses these rituals to relief anxiety. Unfortunately, the anxiety relief is short lived, and the patient must frequently repeat the ritual. The other options are unrelated to the dynamics of compulsive behavior. 30. A patient performs ritualistic hand washing. What should the nurse do to help the patient develop more effective coping strategies? a. Allow the patient to set a hand-washing schedule. b. Encourage the patient to participate in social activities. c. Encourage the patient to discuss hand-washing routines. d. Focus on the patient's symptoms rather than on the patient. ANS: B Because patients with obsessive-compulsive disorder become overly involved in rituals, promoting involvement with other people and activities is necessary to improve the patient's coping strategies. Daily activities prevent the constant focus on anxiety and its symptoms. The other interventions focus on the compulsive symptom. 31. For a patient experiencing panic, which nursing intervention should be first? a. Teach relaxation techniques. b. Administer an anxiolytic medication. c. Provide calm, brief, directive communication. d. Gather a show of force in preparation for gaining physical control. ANS: C Calm, brief, directive verbal interaction can help the patient gain control of the overwhelming feelings and impulses related to anxiety. Patients experiencing panic-level anxiety are unable to focus on reality; thus learning relaxation techniques is virtually impossible. Administering an anxiolytic medication should be considered if providing calm, brief, directive communication is ineffective. Although the patient is disorganized, violence may not be imminent, ruling out the intervention of preparing for physical control until other less-restrictive measures are proven ineffective. 32. Which finding indicates that a patient with moderate-to-severe anxiety has successfully lowered the anxiety level to mild? The patient: a. Asks, "What's the matter with me?" b. Stays in a room alone and paces rapidly. c. Can concentrate on what the nurse is saying. d. States, "I don't want anything to eat. My stomach is upset." ANS: C The ability to concentrate and attend to reality is increased slightly in mild anxiety and decreased in moderate-, severe-, and panic-level anxiety. Patients with high levels of anxiety often ask, "What's the matter with me?" Staying in a room alone and pacing suggest moderate anxiety. Expressing a lack of hunger is not necessarily a criterion for evaluating anxiety. 33. A patient tells the nurse, "I don't go to restaurants because people might laugh at the way I eat or I could spill food and be laughed at." The nurse assesses this behavior as consistent with: a. Acrophobia b. Agoraphobia c. Social phobia d. Posttraumatic stress disorder ANS: C The fear of a potentially embarrassing situation represents a social phobia. Acrophobia is the fear of heights. Agoraphobia is the fear of a place in the environment. Posttraumatic stress disorder is associated with a major traumatic event. 34. A patient checks and rechecks electrical cords related to an obsessive thought that the house may burn down. The nurse and patient explore the likelihood of an actual fire. The patient states that a house fire is not likely. This counseling demonstrates the principles of: a. Flooding b. Desensitization c. Relaxation technique d. Cognitive restructuring ANS: D Cognitive restructuring involves the patient in testing automatic thoughts and drawing new conclusions. Desensitization involves a graduated exposure to a feared object. Relaxation training teaches the patient to produce the opposite of the stress response. Flooding exposes the patient to a large amount of undesirable stimuli in an effort to extinguish the anxiety response. 35. A patient has a fear of public speaking. The nurse should be aware that social phobias are often treated with which type of medication? a. (beta)-blockers. b. Antipsychotic medications. c. Tricyclic antidepressant agents. d. Monoamine oxidase inhibitors. ANS: A Beta-blockers, such as propranolol, are often effective in preventing symptoms of anxiety associated with social phobias. Neuroleptic medications are major tranquilizers and not useful in treating social phobias. Tricyclic antidepressants are rarely used because of their side effect profile. MAOIs are administered for depression and only by individuals who can observe the special diet required. 36. A patient tells the nurse, "I wanted my health care provider to prescribe diazepam (Valium) for my anxiety disorder, but buspirone (BuSpar) was prescribed instead. Why?" The nurse's reply should be based on the knowledge that buspirone: a. Does not produce blood dyscrasias. b. Does not cause dependence. c. Can be administered as needed. d. Is faster acting than diazepam. ANS: B Buspirone is considered effective in the long-term management of anxiety because it is not habituating. Because it is long acting, buspirone is not valuable as an as-needed or as a fast-acting medication. The fact that buspirone does not produce blood dyscrasias is less relevant in the decision to prescribe buspirone. 1. A child is placed in a foster home after being removed from parental contact because of abuse. The child is apprehensive and overreacts to environmental stimuli. The foster parents ask the nurse how to help the child. What should the nurse recommend? Select all that apply. a. Use a calm manner and low voice. b. Maintain simplicity in the environment. c. Avoid repetition in what is said to the child. d. Minimize opportunities for exercise and play. e. Explain and reinforce reality to avoid distortions. ANS: A, B, E The child can be hypothesized to have moderate-to-severe trait (chronic) anxiety. A calm manner calms the child. A simple, structured, predictable environment is less anxiety provoking and reduces overreaction to stimuli. Calm, simple explanations that reinforce reality validate the environment. Repetition is often needed when the child is unable to concentrate because of elevated levels of anxiety. Opportunities for play and exercise should be provided as avenues to reduce anxiety. Physical movement helps channel and lower anxiety. Play also helps by allowing the child to act out concerns. 2. A nurse plans health teaching for a patient with generalized anxiety disorder who takes lorazepam (Ativan). What information should be included? Select all that apply. a. Use caution when operating machinery. b. Allowed tyramine-free foods in diet. c. Understand the importance of caffeine restriction. d. Avoid alcohol and other sedatives. e. Take the medication on an empty stomach. ANS: A, C, D Caffeine is a central nervous system stimulant that acts as an antagonist to the benzodiazepine lorazepam. Daily caffeine intake should be reduced to the amount contained in one cup of coffee. Benzodiazepines are sedatives, thus the importance of exercising caution when driving or using machinery and the importance of not using other central nervous system depressants such as alcohol or sedatives to avoid potentiation. Benzodiazepines do not require a special diet. Food will reduce gastric irritation from the medication. 3. Which assessment questions are most appropriate to ask a patient with possible obsessive-compulsive disorder? Select all that apply. a. "Have you been a victim of a crime or seen someone badly injured or killed?" b. "Are there certain social situations that cause you to feel especially uncomfortable?" c. "Do you have to do things in a certain way to feel comfortable?" d. "Is it difficult to keep certain thoughts out of awareness?" e. "Do you do certain things over and over again?" ANS: C, D, E The correct questions refer to obsessive thinking and compulsive behaviors. The incorrect responses are more pertinent to a patient with suspected posttraumatic stress disorder or with suspected social phobia. Text book questions 1. Friends invite an adult diagnosed with type 2 diabetes to go on a mountain hike next week. The adult replies, “I cant go because I dont have any hiking shoes.” In actuality, this adult fears difficulty with blood glucose management during strenuous activity. Which defense mechanism is evident? Rationalization 2. A nurse analyzes reports from four adult patients of frightening events they encountered. which patients report most clearly indicates that the resulting fear was mentally healthy? C. I was at home when a storm with heavy thunder and lightning lasted over an hour 3. A nursing student arrives late for a clinical experience and is not wearing the correct attire. When the instructor privately criticizes the behavior, the student responds, "I am always the one who gets caught. Youre going to cause me to fail." Select the instructors best response. D. the requirements for this experience were discussed during our orientation 4. Select the best example of altruism D. an individual with a long standing fear of animals volunteers at a community animal shelter 5. An outpatient psychiatric nurse assesses a patient diagnosed with hoarding disorder. The patient has lost 12 pounds in the past two months, appears disheveled, and is wearing dirty clothing with poor hygiene. What is the nurses priority action? D. schedule a home visit to assess the safety of the patients living conditions Chapter 12 psych 1. A medical-surgical nurse works with a patient diagnosed with a somatic system disorder. Care planning is facilitated by understanding that the patient will probably: b. Be resistant to accepting psychiatric help. 2. A patient has blindness related to a functional neurological (conversion) disorder but is unconcerned about this problem. Which understanding should guide the nurse’s planning for this patient? The patient is: b. relieving anxiety through the physical symptom. 3. A patient has blindness related to a functional neurological (conversion) disorder. To help the patient eat, the nurse should: b. expect the patient to feed him- or herself after explaining the arrangement of the food on the tray. 4. A patient with blindness related to a functional neurological (conversion) disorder says, “All the doctors and nurses in this hospital stop by often to check on me. Too bad people outside the hospital don’t find me interesting.” Which nursing diagnosis is most relevant?b. Chronic low self-esteem 5. To assist a patient with a somatic system disorder, a nursing intervention of high priority is to: c. shift the focus from somatic symptoms to feelings. 6. A patient who fears serious heart disease was referred to the mental health center by a cardiologist after extensive diagnostic evaluation showed no physical illness. The patient says, “I have tightness in my chest and my heart misses beats. I’m frequently absent from work. I don’t go out much because I need to rest.” Which health problem is most likely? d. Prominent health anxiety (hypochondriasis) 7. A nurse assessing a patient with a somatic system disorder is most likely to note that the patient: d. Has altered comfort and activity needs 8. To plan effective care for patients with somatic system disorders, the nurse should understand that patients have difficulty giving up the symptoms because the symptoms: d. Provide relief from health anxiety. 9. A patient with a somatic symptom disorder has the nursing diagnosis: Interrupted family processes, related to patient’s disabling symptoms as evidenced by spouse and children assuming roles and tasks that previously belonged to patient. An appropriate outcome is that the patient will: b. Demonstrate a resumption of former roles and tasks. 10. A woman is 5’7” tall, weighs 160 pounds, and wears a size 8 shoe. She says, “My feet are huge. I’ve asked three orthopedists to surgically reduce my feet.” The patient tries to buy shoes to make her feet look smaller, and in social settings conceals both feet under a table or chair. Which health problem is likely? c. Body dysmorphic disorder 11. Which assessment finding best supports dissociative fugue? The patient states: a. “I cannot recall why I’m living in this town.” 12. A college student observes a roommate going out wearing uncharacteristically seductive clothing, returning 12 to 24 hours later and sleeping for 8 to 12 hours. At other times, the roommate sits on the floor speaking like a young child. Which health problem should be considered? b. Dissociative identity disorder 13. A nurse assesses a patient diagnosed with functional neurological (conversion) disorder. Which comment is most likely from this patient? a. “Since my father died, I’ve been short of breath and had sharp pains that go down my left arm, but I think it’s just indigestion.” 14. A nurse counsels a patient diagnosed with body dysmorphic disorder. Which nursing diagnosis would be a priority for the plan of care?b. Risk for suicide 15. Select the correct etiology to complete this nursing diagnosis for a patient with dissociative identity disorder. Disturbed personal identity, related to: d. cognitive distortions associated with unresolved childhood abuse issues. 16. For a patient with dissociative amnesia, complete this outcome: Within 4 weeks, the patient will demonstrate an ability to execute complex mental processes by: d. describing previously forgotten experiences.” 17. A person comes to the clinic reporting, “I wear a scarf across my lower face when I go out but because of my ugly appearance.” Assessment reveals an average appearance with no actual disfigurement. Which problem is most likely? b. Body dysmorphic disorder 18. A nurse counseling a patient with a dissociative identity disorder should understand that the assessment of highest priority is: a. Risk for self-harm 19. A patient says, “I feel detached and weird all the time, like I’m looking at life through a cloudy window. Everything seems unreal. These feelings really interfere with my work and study.” Which term should the nurse use to document this complaint? a. Depersonalization 20. A patient reports fears of having cervical cancer and says to the nurse, “I’ve had Pap smears by six different doctors. The results are normal, but I’m sure that’s because of errors in the laboratory.” Which disorder would the nurse suspect? b. Prominent health anxiety (hypochondriasis) 21. A patient with predominant pain disorder says, “My pain is from an undiagnosed injury. I can’t take care of myself. I need pain medicine six or seven times a day. I feel like a baby because my family has to help me so much.” It is important for the nurse to assess: c. Secondary gains 22. The causes of somatic system disorders may be related to: a. faulty perceptions of body sensations. 23. An essential difference between somatic system disorders and dissociative disorders is: d. Dissociative disorders entail stress-related disruptions of memory, consciousness, or identity, whereas somatic system disorders involve the expression of psychologic stress through somatic symptoms. 24. A patient says, “I know I have a brain tumor despite the results of the magnetic resonance image (MRI). The radiologist is wrong. People who have brain tumors vomit, and yesterday I vomited all day.” Which response by the nurse fosters cognitive restructuring? b. “Let’s see whether any other explanations for your vomiting are possible.” 25. Which treatment modality should a nurse recommend to help a patient with pain disorder cope more effectively? b. Relaxation 26. A patient with depersonalization disorder tells the nurse, “It’s starting again. I feel as though I’m going to float away.” The nurse should help the patient by: d. helping the patient focus on the here and now. 27. A patient with predominant pain disorder has been in treatment for 4 weeks. The patient says, “Although I’m still having pain, I notice it less and am able to perform more activities.” The nurse should evaluate the treatment plan as: c. partially successful. 2. A nurse assesses a patient suspected to have complex somatic system disorder. Which findings support the diagnosis? Select all that apply. a. Patient is a woman. b. Patient reports frequent syncope. c. Patient complains of heavy menstrual bleeding. e. Patient reports back pain, painful urination, frequent diarrhea, and hemorrhoids. 3. A patient’s roommate has observed the patient behaving in uncharacteristic ways, but the patient cannot remember the episodes. A dissociative identity disorder is suspected. Which questions are most relevant to the assessment of this patient? Select all that apply. c. “Have you ever found yourself someplace and did not know how you got there?” d. “Are your memories of childhood clear and complete, or do you have blank spots?” e. “Have you ever found new things in your belongings that you can’t remember buying?” 1. A patient with predominant pain disorder says, “Why has God chosen me to be sick all the time and unable to provide for my family? The burden on my family is worse than the pain I bear.” Which nursing diagnoses apply to this patient? Select all that apply. a. Spiritual distress e. Ineffective role performance Text book questions 1.A patient at a general medical clinic tells the nurse i have so many ailments that i need to see six different doctors. none of them has discovered what is really wrong with me. Which comment should the nurse offer next? Lets review all the medications you currently take. 2. A combat veteran from two tours of the war in Afghanistan tells the nurse, some guys in my unit have posttraumatic stress disorder, but i never had any problems other than my hearing is not as good as it once was. Which explanation for this comment should the nurse consider? The veteran may have amnesia associated with the combat experience. 3. A patient diagnosed with dissociative identity disorder is hospitalized on an acute care psychiatric unit after a suicide attempt. During a team meeting, which staff nurses comment should prompt the nursing supervisor to intervene? I find myself more fascinated and engaged with this patient than others. 4. A nurse in an outpatient medical clinic talks to a patient with a long history of malingering and doctor-shopping. The patient continues to express complaints of multiple problems. Select the nurses best comment to the patient. The treatment team discussed your case and wants to begin a special case management program for you. 5. A patient in the emergency department was seen for the third time in a month with complaints of tremors and paresthesia in the lower extremities. Conversion disorder was diagnosed. While preparing for discharge, the patient says, now im having chest pain but its probably nothing. How should the nurse respond? Interrupt the discharge and arrange additional medical evaluation of the patient. Chapter 13 Personality Disorders 1. A physical therapist recently convicted of multiple counts of Medicare fraud says to a nurse, "Sure I overbilled. Why not? Everyone takes advantage of the government. They have so many rules - no one can follow them." These statements show: a. shame. b. suspiciousness. c. superficial remorse. d. lack of guilt feelings. ANS: D Rationalization is being used to explain behavior and deny wrongdoing. The individual who does not believe he or she has done anything wrong will not exhibit anxiety, remorse, or guilt about the act. The patient's remarks cannot be assessed as shameful. Lack of trust or concern that others are determined to cause harm is not evident. 2. Which intervention is appropriate for a patient with an antisocial personality disorder who frequently manipulates others? a. Refer the patient's requests and questions related to care to the case manager. b. Encourage the patient to discuss his or her feelings of fear and inferiority. c. Provide negative reinforcement for acting-out behavior. d. Ignore, rather than confront, inappropriate behavior. ANS: A Manipulative patients frequently make requests of many different staff members, hoping someone will give in. Having only one decision-maker provides consistency and avoids the potential for playing one staff member against another. Positive reinforcement of appropriate behaviors is more effective than negative reinforcement. The behavior should not be ignored - judicious use of confrontation is necessary. Patients with antisocial personality disorders rarely have feelings of fear and inferiority. 3. As a nurse prepares to administer a medication to a patient with a borderline personality disorder, the patient says, "Just leave it on the table. I'll take it when I finish combing my hair." What is the nurse's best response? a. Reinforce this assertive action by the patient. Leave the medication on the table as requested. b. Respond to the patient, "I'm worried that you might not take it. I'll come back later." c. Say to the patient, "I must watch you take the medication. Please take it now." d. Ask the patient, "Why don't you want to take your medication now?" ANS: C The individual with a borderline personality disorder characteristically demonstrates manipulative, splitting, and self-destructive behaviors. Consistent limit setting is vital for the patient's safety, as well as to prevent splitting other staff members. "Why" questions are not therapeutic. 4. What is an appropriate initial outcome for a patient with a personality disorder who frequently manipulates others? The patient will: a. Identify when feeling angry. b. Use manipulation only to get legitimate needs met. c. Acknowledge manipulative behavior when it is called to his or her attention. d. Accept fulfillment of his or her requests within an hour rather than immediately. ANS: C Acknowledging manipulative behavior is an early outcome that paves the way for taking greater responsibility for controlling manipulative behavior at a later time. Identifying anger relates to anger and aggression control. Using manipulation to get legitimate needs is an inappropriate outcome. The patient will ideally use assertive behavior to promote the fulfillment of legitimate needs. Accepting fulfillment of requests within an hour rather than immediately relates to impulsivity and immediacy control. 5. Consider these comments to three different nurses by a patient with an antisocial personality disorder: "You're a better nurse than the day shift nurse said you were": "Another nurse said you don't do your job right": "You think you're perfect, but I've seen you make three mistakes." Collectively, these interactions can be assessed as: a. Seductive b. Detached c. Manipulative d. Guilt producing ANS: C Patients manipulate and control staff members in various ways. By keeping staff members off balance or fighting among themselves, the person with an antisocial personality disorder is left to operate as he or she pleases. Seductive behavior has sexual connotations. The patient is displaying the opposite of detached behavior. Guilt is not evidenced in the comments. 6. A nurse reports to the interdisciplinary team that a patient with an antisocial personality disorder lies to other patients, verbally abuses a patient with dementia, and flatters the primary nurse. This patient is detached and superficial during counseling sessions. Which behavior most clearly warrants limit setting? a. Flattering the nurse b. Lying to other patients c. Verbal abuse of another patient d. Detached superficiality during counseling ANS: C Limits must be set in areas in which the patient's behavior affects the rights of others. Limiting verbal abuse of another patient is a priority intervention. The other concerns should be addressed during therapeutic encounters. 7. A patient with borderline personality disorder has a history of self-mutilation and suicide attempts. The patient reveals feelings of depression and anger with life. The psychiatrist suggests the use of a medication. Which type of medication should the nurse expect? a. Selective serotonin reuptake inhibitor (SSRI) b. Monoamine oxidase inhibitor (MAOI) c. Benzodiazepine d. Antipsychotic ANS: A SSRIs are used to treat depression. Many patients with borderline personality disorder are fearful of taking something over which they have little control. Because SSRIs have a good side effect profile, the patient is more likely to comply with the medication. Low-dose antipsychotic or anxiolytic medications are not supported by the data given in this scenario. MAOIs require great diligence in adherence to a restricted diet and are rarely used for patients who are impulsive. 8. A patient's spouse filed charges of battery. The patient says, "I'm sorry for what I did. I need psychiatric help." The patient has a long history of acting-out behaviors and several arrests. Which statement by the patient suggests an antisocial personality disorder? a. "I have a quick temper, but I can usually keep it under control." b. "I've done some stupid things in my life, but I've learned a lesson." c. "I'm feeling terrible about the way my behavior has hurt my family." d. "I hit because I'm tired of being nagged. My spouse deserves the beating." ANS: D The patient with an antisocial personality disorder often impulsively acts out feelings of anger and feels no guilt or remorse. Patients with antisocial personality disorders rarely seem to learn from experience or feel true remorse. Problems with anger management and impulse control are common. 9. What is the priority nursing diagnosis for a patient with antisocial personality disorder who has made threats against staff, ripped art off the walls, and thrown objects? a. Disturbed sensory perception-auditory b. Risk for other-directed violence c. Ineffective denial d. Ineffective coping ANS: B Violence against property, along with threats to harm staff, makes this diagnosis the priority. Patients with antisocial personality disorders rarely have psychotic symptoms. When patients with antisocial personality disorders use denial, they use it effectively. Although ineffective coping applies, the risk for violence is a higher priority. 10. When a patient with a personality disorder uses manipulation to get his or her needs met, the staff decides to apply limit-setting interventions. What is the correct rationale for this action? a. It provides an outlet for feelings of anger and frustration. b. It respects the patient's wishes so assertiveness will develop. c. External controls are necessary while internal controls are developed. d. Anxiety is reduced when staff members assume responsibility for the patient's behavior. ANS: C A lack of internal controls leads to manipulative behaviors such as lying, cheating, conning, and flattering. To protect the rights of others, external controls must be consistently maintained until the patient is able to behave appropriately. 11. A patient with borderline personality disorder and a history of self-mutilation has now begun dialectical behavior therapy on an outpatient basis. Counseling focuses on self-harm behavior management. Today the patient telephones to say, "I'm feeling empty and want to cut myself." The nurse should: a. arrange for emergency inpatient hospitalization. b. send the patient to the crisis intervention unit for 8 to 12 hours. c. assist the patient to identify the trigger situation and choose a coping strategy. d. advise the patient to take an antianxiety medication to decrease the anxiety level. ANS: C The patient has responded appropriately to the urge for self-harm by calling a helping individual. A component of dialectical behavior therapy is telephone access to the therapist for "coaching" during crises. The nurse can assist the patient to choose an alternative to self-mutilation. The need for a protective environment may not be necessary if the patient is able to use cognitive strategies to determine a coping strategy that reduces the urge to mutilate. Taking a sedative and going to sleep should not be the first-line intervention, sedation may reduce the patient's ability to weigh alternatives to mutilating behavior. 12. The most challenging nursing intervention with patients with personality disorders who use manipulation to get their needs met is: a. supporting behavioral change. b. monitoring suicide attempts. c. maintaining consistent limits. d. using aversive therapy. ANS: C Maintaining consistent limits is by far the most difficult intervention because of the patient's superior skills at manipulation. Supporting behavioral change and monitoring patient safety are less difficult tasks. Aversive therapy would probably not be part of the care plan. Positive reinforcement strategies for acceptable behavior are more effective than aversive techniques. 13. The history shows that a newly admitted patient has impulsivity. The nurse would expect behavior characterized by: a. adherence to a strict moral code. b. manipulative, controlling strategies. c. postponing gratification to an appropriate time. d. little time elapsed between thought and action. ANS: D The impulsive individual acts in haste without taking time to consider the consequences of the action. None of the other options describes impulsivity. 14. A patient tells a nurse, "I sometimes get into trouble because I make quick decisions and act on them." A therapeutic response would be: a. "Let's consider the advantages of being able to stop and think before acting." b. "It sounds as though you've developed some insight into your situation." c. "I'll bet you have some interesting stories to share about overreacting." d. "It's good that you're showing readiness for behavioral change." ANS: A The patient is showing openness to learning techniques for impulse control. One technique is to teach the patient to stop and think before acting impulsively. The patient can then be taught to evaluate the outcomes of possible actions and choose an effective action. The incorrect responses shift the encounter to a social level or are judgmental. 15. A patient with borderline personality disorder is hospitalized several times after self-mutilating episodes. The patient remains impulsive. Dialectical behavior therapy starts on an outpatient basis. Which nursing diagnosis is the focus of this therapy? a. Risk for self-directed violence b. Impaired skin integrity c. Risk for injury d. Powerlessness ANS: A Risk for self-mutilation is a nursing diagnosis relating to patient safety needs and is therefore a high priority. Impaired skin integrity and powerlessness may be appropriate foci for care but are not the priority or related to this therapy. Risk for injury implies accidental injury, which is not the case for the patient with borderline personality disorder. 16. Which statement made by a patient with borderline personality disorder indicates the treatment plan is effective? a. "I think you are the best nurse on the unit." b. "I'm never going to get high on drugs again." c. "I hate my doctor for not giving me what I ask for." d. "I felt empty and wanted to cut myself, so I called you." ANS: D Seeking a staff member instead of impulsively self-mutilating shows an adaptive coping strategy. The incorrect responses demonstrate idealization, devaluation, and wishful thinking. 17. When preparing to interview a patient with narcissistic personality disorder, a nurse can anticipate the assessment findings will include: a. preoccupation with minute details, perfectionist. b. charm, drama, seductiveness, seeking admiration. c. difficulty being alone, indecisive, submissiveness. d. grandiosity, self-importance, and a sense of entitlement. ANS: D According to the Diagnostic and Statistical Manual of Mental Disorders (fourth edition, text revision) (DSM-IV-TR), the characteristics of grandiosity, self-importance, and a sense of entitlement are consistent with narcissistic personality disorder. Charm, drama, seductiveness, and admiration seeking are observed in patients with histrionic personality disorder. Preoccupation with minute details and perfectionism are observed in individuals with obsessive-compulsive personality disorder. Patients with dependent personality disorder often express difficulty being alone and are indecisive and submissive. 18. For which behavior would limit setting be most essential? The patient: a. clings to the nurse and asks for advice about inconsequential matters. b. is flirtatious and provocative with staff members of the opposite sex. c. is hypervigilant and refuses to attend unit activities. d. urges a suspicious patient to hit anyone who stares. ANS: D This option is an example of a manipulative behavior. Because manipulation violates the rights of others, limit setting is absolutely necessary. Furthermore, limit setting is necessary in this case because the safety of at least two other patients is at risk. Limit setting may be occasionally used with dependent behavior (clinging to the nurse) and histrionic behavior (flirting with staff members), but other therapeutic techniques are also useful. Limit setting is not needed for a patient who is hypervigilant and refuses to attend unit activities. Rather, the need to develop trust is central to patient compliance. 19. A nurse in the emergency department tells an adult, "Your mother had a severe stroke." The adult tearfully says, "Who will take care of me now? My mother always told me what to do, what to wear, and what to eat. I need someone to reassure me when I get anxious." Which term best describes this behavior? a. Histrionic b. Dependent c. Narcissistic d. Borderline ANS: B The main characteristic of the dependent personality is a pervasive need to be taken care of that leads to submissive behaviors and a fear of separation. Histrionic behavior is characterized by flamboyance, attention seeking, and seductiveness. Narcissistic behavior is characterized by grandiosity and exploitive behavior. Patients with borderline personality disorder demonstrate separation anxiety, impulsivity, and splitting. 20. Others describe a worker as very shy and lacking in self-confidence. This worker stays in an office cubicle all day, and never comes out for breaks or lunch. Which term best describes this behavior? a. Avoidant b. Dependent c. Histrionic d. Paranoid ANS: A Patients with avoidant personality disorder are timid, socially uncomfortable, and withdrawn and avoid situations in which they might fail. They believe themselves to be inferior and unappealing. Individuals with dependent personality disorder are clinging, needy, and submissive. Individuals with histrionic personality disorder are seductive, flamboyant, shallow, and attention seeking. Individuals with paranoid personality disorder are suspicious and hostile and project blame. 21. What is the priority intervention for a nurse beginning to work with a patient with a schizotypal personality disorder? a. Respect the patient's need for periods of social isolation. b. Prevent the patient from violating the nurse's rights. c. Engage the patient in many community activities. d. Teach the patient how to match clothing. ANS: A Patients with schizotypal personality disorder are eccentric and often display perceptual and cognitive distortions. They are suspicious of others and have considerable difficulty trusting. They become highly anxious and frightened in social situations, thus the need to respect their desire for social isolation. Teaching the patient to match clothing is not the priority intervention. Patients with schizotypal personality disorder rarely engage in behaviors that violate the nurse's rights or exploit the nurse. 22. A patient with borderline personality disorder self-inflicted wrist lacerations after gaining new privileges on the unit. The cause of the self-mutilation is probably related to: a. inherited disorder that manifests itself as an incapacity to tolerate stress b. use of projective identification and splitting to bring anxiety to manageable levels c. constitutional inability to regulate affect, predisposing to psychic disorganization d. fear of abandonment associated with progress toward autonomy and independence ANS: D Fear of abandonment is a central theme for most patients with borderline personality disorder. This fear is often exacerbated when patients with borderline personality disorder experience success or growth. 23. A patient with borderline personality disorder has self-inflicted wrist lacerations. The health care provider prescribes daily dressing changes. The nurse performing this care should: a. encourage the patient to express anger. b. provide care in a matter-of-fact manner. c. be very kind, sympathetic, and concerned. d. offer to listen to the patient's feelings about cutting. ANS: B A matter-of-fact approach does not provide the patient with positive reinforcement for self-mutilation. The goal of providing emotional consistency is supported by this 24. A nurse set limits for a patient with a borderline personality disorder. The patient tells the nurse, "You used to care about me. I thought you were wonderful. Now I can see I was mistaken. You're terrible." This outburst can be assessed as: a. denial. b. splitting. c. reaction formation. d. separation-individuation strategies. ANS: B Splitting involves loving a person and then hating the person. The patient is unable to recognize that an individual can have both positive and negative qualities. Denial is an unconscious motivated refusal to believe something. Reaction formation involves unconsciously doing the opposite of a forbidden impulse. Separation-individuation strategies refer to childhood behaviors related to developing independence from the caregiver. 25. Which characteristic of individuals with personality disorders makes it most necessary for staff to schedule frequent meetings? a. Ability to achieve true intimacy b. Flexibility and adaptability to stress c. Ability to evoke interpersonal conflict d. Inability to develop trusting relationships ANS: C Frequent team meetings are held to counteract the effects of the patient's attempts to split staff and set them against one another, causing interpersonal conflict. Patients with personality disorders are inflexible and demonstrate maladaptive responses to stress. They are usually unable to develop true intimacy with others and are unable to develop trusting relationships. Although problems with trust may exist, it is not the characteristic that requires frequent staff meetings. 26. Which commonality would be most applicable to the patient with a personality disorder? The patient: a. demonstrates behaviors that cause distress to self rather than to others. b. has self-esteem issues, despite his or her outward presentation. c. usually becomes psychotic when exposed to stress. d. does not experience real distress from symptoms. ANS: B Self-esteem issues are present, despite patterns of withdrawal, grandiosity, suspiciousness, or unconcern. They seem to relate to early life experiences and are reinforced through unsuccessful experiences in loving and working. Personality disorders involve lifelong, inflexible, dysfunctional, and deviant patterns of behavior that cause distress to others and, in some cases, to self. Patients with personality disorders may experience very real anxiety and distress when stress levels rise. Some individuals with personality disorders, but not all, may decompensate and show psychotic behaviors under stress. 1. A nurse plans the care for an individual diagnosed with antisocial personality disorder. Which characteristic behaviors will the nurse expect? Select all that apply. a. Reclusive behavior b. Callous attitude c. Perfectionism d. Aggression e. Clinginess f. Anxiety ANS: B, D Individuals with antisocial personality disorders characteristically demonstrate manipulative, exploitative, aggressive, callous, and guilt-instilling behaviors. Individuals with antisocial personality disorders are more extroverted than reclusive, rarely show anxiety, and rarely demonstrate clinging or dependent behaviors. Individuals with antisocial personality disorders are more likely to be impulsive than to be perfectionists. 2. For which patients with personality disorders would a family history of similar problems be most likely? Select all that apply. a. Obsessive-compulsive b. Antisocial c. Dependent d. Schizotypal e. Narcissistic ANS: A, B, D Some personality disorders have evidence of genetic links. Therefore the family history would show other family members with similar traits. Heredity plays a role in schizotypal and antisocial problems, as well as obsessive-compulsive personality disorder. Textbook Questions 1. A person shoplifts merchandise from a community cancer thrift shop. When confronted, the thief replies “All this stuff was donated, so I can take it.” This comment suggest features of which personality disorder? -A. Antisocial 2. After a power outage, a facility must serve a dinner of sandwiches and fruit to patients. Which comments is most likely from a patient diagnosed with a narcissistic personality disorder? - C. “You should have ordered a to-go meal from these circumstances. 3. A nurse plans care for a patient diagnosed with borderline personality disorder. Which nursing diagnosis is most likely to apply to this patient? -”Ineffective relationship related to frequent splitting.” 4. The nurse assesses a new patient suspected of recurrent suspected of having a schizotypal personality disorder. Which assessment question is this patient most likely to answer affirmatively? - “Is anyone in your family diagnosed with a mental illness?” 5. A mental health nurse assesses a patient diagnosed with an antisocial personality disorder. Which comorbid problem is most important for the nurse to include in the assessment? -Alcohol use and abuse. Chapter 15 Depression 1. A patient became severely depressed when the last of six children moved out of the home 4 months ago. The patient repeatedly says, "No one cares about me. I'm not worth anything." Which response by the nurse would be the most helpful? a. "Things will look brighter soon. Everyone feels down once in a while." b. "The staff here cares about you and wants to try to help you get better." c. "It is difficult for others to care about you when you repeatedly say the same negative things." d. "I'll sit with you for 10 minutes now and return for 10 minutes at lunchtime and again at 2:30 this afternoon." ANS: D Spending time with the patient at intervals throughout the day shows acceptance by the nurse and helps the patient establish a relationship with the nurse. The therapeutic technique is called offering self. Setting definite times for the therapeutic contacts and keeping the appointments show predictability on the part of the nurse, an element that fosters the building of trust. The incorrect responses would be difficult for a person with profound depression to believe, provide trite reassurance, and are counterproductive. The patient is unable to say positive things at this point. 2. A patient became depressed after the last of six children moved out of the home 4 months ago. The patient has been self-neglectful, slept poorly, lost weight, and repeatedly says, "No one cares about me anymore. I'm not worth anything." Select an appropriate initial outcome for the nursing diagnosis Situational low self-esteem, related to feelings of abandonment. The patient will: a. verbalize realistic positive characteristics about self by (date) . b. consent to take antidepressant medication regularly by (date) . c. initiate social interaction with another person daily by (date) . d. identify two personal behaviors that alienate others by (date) . ANS: A Low self-esteem is reflected by making consistently negative statements about self and self-worth. Replacing negative cognitions with more realistic appraisals of self is an appropriate intermediate outcome. The incorrect options are not as clearly related to the nursing diagnosis. Outcomes are best when framed positively; identifying two personal behaviors that might alienate others is a negative concept. 3. A patient with major depression says, "No one cares about me anymore. I'm not worth anything." The nurse wants to reinforce positive self-esteem. Today, the patient is wearing a new shirt and has neat, clean hair. Which remark is most appropriate? a. "You look nice this morning." b. "You're wearing a new shirt." c. "I like the shirt you're wearing." d. "You must be feeling better today." ANS: B Patients with depression usually see the negative side of things. The meaning of compliments may be altered to "I didn't look nice yesterday" or "They didn't like my other shirt." Neutral comments such as an observation avoid negative interpretations. Saying "You look nice" or "I like your shirt" gives approval (nontherapeutic techniques). Saying "You must be feeling better today" is an assumption, which is nontherapeutic. 4. An adult with depression was treated with medication and cognitive behavioral therapy. The patient now recognizes how passivity contributed to the depression. Which intervention should the nurse suggest? a. Social skills training b. Relaxation training classes c. Use of complementary therapy d. Learning desensitization techniques ANS: A Social skills training is helpful in treating and preventing the recurrence of depression. Training focuses on assertiveness and coping skills that lead to positive reinforcement from others and the development of a patient's support system. The use of complementary therapy refers to adjunctive therapies such as herbals. Assertiveness would be of greater value than relaxation training because passivity is a concern. Desensitization is used in the treatment of phobias. 5. A priority intervention for a patient with major depression is: a. distracting the patient from self-absorption. b. carefully and unobtrusively observing the patient around the clock. c. allowing the patient to spend long periods alone in meditation. d. offering opportunities for the patient to assume a leadership role in the therapeutic milieu. ANS: B Approximately two thirds of people with depression contemplate suicide. Patients with depression who exhibit feelings of worthlessness are at higher risk. Regularly planned observations of the patient with depression may prevent a suicide attempt on the unit. 6. When counseling patients with major depression, an advanced practice nurse will address the negative thought patterns by using: a. psychoanalytic therapy. b. desensitization therapy. c. cognitive behavioral therapy. d. alternative and complementary therapies. ANS: C Cognitive behavioral therapy attempts to alter the patient's dysfunctional beliefs by focusing on positive outcomes rather than negative attributions. The patient is also taught the connection between thoughts and resultant feelings. Research shows that cognitive behavioral therapy involves the formation of new connections among nerve cells in the brain and that it is at least as effective as medication. Evidence does not support superior outcomes for the other psychotherapeutic modalities mentioned. 7. A patient says to the nurse, "My life doesn't have any happiness in it anymore. I once enjoyed holidays, but now they're just another day." How would the nurse document the complaint? a. Dysthymia b. Anhedonia c. Euphoria d. Anergia ANS: B Anhedonia is a common finding in many types of depression and refers to feelings of a loss of pleasure in formerly pleasurable activities. Dysthymia is a diagnosis. Euphoria refers to an elated mood. Anergia means without energy. 8. A patient with depression is taking a tricyclic antidepressant. The patient says, "I don't think I can keep taking these pills. They make me so dizzy, especially when I stand up." The nurse should: a. explain how to manage hypotension, and educate the patient that side effects go away after several weeks. b. tell the patient that the side effects are a minor inconvenience compared with the feelings of depression. c. withhold the drug, force oral fluids, and notify the health care provider to examine the patient. d. update the patient's mental status examination. ANS: A Drowsiness, dizziness, and postural hypotension usually subside after the first few weeks of therapy with tricyclic antidepressants. Postural hypotension can be managed by teaching the patient to stay well hydrated and rise slowly. Knowing these facts may be enough to convince the patient to remain medication compliant. The minor inconvenience of side effects as compared with feelings of depression is a convincing reason to remain on the medication. Withholding the drug, forcing oral fluids, and having the health care provider examine the patient are unnecessary steps. Independent nursing action is appropriate. Updating a mental status examination is unnecessary. 9. A patient with depression is receiving imipramine (Tofranil) 200 mg every night at bedtime. Which assessment finding would prompt the nurse to collaborate with the health care provider regarding potentially hazardous side effects of this drug? a. Dry mouth b. Blurred vision c. Nasal congestion d. Urinary retention ANS: D All the side effects mentioned are the result of the anticholinergic effects of the drug. Only urinary retention and severe constipation warrant immediate medical attention. Dry mouth, blurred vision, and nasal congestion may be less troublesome as therapy continues. 10. A patient with depression tells the nurse, "Bad things that happen are always my fault." To assist the patient in reframing this overgeneralization, the nurse should respond: a. "I really doubt that one person can be blamed for all the bad things that happen." b. "Let's look at one bad thing that happened to see if another explanation exists." c. "You are being exceptionally hard on yourself when you imply you are a jinx." d. "What about the good things that happen; are any of them ever your fault?" ANS: B By questioning a faulty assumption, the nurse can help the patient look at the premise more objectively and reframe it as a more accurate representation of fact. The incorrect responses cast doubt but do not require the patient to evaluate the statement. 11. A nurse worked with a patient with major depression who displayed severely withdrawn behavior and psychomotor retardation. After 3 weeks, the patient did not improve. The nurse is at risk for feelings of: a. overinvolvement. b. guilt and despair. c. interest and pleasure. d. ineffectiveness and frustration. ANS: D Nurses may have expectations for self and patients that are not wholly realistic, especially regarding the patient's progress toward health. Unmet expectations result in feelings of ineffectiveness, anger, or frustration. Nurses rarely become overinvolved with patients with depression because of the patient's resistance. Guilt and despair might be observed when the nurse experiences patient feelings because of empathy. Interest is possible but not the most likely result. 12. A patient with depression begins selective serotonin reuptake inhibitor (SSRI) antidepressant therapy, and priority information is given to the patient and family. This information should include a directive to: a. avoid exposure to bright sunlight. b. report increased suicidal thoughts. c. restrict sodium intake to 1 g daily. d. maintain a tyramine-free diet. ANS: B Some evidence indicates that suicidal ideation may worsen at the beginning of antidepressant therapy; thus close monitoring is necessary. Avoiding exposure to bright sunlight and restricting sodium intake are unnecessary. Tyramine restriction is associated with monoamine oxidase inhibitor (MAOI) therapy. 13. A nurse teaching a patient about a tyramine-restricted diet would approve which meal? a. Mashed potatoes, ground beef patty, corn, green beans, apple pie b. Avocado salad, ham, creamed potatoes, asparagus, chocolate cake c. Macaroni and cheese, hot dogs, banana bread, caffeinated coffee d. Noodles with cheddar cheese sauce, smoked sausage, lettuce salad, yeast rolls ANS: A The correct answer describes a meal that contains little tyramine. Vegetables and fruits contain little or no tyramine, and fresh ground beef and apple pie should be safe. The other meals contain various amounts of tyramine-rich foods or foods that contain vasopressors: avocados, ripe bananas (banana bread), sausages and hot dogs, smoked meat (ham), cheddar cheese, yeast, caffeine drinks, and chocolate. 14. What is the focus of priority nursing interventions for the period immediately after electroconvulsive therapy treatment? a. Supporting physiologic stability b. Reducing disorientation and confusion c. Establishing random eye movement latency d. Assisting the patient to identify and test negative thoughts ANS: A During the immediate posttreatment period, the patient is recovering from general anesthesia, hence the need to establish and support physiologic stability. Establishing random eye movement latency is neither possible nor a priority. Reducing disorientation and confusion is an acceptable intervention but not the priority. Assisting the patient in identifying and testing negative thoughts is inappropriate in the immediate posttreatment period because the patient may be confused. 15. A nurse provided medication education for a patient who takes phenelzine (Nardil) for depression. Which behavior indicates effective learning? The patient: a. Monitors sodium intake and weight daily. b. Wears support stockings and elevates the legs when sitting. c. Consults the pharmacist when selecting over-the-counter medications. d. Can identify foods with high selenium content, which should be avoided. ANS: C Over-the-counter medicines may contain vasopressor agents or tyramine, a substance that must be avoided when the patient takes MAOI antidepressants. Medications for colds, allergies, or congestion or any preparation that contains ephedrine or phenylpropanolamine may precipitate a hypertensive crisis. MAOI antidepressant therapy is unrelated to the need for sodium limitation, support stockings, or leg elevation. MAOIs interact with tyramine-containing foods, not selenium, to produce dangerously high blood pressure. 16. A patient's employment is terminated and major depression results. The patient says to the nurse, "I'm not worth the time you spend with me. I'm the most useless person in the world." Which nursing diagnosis applies? a. Powerlessness b. Defensive coping c. Situational low self-esteem d. Disturbed personal identity ANS: C The patient's statements express feelings of worthlessness and most clearly relate to the nursing diagnosis of Situational low self-esteem. Insufficient information exists to lead to other diagnoses. 17. A patient with depression does not interact with others except when addressed and then only in monosyllables. The nurse wants to show nonjudgmental acceptance and support for the patient. Select the nurse's most effective action. a. Make observations. b. Ask the patient direct questions. c. Phrase questions to require "yes" or "no" answers. d. Frequently reassure the patient to reduce guilt feelings. ANS: A Making observations about neutral topics such as the environment draws the patient into the reality around him or her but places no burdensome expectations on the patient for answers. Acceptance and support are shown by the nurse's presence. Direct questions may make the patient feel that the encounter is an interrogation. Open-ended questions are preferable if the patient is able to participate in dialog. Platitudes are never acceptable; they minimize patient feelings and can increase feelings of worthlessness. 18. A patient being treated for depression has taken 300 mg amitriptyline (Elavil) daily for a year. The patient calls the case manager at the clinic and says, "I stopped taking my antidepressant 2 days ago. Now I am having cold sweats, nausea, a rapid heartbeat, and nightmares." The nurse should advise the patient: a. "Go to the nearest emergency department immediately." b. "Do not to be alarmed. Take two aspirin and drink plenty of fluids." c. "Take one dose of the antidepressant. Come to the clinic to see the health care provider." d. "Resume taking the antidepressant for 2 more weeks, and then discontinue them again." ANS: C The patient has symptoms associated with abrupt withdrawal of the tricyclic antidepressant. Taking a dose of the drug will ameliorate the symptoms. Seeing the health care provider will allow the patient to discuss the advisability of going off the medication and to be given a gradual withdrawal schedule if discontinuation is the decision. This situation is not a medical emergency, although it calls for medical advice. Resuming taking the antidepressant for 2 more weeks and then discontinuing again would produce the same symptoms the patient is experiencing. 19. Which documentation indicates the treatment plan of a patient with major depression was effective? a. Slept 6 hours uninterrupted. Sang with activity group. Anticipates seeing grandchild. b. Slept 10 hours uninterrupted. Attended craft group; stated "project was a failure, just like me." c. Slept 5 hours with brief interruptions. Personal hygiene adequate with assistance. Weight loss of 1 pound. d. Slept 7 hours uninterrupted. Preoccupied with perceived inadequacies. States, "I feel tired all the time." ANS: A Sleeping 6 hours, participating in a group activity, and anticipating an event are all positive happenings. All the other options show at least one negative finding. 20. A woman gave birth to a healthy newborn 1 month ago. The patient now reports she cannot cope and is unable to sleep or eat. She says, "I feel like a failure. I can't take care of my baby. This baby is the root of my problems." The priority nursing diagnosis is: a. Insomnia b. Ineffective coping c. Situational low self-esteem d. Risk for other-directed violence ANS: D When the mother with depression of postpartum onset has ruminations or delusional thoughts about the infant, the risk for harming the infant is increased; thus it becomes the priority diagnosis. The other diagnoses are relevant but are of lower priority. 21. A patient with depression repeatedly tells staff members, "I have cancer. It's my punishment for being a bad person." Diagnostic tests reveal no cancer. Select the priority nursing diagnosis. a. Powerlessness b. Risk for suicide c. Stress overload d. Spiritual distress ANS: B A patient with depression who feels so worthless as to believe cancer is deserved is at risk for suicide. Safety concerns take priority over the other diagnoses listed. 22. Which beverage should the nurse offer to a patient with depression who refuses solid food? a. Tomato juice b. Orange juice c. Hot tea d. Milk ANS: D Milk is the only beverage listed that provides protein, fat, and carbohydrates. In addition, milk is fortified with vitamins. 23. During a psychiatric assessment, the nurse observes a patient's facial expressions that are without emotion. The patient says, "Life feels so hopeless to me. I've been feeling sad for several months." How should the nurse document the patient's affect and mood? a. Affect depressed; mood flat b. Affect flat; mood depressed c. Affect labile; mood euphoric d. Affect and mood are incongruent ANS: B Mood is a person's self-reported emotional feeling state. Affect is the emotional feeling state that is outwardly observable by others. 24. A disheveled patient with severe depression and psychomotor retardation has not showered for several days. The nurse should: a. avoid forcing the issue. b. bring up the issue at the community meeting. c. calmly tell the patient, "You must bathe daily." d. firmly and neutrally assist the patient with showering. ANS: D When patients are unable to perform self-care activities, staff members must assist them rather than ignore the issue. Better grooming increases self-esteem. Calmly telling the patient to bathe daily and bringing up the issue at a community meeting are punitive. 25. A patient was started on escitalopram (Lexapro) 5 days ago and now says, "This medicine isn't working." The nurse's best intervention would be to: a. discuss with the health care provider the need to change medications. b. reassure the patient that the medication will be effective soon. c. explain the time lag before antidepressants relieve symptoms. d. critically assess the patient for symptom relief. ANS: C Escitalopram is an SSRI antidepressant. Between 1 and 3 weeks of treatment are usually necessary before a relief of symptoms occurs. This information is important to share with patients. 26. A nurse is caring for a patient with low self-esteem. Which nonverbal communication should the nurse anticipate? a. Arms crossed b. Staring at the nurse c. Smiling inappropriately d. Eyes pointed downward ANS: D Nonverbal communication is usually considered more powerful than verbal communication. Downward-casted eyes suggest feelings of worthlessness or hopelessness. 27. A patient with major depression was hospitalized for 8 days. Treatment included six electroconvulsive therapy sessions and aggressive dose adjustments of antidepressant medications. The patient owns a small business and was counseled not to make major decisions for a month. Select the correct rationale for this counseling. a. Temporary memory impairments and confusion are associated with electroconvulsive therapy. b. Antidepressant medications alter catecholamine levels, which impair decision-making abilities. c. Antidepressant medications may cause confusion related to a limitation of tyramine in the diet. d. The patient needs time to reorient him- or herself to a pressured work schedule. ANS: A Recent memory impairment or confusion or both are often present during and for a short time after electroconvulsive therapy. An inappropriate business decision might be made because of forgotten and important details. The incorrect responses contain rationales that are untrue. The patient needing time to reorient him- or herself to a pressured work schedule is less relevant than the correct rationale. 28. A nurse instructs a patient taking a drug that inhibits the action of a monoamine oxidase (MAO) to avoid certain foods and drugs because of the risk of: a. hypotensive shock. b. hypertensive crisis. c. cardiac dysrhythmia. d. cardiogenic shock. ANS: B Patients taking MAOI-inhibiting drugs must be on a tyramine-free diet to prevent hypertensive crisis. In the presence of MAOIs, tyramine is not destroyed by the liver and, in high levels, produces intense vasoconstriction, resulting in elevated blood pressure. 1. The admission note indicates a patient with depression has anergia and anhedonia. For which measures should the nurse plan? Select all that apply. a. Channeling excessive energy b. Reducing guilty ruminations c. Instilling a sense of hopefulness d. Assisting with self-care activities e. Accommodating psychomotor retardation ANS: C, D, E Anhedonia refers to the inability to find pleasure or meaning in life; thus planning should include measures to accommodate psychomotor retardation, assist with activities of daily living, and instill hopefulness. Anergia is the lack of energy, not excessive energy. Anhedonia does not necessarily imply the presence of guilty ruminations. 2. A student nurse caring for a patient with depression reads in the patient's medical record, "This patient shows vegetative signs of depression." Which nursing diagnoses most clearly relate to the vegetative signs? Select all that apply. a. Imbalanced nutrition: less than body requirements b. Chronic low self-esteem c. Sexual dysfunction d. Self-care deficit e. Powerlessness f. Insomnia ANS: A, C, D, F Vegetative signs of depression are alterations in the body processes necessary to support life and growth, such as eating, sleeping, elimination, and sexual activity. These diagnoses are more closely related to vegetative signs than to diagnoses associated with feelings about self. 3. A patient with major depression will begin electroconvulsive therapy tomorrow. Which interventions are routinely implemented before the treatment? Select all that apply. a. Administer pretreatment medication 30 to 45 minutes before treatment. b. Withhold food and fluids for a minimum of 6 hours before treatment. c. Remove dentures, glasses, contact lenses, and hearing aids. d. Restrain the patient in bed with padded limb restraints. e. Assist the patient to prepare an advance directive. ANS: A, B, C The correct interventions reflect routine electroconvulsive therapy preparation, which is similar to preoperative preparation: sedation and anticholinergic medication before anesthesia, maintaining nothing-by-mouth status to prevent aspiration during and after treatment, airway maintenance, and general safety by removing prosthetic devices. Restraint is not part of the pretreatment protocol. An advance directive is prepared independent of this treatment. 4. A patient with major depression shows vegetative signs of depression. Which nursing actions should be implemented? Select all that apply. a. Offer laxatives, if needed. b. Monitor food and fluid intake. c. Provide a quiet sleep environment. d. Eliminate all daily caffeine intake. e. Restrict the intake of processed foods. ANS: A, B, C The correct options promote a normal elimination pattern. Although excessive intake of stimulants such as caffeine may make the patient feel jittery and anxious, small amounts may provide useful stimulation. No indication exists that processed foods should be restricted. 5. A patient being treated with paroxetine (Paxil) 50 mg/day orally for depression reports to the clinic nurse, "I took a few extra tablets earlier in the day and now I feel bad." Which assessments are most critical? Select all that apply. a. Vital signs b. Urinary frequency c. Increased suicidal ideation d. Presence of abdominal pain and diarrhea e. Hyperactivity or feelings of restlessness ANS: A, D, E The patient is taking the maximum dose of this SSRI and has ingested an additional unknown amount of the drug. Central serotonin syndrome must be considered. Symptoms include abdominal pain, diarrhea, tachycardia, elevated blood pressure, hyperpyrexia, increased motor activity, and muscle spasms. Central serotonin syndrome may progress to a full medical emergency if not treated early. Although assessing for suicidal ideation is never inappropriate, in this situation physiologic symptoms should be the initial focus. The patient may have urinary retention, but frequency would not be expected. 1. Which changes in brain biochemical function is most associated with suicidal behavior? a. Dopamine excess b. Serotonin deficiency c. Acetylcholine excess d. Gamma-aminobutyric acid deficiency ANS: B Research suggests that low levels of serotonin may play a role in the decision to commit suicide. The other neurotransmitter alterations have not been implicated in suicidal crises. 2. A college student failed two tests. Afterward, the student cried for hours and then tried to telephone a parent but got no answer. The student then gave several expensive sweaters to a roommate. Which behavior provides the strongest clue of an impending suicide attempt? a. Calling parents b. Excessive crying c. Giving away sweaters d. Staying alone in dorm room ANS: C Giving away prized possessions may signal that the individual thinks he or she will have no further need for the items, such as when a suicide plan has been formulated. Calling parents and crying do not provide clues to suicide, in and of themselves. Remaining in the dormitory would be an expected behavior because the patient has nowhere else to go. 3. A nurse uses the SAD PERSONS scale to interview a patient. This tool provides data relevant to: a. current stress level. b. mood disturbance. c. suicide potential. d. level of anxiety. ANS: C The SAD PERSONS tool evaluates 10 major risk factors in suicide potential: sex, age, depression, previous attempt, ethanol use, rational thinking loss, social supports lacking, organized plan, no spouse, and sickness. The tool does not have appropriate categories to provide information on the other options listed. 4. A person intentionally overdoses on antidepressant drugs. Which nursing diagnosis has the highest priority? a. Powerlessness b. Social isolation c. Risk for suicide d. Compromised family coping ANS: C This diagnosis is the only one with life-or-death ramifications and is therefore higher in priority than the other options. 5. A person attempts suicide by overdose, is treated in the emergency department, and is then hospitalized. What is the best initial outcome? The patient will: a. verbalize a will to live by the end of the second hospital day. b. describe two new coping mechanisms by the end of the third hospital day. c. accurately delineate personal strengths by the end of first week of hospitalization. d. exercise suicide self-restraint by refraining from gestures or attempts to kill self for 24 hours. ANS: D Suicide self-restraint relates most directly to the priority problem of risk for self-directed violence. The other outcomes are related to hope, coping, and self-esteem. 6. A college student who attempts suicide by overdose is hospitalized. When the parents are contacted they respond, "There must be a mistake. This could not have happened. We've given our child everything." The parents' reaction reflects: a. denial. b. anger. c. anxiety. d. rescue feelings. ANS: A The parents' statements indicate denial. Denial or minimization of suicidal ideation or attempts is a defense against uncomfortable feelings. Family members are often unable to acknowledge suicidal ideation in someone close to them. The feelings suggested in the distracters are not clearly described in the scenario. 7. An adolescent tells the school nurse, "My friend threatened to take an overdose of pills." The nurse talks to the friend who verbalized the suicide threat. The most critical question for the nurse to ask would be: a. "Why do you want to kill yourself?" b. "Do you have access to medications?" c. "Have you been taking drugs and alcohol?" d. "Did something happen with your parents?" ANS: B The nurse must assess the patient's access to the means to carry out the plan and, if there is access, alert the parents to remove them from the home. The other questions may be important to ask but are not the most critical. 8. An adult attempts suicide after declaring bankruptcy. The patient is hospitalized and takes an antidepressant medication for 5 days. The patient is now more talkative and shows increased energy on the unit. Select the highest priority nursing intervention. a. Supervise the patient 24 hours a day. b. Begin discharge planning for the patient. c. Refer the patient to art and music therapists. d. Consider the discontinuation of suicide precautions. ANS: A The patient now has more energy and may have decided on suicide, especially considering the history of the prior suicide attempt. The patient must be supervised 24 hours per day; the patient is still a suicide risk. 9. A nurse and patient construct a no-suicide contract. Select the preferable wording. a. "I will not try to harm myself during the next 24 hours." b. "I will not make a suicide attempt while I am hospitalized." c. "For the next 24 hours, I will not kill or harm myself in any way." d. "I will not kill myself until I call my primary nurse or a member of the staff." ANS: C The correct answer leaves no loopholes. The wording about not harming oneself and not making an attempt leaves loopholes or can be ignored by the patient who thinks, "I am not going to harm myself, I am going to kill myself," or "I am not going to attempt suicide, I am going to commit suicide." A patient may call a therapist and leave the telephone to carry out the suicidal plan. 10. A tearful, anxious patient at the outpatient clinic reports, "I should be dead." The initial task of the nurse conducting the assessment interview is to: a. assess the lethality of a suicide plan. b. encourage expression of anger. c. establish a rapport with the patient. d. determine risk factors for suicide. ANS: C Establishing rapport will allow the nurse to obtain relevant assessment data such as the presence of a suicide plan, the lethality of a suicide plan, and the presence of risk factors for suicide. 11. Select the most helpful response for a nurse to make when a patient being treated as an outpatient states, "I am considering committing suicide." a. "I'm glad you shared this. Please do not worry. We will handle it together." b. "I think you should admit yourself to the hospital to get help." c. "We need to talk about the good things you have to live for." d. "Bringing this up is a very positive action on your part." ANS: D This response gives the patient reinforcement and validation for making a positive response rather than acting out the suicidal impulse. It gives neither advice nor false reassurance, and it does not imply stereotypes such as, "You have a lot to live for." It uses the patient's ambivalence and sets the stage for more realistic problem-solving strategies. 12. Which intervention should a nurse recommend for the distressed family and friends of someone who has committed suicide? a. Participating in reminiscence therapy b. Completing a psychologic postmortem assessment c. Attending a self-help group for survivors d. Contracting for two sessions of group therapy ANS: C Survivors need outlets for their feelings about the loss and the deceased person. Self-help groups provide peer support while survivors work through feelings of loss, anger, and guilt. Psychologic postmortem assessment would not provide the support necessary to work through feelings of loss associated with the suicide of a family member. Reminiscence therapy is not geared to loss resolution. Contracting for two sessions of group therapy would probably not provide sufficient time to work through the issues associated with a death by suicide. 13. Which statement provides the best rationale for why a nurse should closely monitor a severely depressed patient during antidepressant medication therapy? a. As depression lifts, physical energy becomes available to carry out suicide. b. Suicide may be precipitated by a variety of internal and external events. c. Suicidal patients have difficulty using social supports. d. Suicide is an impulsive act. ANS: A Antidepressant medication has the objective of relieving depression. The risk for suicide is greater as the depression lifts, primarily because the patient has more physical energy at a time when he or she may still have suicidal ideation. The other options have little to do with nursing interventions relating to antidepressant medication therapy. 14. A nurse assesses a patient who reports a 3-week history of depression and crying spells. The patient says, "My business is bankrupt, and I was served with divorce papers." Which subsequent statement by the patient alerts the nurse to a concealed suicidal message? a. "I wish I were dead." b. "Life is not worth living." c. "I have a plan that will fix everything." d. "My family will be better off without me." ANS: C Verbal clues to suicide may be overt or covert. The incorrect options are overt references to suicide. The correct option is more veiled. It alludes to the patient's suicide as being a way to "fix everything" but does not say it outright. 15. A depressed patient says, "Nothing matters anymore." What is the most appropriate response by the nurse? a. "Are you having thoughts of suicide?" b. "I am not sure I understand what you are trying to say." c. "Try to stay hopeful. Things have a way of working out." d. "Tell me more about what interested you before you began feeling depressed." ANS: A The nurse must make overt what is covert; that is, the possibility of suicide must be openly addressed. Often, patients feel relieved to be able to talk about suicidal ideation. 16. A nurse counsels a patient with recent suicidal ideation. Which is the nurse's most therapeutic comment? a. "Let's make a list of all your problems and think of solutions for each one." b. "I'm happy you're taking control of your problems and trying to find solutions." c. "When you have bad feelings, try to focus on positive experiences from your life." d. "Let's consider which problems are most important and which are less important." ANS: D The nurse helps the patient develop effective coping skills. He or she assists the patient to reduce the overwhelming effects of problems by prioritizing them. The incorrect options continue to present overwhelming approaches to problem solving. 17. When assessing a patient's plan for suicide, what aspect has priority? a. Patient's financial and educational status b. Patient's insight into suicidal motivation c. Availability of means and lethality of method d. Quality and availability of patient's social support ANS: C If a person has definite plans that include choosing a method of suicide readily available, and if the method is one that is lethal (i.e., will cause the person to die with little probability for intervention), the suicide risk is considered high. These areas provide a better indication of risk than the areas mentioned in the other options. 18. Which understanding about individuals who attempt suicide will help a nurse plan the care for a suicidal patient? Every suicidal person should be considered: a. mentally ill. b. intent on dying. c. cognitively impaired. d. experiencing hopelessness. ANS: D Hopelessness is the characteristic common among people who attempt suicide. The incorrect options reflect myths about suicide. Not all who attempt suicide are intent on dying. Not all are mentally ill or cognitively impaired. 19. Which statement by a patient during an assessment interview should alert the nurse to the patient's need for immediate, active intervention? a. "I am mixed up, but I know I need help." b. "I have no one to turn to for help or support." c. "It is worse when you are a person of color." d. "I tried to get attention before I shot myself." ANS: B Lack of social support and social isolation increase the suicide risk. The willingness to seek help lowers the risk. Being a person of color does not suggest a higher risk; more whites commit suicide than do individuals of other racial groups. Attention seeking is not correlated with a higher risk of suicide. 20. The feeling experienced by a patient that should be assessed by the nurse as most predictive of elevated suicide risk is: a. hopelessness. b. sadness. c. elation. d. anger. ANS: A Of the feelings listed, hopelessness is most closely associated with increased suicide risk. Depression, aggression, impulsivity, and shame are other feelings noted as risk factors for suicide. 21. Four individuals have given information about their suicide plans. Which plan evidences the highest suicide risk? a. Jumping from a 100-foot-high railroad bridge located in a deserted area late at night b. Turning on the oven and letting gas escape into the apartment during the night c. Cutting the wrists in the bathroom while the spouse reads in the next room d. Overdosing on aspirin with codeine while the spouse is out with friends ANS: A This is a highly lethal method with little opportunity for rescue. The other options are lower lethality methods with higher rescue potential. 22. Which individual in the emergency department should be considered at the highest risk for completing suicide? a. An adolescent Asian-American girl with superior athletic and academic skills who has asthma b. A 38-year-old single African-American female church member with fibrocystic breast disease c. A 60-year-old married Hispanic man with 12 grandchildren who has type 2 diabetes d. A 79-year-old single white man with cancer of the prostate gland ANS: D High-risk factors include being an older adult, single, and male and having a co-occurring medical illness. Cancer is one of the somatic conditions associated with increased suicide risk. Protective factors for African-American women and Hispanic individuals include strong religious and family ties. Asian Americans have a suicide rate that increases with age. 23. A nurse answers a suicide crisis line. A caller says, "I live alone in a home several miles from my nearest neighbors. I have been considering suicide for 2 months. I have had several drinks and now my gun is loaded. I'm going to shoot myself in the heart." How would the nurse assess the lethality of this plan? a. No risk b. Low level c. Moderate level d. High level ANS: D The patient has a highly detailed plan, a highly lethal method, the means to carry it out, lowered impulse control because of alcohol ingestion, and a low potential for rescue. 24. A staff nurse tells another nurse "I evaluated a new patient using the SAD PERSONS scale and got a score of 10. I'm wondering if I should send the patient home." Select the best reply by the second nurse. a. "That action would seem appropriate." b. "A score over 8 requires immediate hospitalization." c. "I think you should strongly consider hospitalization for this patient." d. "Give the patient a follow-up appointment. Hospitalization may be needed soon." ANS: B A SAD PERSONS scale score of 0 to 5 suggests home care with follow-up. A score of 6 to 8 requires psychiatric consultation. A score over 8 calls for hospitalization. 25. A patient who was hospitalized for 2 weeks committed suicide during the night. Which initial measure will be most helpful for staff members and other patients regarding this event? a. Request the public information officer to make an announcement to the local media. b. Hold a staff meeting to express feelings and plan the care for other patients. c. Ask the patient's roommate not to discuss the event with other patients. d. Quickly discharge as many patients as possible to prevent panic. ANS: B Interventions should be aimed at helping the staff and patients come to terms with the loss and to grow because of the incident. Then, a community meeting should be scheduled to allow other patients to express their feelings and request help. Staff members should be prepared to provide additional support and reassurance to patients and should seek opportunities for peer support. The incorrect options will not control information or may result in unsafe care. 26. A severely depressed patient who has been on suicide precautions tells the nurse, "I am feeling a lot better, so you can stop watching me. I have taken too much of your time already." Which is the nurse's best response? a. "I wonder what this sudden change is all about. Please tell me more." b. "I am glad you are feeling better. The team will consider your request." c. "You should not try to direct your care. Leave that to the treatment team." d. "Because we are concerned about your safety, we will continue with our plan." ANS: D When a patient seeks to have precautions lifted by professing to feel better, the patient may be seeking greater freedom in which to attempt suicide. Changing the treatment plan requires careful evaluation of outcome indicators by the staff. The incorrect options will not cause the patient to admit to a suicidal plan, do not convey concern for the patient, or suggest that the patient is not a partner in the care process. 27. A new nurse says to a peer, "My newest patient has schizophrenia. At least I won't have to worry about suicide risk." Which response by the peer would be most helpful? a. "Let's reconsider your plan. Suicide risk is high in patients with schizophrenia." b. "Suicide is a risk for any patient with schizophrenia who uses alcohol or drugs." c. "Patients with schizophrenia are usually too disorganized to attempt suicide." d. "Visual hallucinations often prompt suicide among patients with schizophrenia." ANS: A Up to 10% of patients with schizophrenia die from suicide, usually related to depressive symptoms occurring in the early years of the illness. Depressive symptoms are related to suicide among patients with schizophrenia. Patients with schizophrenia usually have auditory, not visual, hallucinations. Although the use of drugs and alcohol compounds the risk for suicide, it is independent of schizophrenia. 28. The parents of identical twins ask a nurse for advice. One twin committed suicide a month ago. Now the parents are concerned that the other twin may also have suicidal tendencies. Which reply by the nurse would be most helpful? a. "Genetics are associated with suicide risk. Monitoring and support are important." b. "Apathy underlies suicide. Instilling motivation is the key to health maintenance." c. "Your child is unlikely to act out suicide when identifying with a suicide victim." d. "Fraternal twins are at higher risk for suicide than identical twins." ANS: A Twin studies suggest the presence of genetic factors in suicide; however, separating genetic predisposition to suicide from predisposition to depression or alcoholism is difficult. Primary interventions can be helpful in promoting and maintaining health and possibly counteracting the genetic load. The incorrect options are untrue statements or oversimplifications. 1. A college student failed two examinations. The student cried for hours and then tried to call a parent but got no answer. The student then suspended access to his social networking web site. Which suicide risk factors are present? Select all that apply. a. History of earlier suicide attempt b. Co-occurring medical illness c. Recent stressful life event d. Self-imposed isolation e. Shame or humiliation ANS: C, D, E Failing examinations in the academic major constitutes a recent stressful life event. Shame and humiliation related to the failure can be hypothesized. The inability to contact parents can be seen as a recent lack of social support, as can the roommate's absence from the dormitory. Terminating access to one's social networking site represents self-imposed isolation. This scenario does not provide data regarding a history of an earlier suicide attempt, a family history of suicide, or of co-occurring medical illness. 2. A patient with suicidal impulses is on the highest level of suicide precautions. Which measures should the nurse incorporate into the patient's plan of care? Select all that apply. a. Allow no glass or metal on meal trays. b. Remove all potentially harmful objects from the patient's possession. c. Maintain arm's length, one-on-one nursing observation around the clock. d. Check the patient's whereabouts every hour. Make verbal contact at least three times each shift. e. Check the patient's whereabouts every 15 minutes, and make frequent verbal contacts. f. Keep the patient within visual range while he or she is awake. Check every 15 to 30 minutes while the patient is sleeping. ANS: A, B, C One-on-one observation is necessary for anyone who has limited control over suicidal impulses. Plastic dishes on trays and the removal of potentially harmful objects from the patient's possession are measures included in any level of suicide precautions. The remaining options are used in less stringent levels of suicide precautions. 3. A nurse assesses five newly hospitalized patients. Which patients have the highest suicide risk? Select all that apply. a. 82-year-old white man b. 17-year-old white female adolescent c. 39-year-old African-American man d. 29-year-old African-American woman e. 22-year-old man with traumatic brain injury ANS: A, B, E Whites have suicide rates almost twice those of nonwhites, and the rate is particularly high for older adult men, adolescents, and young adults. Other high risk groups include young African-American men, Native-American men, older Asian Americans, and persons with traumatic brain injury. 4. A nurse assesses the health status of soldiers returning from Afghanistan. Screening for which health problems will be a priority? Select all that apply. a. Schizophrenia b. Eating disorder c. Traumatic brain injury d. Seasonal affective disorder e. Posttraumatic stress disorder ANS: C, E Traumatic brain injury and posttraumatic stress disorder each occur in approximately 20% of soldiers returning from Afghanistan. Some soldiers have both problems. The incidence of disorders identified in the distracters would be expected to parallel the general population. Textbook Questions 1. A 28 year old second grade teacher is diagnosed with major depressive disorder. She grew up in Texas but moved to Alaska 10 yrs ago to separate from an abusive mother. Her father died by suicide when she was 12 yrs old. Which combination of factors in this scenario best demonstrates the stress-diathesis model? -Family history of mental illness coupled with history of abuse. 2. A patient tells the nurse “No matter what I do, I feel like there’s always a dark cloud following me.” Select the nurse’s initial action. -Say to the patient “Tell me more about what you mean by ‘a dark cloud’.” 3. A patient experiencing depression says to the nurse, “My health care provider said I need ‘talk’ therapy but I think I need prescription for an antidepressant medication. What should I do?” Select the nurse's best response. -”Lets consider some ways to address your concerns with your healthcare provider.” 4. The nurse cares for a hospitalized adolescent diagnosed with major depressive disorder. The healthcare provider prescribes a low-dose antidepressant. In consideration of published warning about use of antidepressant medication in younger patients, which action should the nurse employ? -Monitor the adolescent closely for evidence of adverse effects, particularly suicidal thinking or behavior. 5. Over the past 2 months a patient made eight suicide attempts with increasing lethality. The healthcare provider informs the patient and family that electroconvulsive therapy (ECT) is needed. The family whispers to the nurse “Isn’t this a dangerous treatment?” How should the nurse reply? - “Electroconvulsive therapy is very effective when urgent help is needed. You family member was carefully evaluated for possible risks.” Chapter 16 Bipolar Text end chapter q’s 1. A patient has long history of bipolar disorder with frequent episodes of mania secondary to stopping prescribed meds. The patient says “I will use my whole check next month to buy lottery tickets. Winning will solve my money problems” Select nurse’s best action. –confer with the treatment team about appointing a legal guardian for the patient 2. Which comment by a patient diagnosed with bipolar disorder best indicates the patient is experiencing mania? –“Yesterday I made 487 posts on my social network page” 3. A community mental health nurse counsels a group of pts about the upcoming flu season. What instruction does the nurse provide for pts who are prescribed lithium? –“Stop taking your medicine and contact me if you have nausea, vomit and/or diarrhea” 4. A patient was diagnosed with bipolar disorder many years ago. The patient tells nurse “When I have a manic episode, there’s always a feeling of gloom behind it and I know I will soon be totally depressed.” What’s nurse’s best response? –“Your comment indicates you have an understanding and insight about your disorder” 5. A patient diagnosed with bipolar disorder lives in the community and is showing early signs of mania. The patient says “I need to go visit my daughter but she lives across the country. I put some requests on the internet to get a ride. I’m sure someone will take me” what is nurse’s most therapeutic response? –“I’m concerned about your safety when meeting or riding with strangers” Chapter 17 Schizophrenia - Text end chapter q’s 1. A patient smiles broadly at the nurse and says “look at my clean teeth. I brush them with scouring power because the label said it brightens and whitens everything” Which term should the nurse include when documenting this encounter? –concrete thinking 2. A patient diagnosed with schizophrenia says “I hear the voices every day. They always say bad things about me.” Which action by the nurse has the highest priority? – assess the patient for suicidal thinking/plans 3. Three days after beginning a new regime of Haldol 10 mg BID, the nurse observes that a hospitalized patient is drooling, has stiff and extended extremities, and has skin that is damp and hot to the touch. The patient has difficulty responding verbally to the nurse. What is the nurse’s correct analysis and action in this situation? –Neuroleptic malignant syndrome has developed; prepare the patient for immediate transfer to a medical unit 4. A patient diagnosed with schizophrenia complains to the nurse about persistent feelings of restlessness and says “I feel like I need to move all the time.” What is the nurse’s next action? – assess the patient for other EPS symptoms 5. A nurse begins a therapeutic relationship with a patient diagnosed with schizophrenia. The patient has severe paranoia. Which comment by the nurse is most appropriate? –as you get to know me better, I hope you will feel comfortable talking to me Ch 19 Substance Abuse - Text end chapter q’s 1. A young adult has heavily abused alcohol and prescription drugs since mid-adolescence. This person now has ataxic gait and uses a cane. Which comment by nurse presents reality while demonstrating compassion? -Addiction is powerful. You are young yet cannot walk without a cane. Make changes or your health will continue to suffer. 2. Nurse at local medical clinic reviews phoned-in requests from pts for prescription refills. As the nurse confers w the HCP about which prescription refill requests should be authorized, which refill request should be considered 1st? –Lorazepam (Ativan) 1 mg PO BID for an adult who has taken it daily for 3 years for episodes of anxiety (sudden withdrawal from benzo can cause death) 3. A patient tells nurse “After many years, I finally quit smoking. Now I use e-cigs only.” Which response should nurse provide? –“Congrats on quitting, but e-cigs have nicotine and other bad chemicals” 4. A young adult tells nurse, “I have new prescription for medical marijuana. I use it several times a day for my frequent muscle spasms.” Which info should nurse provide 1st to patient? –Instructions about safety issues associated with driving or operating machinery (bc of distortions and hallucinations) 5. A nurse teaches a patient diagnosed with alcohol addiction about a new prescription for naltrexone (Revia) which comment by patient indicates teaching was effective? –“This med is one part of a bigger treatment plan to help me stay sober” PHARM KEE Chapter 5 text end q’s 1. When caring for a patient recovering from an episode of opioid toxicity, the nurse determines that the patient has an addiction to the drug based on which finding? - Craving that results in drug-seeking behaviors 2. While teaching the parents of an adolescent who has been using marijuana, the nurse explains that the euphoria that results from the use of abused psychoactive substances is believed to be caused by which factor? - Stimulation of the dopamine pathways in the pleasure areas of the brain 3. A patient hospitalized with a fractured femur following an automobile accident develops diarrhea and vomiting with abdominal cramps, chills with goose bumps, and dilated pupils. The nurse suspects that the patient is experiencing which reaction? - Opioid withdrawal 4. Which agent(s) will the nurse anticipate administering to a patient who has been admitted with acute alcohol intoxication? (SATA) - Thiamine, Lorazepam (Ativan), Intravenous glucose solution 5. A patient is admitted to the emergency department with acute cocaine toxicity. Which is the most important intervention by the nurse? - Institute cardiac monitoring, and obtain frequent blood pressures 6. A nurse observes another nurse taking oral opioids from the medication room at the hospital. Which is the best action by the nurse? - Report the finding to the nursing supervisor to enable the nurse’s participation in a diversion program 7. A patient is to start disulfiram (Antabuse) to help with alcohol abuse. The nurse providing medication education about the drug will include which topics in the education plan? (SATA) - Importance of taking this medication every day, That better results are experienced when a support group helps with adhering to treatment, Common food and hygiene products containing alcohol, That disulfiram works by disrupting the metabolism of alcohol, That use of alcohol with disulfiram may cause nausea and vomiting and may even be fatal 8. A patient in the hospital is experiencing methamphetamine withdrawal. What does the nurse expect the symptoms and treatment to be? - Hypersomnia, irritability; treated by supportive care including pushing foods and fluids PHARM KEE chapter 6 text end q’s 1. What provisions of the Dietary Supplement Health and Education Act of 1994 are most important for the nurse to know related to patient health teaching? (SATA) - Clarified marketing regulations, reclassified herbs as dietary supplements, Stated that herbal products can be marketed with suggested dosages 2. The nurse discovers that a patient has recently decided to take four herbal preparations. Which action will the nurse take first? - Instruct the patient to inform the health care provider of all products taken 3. Labeling of herbal products is important. Which is an appropriate claim for an herbal product? - “Helps increase blood flow to the extremities” 4. The nurse is reviewing a patient’s current medications. Which herbal products interfere with the action of anticoagulants? (SATA) - Feverfew, Ginger, Licorice 5. A patient has multiple prescription medications and is not able to recall which herbal product he uses for relief of migraine headaches. He notes that it is a serum antagonist and asks the nurse to assist him. What is the best response by the nurse? - “Feverfew” 6. A patient is being followed by a cardiovascular clinic and takes garlic, which is reported to decrease cholesterol, triglycerides, blood pressure, and blood-clotting capability. Which patient statement indicates a need for further teaching? (SATA) - “I can just take garlic for my heart problems.”, “”Garlic is very effective in preventing depression” 7. The nurse notes that many patients are taking herbal remedies for relief of depression and anxiety. Which herbal preparation has at least 10 pharmacologically active components for relief of depression and anxiety? - St. John’s wort 8. A patient who is pregnant asks the nurse, “What herbal products can I take?” What is the best response by the nurse? - “Discuss this with your health care provider” 9. A patient reports taking aloe, ginkgo, and licorice on a regular basis. Which patient statement indicates a need for health reaching about potential drug interactions? - “Aloe decreases the effect of my digoxin” PHARM KEE chapter 28 text q’s Antidepressants and Mood Stabilizers 1. A patient is admitted with bipolar affective disorder. The nurse acknowledges that which medication is used to treat this disorder for some patients in place of lithium? - divalproex (Depakote) 2. The nurse realizes that some herbs interact with selective serotonin reuptake inhibitors. Which herb interaction may cause serotonin syndrome? - St. John’s Wort 3. A selective serotonin reuptake inhibitor (SSRI) is prescribed for a patient. The nurse knows that which drugs is an SSRI? - paroxetine (Paxil) 4. A patient is taking tranycypromine sulfate (Parnate) for depression. What advice should the nurse include in the teaching plan for this medication? - Instruct the patient to avoid beer and cheddar cheese 5. Which statement is true concerning lithium? - Concurrent NSAIDs may increase lithium levels 6. When a patient is taking an antidepressant, what should the nurse do? (SATA) - Monitor the patient for suicidal tendencies, Observe the patient for orthostatic hypotension, Teach the patient to take the drug with food if GI distress occurs, Tell the patient that the drug may not have full effectiveness for 1-2 weeks 7. A patient is taking lithium. The nurse should be aware of the importance of which nursing intervention(s)? (SATA) - Observe the patient for motor tremors, Monitor the patient for orthostatic hypotension, Advise the patient to avoid caffeinated foods and beverages, Teach the patient to take lithium with meals to decrease gastric irritation PHARM study guide Chapter 5 1. The patient presents to the ED under the custody of local law enforcement. The patient reportedly swallowed a small balloon full of cocaine. What clinical manifestations would the nurse expect to see if the balloon ruptured? - dilated pupils and diaphoresis 2. A patient is taking Tricyclic antidepressants as a prophylactic medicine for migraines. She also abuses cocaine. What effect may result from the interaction b/t these two substances? - Dysrhythmias 3. Which medication can be given to aid a pt with opioid withdrawal? - Methadone (Dolophone) 4. Which drug contains the only Food and Drug Administration-approved cannabis preparation? - Dronabinol (Marinol) 5. The patient has decided to quit smoking. What key point(s) must the nurse include in the teaching plan? (SATA) - assess that the patient is motivated to quit, Help the patient ID triggers or high-risk situations, Provide the patient with a list of OTC and prescription options for smoking cessation aids 6. What percentage of nurses abuse drugs and demonstrate impaired practice due to that abuse? - 3-6% 7. A mom brings her 13 year old daughter to the ED because she is “acting strange”. The mother states she found a package of “bath salts: in her daughter’s bedroom. “Bath salts” are classified as which type of drug? - Synthetic stimulant PHARM KEE STUDY GUIDE CH 6 HERBALS 1. Which herb is commonly used for external treatment of insect bites and minor burns? - Aloe vera 2. The patient presents to the clinic with complaints of abdominal discomfort and nausea. When obtaining the health history, the nurse inquiries about herbal preparations. Which herb would the nurse recognize as 1 that provides relief of digestive and gastrointestinal distress? - Chamomile 3. What is the most commonly prescribed herbal remedy worldwide? - Ginkgo biloba 4. The pregnant patient has been complaining of a sore in her mouth. Her grandmother suggested she try “something natural” due to her pregnancy. Which preparation does the nurse recognize as unsafe due to causing uterine irritability? - Goldenseal 5. The patient drinks 3 to 4 alcoholic drinks per day and reports that he takes a variety of herbal preps. Which herb does the nurse recognize as dangerous for this patient given his history? - Kava 6. The patient has just started taking warfarin (Coumadin) for Atrial fibrillation. Health teaching for this patient would include info on which herbal product(s)? (SATA) - evening primrose, Garlic, Ginseng, Hawthorn 7. The nurse is concerned that a patient may be abusing ginseng. What will the nurse assess in a patient with ginseng abuse syndrome? (SATA) - Edema, Reflexes 8. Which statement(s) by the patient reflect(s) prudent use of herbs? (SATA) - “Do not take a large quantity of any one herbal product”, “Do not give herbs to infants or young children” 9. The nurse is caring for a patient who takes a variety of herbal products and is starting a RX antidiabetic medication. Which herb(s) will change the effect of the antidiabetic drug? (SATA) - Cocoa, Dandelion, Garlic 10. The patient tells the nurse he is taking ginkgo. Which medication(s) has/have negative interactions with ginkgo? (SATA) - Antiplatelet agents, Nifedipine (Procardia), Omeprazole (Prilosec) 11. The patient has a history of HTN, A-fib, COPD, and insomnia. She tells the nurse at discharge “I really love the taste of licorice. It motivates me to walk every day.” What medication alteration(s) is/are seen when given in combination with licorice? (SATA) - Antihypertensive meds effects are decreased, Corticosteroid effects are increased, Digoxin effects are increased PHARM KEE STUDY GUIDE CH 28 ANTIDEPRESSANTS/mood stabilizers 1. The 12 year old patient has been evaluated for enuresis. After home remedies and other alternatives have been explored, which medication does the nurse know may be prescribed to treat this condition? - Imipramine (Tofranil) 2. The patient has been taking phenelzine (Nardil) for several months and his chronic anxiety and fear have not improved. What is the maximum daily dose for this medication? - 90 mg/d 3. The patient has been prescribed amitriptyline (Elavil) as an adjunct to therapy for depression. What info will the nurse include in the health teaching regarding this med? - “Stand up slowly because your BP can drop suddenly” 4. Which food(s) or beverages(s) is/are contraindicated in a patient prescribed isocarboxazid (Marplan)? (SATA) - Bananas, Chocolate, Wine 5. The patient has a history of depression and is taking fluoxetine (Prozac). The patient presents to the ED complaining of a severe headache. She is diaphoretic and is unable to sit still. Her family tells the nurse that the patient has been taking “some herb”. Which herb does the nurse suspect? - St. John’s Wort 6. Which is an advantage of taking SSRI’s over TCAs? - Less sedation 7. Which nursing intervention is most important for a patient taking lithium? - Monitoring for excessive thirst, weight gain, and increased urination 8. The patient has been diagnosed with bipolar disorder and is acutely manic. The patient is currently taking lithium (LIthobid). What laboratory value is the nurse particularly concerned with? - BUN 9. The patient has been taking lithium (Lithobid) 1800mg/day in 3 divided doses for 10 days. He remains agitated and hyperactive, with a lithium level of 0.8 mEq/L. He complains of feeling slow and thirsty. What does the nurse suspect is occurring? - The patient’s lithium level is subtherapeutic, and he is still manic 10. The nurse is teaching the patient about lithium (Eskalith). Which statement by the patient indicates a need for more education? - “If I stop my medication, the depression will return” 11. The patient has been prescribed venlafaxine (Effexor) for generalized anxiety disorder. Which statement by the patient indicates the need for further health teaching? - “It is OK if I keep taking my herbal medications for my depression and anxiety” 12. The patient has a history of bipolar disorder and takes lithium (Eskalith). She tells her health care provider that she would like to become pregnant in the near future. What is/are the concern(s) with taking lithium while pregnant? (SATA) - Congenital anomaly PHARM CH 5 Chapter 05: Drugs of Abuse MULTIPLE CHOICE 1. The nurse is teaching a group in the community about drug abuse. Which statement by the nurse is correct? a. “Cue-induced cravings eventually disappear after long periods of abstinence by the person addicted to drugs.” b. “Drug abuse and drug addiction are synonymous terms, describing dependence on drugs.” c. “Drug addiction is characterized by emotional, mental, and sometimes physical dependence.” d. “Drug addiction occurs when physical dependence is present.” ANS: C Drug addiction occurs when emotional and mental dependence on a drug are present. Although physical dependence may often occur, it is not always present. Cue-induced cravings may diminish after long abstinence but do not disappear completely. Drug abuse may occur without addiction. 2. The nurse is caring for a patient who is being treated for chronic alcohol intoxication. The nurse notes that the patient’s serum alcohol level is 0.40 mg%. The patient is awake and talkative even though this is a potentially lethal dose. The nurse recognizes this as alcohol a. addiction. b. dependence. c. misuse. d. tolerance. ANS: D Intoxication is a state of being influenced by a drug or other substance and may be a very small amount in the drug-naïve person or a potentially lethal amount in the chronic user. This person has developed tolerance to alcohol and is able to have a potentially lethal amount without severe effects. Addiction describes a state of emotional, mental, and physical dependence on a drug. Dependence describes physical need for the drug such that when the drug is stopped, withdrawal symptoms occur. Misuse refers to using a drug or substance to excess. 3. What does the nurse understand must occur in order to produce withdrawal syndrome? a. Addiction b. Craving c. Drug tolerance d. Physical dependence ANS: D Patients who develop a physical dependence on a drug will experience withdrawal syndrome when the drug is stopped. Addiction and cravings can occur without physical dependence. Tolerance refers to a decrease in drug effects with repeated use. 4. The nurse is counseling a patient who wants to stop smoking. Which statement by the nurse is correct? a. “Bupropion (Zyban) is effective and does not have serious adverse effects.” b. “Nicotine replacement therapies are effective and eliminate the need for behavioral therapy.” c. “Varenicline (Chantix) may be used short-term for 1 to 2 months.” d. “You may experience headaches and increased appetite for several months after stopping smoking. ANS: D Headaches and increased appetite are common during nicotine withdrawal and may last for several months. Bupropion is effective but has many serious effects. Nicotine replacement therapy does not eliminate the need for behavioral therapy. Varenicline is used for at least 4 months. 5. A patient with asthma has been using a nicotine transdermal 24-hour patch for 3 weeks to quit smoking. The patient reports having difficulty sleeping. What action will the nurse take? a. Ask the provider for a prescription for Nicotrol NS. b. Recommend removing the patch at bedtime. c. Suggest using an 18-hour patch instead. d. Tell the patient to stop the patch and join a support group. ANS: C The patient should try an 18-hour patch to help with sleep. Nicotrol is not a good option for patients with asthma. 6. The nurse is discussing smoking cessation with a nurse colleague who smokes. Which statement indicates a readiness to quit smoking? a. “I don’t smoke around my children or inside the house.” b. “I want to stop smoking, but I will need help to do it.” c. “I will quit so my co-workers will stop harassing me about it.” d. “If I cut down gradually, I should be able to quit.” ANS: B Patients exhibit readiness when they state a desire to quit along with a request for professional assistance. Other factors, such as children or co-workers, do not indicate a desire to quit. 7. A patient is using the Commit lozenge 2 mg to help quit smoking and reports nausea and indigestion. The nurse will instruct the patient to perform which action? a. Allow the lozenge to dissolve slowly over 20 to 30 minutes. b. Chew the lozenge thoroughly before swallowing it. c. Increase to 4 mg and use less often. d. Take the lozenge with food and a full glass of water. ANS: A The patient should allow the lozenge to dissolve slowly. Chewing or swallowing the lozenge increases gastrointestinal side effects. Increasing the dose and decreasing the frequency are not recommended. 8. A patient is brought to the emergency department by a family member. The patient reports seeing colored lights and describes feeling bugs crawling under the skin. The nurse suspects that this patient is abusing which drug? a. Alcohol b. Cocaine c. LSD d. Methamphetamine ANS: B A stimulant psychosis can occur with chronic use of any stimulant and, with cocaine, progresses to visual hallucinations of colored lights and tactile hallucinations of bugs crawling under the skin. These are not signs of abuse with alcohol, LSD, or methamphetamine. 9. The nurse is caring for a patient who is chronically irritable and anxious and prone to violent behaviors. The patient has several teeth missing and has dental caries in the remaining teeth. The nurse suspects previous chronic use of which drug? a. Alcohol b. Cocaine c. LSD d. Methamphetamine ANS: D Patients previously exposed to methamphetamine use will exhibit these symptoms, and the physical effects of extended methamphetamine use are notable tooth decay and dermatologic deterioration. 10. The nurse is teaching a patient who has completed detoxification for alcohol abuse who will be discharged home with a prescription for disulfiram (Antabuse). Which statement by the patient indicates understanding of the teaching? a. “Even topical products containing alcohol can have serious adverse effects while I am taking this drug.” b. “If I experience drowsiness or skin rash, I should discontinue this drug immediately.” c. “It is safe to take a product containing alcohol one week after the last dose of disulfiram.” d. “This drug acts by blocking the pleasurable effects of alcohol.” ANS: A Disulfiram causes an unpleasant and potentially fatal reaction if alcohol is consumed while taking it and can even occur with topical products containing alcohol. Drowsiness and skin rash aren’t common adverse effects. The effects of disulfiram do not wear off for up to 2 weeks after the last dose. It does not block the pleasurable effects of alcohol. 11. A patient who has a long history of alcohol abuse is admitted to the hospital for detoxification. In addition to medications needed to treat withdrawal symptoms, the nurse will anticipate giving intravenous a. dopamine to restore blood pressure. b. fluid boluses to treat dehydration. c. glucose to prevent hypoglycemia. d. thiamine to treat nutritional deficiency. ANS: D Thiamine should be given to prevent Wernicke’s encephalopathy in patients treated for alcoholism. If glucose is indicated, the thiamine should be given first. Other treatments are given as indicated. 12. A patient arrives in the emergency department in an acute state of alcohol intoxication and reports chronic consumption of “several six packs” of beer every day for the past year. The nurse anticipates administering which medication or treatment? a. Chlordiazepoxide (Librium) b. Disulfiram (Antabuse) c. Gastric lavage d. Vasoconstrictors ANS: A To prevent acute withdrawal and delirium tremens, a long-acting benzodiazepine, such as chlordiazepoxide, is given. Disulfiram would cause an acute drug interaction. Gastric lavage should no longer be performed, and vasoconstrictors are not indicated. 13. A patient who is unconscious arrives in the emergency department with clammy skin and constricted pupils. The nurse assesses a respiratory rate of 8 to 10 breaths per minute. The paramedics report obvious signs of drug abuse in the patient’s home. The nurse suspects that this patient has had an overdose of which substance? a. Alcohol b. LSD c. An opioid d. Methamphetamine ANS: C Opioid overdose is characterized by constricted pupils and respiratory depression. 14. A patient is brought to the emergency department after ingesting an overdose of lorazepam (Ativan) several hours prior. The patient has a respiratory rate of 6 to 10 breaths per minute and is unconscious. The nurse will prepare to perform which action? a. Administer activated charcoal. b. Give flumazenil (Romazicon). c. Give naloxone (Narcan). d. Perform gastric lavage. ANS: B Flumazenil is the antidote for benzodiazepine overdose. Activated charcoal is used for asymptomatic patients who have recently consumed the drug. Gastric lavage should no longer be performed for treatment. 15. A patient with a history of opioid abuse will be discharged home with buprenorphine to help prevent relapse. Which product will the nurse anticipate the provider to order? a. Buprenex b. Suboxone c. Subutex d. Vivitrol ANS: A Buprenex is an agonist-antagonist opioid that can be used for detoxification and maintenance therapy because it has a low potential for abuse. Suboxone and Subutex have abuse potential. Vivitrol does not contain buprenorphine and does not prevent cravings. 16. The nurse is teaching a patient who will be discharged home with naltrexone (ReVia) after treatment for opioid addiction. What information will the nurse include in the teaching for this patient? a. “This drug will help control cravings.” b. “You may take this drug once weekly.” c. “ReVia blocks the pleasurable effects of opioids.” d. “If you discontinue this drug abruptly, you will have withdrawal symptoms.” ANS: C ReVia acts by blocking the pleasurable effects of opioids. It can precipitate withdrawal when given to opioid-dependent patients. This drug does not control cravings, and it is taken once daily or every other day. PHARM CH 6 Chapter 06: Herbal Therapies MULTIPLE CHOICE 1. A family member expresses concern that a patient is taking several herbal remedies and worries that they may be unsafe. The nurse will respond by saying that herbs a. are classified as medications by the Dietary Supplement Health and Education Act of 1994. b. are regulated by the government and are determined to be safe. c. aren’t usually effective but are generally harmless. d. should be discussed with the patient’s provider in conjunction with other medications. ANS: D Herbs are sometimes useful but can also be useless or dangerous. There are two types of monographs under development to compile information about these substances, but there are no agencies that regulate safety and efficacy. Patients should always tell providers if they are taking any herbal remedies since there are known drug-herbal interactions and side effects. 2. A pregnant woman tells the nurse that she is taking ginger to reduce morning sickness. What will the nurse tell this patient? a. “Ginger can cause fetal birth defects.” b. “Ginger is not safe during pregnancy.” c. “Ginger can cause abortion in low doses.” d. “Ginger may be taken in low doses for up to 4 days.” ANS: D Ginger may be taken during pregnancy for morning sickness, but only on a short-term, low-dose basis. There is no indication that it causes fetal birth defects. Ginger is an abortifacient in large amounts. 3. A patient asks the nurse about an herbal supplement and reports that it has a USP seal of approval. The nurse explains that this indicates a. identity, potency, purity, and labeling accuracy. b. premarket testing for safety and efficacy. c. structure and function claims may be made. d. the supplement’s ability to prevent and treat disease. ANS: A The USP “seal of approval” is a fee-based test and reports on identity, potency, purity, and labeling accuracy. It does not indicate premarket research on safety and accuracy, does not allow manufacturers to make claims about the function of the products, and does not indicate the substance’s ability to prevent and treat disease. 4. A woman reports using aloe vera to treat constipation. Which response by the nurse is correct? a. “Aloe vera is for external use only.” b. “Please tell me if you are taking cardiac medications or diuretics.” c. “Side effects of aloe vera are common.” d. “You may experience a decrease in menstrual flow while taking aloe vera.” ANS: B Patients taking aloe vera should consult with their provider if taking cardiac medications or diuretics. It is for internal and external use. Side effects are rare except with long-term use or in large doses. Aloe vera can increase menstrual flow. 5. A woman who is experiencing hot flashes associated with menopause asks the nurse about using black cohosh. Which response by the nurse is correct? a. “Black cohosh may be used long term in place of hormone replacement therapy.” b. “Black cohosh may contribute to iron toxicity.” c. “Black cohosh may interact with antihypertensive drugs.” d. “Black cohosh treats menopausal symptoms without altering hormone levels.” ANS: C Black cohosh may increase the action of antihypertensive medications. It should not be used longer than 6 months. It may decrease iron absorption, contributing to iron deficiency. Black cohosh suppresses luteinizing hormone and optimizes estrogen levels. 6. A patient who has HIV asks the nurse about taking echinacea to improve immune function. What will the nurse tell this patient? a. “The root extract is useful for treating upper respiratory and urinary tract infections.” b. “This use is currently being studied in patients who have HIV.” c. “Use it as needed when antibiotics fail to treat your infections.” d. “You may use it safely up to 8 weeks at a time as a preventive medication.” ANS: B The use of echinacea to stimulate the immune system of patients with HIV is being investigated, but its use is currently not recommended. 7. The nurse is counseling a female patient who reports taking feverfew to treat premenstrual syndrome and migraines. Which statement by the patient indicates understanding of the teaching? a. “I should experience immediate effects with this herb.” b. “I should not take feverfew if I get pregnant.” c. “I should take feverfew with nonsteroidal anti-inflammatory drugs (NSAIDs) to enhance its effects.” d. “If I develop gastrointestinal (GI) upset, I should stop taking feverfew immediately.” ANS: B Feverfew should be avoided during pregnancy. Patients may not experience effects for 4 to 6 weeks. Patients should not take feverfew with NSAIDs without consulting the provider. Discontinuing feverfew abruptly can cause rebound headache. 8. The nurse is providing preoperative education to a patient who will have surgery in several weeks. The patient denies taking anticoagulant medications but reports using herbal supplements. Which herb would cause the nurse to be concerned? a. Echinacea b. Ginkgo biloba c. Kava d. Sage ANS: B Ginkgo can prolong bleeding time and therefore should be discontinued 2 weeks prior to surgery. 9. The nurse provides counseling to a patient who reports taking kava to treat anxiety-related symptoms. Which statement by the patient indicates understanding of the teaching? a. “I may take kava with lorazepam (Ativan).” b. “I may develop a serious skin disorder while taking kava.” c. “I should avoid alcohol while taking kava.” d. “I should avoid St. John’s wort and valerian while taking kava.” ANS: C Kava can cause drowsiness and may cause liver damage. Patients should be cautioned against drinking alcohol while taking kava. Kava increases the effect of other benzodiazepines, so it should not be taken with lorazepam. It does not cause skin disorders. It may be taken with St. John’s wort and valerian. 10. The spouse of a patient who is an alcoholic asks the nurse about dietary supplements that may help prevent liver disease. Which herb will the nurse suggest the patient discuss with a provider who has prescriptive authority? a. Ginkgo biloba b. Kava c. Milk thistle d. Sage ANS: C Milk thistle can prevent damage to liver cells and stimulates regeneration of liver cells. 11. A patient reports taking St. John’s wort to treat symptoms of depression and asks the nurse how to use this product safely and effectively. Which response by the nurse is correct? a. “Apply sunscreen while taking St. John’s wort.” b. “It is safe to take St. John’s wort with prescription antidepressants.” c. “St. John’s wort does not affect nutrition.” d. “You should take St. John’s wort as needed when symptoms occur.” ANS: A St. John’s wort can cause photosensitivity, so patients should be counseled to use sunscreen. It should not be taken with prescription antidepressants because it increases the risk of suicidal ideation. It interferes with the absorption of iron and other minerals. Effects do not occur for 4 to 8 weeks, so it cannot be taken as needed. 12. A man is diagnosed with benign prostatic hypertrophy and wants to avoid surgery. He asks the nurse about using saw palmetto. What information will the nurse include when teaching this patient about this herb? a. “Gastric disturbances are common while using this herb.” b. “Saw palmetto may help treat erectile dysfunction (ED) as well.” c. “Use saw palmetto for up to 30 days and stop taking it when effects occur.” d. “You should stop taking the herb 1 to 2 weeks prior to prostate-specific antigen (PSA) testing.” ANS: D Saw palmetto can cause a false-negative test result for PSA, so patients should stop taking it 1 to 2 weeks prior to this test. GI effects are rare. It is not effective in treating ED. Effects usually don’t occur until 30 days, and it may be taken long term. 13. A patient is taking valerian, or “herbal valium,” to induce sleep. What will the nurse teach this patient about this herb? a. Habituation and addiction are likely. b. Hangover effects are common with usual doses. c. Liver function tests must be monitored with long-term use. d. Valerian has a high risk for overdose. ANS: C Liver function tests must be monitored with long-term use, and valerian should be discontinued if these are elevated. Habituation and addiction are rare. Hangover effects occur with high doses. There is no increased risk for overdose. 14. A patient asks the nurse about the safety of herbal products in general. Which response by the nurse is correct? a. “Consumers should research products and their manufacturers before taking.” b. “Manufacturers are required to list interactions of herbs with drugs and food.” c. “Products manufactured for drug and grocery store chains are safe.” d. “Toxicological analysis is required of all commercial herbal products.” ANS: A There are no comprehensive regulations of herbal supplements regarding safety and efficacy, so consumers should research herbs and product manufacturers. Companies manufacturing for drug and grocery stores are suspect and do not always list all ingredients on their labels. 15. Which information can consumers expect to be included in labeling of herbal products? a. Actions and uses b. Interactions and precautions c. Scientific name of the product d. Safety and efficacy study results ANS: C Manufacturers should list the scientific name of the product and the parts of the plant used in preparation. They are not required to list actions, uses, interactions, precautions, and any results of safety or efficacy studies. MULTIPLE RESPONSE 1. Which herbal remedies are often used for gastrointestinal disorders? (Select all that apply.) a. Chamomile b. Cranberry c. Dong quai d. Echinacea e. Ginger f. Peppermint ANS: A, E, F Chamomile, ginger, and peppermint are often used to treat gastrointestinal disorders. PHARM CH 28 Chapter 28: Antidepressants and Mood Stabilizers MULTIPLE CHOICE 1. A nurse performs a medication history on a newly admitted patient. The patient reports taking amitriptyline (Elavil) 75 mg at bedtime for 6 weeks to treat depression. The patient reports having continued fatigue, lack of energy, and depressed mood. The nurse will contact the provider to discuss which intervention? a. Beginning to taper the amitriptyline b. Changing to a morning dose schedule c. Giving the amitriptyline twice daily d. Increasing the dose of amitriptyline ANS: A The response to tricyclic antidepressants (TCAs) should occur after 2 to 4 weeks of therapy. If there is no improvement at this time, the TCA should be gradually withdrawn and an SSRI prescribed. TCAs should never be stopped abruptly. TCAs cause fatigue and drowsiness, so they should be given at bedtime. Changing the dose or the dosing schedule are not indicated. 2. The nurse is teaching a patient who will begin taking doxepin (Sinequan) to treat depression. Which statement by the patient indicates a need for further teaching? a. “I should expect results within 2 to 4 weeks.” b. “I should increase fluids and fiber while taking this medication.” c. “I should take care when rising from a sitting to standing position.” d. “I will take the medication in the morning before breakfast.” ANS: D Tricyclic antidepressants (TCAs) should begin to show effects within 1 to 4 weeks. Tricyclic antidepressants are known to cause orthostatic hypotension and constipation, so patients should be counseled on how to minimize these effects. TCAs should be taken at bedtime because of their tendency to cause drowsiness. 3. A patient who is taking amitriptyline (Elavil) reports constipation and dry mouth. The nurse will give the patient which instruction? a. Increase fluid intake. b. Notify the provider. c. Request another antidepressant. d. Stop taking the medication immediately. ANS: A Constipation and dry mouth are common side effects of tricyclic antidepressants (TCAs), and patients should be taught to manage these symptoms. There is no need to notify the provider or to switch medications unless the side effects become too uncomfortable. Patients should not stop taking TCAs abruptly. 4. A patient who has had a loss of interest in most activities, weight loss, and insomnia is diagnosed with a major depressive disorder and will begin taking fluoxetine (Prozac) daily. The patient asks about the weekly dosing that a family member follows. What will the nurse tell the patient about a weekly dosing regimen? a. It can be used after daily maintenance dosing proves effective and safe. b. It is used after a trial of tricyclic antidepressant medication fails. c. It is not effective for this type of depression and its symptoms. d. It will cause more adverse effects than daily dosing regimens. ANS: A Before weekly dosing is begun, the patient should respond to a daily maintenance dose of 20 mg/day without serious effects. It is not necessary to undergo a trial of tricyclic antidepressants (TCAs). Weekly dosing is used for this type of depression, and although it may have some adverse effects, these are not more common than with daily dosing. 5. A patient has been taking sertraline (Zoloft) 20 mg/mL oral concentrate, 1 mL daily for several weeks and reports being unable to sleep well. What will the nurse do next? a. Ask the patient what time of day the medication is taken. b. Counsel the patient to take the medication at bedtime. c. Recommend asking the provider about weekly dosing. d. Suggest that the patient request a lower dose. ANS: A Selective serotonin reuptake inhibitors (SSRIs) cause nervousness and insomnia. Patients can minimize these effects by taking the drug in the morning. The nurse should assess this with this patient. Taking the medication at bedtime will only increase the insomnia. Requesting a lower dose 6. A patient has been taking paroxetine (Paxil) 20 mg per day for 2 weeks and reports headaches, nervousness, and poor appetite. Which action will the nurse take? a. Counsel the patient to take the medication with food. b. Reassure the patient that these side effects will decrease over time. c. Suggest that the patient discuss a lower dose with the provider. d. Tell the patient to stop taking the drug and contact the provider. ANS: B These are common side effects of SSRIs and will subside over time. Taking the medication with food will not affect these side effects. Lowering the dose is not indicated. Patients should not abruptly stop taking SSRIs. 7. A patient who has been diagnosed with social anxiety disorder will begin taking venlafaxine (Effexor). The nurse who performs a medication and dietary history will be concerned about ingestion of which substance or drug? a. Coffee b. Grapefruit juice c. Oral hypoglycemic drug d. St. John’s wort ANS: D The concurrent interaction of venlafaxine and St. John’s wort may increase the risk of serotonin syndrome and neuroleptic malignant syndrome. Oral hypoglycemic drugs are concerning for patients who take lithium. Coffee and grapefruit juice is to be avoided by patients who take monoamine oxidase inhibitors 8. A male patient has been taking venlafaxine (Effexor) 37.5 mg daily for 2 weeks and reports ejaculation dysfunction and urinary retention. What action will the nurse take? a. Contact the provider to discuss decreasing the dose. b. Reassure the patient that these are common side effects. c. Report potential serious adverse effects to the provider. d. Withhold the dose until the provider is notified. ANS: B Venlafaxine can cause ejaculation dysfunction and urinary retention, and these side effects tend to be transient and treatable. Decreasing the dose is not indicated, and these are not serious adverse effects. Withholding the dose is not indicated. 9. A patient who has been taking a monoamine oxidase (MAO) inhibitor for several months will begin taking amoxapine (Asendin) instead of the MAO inhibitor. The nurse will counsel the patient to begin taking the amoxapine a. along with the MAO inhibitor for several months. b. at least 14 days after discontinuing the MAO inhibitor. c. the day after the last dose of the MAO inhibitor. d. while withdrawing the MAO inhibitor over several weeks. ANS: B Amoxapine is an atypical antidepressant that should not be taken with MAO inhibitors and should not be used within 14 days of taking an MAO inhibitor. DIF: COGNITIVE LEVEL: Understanding (Comprehension) REF: Page 389 TOP: NURSING PROCESS: Planning MSC: NCLEX: Physiological Integrity: Pharmacological and Parenteral Therapies 10. A patient who has been diagnosed with depression asks why the provider has not ordered a monoamine oxidase (MAO) inhibitor to treat the disorder. The nurse will explain to the patient that MAO inhibitors a. are more expensive than other antidepressants. b. are no longer approved for treating depression. c. can cause profound hypotension. d. require strict dietary restrictions. ANS: D MAO inhibitors have many food and drug interactions that can be fatal, and patients must adhere to strict dietary restrictions while taking these drugs. They are not more expensive than the newer antidepressants. They remain approved for treating depression. MAO inhibitors cause profound hypertension. DIF: COGNITIVE LEVEL: Understanding (Comprehension) REF: Page 389 TOP: NURSING PROCESS: Planning MSC: NCLEX: Physiological Integrity: Pharmacological and Parenteral Therapies 11. A patient who takes a monoamine oxidase (MAO) inhibitor asks the nurse about taking over-the-counter medications to treat cold symptoms. Which medication will the nurse counsel the patient to avoid while taking an MAO inhibitor? a. Diphenhydramine b. Guaifenesin c. Pseudoephedrine d. Saline nasal spray ANS: C MAO inhibitors can cause hypertensive crises, which can be fatal when taken with sympathomimetic drugs such as pseudoephedrine. DIF: COGNITIVE LEVEL: Applying (Application) REF: Page 391 TOP: NURSING PROCESS: Planning MSC: NCLEX: Physiological Integrity: Pharmacological and Parenteral Therapies 12. A patient who has a major depressive disorder has been taking fluoxetine (Prozac) 20 mg daily for 3 months and reports improved mood, less fatigue, and an increased ability to concentrate. The patient’s side effects have diminished. What will the nurse counsel this patient to discuss with the provider? a. Changing to once-weekly dosing b. Decreasing the dose to 10 mg daily c. Discontinuing the medication d. Increasing the dose to 30 mg daily ANS: A Once patients have demonstrated control of symptoms with decreased side effects on the maintenance dose of 20 mg daily, patients may begin once-weekly dosing. The 20-mg dose is maintenance dosing, so decreasing or increasing the dose is not indicated. Patients should not stop taking the medication abruptly. DIF: COGNITIVE LEVEL: Applying (Application) REF: Page 388 TOP: NURSING PROCESS: Planning MSC: NCLEX: Physiological Integrity: Pharmacological and Parenteral Therapies 13. A patient who has been diagnosed with a major depression disorder has been ordered to take doxepin (Sinequan). The nurse will contact the provider if the patient’s medical history reveals a history of which condition? a. Asthma b. Glaucoma c. Hypertension d. Hypoglycemia ANS: B Antidepressants, such as doxepin, that cause anticholinergic-like symptoms are contraindicated if the patient has glaucoma. DIF: COGNITIVE LEVEL: Applying (Application) REF: Page 394 TOP: NURSING PROCESS: Assessment MSC: NCLEX: Physiological Integrity: Pharmacological and Parenteral Therapies 14. The nurse is preparing to administer a dose of lithium (Lithibid) to a patient who has been taking the drug as maintenance therapy to treat bipolar disorder. The nurse assesses the patient and notes tremors and confusion. The patient’s latest serum lithium level was 2 mEq/L. Which action will the nurse take? a. Administer the dose. b. Hold the dose and notify the provider. c. Request an order for a higher dose. d. Request an order for a lower dose. ANS: B The patient has symptoms of lithium toxicity, and the serum drug level is in toxic range. The nurse should hold the dose and notify the provider. DIF: COGNITIVE LEVEL: Applying (Application) REF: Page 396 TOP: NURSING PROCESS: Evaluation MSC: NCLEX: Physiological Integrity: Pharmacological and Parenteral Therapies 15. The nurse assesses a patient who is taking lithium (Lithibid) and notes a large output of clear, dilute urine. The nurse suspects which cause for this finding? a. Cardiovascular complications b. Expected lithium side effects c. Increased mania d. Lithium toxicity ANS: D An increased output of dilute urine is a sign of lithium toxicity. DIF: COGNITIVE LEVEL: Understanding (Comprehension) REF: Page 397 TOP: NURSING PROCESS: Evaluation MSC: NCLEX: Physiological Integrity: Pharmacological and Parenteral Therapies 16. The nurse provides teaching for a patient who will begin taking lithium (Lithibid). Which statement by the patient indicates understanding of the teaching? a. “I may drink tea or cola but not coffee.” b. “I may stop taking the drug when mania symptoms subside.” c. “I should consume a sodium-restricted diet.” d. “I should drink 2 to 3 liters of fluid each day.” ANS: D Patients taking lithium should be encouraged to maintain adequate fluid intake of 2 to 3 L/day initially and then 1 to 2 L/day as maintenance. Patients should not drink any caffeine-containing drinks, including tea and cola. Patients must continue taking lithium even when symptoms subside, or else symptoms will recur. It is not necessary to consume a sodium-restricted diet. DIF: COGNITIVE LEVEL: Applying (Application) REF: Page 397 TOP: NURSING PROCESS: Nursing Intervention: Patient Teaching MSC: NCLEX: Physiological Integrity: Pharmacological and Parenteral Therapies 17. A patient who has recently begun taking lithium (Lithibid) calls the clinic to report nausea, vomiting, anorexia, and drowsiness. What will the nurse do next? a. Contact the provider to obtain an order for a serum lithium level. b. Reassure the patient that these symptoms are common and transient. c. Tell the patient that the lithium dose is probably too low. d. Tell the patient to stop taking the medication immediately. ANS: A Early symptoms of lithium toxicity include nausea and vomiting, anorexia, and drowsiness. The nurse should obtain an order for a lithium level to evaluate this. Patients should be encouraged to report these symptoms if they occur. Patients should never be counseled to stop the medication abruptly. DIF: COGNITIVE LEVEL: Applying (Application) REF: Page 397 TOP: NURSING PROCESS: Evaluation MSC: NCLEX: Physiological Integrity: Pharmacological and Parenteral Therapies 18. The nurse is preparing to administer paroxetine HCl (Paxil) to a 70-year-old patient. The nurse understands that this patient may require a. a decreased dose. b. an increased dose. c. every other day dosing. d. more frequent dosing. ANS: A Older adults usually need a lower dose of antidepressants. DIF: COGNITIVE LEVEL: Understanding (Comprehension) REF: Page 392 TOP: NURSING PROCESS: Assessment/Planning MSC: NCLEX: Physiological Integrity: Pharmacological and Parenteral Therapies 19. A patient who has a history of migraine headaches is diagnosed with bipolar disorder. The nurse might expect the provider to order which medication for this patient? a. Carbamazapine (Tegretol) b. Divalproex (Valproate) c. Lamotrigine (Lamictal) d. Lithium citrate (Eskalith) ANS: B All of these medications may be used to treat bipolar disorder, but divalproex is also indicated for migraine prophylaxis. DIF: COGNITIVE LEVEL: Understanding (Comprehension) REF: Page 394 TOP: NURSING PROCESS: Planning MSC: NCLEX: Physiological Integrity: Pharmacological and Parenteral Therapies MULTIPLE RESPONSE 1. The nurse is teaching a patient about foods to avoid when taking isocarboxazid (Marplan). Which foods will the nurse instruct the patient to avoid? (Select all that apply.) a. Bananas b. Bread c. Eggs d. Red wine e. Sausage f. Yogurt ANS: A, D, E, F Aged cheeses and wines are the chief foods that are prohibited. Any food containing tyramine, which has sympathomimetic effects, can cause a hypertensive crisis. This includes bananas, sausage, and yogurt. PSYCH CH 16 Chapter 16: Bipolar Spectrum Disorders Test Bank MULTIPLE CHOICE 1. A person is directing traffic on a busy street and shouting, “To work, you jerk, for perks,” and making obscene gestures at cars. The person has not slept or eaten for 3 days. What features of mania are evident? a. Increased muscle tension and anxiety b. Vegetative signs and poor grooming c. Poor judgment and hyperactivity d. Cognitive deficit and sad mood ANS: C Hyperactivity (directing traffic) and poor judgment (putting self in a dangerous position) are characteristic of manic episodes. The distracters do not specifically apply to mania. DIF: Cognitive Level: Application REF: Pages: 281-282 TOP: Nursing Process: Assessment MSC: NCLEX: Psychosocial Integrity 2. A patient with bipolar disorder is dressed in a red leotard and brightly colored scarves. The patient says, “I’ll punch you, munch you, crunch you,” while twirling and shadowboxing. Then the patient says gaily, “Do you like my scarves? Here…they are my gift to you.” How should the nurse document the patient’s mood? a. Labile and euphoric b. Irritable and belligerent c. Highly suspicious and arrogant d. Excessively happy and confident ANS: A The patient has demonstrated angry behavior and pleasant, happy behavior within seconds of each other. Excessive happiness indicates euphoria. Mood swings are often rapid and seemingly without understandable reason in patients who are manic. These swings are documented as labile. Irritability, belligerence, excessive happiness, and confidence are not entirely correct terms for the patient’s mood. A high level of suspicion is not evident. DIF: Cognitive Level: Application REF: Pages: 279-282 TOP: Nursing Process: Assessment MSC: NCLEX: Psychosocial Integrity 3. A person is directing traffic on a busy street and rapidly shouting, “To work, you jerk, for perks,” and making obscene gestures at cars. The person has not slept or eaten for 3 days. Which assessment findings will have priority concern for this patient’s plan of care? a. Insulting, provocative behavior b. Pressured speech and grandiosity c. Hyperactivity; not eating and sleeping d. Poor concentration and decision making ANS: C Hyperactivity, poor nutrition, hydration, and not sleeping take priority in terms of the needs listed because they threaten the physical integrity of the patient. The other behaviors are less threatening to the patient’s life. DIF: Cognitive Level: Analysis REF: Pages: 283-285 TOP: Nursing Process: Planning MSC: NCLEX: Safe, Effective Care Environment 4. A patient with mania has not eaten or slept for 3 days. Which nursing diagnosis has priority? a. Risk for injury b. Ineffective coping c. Impaired social interaction d. Ineffective therapeutic regimen management ANS: A Although each of the nursing diagnoses listed is appropriate for a patient having a manic episode, the priority lies with the patient’s physiologic safety. Hyperactivity and poor judgment place the patient at risk for injury. DIF: Cognitive Level: Application REF: Pages: 283-285 TOP: Nursing Process: Diagnosis| Nursing Process: Analysis MSC: NCLEX: Safe, Effective Care Environment 5. A patient with bipolar disorder is hyperactive after discontinuing lithium and has not slept for 3 days. The patient threatens to hit another patient. Which comment by the nurse is appropriate? a. “Stop that! No one did anything to provoke an attack by you.” b. “If you do that one more time, you will be secluded immediately.” c. “Do not hit anyone. If you are unable to control yourself, we will help you.” d. “You know we will not let you hit anyone. Why do you continue this behavior?” ANS: C When the patient is unable to control his or her behavior and violates or threatens to violate the rights of others, limits must be set in an effort to de-escalate the situation. Limits should be set in simple, concrete terms. The first response does not offer appropriate assistance to the patient. The second response threatens the patient with seclusion as punishment. The fourth response asks a rhetorical question. DIF: Cognitive Level: Application REF: Pages: 286-289 TOP: Nursing Process: Implementation MSC: NCLEX: Safe, Effective Care Environment 6. This nursing diagnosis applies to a patient with mania: Imbalanced nutrition: less than body requirements related to insufficient caloric intake and hyperactivity as evidenced by 5-pound weight loss in 4 days. Select the most appropriate outcome. The patient will: a. ask staff for assistance with feeding within 4 days. b. drink six servings of a high-calorie, highprotein drink each day. c. consistently sit with others for at least 30 minutes at mealtime within 1 week. d. consistently wear appropriate attire for age and sex within 1 week while in the psychiatric unit. ANS: B 7. A patient with mania relapsed after discontinuing lithium. New orders are written to resume lithium twice daily and begin olanzapine (Zyprexa). The addition of olanzapine to the medication regimen will: a. minimize the side effects of lithium. b. bring hyperactivity under rapid control. c. enhance the antimanic actions of lithium. d. be used for long-term control of hyperactivity. ANS: B 8. A patient with bipolar disorder has rapid cycles. The health care provider prescribes an anticonvulsant medication. To prepare teaching materials, which drug should the nurse anticipate will be prescribed? a. phenytoin (Dilantin) b. clonidine (Catapres) c. carbamazepine (Tegretol) d. chlorpromazine (Thorazine) ANS: C Some patients with bipolar disorder, especially those who have only short periods between episodes, have a favorable response to the anticonvulsants carbamazepine and valproate. Phenytoin is also an anticonvulsant but is not used for mood stabilization. Carbamazepine seems to work better in patients with rapid cycling and in severely paranoid, angry patients with manic episodes. DIF: Cognitive Level: Application REF: Pages: 292-293 TOP: Nursing Process: Planning MSC: NCLEX: Physiological Integrity 9. The cause of bipolar disorder has not been determined, but: a. several factors, including genetics, are implicated. b. brain structures were altered by stresses early in life. c. excess norepinephrine is probably a major factor. d. excess sensitivity in dopamine receptors may exist. ANS: A At this time, the interplay of complex independent variables is most likely the best explanation of the cause for bipolar disorder. Various theories implicate genetics, endocrine imbalance, early stress, and neurotransmitter imbalances. DIF: Cognitive Level: Comprehension REF: Pages: 278-279 TOP: Nursing Process: Implementation MSC: NCLEX: Health Promotion and Maintenance 10. The spouse of a patient with bipolar disorder asks what evidence supports the possibility of genetic transmission of bipolar disorders. Select the nurse’s best response. a. “A high proportion of patients with bipolar disorders are found among creative writers.” b. “A higher rate of relatives with bipolar disorder is found among patients with bipolar disorder.” c. “Patients with bipolar disorder have higher rates of relatives who respond in an exaggerated way to daily stresses.” d. “More individuals with bipolar disorder come from high socioeconomic and educational backgrounds.” ANS: B Evidence of genetic transmission is supported when twins or relatives of patients with a particular disorder also show an incidence of the disorder that is higher than the incidence in the general public. The incorrect options do not support the theory of genetic transmission of bipolar disorder. DIF: Cognitive Level: Application REF: Page: 278 TOP: Nursing Process: Implementation MSC: NCLEX: Health Promotion and Maintenance 11. A patient with bipolar disorder commands other patients, “Get me a book. Take this stuff out of here,” and other similar demands. The nurse wants to interrupt this behavior without entering into a power struggle. Select the best initial approach by the nurse. a. Distraction: “Let’s go to the dining room for a snack.” b. Humor: “How much are you paying servants these days?” c. Limit setting: “You must stop ordering other patients around.” d. Honest feedback: “Your controlling behavior is annoying others.” ANS: A The distractibility characteristic of manic episodes can assist the nurse to direct the patient toward more appropriate, constructive activities without entering into a power struggle. Humor usually backfires by either encouraging the patient or inciting anger. Limit setting and honest feedback may seem heavy-handed to a labile patient and may incite anger. DIF: Cognitive Level: Application REF: Page: 287 TOP: Nursing Process: Implementation MSC: NCLEX: Psychosocial Integrity 12. A nurse receives this laboratory result: lithium level 1 mEq/L. This result is: a. within therapeutic limits b. below therapeutic limits c. above therapeutic limits d. incorrect because of inaccurate testing ANS: A The normal range for a blood sample taken 8 to 12 hours after the last dose of lithium is 0.4 to 1 mEq/L. DIF: Cognitive Level: Comprehension REF: Page: 291 TOP: Nursing Process: Assessment MSC: NCLEX: Physiological Integrity 13. Consider these three drugs: divalproex (Depakote), carbamazepine (Tegretol), and gabapentin (Neurontin). Which drug also belongs to this group? a. clonazepam (Klonopin) b. risperidone (Risperdal) c. lamotrigine (Lamictal) d. aripiprazole (Abilify) ANS: C The three drugs in the stem of this question are all anticonvulsants. Lamotrigine is also an anticonvulsant. Clonazepam is an anxiolytic; aripiprazole and risperidone are antipsychotic drugs. DIF: Cognitive Level: Analysis REF: Page: 294 TOP: Nursing Process: Implementation MSC: NCLEX: Physiological Integrity 14. When a hyperactive patient with acute mania is hospitalized, what initial nursing intervention is a priority? a. Allow the patient to act out his or her feelings. b. Set limits on patient behavior as necessary. c. Provide verbal instructions to the patient to remain calm. d. Restrain the patient to reduce hyperactivity and aggression. ANS: B This intervention provides support through the nurse’s presence and provides structure as necessary while the patient’s control is tenuous. Acting out may lead to the loss of behavioral control. The patient will probably be unable to focus on instructions and comply. Restraint is used only after other interventions have proved ineffective. DIF: Cognitive Level: Application REF: Pages: 286-289 TOP: Nursing Process: Planning MSC: NCLEX: Safe, Effective Care Environment 15. At a unit meeting, staff members discuss the decor for a special room for patients with mania. Select the best option. a. Extra-large window with a view of the street b. Neutral walls with pale, simple accessories c. Brightly colored walls and print drapes d. Deep colors for walls and upholstery ANS: B The environment for a patient with mania should be as simple and nonstimulating as possible. Patients with mania are highly sensitive to environmental distractions and stimulation. DIF: Cognitive Level: Application REF: Pages: 285-287 TOP: Nursing Process: Implementation MSC: NCLEX: Psychosocial Integrity 16. A patient with acute mania has exhausted the staff members by noon. The patient has joked, manipulated, insulted, and fought all morning. Staff members are feeling defensive and fatigued. Which is the best action? a. Confer with the health care provider to consider the use of seclusion for this patient. b. Hold a staff meeting to discuss consistency and limit-setting approaches. c. Conduct a meeting with all patients to discuss the behavior. d. Explain to the patient that the behavior is unacceptable. ANS: B When staff members are at their wits’ end, the patient has succeeded in keeping the environment unsettled and avoided outside controls on behavior. Staff meetings can help minimize staff splitting and feelings of anger, helplessness, confusion, and frustration. DIF: Cognitive Level: Application REF: Pages: 286-287 TOP: Nursing Process: Implementation MSC: NCLEX: Safe, Effective Care Environment 17. A patient with acute mania undresses in the group room and dances. The nurse’s first intervention would be to: a. quietly ask the patient, “Why don’t you put on your clothes?” b. firmly tell the patient, “Stop dancing, and put on your clothing.” c. put a blanket around the patient, and walk with the patient to a quiet room. d. allow the patient stay in the group room. Move the other patients to a different area. ANS: C Patients must be protected from the embarrassing consequences of their poor judgment whenever possible. Protecting the patient from public exposure by matter-of-factly covering the patient and removing him or her from the area with a sufficient number of staff members to avoid argument and provide control is an effective approach. DIF: Cognitive Level: Application REF: Page: 287|Page: 290 TOP: Nursing Process: Implementation MSC: NCLEX: Physiological Integrity 18. A patient with acute mania waves a newspaper and says, “I must have my credit card and use the computer right now. A store is having a big sale and I need to order 10 dresses and four pairs of shoes.” Select the nurse’s most appropriate intervention. a. Suggest to the patient to have a friend do the shopping and bring purchases to the unit. b. Invite the patient to sit with the nurse and look at new fashion magazines. c. Tell the patient that computer use is not allowed until self-control improves. d. Ask whether the patient has enough money to pay for the purchases. ANS: B Situations such as this offer an opportunity to use the patient’s distractibility to the staff’s advantage. Patients become frustrated when staff members deny requests that the patient sees as entirely reasonable. Distracting the patient can avoid power struggles. Suggesting that a friend do the shopping would not satisfy the patient’s need for immediacy and would ultimately result in the extravagant expenditure. Asking whether the patient has enough money would likely precipitate an angry response. DIF: Cognitive Level: Application REF: Page: 287 TOP: Nursing Process: Implementation MSC: NCLEX: Psychosocial Integrity 19. A patient with bipolar disorder, who is being treated on an outpatient basis, takes lithium carbonate 300 mg three times daily. The patient complains of nausea. To reduce the nausea, the nurse can suggest that the lithium be taken with: a. meals b. an antacid c. an antiemetic medication d. a large glass of juice ANS: A Some patients find that taking lithium with meals diminishes nausea. The incorrect options are less helpful. DIF: Cognitive Level: Application REF: Page: 292 TOP: Nursing Process: Implementation MSC: NCLEX: Physiological Integrity 20. A health teaching plan for a patient taking lithium should include instructions to: a. Maintain normal salt and fluids in the diet. b. Drink twice the usual daily amount of fluids. c. Double the lithium dose if diarrhea or vomiting occurs. d. Avoid eating aged cheese, processed meats, and red wine. ANS: A Sodium depletion and dehydration increase the chance for developing lithium toxicity. The other options offer inappropriate information. DIF: Cognitive Level: Application REF: Page: 292 TOP: Nursing Process: Planning MSC: NCLEX: Physiological Integrity 21. Which nursing diagnosis would most likely apply to both a patient with depression and one with acute mania? a. Deficient diversional activity b. Disturbed sleep pattern c. Fluid volume excess d. Defensive coping ANS: B 22. Which dinner menu is best suited for the patient with bipolar disorder experiencing acute mania? a. Spaghetti and meatballs, salad, and a banana b. Beef and vegetable stew, a roll, and chocolate pudding c. Broiled chicken breast on a roll, an ear of corn, and an apple d. Chicken casserole, green beans, and flavored gelatin with whipped cream ANS: C 23. Outcome identification for the treatment plan of a patient with grandiose thinking associated with acute mania focuses on: a. developing an optimistic outlook b. self-control of distorted thinking c. maintaining an interest in the environment d. stabilizing the sleep pattern ANS: B 24. Which documentation indicates that the treatment plan for a patient with acute mania has been effective? a. “Converses without interrupting, clothing matches, participates in activities.” b. “Irritable, suggestible, distractible, napped for 10 minutes in afternoon.” c. “Attention span short, writing copious notes, intrudes in conversations.” d. “Heavy makeup, seductive toward staff, pressured speech.” ANS: A The descriptors given indicate the patient is functioning at an optimal level, using appropriate behavior, and thinking without becoming overstimulated by unit activities. The incorrect options reflect manic behavior. DIF: Cognitive Level: Application REF: Page: 296 TOP: Nursing Process: Evaluation MSC: NCLEX: Psychosocial Integrity 25. A patient with mania dances around the unit, seldom sits, monopolizes conversations, interrupts, and intrudes. Which nursing intervention will best assist the patient with energy conservation? a. Monitor physiologic functioning. b. Provide a subdued environment. c. Supervise personal hygiene. d. Observe for mood changes. ANS: B All the options are reasonable interventions with a patient with acute mania, but providing a subdued environment directly relates to the outcome of energy conservation by decreasing stimulation and helping balance activity and rest. DIF: Cognitive Level: Application REF: Pages: 288-290 TOP: Nursing Process: Planning MSC: NCLEX: Physiological Integrity 26. A patient with bipolar disorder was hospitalized 7 days earlier and has been taking lithium 600 mg three times daily. Staff members observe increased agitation, pressured speech, poor personal hygiene, hyperactivity, and bizarre clothing. What is the nurse’s best intervention? a. Educate the patient about the proper ways to perform personal hygiene and coordinate clothing. b. Continue to monitor and document the patient’s speech patterns and motor activity. c. Ask the health care provider to prescribe an increased dose and frequency of lithium. d. Consider the need to check the lithium level. The patient may not be swallowing medications. ANS: D 27. A patient with acute mania has disrobed in the hall three times in 2 hours. The nurse should: a. direct the patient to wear clothes at all times. b. ask if the patient finds clothes bothersome. c. tell the patient that others feel embarrassed. d. arrange for one-on-one supervision. ANS: D 28. A patient with acute mania is dancing atop the pool table in the recreation room. The patient waves a cue in one hand and says, “I’ll throw the pool balls if anyone comes near me.” The nurse’s first intervention is to: a. tell the patient, “You need to be secluded.” b. clear the room of all other patients. c. help the patient down from the table. d. assemble a show of force. ANS: B Safety is of primary importance. Once other patients are out of the room, a plan for managing this patient can be implemented. DIF: Cognitive Level: Analysis REF: Pages: 285-287 TOP: Nursing Process: Planning MSC: NCLEX: Safe, Effective Care Environment 29. After hospital discharge, what is the priority intervention for a patient with bipolar disorder, who is taking antimanic medication, and for the patient’s family? a. Decreasing physical activity b. Increasing food and fluids c. Meeting self-care needs d. Psychoeducation ANS: D During the continuation phase of treatment for bipolar disorder, the physical needs of the patient are not as important an issue as they were during the acute episode. After hospital discharge, the treatment focuses on maintaining medication compliance and preventing a relapse, both of which are fostered by ongoing psychoeducation. DIF: Cognitive Level: Application REF: Pages: 293-296 TOP: Nursing Process: Planning MSC: NCLEX: Psychosocial Integrity 30. A patient receiving lithium should be assessed for which evidence of complications? a. Pharyngitis, mydriasis, and dystonia b. Alopecia, purpura, and drowsiness c. Diaphoresis, weakness, and nausea d. Ascites, dyspnea, and edema ANS: C Diaphoresis, weakness, and nausea are early signs of lithium toxicity. Problems mentioned in the incorrect options are unrelated to lithium therapy. DIF: Cognitive Level: Analysis REF: Pages: 290-291 TOP: Nursing Process: Assessment MSC: NCLEX: Physiological Integrity 31. A patient with bipolar disorder is in the maintenance phase of treatment. The patient asks, “Do I have to keep taking this lithium even though my mood is stable now?” Select the nurse’s most appropriate response. a. “You will be able to stop the medication in approximately 1 month.” b. “Taking the medication every day helps prevent relapses and recurrences.” c. “Usually patients take this medication for approximately 6 months after discharge.” d. “It’s unusual that the health care provider hasn’t already stopped your medication.” ANS: B 32. A patient with bipolar disorder and who takes lithium telephones the nurse at the clinic to say, “I’ve had severe diarrhea for 4 days. I feel very weak and unsteady when I walk. My usual hand tremor has gotten worse. What should I do?” The nurse should advise the patient: a. “Restrict food and fluids for 24 hours, and stay in bed.” b. “Have someone bring you to the clinic immediately.” c. “Drink a large glass of water with 1 teaspoon of salt added.” d. “Take an over-the-counter antidiarrheal medication hourly until the diarrhea subsides.” ANS: B The symptoms described suggest lithium toxicity. The patient should have a lithium level drawn and may require further treatment. Because neurologic symptoms are present, the patient should not drive and should be accompanied by another person. The incorrect options will not ameliorate the patient’s symptoms. DIF: Cognitive Level: Application REF: Pages: 290-291 TOP: Nursing Process: Implementation MSC: NCLEX: Physiological Integrity 33. Lithium is prescribed for a new patient. Which information from the patient’s history indicates that monitoring serum concentrations of the drug will be challenging and critical? a. Arthritis b. Epilepsy c. Psoriasis d. Congestive heart failure ANS: D The patient with congestive heart failure will likely need diuretic drugs, which will complicate the maintenance of the fluid balance necessary to avoid lithium toxicity. DIF: Cognitive Level: Analysis REF: Pages: 291-293 TOP: Nursing Process: Assessment MSC: NCLEX: Physiological Integrity MULTIPLE RESPONSE 1. Which suggestions are appropriate for the family of a patient with bipolar disorder who is being treated as an outpatient during a hypomanic episode? Select all that apply. a. Provide structure. b. Limit credit card access. c. Encourage group-social interaction. d. Suggest limiting work to half days. e. Monitor the patient’s sleep patterns. ANS: A, B, E 2. A nurse prepares the plan of care for a patient having a manic episode. Which nursing diagnoses are most likely? Select all that apply. a. Imbalanced nutrition: more than body requirements b. Disturbed thought processes c. Sleep deprivation d. Chronic confusion e. Social isolation ANS: B, C 3. A patient tells the nurse, “I am so ashamed of being bipolar. When I’m manic, my behavior embarrasses my family. Even if I take my medication, there’s no guarantee I won’t have a relapse. I am such a burden to my family.” These statements support which nursing diagnoses? Select all that apply. a. Powerlessness b. Defensive coping c. Chronic low self-esteem d. Impaired social interaction e. Risk-prone health behavior ANS: A, C Chronic low self-esteem and powerlessness are interwoven in the patient’s statements. No data support the other diagnoses. DIF: Cognitive Level: Analysis REF: Pages: 283-285 TOP: Nursing Process: Diagnosis| Nursing Process: Analysis MSC: NCLEX: Psychosocial Integrity PSYCH CH 17 .Chapter 17: Schizophrenia Disorders and Other Psychotic Disorders Test Bank MULTIPLE CHOICE 1. A person has had difficulty keeping a job because of arguing with coworkers and accusing them of conspiracy. Today the person shouts, “They’re all plotting to destroy me. Isn’t that true?” Select the nurse’s most therapeutic response. a. “Everyone here is trying to help you. No one wants to harm you.” b. “Feeling that people want to destroy you must be very frightening.” c. “No, that is not true. People here are trying to help you if you will let them.” d. “Staff members are health care professionals who are qualified to help you.” ANS: B 2. A newly admitted patient diagnosed with paranoid schizophrenia is hypervigilant and constantly scans the environment. The patient states, “I saw two doctors talking in the hall. They were plotting to kill me.” The nurse may correctly assess this behavior as: a. echolalia b. idea of reference c. delusion of infidelity d. auditory hallucination ANS: B Ideas of reference are misinterpretations of the verbalizations or actions of others that give special personal meanings to these behaviors; for example, when seeing two people talking, the individual assumes they are talking about him or her. The other terms do not correspond with the scenario. DIF: Cognitive Level: Comprehension REF: Page: 306 TOP: Nursing Process: Assessment MSC: NCLEX: Psychosocial Integrity 3. A patient diagnosed with paranoid schizophrenia says, “My co-workers are out to get me. I also saw two doctors plotting to kill me.” How does this patient perceive the environment? a. Disorganized b. Dangerous c. Supportive d. Bizarre ANS: B The patient sees the world as hostile and dangerous. This assessment is important because the nurse can be more effective by using empathy to respond to the patient. Data are not present to support any of the other options. DIF: Cognitive Level: Comprehension REF: Page: 309|Page: 316 TOP: Nursing Process: Assessment MSC: NCLEX: Psychosocial Integrity 4. When a patient with paranoid schizophrenia was discharged 6 months ago, haloperidol (Haldol) was prescribed. The patient now says, “I stopped taking those pills. They made me feel like a robot.” What common side effects should the nurse validate with the patient? a. Sedation and muscle stiffness b. Sweating, nausea, and diarrhea c. Mild fever, sore throat, and skin rash d. Headache, watery eyes, and runny nose ANS: A Typical antipsychotic drugs often produce sedation and extrapyramidal side effects such as stiffness and gait disturbance, effects the patient might describe as making him or her feel like a “robot.” The side effects mentioned in the other options are usually not associated with typical antipsychotic therapy or would not have the effect described by the patient. DIF: Cognitive Level: Application REF: Pages: 320-325 TOP: Nursing Process: Assessment MSC: NCLEX: Physiological Integrity 5. A nurse works with a patient with paranoid schizophrenia regarding the importance of medication management. The patient repeatedly says, “I don’t like taking pills.” Family members say they feel helpless to foster compliance. Which treatment strategy should the nurse discuss with the health care provider? a. Use of a long-acting antipsychotic preparation b. Addition of a benzodiazepine, such as lorazepam (Ativan) c. Adjunctive use of an antidepressant, such as amitriptyline (Elavil) d. Prolonged hospitalization; this patient is not ready for discharge ANS: A Medications such as fluphenazine decanoate and haloperidol decanoate are long-acting forms of antipsychotic medications. They are administered by depot injection every 2 to 4 weeks, thus reducing daily opportunities for noncompliance. The other options do not address the patient’s dislike of taking pills. DIF: Cognitive Level: Application REF: Pages: 320-325 TOP: Nursing Process: Planning MSC: NCLEX: Physiological Integrity 6. A patient’s care plan includes monitoring for auditory hallucinations. Which assessment findings suggest the patient may be hallucinating? a. Aloofness, haughtiness, suspicion b. Darting eyes, tilted head, mumbling to self c. Elevated mood, hyperactivity, distractibility d. Performing rituals, avoiding open places ANS: B Clues to hallucinations include looking around the room as though to find the speaker; tilting the head to one side as though intently listening; and grimacing, mumbling, or talking aloud as though responding conversationally to someone. DIF: Cognitive Level: Application REF: Pages: 307-308 TOP: Nursing Process: Assessment MSC: NCLEX: Psychosocial Integrity 7. A health care provider considers which antipsychotic medication to prescribe for a patient with schizophrenia who has auditory hallucinations and poor social functioning. The patient is also overweight and has hypertension. Which drug should the nurse advocate? a. clozapine (Clozaril) b. ziprasidone (Geodon) c. olanzapine (Zyprexa) d. aripiprazole (Abilify) ANS: D TOP: Nursing Process: Planning MSC: NCLEX: Physiological Integrity 8. A patient with schizophrenia tells the nurse, “I eat skiller. Tend to end. Easter. It blows away. Get it?” Select the nurse’s best response. a. “Nothing you are saying is clear.” b. “Your thoughts are very disconnected.” c. “Try to organize your thoughts, and then tell me again.” d. “I am having difficulty understanding what you are saying.” ANS: D When a patient’s speech is loosely associated, confused, and disorganized, pretending to understand is useless. The nurse should tell the patient that he or she is having difficulty understanding what the patient is saying. If a theme is discernible, ask the patient to talk about the theme. The incorrect options tend to place blame for the poor communication with the patient. The correct response places the difficulty with the nurse rather than being accusatory. DIF: Cognitive Level: Application REF: Pages: 306-307 TOP: Nursing Process: Implementation MSC: NCLEX: Psychosocial Integrity 9. A patient with catatonic schizophrenia exhibits little spontaneous movement and demonstrates waxy flexibility. Which patient needs are of priority importance? a. Psychosocial b. Physiologic c. Self-actualization d. Safety and security ANS: B 10. A patient with catatonic schizophrenia is semistuporous, demonstrates little spontaneous movement, and has waxy flexibility. The patient’s activities of daily living are severely compromised. An appropriate outcome is that the patient will: a. demonstrate increased interest in the environment by the end of week 1. b. perform self-care activities with coaching by the end of day 3. c. gradually take the initiative for self-care by the end of week 2. d. accept tube feeding without objection by day 2. . ANS: B 11. A nurse observes a patient who is in a catatonic state and standing immobile, facing the wall with one arm extended in a salute. The patient remains immobile in this position for 15 minutes, moving only when the nurse gently lowers the arm. What is the name of this phenomenon? a. Echolalia b. Waxy flexibility c. Depersonalization d. Thought withdrawal ANS: B 12. Which patient with schizophrenia would be expected to have the lowest score in global assessment of functioning? a. 39 years old; paranoid ideation since age 35 years b. 32 years old; diagnosed as catatonic at age 24 years; stable for 3 years c. 19 years old; diagnosed with undifferentiated schizophrenia at age 17 d. 40 years old; disorganized schizophrenia since age 18; frequent relapses ANS: D 13. A patient with delusions of persecution about being poisoned has refused all hospital meals for 3 days. Which intervention is most likely to be acceptable to the patient? a. Allowing the patient to have supervised access to food vending machines b. Allowing the patient to telephone a local restaurant to deliver meals c. Offering to taste each portion on the tray for the patient d. Providing tube feedings or total parenteral nutrition ANS: A 14. A community mental health nurse wants to establish a relationship with a very withdrawn patient diagnosed with schizophrenia. The patient lives at home with a supportive family. Select the nurse’s best plan. a. Visit daily for 4 days, then visit every other day for 1 week; stay with the patient for 20 minutes; accept silence; state when the nurse will return. b. Arrange to spend 1 hour each day with the patient; focus on asking questions about what the patient is thinking or experiencing; avoid silences. c. Visit twice daily; sit beside the patient with a hand on the patient’s arm; leave if the patient does not respond within 10 minutes. d. Visit every other day; remind the patient of the nurse’s identity; encourage the patient to talk while the nurse works on the nurse will return. ANS: A 15. Withdrawn patients with schizophrenia: a. Universally fear sexual involvement with therapists. b. Are socially disabled by the positive symptoms of schizophrenia. c. Exhibit a high degree of hostility as evidenced by rejecting behavior. d. Avoid relationships because they become anxious with emotional closeness. ANS: D TOP: Nursing Process: Evaluation MSC: NCLEX: Psychosocial Integrity 16. A newly admitted patient with schizophrenia says, “The voices are bothering me. They yell and tell me I’m bad. I have got to get away from them.” Select the nurse’s most helpful reply. a. “Do you hear the voices often?” b. “Do you have a plan for getting away from the voices?” c. “I’ll stay with you. Focus on what we are talking about, not the voices.” d. “Forget the voices, and ask some other patients to play cards with you.” ANS: C 17. A patient with schizophrenia has taken fluphenazine (Prolixin) 5 mg orally twice daily for 3 weeks. The nurse now assesses a shuffling propulsive gait, a masklike face, and drooling. Which term applies to these symptoms? a. Neuroleptic malignant syndrome b. Hepatocellular effects c. Pseudoparkinsonism d. Akathisia ANS: C Pseudoparkinsonism induced by antipsychotic medication mimics the symptoms of Parkinson disease. It frequently appears within the first month of treatment. Hepatocellular effects would produce abnormal liver test results. Neuroleptic malignant syndrome is characterized by autonomic instability. Akathisia produces motor restlessness. DIF: Cognitive Level: Application REF: Pages: 325-326 TOP: Nursing Process: Assessment MSC: NCLEX: Physiological Integrity 18. A patient with schizophrenia is acutely disturbed and violent. After several doses of haloperidol (Haldol), the patient is calm. Two hours later the nurse sees the patient’s head rotated to one side in a stiff position; the lower jaw is thrust forward, and the patient is drooling. Which problem is most likely? a. Acute dystonic reaction b. Tardive dyskinesia c. Waxy flexibility d. Akathisia ANS: A 19. An acutely violent patient with schizophrenia receives several doses of haloperidol (Haldol). Two hours later the nurse notices the patient’s head rotated to one side in a stiffly fixed position; the lower jaw is thrust forward, and the patient is drooling. Which intervention by the nurse is indicated? a. Administer diphenhydramine (Benadryl) 50 mg IM from the PRN medication administration record. b. Reassure the patient that the symptoms will subside. Practice relaxation exercises with the patient. c. Give trihexyphenidyl (Artane) 5 mg orally at the next regularly scheduled medication administration time. d. Administer atropine sulfate 2 mg subcut from the PRN medication administration record. ANS: A 20. A patient has taken trifluoperazine (Stelazine) 30 mg/day orally for 3 years. The clinic nurse notes that the patient grimaces and constantly smacks both lips. The patient’s neck and shoulders twist in a slow, snakelike motion. Which problem would the nurse suspect? a. Agranulocytosis b. Tardive dyskinesia c. Tourette syndrome d. Anticholinergic effects ANS: B Tardive dyskinesia is a neuroleptic-induced condition involving the face, trunk, and TOP: Nursing Process: Evaluation MSC: NCLEX: Physiological Integrity 21. A nurse sits with a patient diagnosed with disorganized schizophrenia. The patient starts to laugh uncontrollably, although the nurse has not said anything funny. Select the nurse’s best response. a. “Why are you laughing?” b. “Please share the joke with me.” c. “I don’t think I said anything funny.” d. “You’re laughing. Tell me what’s happening.” ANS: D 22. Which symptoms are expected for a patient with disorganized schizophrenia? a. Extremes of motor activity, from excitement to stupor b. Social withdrawal and ineffective communication c. Severe anxiety with ritualistic behavior d. Highly suspicious, delusional behavior ANS: B 23. What assessment findings mark the prodromal stage of schizophrenia? a. Withdrawal, misinterpreting, poor concentration, and preoccupation with religion b. Auditory hallucinations, ideas of reference, thought insertion, and broadcasting c. Stereotyped behavior, echopraxia, echolalia, and waxy flexibility d. Loose associations, concrete thinking, and echolalia neologisms ANS: A 24. A patient with schizophrenia says, “Contagious bacteria are everywhere. When they get in your body, you will be locked up with other infected people.” Which problem is evident? a. Poverty of content b. Concrete thinking c. Neologisms d. Paranoia ANS: D The patient’s unrealistic fear of harm indicates paranoia. Neologisms are invented words. Concrete thinking involves literal interpretation. Poverty of content refers to an inadequate fund of information. DIF: Cognitive Level: Application REF: Page: 309 TOP: Nursing Process: Assessment MSC: NCLEX: Physiological Integrity 25. A patient diagnosed with paranoid schizophrenia angrily tells a nurse, “You act like a homosexual. No one trusts you or wants to be around you.” Select the most likely analysis. The patient: a. is showing reaction formation in response to feelings of abandonment. b. is unleashing unconscious, hostile feelings toward the nurse. c. is dwelling on others’ shortcomings, thus placing them on the defensive. d. may be projecting homosexual urges. ANS: D 26. A patient diagnosed with disorganized schizophrenia says, “It’s beat. Time to eat. No room for the cat.” What type of verbalization is evident? a. Neologism b. Idea of reference c. Thought broadcasting d. Associative looseness ANS: D 27. A patient with schizophrenia has taken a conventional antipsychotic medication for a year. Hallucinations are less intrusive, but the patient continues to have apathy, poverty of thought, and social isolation. The nurse expects a change to which medication? a. haloperidol (Haldol) b. olanzapine (Zyprexa) c. chlorpromazine (Thorazine) d. diphenhydramine (Benadryl) ANS: B Olanzapine is an atypical antipsychotic medication that targets both positive and negative symptoms of schizophrenia. Haloperidol and chlorpromazine are conventional antipsychotic agents that target only positive symptoms. Diphenhydramine is an antihistamine. DIF: Cognitive Level: Analysis REF: Page: 320 TOP: Nursing Process: Planning MSC: NCLEX: Physiological Integrity 28. The family of a patient with undifferentiated schizophrenia is unfamiliar with the illness and the family’s role in recovery. Which type of therapy should the nurse recommend? a. Psychoeducational b. Psychoanalytic c. Transactional d. Family ANS: A 29. A patient with schizophrenia has been stable for a year; however, the family now reports the patient is tense, sleeps 3 to 4 hours per night, and has difficulty concentrating. The patient says, “Volmers are hiding in the house.” The nurse can correctly assess this information as an indication of: a. need for psychoeducation b. medication noncompliance c. chronic deterioration d. relapse ANS: D Signs of potential relapse include feeling tense, difficulty concentrating, trouble sleeping, increased withdrawal, and increased bizarre or magical thinking. Medication noncompliance may not be implicated. Relapse can occur even when the patient is regularly taking his or her medication. Psychoeducation is more effective when the patient’s symptoms are stable. Chronic deterioration is not the best explanation. DIF: Cognitive Level: Analysis REF: Page: 319 TOP: Nursing Process: Assessment MSC: NCLEX: Psychosocial Integrity 30. A patient with schizophrenia begins to talks about “volmers” hiding in the warehouse at work. The term “volmers” should be documented as: a. neologism b. concrete thinking c. thought insertion d. idea of reference ANS: A A neologism is a newly coined word having special meaning to the patient. “Volmer” is not a known common noun. Concrete thinking refers to the inability to think abstractly. Thought insertion refers to thoughts of others that are implanted in one’s mind. An idea of reference is a type of delusion in which trivial events are given personal significance. DIF: Cognitive Level: Comprehension REF: Pages: 306-307 TOP: Nursing Process: Assessment MSC: NCLEX: Psychosocial Integrity 31. A patient with schizophrenia anxiously says, “I can see the left side of my body merging with the wall, then my face appears and disappears in the mirror.” While listening, the nurse should: a. sit close to the patient. b. place an arm protectively around the patient’s shoulders. c. place a hand on the patient’s arm and exert light pressure. d. maintain a normal social interaction distance from the patient. ANS: D The patient is describing phenomena that indicate personal boundary difficulties. The nurse should maintain an appropriate social distance and not touch the patient, because the patient is anxious about the inability to maintain ego boundaries and merging with or being swallowed by the environment. Physical closeness or touch could precipitate panic. DIF: Cognitive Level: Application REF: Pages: 307-308 TOP: Nursing Process: Implementation MSC: NCLEX: Psychosocial Integrity 32. A patient with schizophrenia and auditory hallucinations anxiously tells the nurse, “The voice is telling me to do things.” Select the nurse’s priority assessment question. a. “How long has the voice been directing your behavior?” b. “Does what the voice tells you to do frighten you?” c. “Do you recognize the voice speaking to you?” d. “What is the voice telling you to do?” ANS: D TOP: Nursing Process: Assessment MSC: NCLEX: Safe, Effective Care Environment 33. A patient receiving risperidone (Risperdal) reports severe muscle stiffness at 10:30 AM. By noon, the patient has difficulty swallowing and is drooling. By 4:00 PM, vital signs are body temperature, 102.8° F; pulse, 110 beats per minute; respirations, 26 breaths per minute; and blood pressure, 150/90 mm Hg. The patient is diaphoretic. Select the nurse’s best analysis and action. a. Agranulocytosis. Institute reverse isolation. b. Tardive dyskinesia. Withhold the next dose of medication. c. Cholestatic jaundice. Begin a highprotein, high-cholesterol diet. d. Neuroleptic malignant syndrome. Immediately notify the health care provider. ANS: D TOP: Nursing Process: Implementation MSC: NCLEX: Physiological Integrity 34. A patient with schizophrenia begins a new prescription for lurasidone HCl (Latuda). The patient is 5’, 6” tall and currently weighs 204 pounds. Which topic is most important for the nurse to include in the teaching plan related to this medication? a. How to recognize tardive dyskinesia b. Weight management strategies c. Ways to manage constipation d. Sleep hygiene measures ANS: B Lurasidone HCl (Latuda) is an atypical antipsychotic medication. The incidence of weight gain, diabetes, and high cholesterol is high with this medication. The patient is overweight now, so weight management is especially important. The incidence of tardive dyskinesia is low with atypical antipsychotic medications. Constipation may occur, but it is less important than weight management. This drug usually produces drowsiness. DIF: Cognitive Level: Analysis REF: Pages: 320-323 TOP: Nursing Process: Planning MSC: NCLEX: Physiological Integrity 35. A patient with schizophrenia has auditory hallucinations, delusions of grandeur, poor personal hygiene, and motor agitation. Which assessment finding would the nurse regard as a negative symptom of schizophrenia? a. Auditory hallucinations b. Delusions of grandeur c. Poor personal hygiene d. Motor agitation ANS: C Negative symptoms include apathy, anhedonia, poor social functioning, and poverty of thought. Poor personal hygiene is an example of poor social functioning. The distracters are positive symptoms of schizophrenia. DIF: Cognitive Level: Analysis REF: Pages: 304-309 TOP: Nursing Process: Assessment MSC: NCLEX: Psychosocial Integrity MULTIPLE RESPONSE 1. The family members of a patient newly diagnosed with paranoid schizophrenia state that they do not understand what has caused the illness. The nurse’s response should be based on which models? Select all that apply. a. Neurobiological b. Developmental c. Family theory d. Genetic e. Stress ANS: A, D Compelling evidence exists that schizophrenia is a neurologic disorder probably related to neurochemical abnormalities, neuroanatomical disruption of brain circuits, and genetic vulnerability. Stress, family disruption, and developmental influences may contribute but are not considered single etiologic factors. DIF: Cognitive Level: Comprehension REF: Pages: 301-309 TOP: Nursing Process: Implementation MSC: NCLEX: Physiological Integrity 2. A nurse at the mental health clinic plans a series of psychoeducational groups for persons with schizophrenia. Which two topics would take priority? a. How to complete an application for employment b. The importance of correctly taking your medication c. How to dress when attending community events d. How to give and receive compliments e. Ways to quit smoking ANS: B, E Stabilization is maximized by the adherence to the antipsychotic medication regimen. Because so many patients with schizophrenia smoke cigarettes, this topic relates directly to the patients’ physiologic well-being. The other topics are also important but are not priority topics. DIF: Cognitive Level: Application REF: Page: 300|Pages: 319-320 TOP: Nursing Process: Planning| Nursing Process: Outcomes Identification MSC: NCLEX: Health Promotion and Maintenance 3. A patient with paranoid schizophrenia is hospitalized after arguing with co-workers and threatening to harm them. The patient is aloof and suspicious and says, “Two staff members I saw talking were plotting to kill me.” Based on data gathered at this point, which nursing diagnoses relate? Select all that apply. a. Risk for other-directed violence b. Disturbed thought processes c. Risk for loneliness d. Spiritual distress e. Social isolation ANS: A, B Delusions of persecution and ideas of reference support the nursing diagnosis of Disturbed thought processes. Risk for other-directed violence is substantiated by the patient’s feeling endangered by persecutors. Fearful individuals may strike out at perceived persecutors or attempt self-harm to get away from persecutors. Data are not present to support the other diagnoses. DIF: Cognitive Level: Application REF: Pages: 311-312 TOP: Nursing Process: Diagnosis| Nursing Process: Analysis MSC: NCLEX: Psychosocial Integrity PSYCH CH 19 .Chapter 19: Substance-Related and Addictive Disorders Test Bank MULTIPLE CHOICE 1. An alcohol-dependent patient was hospitalized at 0200 today. When would the nurse expect withdrawal symptoms to peak? a. Between 0800 and 1000 today (6 to 8 hours after drinking stopped) b. Between 0200 tomorrow and hospital day 2 (24 to 48 hours after drinking stopped) c. About 0200 on hospital day 3 (72 hours after drinking stopped) d. About 0200 on hospital day 4 (96 hours after drinking stopped) ANS: B Alcohol withdrawal usually begins 6 to 8 hours after cessation or significant reduction of alcohol intake. It peaks between 24 and 48 hours, then resolves or progresses to delirium. DIF: Cognitive Level: Application REF: Pages: 336-367 TOP: Nursing Process: Assessment MSC: NCLEX: Physiological Integrity 2. A woman in the last trimester of pregnancy drinks 8 to 12 ounces of alcohol daily. The nurse plans for the delivery of an infant who is: a. jaundiced b. dependent on alcohol c. healthy but underweight d. microcephalic and cognitively impaired ANS: D Fetal alcohol syndrome is the result of alcohol’s inhibiting fetal development in the first trimester. The fetus of a woman who drinks that much alcohol will probably have this disorder. Alcohol use during pregnancy is not likely to produce the findings listed in the distracters. DIF: Cognitive Level: Application REF: Page: 363 TOP: Nursing Process: Assessment MSC: NCLEX: Health Promotion and Maintenance 3. A patient was admitted last night with a hip fracture sustained in a fall while intoxicated. The patient points to the Buck traction and screams, “Somebody tied me up with ropes.” The patient is experiencing: a. illusion b. delusion c. hallucinations d. hypnagogic phenomenon ANS: A The patient is misinterpreting a sensory perception when seeing a noose instead of traction. Illusions are common in early withdrawal from alcohol. A delusion is a fixed, false belief. Hallucinations are sensory perceptions occurring in the absence of a stimulus. Hypnagogic phenomena are sensory disturbances that occur between waking and sleeping. DIF: Cognitive Level: Comprehension REF: Page: 367 TOP: Nursing Process: Assessment MSC: NCLEX: Physiological Integrity 4. A patient was admitted 48 hours ago for injuries sustained while intoxicated. The patient is shaky, irritable, anxious, and diaphoretic. The pulse rate is 130 beats per minute. The patient shouts, “Bugs are crawling on my bed. I’ve got to get out of here.” What is the most accurate assessment of the situation? The patient: a. is attempting to obtain attention by manipulating staff. b. may have sustained a head injury before admission. c. has symptoms of alcohol withdrawal delirium. d. is having a recurrence of an acute psychosis. ANS: C Symptoms of agitation, elevated pulse, and perceptual distortions point to alcohol withdrawal delirium, a medical emergency. The findings are inconsistent with manipulative attempts, head injury, or functional psychosis. DIF: Cognitive Level: Application REF: Pages: 366-368 TOP: Nursing Process: Assessment MSC: NCLEX: Physiological Integrity 5. A patient admitted yesterday for injuries sustained in a fall while intoxicated believes bugs are crawling on the bed. The patient is anxious, agitated, and diaphoretic. What is the priority nursing diagnosis? a. Ineffective airway clearance b. Ineffective coping c. Ineffective denial d. Risk for injury ANS: D The clouded sensorium, sensory perceptual distortions, and poor judgment increase the risk for injury. Safety is the nurse’s priority. The scenario does not provide data to support the other diagnoses. DIF: Cognitive Level: Analysis REF: Pages: 372-373 TOP: Nursing Process: Diagnosis| Nursing Process: Analysis MSC: NCLEX: Physiological Integrity 6. A patient admitted yesterday for injuries sustained while intoxicated believes the window blinds are snakes trying to get into the room. The patient is anxious, agitated, and diaphoretic. Which medication can the nurse anticipate the health care provider will prescribe? a. Monoamine oxidase inhibitor, such as phenelzine (Nardil) b. Phenothiazine, such as thioridazine (Mellaril) c. Benzodiazepine, such as lorazepam (Ativan) d. Narcotic analgesic, such as morphine ANS: C Sedation allows for the safe withdrawal from alcohol. Benzodiazepines are the drugs of choice in most regions because of their high therapeutic safety index and anticonvulsant properties. DIF: Cognitive Level: Analysis REF: Pages: 379-380 TOP: Nursing Process: Planning MSC: NCLEX: Physiological Integrity 7. A hospitalized patient, injured in a fall while intoxicated, believes spiders are spinning entrapping webs in the room. The patient is anxious, agitated, and diaphoretic. Which nursing intervention has priority? a. Check the patient every 15 minutes. b. Rigorously encourage fluid intake. c. Provide one-on-one supervision. d. Keep the room dimly lit. ANS: C One-on-one supervision is necessary to promote physical safety until sedation reduces the patient’s feelings of terror. Checks every 15 minutes would not be sufficient to provide for safety. A dimly lit room promotes illusions. Oral fluids are important, but safety is a higher priority. DIF: Cognitive Level: Analysis REF: Pages: 372-373|Pages: 379-380 TOP: Nursing Process: Planning MSC: NCLEX: Safe, Effective Care Environment 8. An alcohol-dependent individual says, “Drinking helps me cope with being a single parent.” Which response by the nurse would help the individual conceptualize the drinking more objectively? a. “Sooner or later, alcohol will kill you. Then what will happen to your children?” b. “I hear a lot of defensiveness in your voice. Do you really believe this?” c. “If you were coping so well, why were you hospitalized again?” d. “Tell me what happened the last time you drank.” ANS: D The individual is rationalizing. The correct response will help the patient see alcohol as a cause of the problems, not the solution. This approach can also help the patient become receptive to the possibility of change. The incorrect responses directly confront and attack defenses against anxiety that the patient still needs. They reflect the nurse’s frustration with the patient. DIF: Cognitive Level: Analysis REF: Pages: 365-366|Pages: 373-375 TOP: Nursing Process: Implementation MSC: NCLEX: Psychosocial Integrity 9. A patient asks for information about Alcoholics Anonymous (AA). Which is the nurse’s best response? a. “It is a self-help group with the goal of sobriety.” b. “It is a form of group therapy led by a psychiatrist.” c. “It is a group that learns about drinking from a group leader.” d. “It is a network that advocates strong punishment for drunk drivers.” ANS: A AA is a peer support group for recovering alcoholics. The goal is to maintain sobriety. Neither professional nor peer leaders are appointed. DIF: Cognitive Level: Comprehension REF: Pages: 372-373|Pages: 378-379 TOP: Nursing Process: Implementation MSC: NCLEX: Psychosocial Integrity 10. Police bring a patient to the emergency department after an automobile accident. The patient is ataxic with slurred speech and mild confusion. The blood alcohol level is 400 mg/dl (0.4 mg %). Considering the relationship between behavior and blood alcohol level, which conclusion can the nurse draw? The patient: a. rarely drinks alcohol. b. has a high tolerance to alcohol. c. has been treated with disulfiram (Antabuse). d. has recently ingested both alcohol and sedative drugs. ANS: B A nontolerant drinker would be in a coma with a blood alcohol level of 400 mg/dl (0.40 mg %). The fact that the patient is walking and talking shows a discrepancy between blood alcohol level and expected behavior. It strongly suggests that the patient’s body has become tolerant to the drug. If disulfiram and alcohol are ingested together, then an entirely different clinical picture would result. The blood alcohol level gives no information about the ingestion of other drugs. DIF: Cognitive Level: Analysis REF: Page: 368 TOP: Nursing Process: Assessment MSC: NCLEX: Physiological Integrity 11. A patient admitted to an alcoholism rehabilitation program says, “I’m just a social drinker. I usually have a drink or two at brunch, a few cocktails in the afternoon, wine at dinner, and several drinks during the evening.” Which defense mechanism is evident? a. Rationalization b. Introjection c. Projection d. Denial ANS: D Minimizing one’s drinking is a form of denial of alcoholism. The patient’s own description indicates that “social drinking” is not an accurate name for the behavior. Projection involves blaming another for one’s faults or problems. Rationalization involves making excuses. Introjection involves taking a quality into one’s own system. DIF: Cognitive Level: Comprehension REF: Pages: 363-366|Pages: 375-377 TOP: Nursing Process: Diagnosis| Nursing Process: Analysis MSC: NCLEX: Psychosocial Integrity 12. A new patient in an alcoholism rehabilitation program says, “I’m just a social drinker. I usually have a drink or two at brunch, a few cocktails in the afternoon, wine at dinner, and a few drinks in the evening.” Which response by the nurse will help the patient view the drinking more honestly? a. “I see,” and use interested silence. b. “I think you may be drinking more than you report.” c. “Being a social drinker involves having a drink or two once or twice a week.” d. “You describe drinking steadily throughout the day and evening. Am I correct?” ANS: D The answer summarizes and validates what the patient reported but is accepting rather than strongly confrontational. Defenses cannot be removed until healthier coping strategies are in place. Strong confrontation does not usually take place so early in treatment. DIF: Cognitive Level: Analysis REF: Pages: 375-377 TOP: Nursing Process: Implementation MSC: NCLEX: Psychosocial Integrity 13. During the third week of treatment, the spouse of a patient in an alcoholism rehabilitation program says, “After discharge, I’m sure everything will be just fine.” Which remark by the nurse will be most helpful to the spouse? a. “It is good that you’re supportive of your spouse’s sobriety and want to help maintain it.” b. “Although sobriety solves some problems, new ones may emerge as one adjusts to living without alcohol.” c. “It will be important for you to structure life to avoid as much stress as possible. You will need to provide social protection.” d. “Remember that alcoholism is a disease of self-destruction. You will need to observe your spouse’s behavior carefully.” ANS: B During recovery, patients identify and use alternative coping mechanisms to reduce their reliance on alcohol. Physical adaptations must occur. Emotional responses, formerly dulled by alcohol, are now fully experienced and may cause considerable anxiety. These changes inevitably have an effect on the spouse and children, who should be given anticipatory guidance and accurate information. DIF: Cognitive Level: Application REF: Pages: 364-365|Pages: 375-379 TOP: Nursing Process: Implementation MSC: NCLEX: Psychosocial Integrity 14. The treatment team plans care for a person diagnosed with paranoid schizophrenia and cannabis abuse. The person has recently used cannabis daily and is experiencing increased hallucinations and delusions. Which principle applies to care planning? a. Consider each diagnosis primary and provide simultaneous treatment. b. The person will benefit from treatment in a residential treatment facility. c. Withdraw the person from cannabis, and then treat the symptoms of schizophrenia. d. Treat the schizophrenia first, and then establish the goals for the treatment of substance abuse. ANS: A Dual diagnoses clinical practice guidelines for both outpatient and inpatient settings suggest that the substance disorder and the psychiatric disorder should both be considered primary and receive simultaneous treatments. Residential treatment may or may not be effective. DIF: Cognitive Level: Analysis REF: Pages: 359-360 TOP: Nursing Process: Planning MSC: NCLEX: Safe, Effective Care Environment 15. When working with a patient beginning treatment for alcohol dependence, what is the nurse’s most therapeutic approach? a. Empathetic, supportive b. Strong, confrontational c. Skeptical, guarded d. Cool, distant ANS: A Support and empathy assist the patient to feel safe enough to start looking at problems. Counseling during the early stage of treatment needs to be direct, open, and honest. The other approaches will increase patient anxiety and cause the patient to cling to defenses. DIF: Cognitive Level: Application REF: Pages: 373-375|Pages: 378-379 TOP: Nursing Process: Implementation MSC: NCLEX: Psychosocial Integrity 16. A patient comes to an outpatient appointment obviously intoxicated. The nurse should: a. explore the patient’s reasons for drinking today. b. arrange admission to an inpatient psychiatric unit. c. coordinate emergency admission to a detoxification unit. d. tell the patient, “We cannot see you today because you’ve been drinking.” ANS: D One cannot conduct meaningful therapy with an intoxicated patient. The patient should be taken home to recover and then make another appointment. DIF: Cognitive Level: Application REF: Pages: 377-378 TOP: Nursing Process: Implementation MSC: NCLEX: Physiological Integrity 17. When a person first begins drinking alcohol, two drinks produce relaxation and drowsiness. After 1 year, four drinks are needed to achieve the same relaxed, drowsy state. Why does this change occur? a. The alcohol is less potent. b. Tolerance develops. c. Antagonistic effects occur. d. Hypomagnesemia develops. ANS: B Tolerance refers to needing higher and higher doses of a drug to produce the desired effect. The potency of the alcohol is stable. Neither hypomagnesemia nor antagonistic effects would account for this change. DIF: Cognitive Level: Comprehension REF: Pages: 362-364|Pages: 367-368 TOP: Nursing Process: Assessment MSC: NCLEX: Physiological Integrity 18. Which statement most accurately describes substance dependence? a. It is a lack of control over use. Tolerance and withdrawal symptoms occur when intake is reduced or stopped. b. It occurs when psychoactive drug use interferes with the work of competing neurotransmitters. c. Symptoms occur when two or more drugs that affect the central nervous system (CNS) have additive effects. d. It involves taking a combination of substances to weaken or inhibit the effect of another drug. ANS: A Psychoactive substance dependence involves a lack of control over use, as well as tolerance and withdrawal symptoms when intake is reduced or stopped. DIF: Cognitive Level: Knowledge REF: Pages: 357-358 TOP: Nursing Process: Implementation MSC: NCLEX: Physiological Integrity 19. A patient who was admitted for a heroin overdose received naloxone (Narcan), which improved the breathing pattern. Two hours later, the patient reports muscle aches, abdominal cramps, gooseflesh and says, “I feel terrible.” Which analysis is correct? a. The patient is exhibiting a prodromal symptom of seizures. b. An idiosyncratic reaction to naloxone is occurring. c. Symptoms of opiate withdrawal are present. d. The patient is experiencing a relapse. ANS: C The symptoms given in the question are consistent with narcotic withdrawal. Early symptoms of narcotic withdrawal are flulike in nature. Seizures are more commonly observed in alcohol withdrawal syndrome. DIF: Cognitive Level: Analysis REF: Pages: 365-368|Page: 370|Page: 377 TOP: Nursing Process: Evaluation MSC: NCLEX: Physiological Integrity 20. In the emergency department, a patient’s vital signs are: blood pressure (BP), 66/40 mm Hg; pulse (P), 140 beats per minute (bpm); and respirations (R), 8 breaths per minute and shallow. The patient overdosed on illegally obtained hydromorphone (Dilaudid). Select the priority outcome. a. Within 8 hours, vital signs will stabilize as evidenced by BP greater than 90/60 mm Hg, P less than 100 bpm, and respirations at or above 12 breaths per minute. b. The patient will be able to describe a plan for home care and achieve a drug-free state before being released from the emergency department. c. The patient will attend daily meetings of Narcotics Anonymous within 1 week of beginning treatment. d. The patient will identify two community resources for the treatment of substance abuse by discharge. ANS: A Hydromorphone (Dilaudid) is an opiate drug. The correct answer is the only one that relates to the patient’s physical condition. It is expected that vital signs will return to normal when the CNS depression is alleviated. The distracters are desired outcomes later in the plan of care. DIF: Cognitive Level: Application REF: Page: 359|Pages: 365-370 TOP: Nursing Process: Outcomes Identification MSC: NCLEX: Physiological Integrity 21. Select the nursing intervention necessary after administering naloxone (Narcan) to a patient with an opiate overdose. a. Monitor the airway and vital signs every 15 minutes. b. Insert a nasogastric tube and test gastric pH. c. Treat hyperpyrexia with cooling measures. d. Insert an indwelling urinary catheter. ANS: A Narcotic antagonists such as naloxone quickly reverse CNS depression; however, because the narcotics have a longer span of action than antagonists, the patient may lapse into unconsciousness or require respiratory support again. The incorrect options are measures unrelated to naloxone use. DIF: Cognitive Level: Application REF: Page: 365|Page: 370|Page: 377 TOP: Nursing Process: Planning MSC: NCLEX: Physiological Integrity 22. A nurse worked at a hospital for several months, resigned, and then took a position at another hospital. In the new position, the nurse often volunteers to be the medication nurse. After several serious medication errors, an investigation reveals that the nurse was diverting patient narcotics for self-use. What early indicator of the nurse’s drug use was evident? a. Accepting responsibility for medication errors. b. Seeking to be assigned as the medication nurse. c. Frequent complaints of physical pain. d. High sociability with peers. ANS: B The nurse intent on diverting drugs for personal use often attempts to isolate him- or herself from peers and seeks access to medications. Usually, the person’s appearance will deteriorate, and he or she will blame errors on others. DIF: Cognitive Level: Application REF: Pages: 363-364 TOP: Nursing Process: Evaluation MSC: NCLEX: Psychosocial Integrity 23. A nurse with a history of narcotic dependence is found unconscious in the hospital locker room after overdosing. The nurse is transferred to the inpatient chemical dependence unit for care. Which attitudes or behaviors by nursing staff may be enabling? a. Conveying understanding that pressures associated with nursing practice underlie substance use. b. Pointing out that work problems are the result, but not the cause, of substance dependence. c. Conveying empathy when the nurse discusses fears of disciplinary action by the state board of nursing. d. Providing health teaching about stress management. ANS: A Enabling denies the seriousness of the patient’s problem or supports the patient as he or she shifts responsibility from self to circumstances. The incorrect options are therapeutic and appropriate. DIF: Cognitive Level: Application REF: Pages: 363-364 TOP: Nursing Process: Assessment MSC: NCLEX: Safe, Effective Care Environment 24. Which treatment approach is most appropriate for a patient with antisocial tendencies who has been treated several times for substance addiction but has relapsed? a. One-week detoxification program b. Long-term outpatient therapy c. Twelve-step self-help program d. Residential program ANS: D Residential programs and therapeutic communities have goals of complete change in lifestyle, abstinence from drugs, elimination of criminal behaviors, development of employable skills, self-reliance, and honesty. Residential programs are more effective than outpatient programs for patients with antisocial tendencies. DIF: Cognitive Level: Application REF: Page: 379 TOP: Nursing Process: Planning MSC: NCLEX: Safe, Effective Care Environment 25. Which nursing diagnosis would likely apply to both patients with paranoid schizophrenia and patients with amphetamine-induced psychosis? a. Powerlessness b. Disturbed thought processes c. Ineffective thermoregulation d. Impaired oral mucous membrane ANS: B Both types of patients commonly experience paranoid delusions; thus the nursing diagnosis of Disturbed thought processes is appropriate for both. The incorrect options are not specifically applicable to both. DIF: Cognitive Level: Analysis REF: Page: 369 TOP: Nursing Process: Diagnosis| Nursing Process: Analysis MSC: NCLEX: Psychosocial Integrity 26. Which is an important nursing intervention when giving care to a patient withdrawing from a central nervous system (CNS) stimulant? a. Make physical contact by frequently touching the patient. b. Offer intellectual activities requiring concentration. c. Avoid manipulation by denying the patient’s requests. d. Observe for depression and suicidal ideation. ANS: D Rebound depression occurs with the withdrawal from CNS stimulants, probably related to neurotransmitter depletion. Touch may be misinterpreted if the patient is experiencing paranoid tendencies. Concentration is impaired during withdrawal. Denying requests is inappropriate; maintaining established limits will suffice. DIF: Cognitive Level: Application REF: Pages: 369-370 TOP: Nursing Process: Planning MSC: NCLEX: Safe, Effective Care Environment 27. Which assessment findings best correlate to the withdrawal from central nervous system depressants? a. Dilated pupils, tachycardia, elevated blood pressure, elation b. Labile mood, lack of coordination, fever, drowsiness c. Nausea, vomiting, diaphoresis, anxiety, tremors d. Excessive eating, constipation, headache ANS: C The symptoms of withdrawal from various CNS depressants are similar. Generalized seizures are possible. DIF: Cognitive Level: Comprehension REF: Pages: 367-370 TOP: Nursing Process: Assessment MSC: NCLEX: Physiological Integrity 28. A patient has smoked two packs of cigarettes daily for many years. When the patient does not smoke or tries to cut back, anxiety, craving, poor concentration, and headache result. What does this scenario describe? a. Substance abuse b. Substance intoxication c. Substance dependence d. Recreational use of a social drug ANS: C Nicotine meets the criteria for a substance, the criterion for dependence (tolerance) is present, and withdrawal symptoms are noted with abstinence or a reduction of the dose. The scenario does not meet the criteria for substance abuse, intoxication, or recreational use of a social drug. DIF: Cognitive Level: Comprehension REF: Pages: 357-358|Page: 369 TOP: Nursing Process: Assessment MSC: NCLEX: Physiological Integrity 29. Which assessment findings will the nurse expect in an individual who has just injected heroin? a. Anxiety, restlessness, paranoid delusions b. Heightened sexuality, insomnia, euphoria c. Muscle aching, dilated pupils, tachycardia d. Drowsiness, constricted pupils, slurred speech ANS: D Heroin, an opiate, is a CNS depressant. Blood pressure, pulse, and respirations are decreased, and attention is impaired. The incorrect options describe behaviors consistent with amphetamine use, symptoms of narcotic withdrawal, and cocaine use. DIF: Cognitive Level: Application REF: Pages: 369-370 TOP: Nursing Process: Assessment MSC: NCLEX: Psychosocial Integrity 30. A newly hospitalized patient has needle tracks on both arms. A friend states that the patient uses heroin daily but has not used in the past 24 hours. The nurse should assess the patient for: a. slurred speech, excessive drowsiness, and bradycardia b. paranoid delusions, tactile hallucinations, and panic c. runny nose, yawning, insomnia, and chills d. anxiety, agitation, and aggression ANS: C Early signs and symptoms of narcotic withdrawal resemble symptoms of onset of a flulike illness, minus the temperature elevation. The incorrect options reflect signs of intoxication or CNS depressant overdose and CNS stimulant or hallucinogen use. DIF: Cognitive Level: Application REF: Pages: 369-370 TOP: Nursing Process: Assessment MSC: NCLEX: Physiological Integrity 31. A nurse is called to the home of a neighbor and finds an unconscious person still holding a medication bottle labeled pentobarbital sodium. What is the nurse’s first action? a. Test reflexes b. Check pupils c. Initiate vomiting d. Establish a patent airway ANS: D Pentobarbital sodium is a barbiturate. Maintaining a patent airway is the priority when the patient is unconscious. Assessing neurologic function by testing reflexes and checking pupils can wait. Vomiting should not be induced when a patient is unconscious because of the danger of aspiration. DIF: Cognitive Level: Application REF: Pages: 366-367 TOP: Nursing Process: Implementation MSC: NCLEX: Physiological Integrity 32. An adult in the emergency department states, “I feel restless. Everything I look at wavers. Sometimes I’m outside my body looking at myself. I hear colors. I think I’m losing my mind.” Vital signs are slightly elevated. The nurse should suspect a(n): a. schizophrenic episode b. cocaine overdose c. phencyclidine (PCP) intoxication d. D-lysergic acid diethylamide (LSD) ingestion ANS: D The patient who has ingested LSD often experiences synesthesia (visions in sound), depersonalization, and concerns about going “crazy.” Synesthesia is not common in schizophrenia. CNS stimulant overdose more commonly involves elevated vital signs and assaultive, grandiose behaviors. PCP use commonly causes bizarre or violent behavior, nystagmus, elevated vital signs, and repetitive jerking movements. DIF: Cognitive Level: Application REF: Pages: 370-372 TOP: Nursing Process: Assessment MSC: NCLEX: Psychosocial Integrity 33. In what significant ways is the therapeutic environment different for a patient who has ingested D-lysergic acid diethylamide (LSD) than for a patient who has ingested phencyclidine (PCP)? a. For LSD ingestion, one person stays with the patient and provides verbal support. For PCP ingestion, a regimen of limited contact with staff members is maintained, and continual visual monitoring is provided. b. For PCP ingestion, the patient is placed on one-on-one intensive supervision. For LSD ingestion, a regimen of limited interaction and minimal verbal stimulation is maintained. c. For LSD ingestion, continual moderate sensory stimulation is provided. For PCP ingestion, continual high-level stimulation is provided. d. For LSD ingestion, the patient is placed in restraints. For PCP ingestion, seizure precautions are implemented. ANS: A Patients who have ingested LSD respond well to being “talked down” by a supportive person. Patients who have ingested PCP are very sensitive to stimulation and display frequent, unpredictable, and violent behaviors. Although one person should perform care and talk gently to the patient, no one individual should be alone in the room with the patient. An adequate number of staff members should be gathered to manage violent behavior if it occurs. DIF: Cognitive Level: Analysis REF: Pages: 370-372 TOP: Nursing Process: Planning MSC: NCLEX: Safe, Effective Care Environment 34. When assessing a patient who has ingested flunitrazepam (Rohypnol), the nurse would expect: a. acrophobia b. hypothermia c. hallucinations d. anterograde amnesia ANS: D Flunitrazepam is also known as the date rape drug; it produces disinhibition and a relaxation of voluntary muscles, as well as anterograde amnesia for events that occur. The other options do not reflect symptoms commonly observed after use of this drug. DIF: Cognitive Level: Comprehension REF: Page: 359|Page: 372 TOP: Nursing Process: Assessment MSC: NCLEX: Physiological Integrity 35. A patient is admitted in a comatose state after ingesting 30 capsules of pentobarbital sodium. A friend of the patient says, “Often my friend drinks, along with taking more of the drug than is ordered.” What is the effect of the use of alcohol with this drug? a. The drug’s metabolism is stimulated. b. The drug’s effect is diminished. c. A synergistic effect occurs. d. There is no effect. ANS: C Both pentobarbital and alcohol are CNS depressants and have synergistic effects. Taken together, the action of each would potentiate the other. DIF: Cognitive Level: Comprehension REF: Pages: 364-365 TOP: Nursing Process: Assessment MSC: NCLEX: Physiological Integrity 36. Which medication is the nurse most likely to see prescribed as part of the treatment plan for both a patient in an alcoholism treatment program and a patient in a program for the treatment of opioid addiction? a. methadone (Dolophine) b. bromocriptine (Parlodel) c. disulfiram (Antabuse) d. naltrexone (Revia) ANS: D Naltrexone is useful for treating both opioid and alcohol addictions. As an opioid antagonist, it blocks the action of opioids; because it blocks the mechanism of reinforcement, it also reduces or eliminates alcohol craving. DIF: Cognitive Level: Application REF: Page: 370|Pages: 380-381 TOP: Nursing Process: Planning MSC: NCLEX: Physiological Integrity 37. Select the most appropriate outcome for a patient completing the fourth alcohol detoxification program in 1 year. Before discharge, the patient will a. use rationalization in healthy ways. b. state, “I see the need for ongoing treatment.” c. identify constructive outlets for expression of anger. d. develop a trusting relationship with one staff member. ANS: B The answer refers to the need for ongoing treatment after detoxification and is the best goal related to controlling relapse. The scenario does not provide enough information to know whether anger has been identified as a problem. A trusting relationship, although desirable, would not help the patient maintain sobriety. DIF: Cognitive Level: Analysis REF: Page: 373|Pages: 377-379 TOP: Nursing Process: Outcomes Identification MSC: NCLEX: Psychosocial Integrity 38. Which question has the highest priority when assessing a newly admitted patient with a history of alcohol abuse? a. “Have you ever had blackouts?” b. “When did you have your last drink?” c. “Has drinking caused you any problems?” d. “When did you decide to seek treatment?” ANS: B Learning when the patient had the last drink is essential to knowing when to begin to observe for symptoms of withdrawal. The other questions are relevant but of lower priority. DIF: Cognitive Level: Application REF: Pages: 366-367 TOP: Nursing Process: Assessment MSC: NCLEX: Physiological Integrity 39. A patient in an alcohol rehabilitation program says, “I have been a loser all my life. I’m so ashamed of what I have put my family through. Now, I’m not even sure I can succeed at staying sober.” Which nursing diagnosis applies? a. Chronic low self-esteem b. Situational low self-esteem c. Disturbed personal identity d. Ineffective health maintenance ANS: A Low self-esteem is present when a patient sees him- or herself as inadequate. It is a chronic problem because it is a lifelong feeling for the patient. Data are not present to support the other options. DIF: Cognitive Level: Application REF: Pages: 362-363|Page: 374 TOP: Nursing Process: Diagnosis| Nursing Process: Analysis MSC: NCLEX: Psychosocial Integrity 40. Which documentation indicates that the treatment plan for a patient in an alcohol rehabilitation program was effective? a. Is abstinent for 10 days and states, “I can maintain sobriety 1 day at a time.” Spoke with employer, who is willing to allow the patient to return to work in 3 weeks. b. Is abstinent for 15 days and states, “My problems are under control.” Plans to seek a new job where co-workers will not know history. c. Attends AA daily; states many of the members are “real” alcoholics and states, “I may be able to help some of them find jobs at my company.” d. Is abstinent for 21 days and says, “I know I can’t handle more than one or two drinks in a social setting.” ANS: A The answer reflects the AA beliefs. The incorrect options each contain a statement that suggests early relapse. DIF: Cognitive Level: Analysis REF: Page: 373|Pages: 377-379 TOP: Nursing Process: Evaluation MSC: NCLEX: Psychosocial Integrity 41. Which assessment findings support a nurse’s suspicion that a patient has been using inhalants? a. Perforated nasal septum and hypertension b. Drowsiness, euphoria, and constipation c. Pinpoint pupils and respiratory rate of 12 breaths per minute d. Confusion, mouth ulcers, and ataxia ANS: D Inhalants are usually CNS depressants, giving rise to confusion and ataxia. Mouth ulcers come from the irritation of buccal mucosa by the inhalant. The incorrect options relate to cocaine snorting and opioid use. DIF: Cognitive Level: Application REF: Pages: 370-372 TOP: Nursing Process: Assessment MSC: NCLEX: Physiological Integrity MULTIPLE RESPONSE 1. A patient undergoing alcohol rehabilitation decides to accept disulfiram (Antabuse) therapy to avoid impulsively responding to drinking cues. Which information should be included in the discharge teaching for this patient? Select all that apply. a. Avoid aged cheeses. b. Read labels of all liquid medications. c. Wear sunscreen and avoid bright sunlight. d. Maintain an adequate dietary intake of sodium. e. Avoid breathing fumes of paints, stains, and stripping compounds. ANS: B, E The patient must avoid hidden sources of alcohol. Many liquid medications, such as cough syrups, contain small amounts of alcohol that could trigger an alcohol-disulfiram reaction. Using alcohol-based skin products such as aftershave or cologne; smelling alcohol-laden fumes; and eating foods prepared with wine, brandy, beer, or spirits of any sort may also trigger reactions. The other options do not relate to hidden sources of alcohol. DIF: Cognitive Level: Application REF: Pages: 380-381 TOP: Nursing Process: Planning MSC: NCLEX: Physiological Integrity 2. A nurse can assist a patient and family in which aspects of substance abuse relapse prevention? Select all that apply. a. Rehearsing techniques to handle anticipated stressful situations b. Advising the patient to accept residential treatment if relapse occurs c. Assisting the patient to identify life skills needed for effective coping d. Isolating self from significant others and social situations until sobriety is established e. Teaching the patient about the physical changes to expect as the body adapts to functioning without substances ANS: A, C, E Nurses can be helpful as a patient assesses needed life skills and in providing appropriate referrals. Anticipatory problem solving and role playing are good ways of rehearsing effective strategies for handling stressful situations. The nurse can participate in role playing and help the patient evaluate the usefulness of new strategies. The nurse can also provide valuable information about the physiologic changes that can be expected and the ways in which to cope with these changes. Residential treatment is not usually necessary after relapse. Patients need the support of friends and family to establish and maintain sobriety. DIF: Cognitive Level: Application REF: Page: 373|Pages: 377-379 TOP: Nursing Process: Planning MSC: NCLEX: Psychosocial Integrity 3. While caring for a patient with a methamphetamine overdose, which tasks are the priority focuses of care? Select all that apply. a. Administration of naloxone (Narcan) b. Vitamin B12 and folate supplements c. Restoring nutritional integrity d. Prevention of seizures e. Reduction of fever ANS: D, E Hyperpyrexia and convulsions are common when a patient has overdosed on a CNS stimulant. These problems are life threatening and take priority. Naloxone (Narcan) is administered for opiate overdoses. Vitamin B12 and folate may be helpful for overdoses from solvents, gases, or nitrates. Nutrition is not a priority in an overdose situation. DIF: Cognitive Level: Application REF: Page: 362|Page: 369 TOP: Nursing Process: Planning MSC: NCLEX: Physiological Integrity Lesly Chapter 13 Chapter 13: Personality Disordersg MULTIPLE CHOICE 1. A therapist recently convicted of multiple counts of Medicare fraud says, “Sure I overbilled. Why not? Everyone takes advantage of the government, so I did too.” These statements show: a. shame. b. suspiciousness. c. superficial remorse. d. lack of guilt feelings. ANS: D Rationalization is being used to explain behavior and deny wrongdoing. The individual who does not believe he or she has done anything wrong will not exhibit anxiety, remorse, or guilt about the act. The patient’s remarks cannot be assessed as shameful. Lack of trust or concern that others are determined to cause harm is not evident. DIF: Cognitive Level: Application (Applying) REF: Page: 217 2. Which intervention is appropriate for a patient diagnosed with an antisocial personality disorder who frequently manipulates others? a. Refer the patient’s requests and questions to the case manager. b. Explore the patient’s feelings of fear and inferiority. c. Provide negative reinforcement for acting-out behavior. d. Ignore, rather than confront, inappropriate behavior. ANS: A Manipulative patients frequently make requests of many different staff members, hoping someone will give in. Having only one decision-maker provides consistency and avoids the potential for playing one staff member against another. Positive reinforcement of appropriate behaviors is more effective than negative reinforcement. The behavior should not be ignored; judicious use of confrontation is necessary. Patients with antisocial personality disorders rarely have feelings of fear and inferiority. DIF: Cognitive Level: Application (Applying) REF: Page: 215 | Page: 225 3. As a nurse prepares to administer a medication to a patient diagnosed with a borderline personality disorder, the patient says, “Just leave it on the table. I’ll take it when I finish combing my hair.” What is the nurse’s best response? a. Reinforce this assertive action by the patient. Leave the medication on the table as requested. b. Respond to the patient, “I’m worried that you might not take it. I will come back later.” c. Say to the patient, “I must watch you take the medication. Please take it now.” d. Ask the patient, “Why don’t you want to take your medication now?” ANS: C The individual with a borderline personality disorder characteristically demonstrates manipulative, splitting, and self-destructive behaviors. Consistent limit setting is vital for the patient’s safety, as well as to prevent splitting other staff members. “Why” questions are not therapeutic. DIF: Cognitive Level: Application (Applying) REF: Pages: 217-218 | Page: 225 4. What is an appropriate initial outcome for a patient diagnosed with a personality disorder who frequently manipulates others? The patient will: a. identify when feeling angry. b. use manipulation only to get legitimate needs met. c. acknowledge manipulative behavior when it is called to his or her attention. d. accept fulfillment of his or her requests within an hour rather than immediately. ANS: C Acknowledging manipulative behavior is an early outcome that paves the way for taking greater responsibility for controlling manipulative behavior at a later time. Identifying anger relates to anger and aggression control. Using manipulation to get legitimate needs is an inappropriate outcome. Ideally, the patient will use assertive behavior to promote the fulfillment of legitimate needs. Accepting fulfillment of requests within an hour rather than immediately relates to impulsivity and immediacy control. DIF: Cognitive Level: Analysis (Analyzing) REF: Page: 225 5. Consider these comments made to three different nurses by a patient diagnosed with an antisocial personality disorder: “You’re a better nurse than the day shift nurse said you were”; “Another nurse said you don’t do your job right”; “You think you’re perfect, but I’ve seen you make three mistakes.” Collectively, these interactions can be assessed as: a. seductive. b. detached. c. manipulative. d. guilt producing. ANS: C Patients manipulate and control staff members in various ways. By keeping staff members off balance or fighting among themselves, the person with an antisocial personality disorder is left to operate as he or she pleases. Seductive behavior has sexual connotations. The patient is displaying the opposite of detached behavior. Guilt is not evidenced in the comments. DIF: Cognitive Level: Application (Applying) REF: Page: 217 | Page: 225 6. A nurse reports to the interdisciplinary team that a patient diagnosed with an antisocial personality disorder lies to other patients, verbally abuses a patient diagnosed with dementia, and flatters the primary nurse. This patient is detached and superficial during counseling sessions. Which behavior most clearly warrants limit setting? a. Flattering the nurse b. Lying to other patients c. Verbal abuse of another patient d. Detached superficiality during counseling ANS: C Limits must be set in areas in which the patient’s behavior affects the rights of others. Limiting verbal abuse of another patient is a priority intervention. The other concerns should be addressed during therapeutic encounters. DIF: Cognitive Level: Analysis (Analyzing) REF: Page: 217 | Pages: 223-224 7. A patient diagnosed with borderline personality disorder has a history of self-mutilation and suicide attempts. The patient reveals feelings of depression and anger with life. The psychiatrist suggests the use of a medication. Which type of medication should the nurse expect? a. Selective serotonin reuptake inhibitor (SSRI) b. Monoamine oxidase inhibitor (MAOI) c. Benzodiazepine d. Antipsychotic ANS: A SSRIs are used to treat depression. Many patients with borderline personality disorder are fearful of taking something over which they have little control. Because SSRIs have a good side effect profile, the patient is more likely to comply with the medication. Low-dose antipsychotic or anxiolytic medications are not supported by the data given in this scenario. MAOIs require great diligence in adherence to a restricted diet and are rarely used for patients who are impulsive. DIF: Cognitive Level: Application (Applying) REF: Page: 228 8. A person’s spouse filed charges of battery. The person has a long history of acting-out behaviors and several arrests. Which statement by the person suggests an antisocial personality disorder? a. “I have a quick temper, but I can usually keep it under control.” b. “I’ve done some stupid things in my life, but I’ve learned a lesson.” c. “I’m feeling terrible about the way my behavior has hurt my family.” d. “I hit because I’m tired of being nagged. My spouse deserved the beating.” ANS: D The patient with an antisocial personality disorder often impulsively acts out feelings of anger and feels no guilt or remorse. Patients with antisocial personality disorders rarely seem to learn from experience or feel true remorse. Problems with anger management and impulse control are common. DIF: Cognitive Level: Application (Applying) REF: Page: 217 9. What is the priority nursing diagnosis for a patient diagnosed with antisocial personality disorder who has made threats against staff, ripped art off the walls, and thrown objects? a. Disturbed sensory perception–auditory b. Risk for other-directed violence c. Ineffective denial d. Ineffective coping ANS: B Violence against property, along with threats to harm staff, makes this diagnosis the priority. Patients with antisocial personality disorders rarely have psychotic symptoms. When patients with antisocial personality disorders use denial, they use it effectively. Although ineffective coping applies, the risk for violence is a higher priority. DIF: Cognitive Level: Application (Applying) REF: Page: 217 | Pages: 223-224 10. A patient diagnosed with a personality disorder has used manipulation to get his or her needs met. The staff decides to apply limit-setting interventions. What is the correct rationale for this action? a. It provides an outlet for feelings of anger and frustration. b. It respects the patient’s wishes so assertiveness will develop. c. External controls are necessary while internal controls are developed. d. Anxiety is reduced when staff members assume responsibility for the patient’s behavior. ANS: C A lack of internal controls leads to manipulative behaviors such as lying, cheating, conning, and flattering. To protect the rights of others, external controls must be consistently maintained until the patient is able to behave appropriately. DIF: Cognitive Level: Application (Applying) REF: Pages: 224-225 11. A patient diagnosed with borderline personality disorder and a history of self-mutilation has now begun dialectical behavior therapy (DBT) on an outpatient basis. Counseling focuses on self-harm behavior management. Today the patient telephones to say, “I’m feeling empty and want to cut myself.” The nurse should: a. arrange for emergency inpatient hospitalization. b. send the patient to the crisis intervention unit for 8 to 12 hours. c. assist the patient to identify the trigger situation and choose a coping strategy. d. advise the patient to take an antianxiety medication to decrease the anxiety level. ANS: C The patient has responded appropriately to the urge for self-harm by calling a helping individual. A component of dialectical behavior therapy is telephone access to the therapist for “coaching” during crises. The nurse can assist the patient to choose an alternative to self-mutilation. The need for a protective environment may not be necessary if the patient is able to use cognitive strategies to determine a coping strategy that reduces the urge to mutilate. Taking a sedative and going to sleep should not be the first-line intervention; sedation may reduce the patient’s ability to weigh alternatives to mutilating behavior. REF: Pages: 217-218 | Page: 224 | Page: 228 12. The most challenging nursing intervention for patients diagnosed with personality disorders who use manipulation to get their needs met is: a. supporting behavioral change. b. monitoring suicide attempts. c. maintaining consistent limits. d. using aversive therapy. ANS: C Maintaining consistent limits is by far the most difficult intervention because of the patient’s superior skills at manipulation. Supporting behavioral change and monitoring patient safety are less difficult tasks. Aversive therapy would probably not be part of the care plan; positive reinforcement strategies for acceptable behavior are more effective than aversive techniques. DIF: Cognitive Level: Comprehension (Understanding) REF: Page: 224 13. The history shows that a newly admitted patient has impulsivity. The nurse would expect behavior characterized by: a. adherence to a strict moral code. b. manipulative, controlling strategies. c. postponing gratification to an appropriate time. d. little time elapsed between thought and action. ANS: D The impulsive individual acts in haste without taking time to consider the consequences of the action. None of the other options describes impulsivity. DIF: Cognitive Level: Comprehension (Understanding) REF: Pages: 224-225 14. A patient tells a nurse, “I sometimes get into trouble because I make quick decisions and act on them.” A therapeutic response would be: a. “Let’s consider the advantages of being able to stop and think before acting.” b. “It sounds as though you’ve developed some insight into your situation.” c. “I’ll bet you have some interesting stories to share about overreacting.” d. “It’s good that you’re showing readiness for behavioral change.” ANS: A The patient is showing openness to learning techniques for impulse control. One technique is to teach the patient to stop and think before acting impulsively. The patient can then be taught to evaluate the outcomes of possible actions and choose an effective action. The incorrect responses shift the encounter to a social level or are judgmental. DIF: Cognitive Level: Application (Applying) REF: Pages: 224-225 15. A patient diagnosed with borderline personality disorder is hospitalized several times after self-inflicted lacerations. The patient remains impulsive. Dialectical behavior therapy starts on an outpatient basis. Which nursing diagnosis is the focus of this therapy? a. Risk for self-mutilation b. Impaired skin integrity c. Risk for injury d. Powerlessness ANS: A Risk for self-mutilation is a nursing diagnosis relating to patient safety needs and is therefore a high priority. Impaired skin integrity and powerlessness may be appropriate foci for care but are not the priority or related to this therapy. Risk for injury implies accidental injury, which is not the case for the patient diagnosed with borderline personality disorder. REF: Pages: 217-218 | Page: 224 | Page: 228 16. Which statement made by a patient diagnosed with borderline personality disorder indicates the treatment plan is effective? a. “I think you are the best nurse on the unit.” b. “I’m never going to get high on drugs again.” c. “I hate my doctor for not giving me what I ask for.” d. “I felt empty and wanted to cut myself, so I called you.” ANS: D Seeking a staff member instead of impulsively self-mutilating shows an adaptive coping strategy. The incorrect responses demonstrate idealization, devaluation, and wishful thinking. DIF: Cognitive Level: Analysis (Analyzing) REF: Pages: 217-218 | Page: 228 17. When preparing to interview a patient diagnosed with narcissistic personality disorder, a nurse can anticipate the assessment findings will include: a. preoccupation with minute details; perfectionism. b. charm, drama, seductiveness; seeking admiration. c. difficulty being alone; indecisiveness, submissiveness. d. grandiosity, attention seeking, and arrogance. ANS: D According to the Diagnostic and Statistical Manual of Mental Disorders (DSM-5), the characteristics of grandiosity, attention seeking, and arrogance are consistent with narcissistic personality disorder. Charm, drama, seductiveness, and admiration seeking are observed in patients diagnosed with histrionic personality disorder. Preoccupation with minute details and perfectionism are observed in individuals diagnosed with obsessive-compulsive personality disorder. Patients diagnosed with dependent personality disorder often express difficulty being alone and are indecisive and submissive. DIF: Cognitive Level: Comprehension (Understanding) REF: Page: 219 18. For which behavior would limit setting be most essential? The patient: a. clings to the nurse and asks for advice about inconsequential matters. b. is flirtatious and provocative with staff members of the opposite sex. c. is hypervigilant and refuses to attend unit activities. d. urges a suspicious patient to hit anyone who stares. ANS: D The correct option is an example of a manipulative behavior. Because manipulation violates the rights of others, limit setting is absolutely necessary. Furthermore, limit setting is necessary in this case because the safety of patients is at risk. Limit setting may be occasionally used with dependent behavior (clinging to the nurse) and histrionic behavior (flirting with staff members), but other therapeutic techniques are also useful. Limit setting is not needed for a patient who is hypervigilant and refuses to attend unit activities; rather, the need to develop trust is central to patient compliance. DIF: Cognitive Level: Analysis (Analyzing) REF: Page: 222 19. A nurse in the emergency department tells an adult, “Your mother had a severe stroke.” The adult tearfully says, “Who will take care of me now? My mother always told me what to do, what to wear, and what to eat. I need someone to reassure me when I get anxious.” Which term best describes this behavior? a. Histrionic b. Dependent c. Narcissistic d. Borderline ANS: B The main characteristic of the dependent personality is a pervasive need to be taken care of that leads to submissive behaviors and a fear of separation. Histrionic behavior is characterized by flamboyance, attention seeking, and seductiveness. Narcissistic behavior is characterized by grandiosity and exploitive behavior. Patients with borderline personality disorder demonstrate separation anxiety, impulsivity, and splitting. DIF: Cognitive Level: Comprehension (Understanding) REF: Pages: 220-221 20. Others describe a worker as very shy and lacking in self-confidence. This worker stays in an office cubicle all day and never comes out for breaks or lunch. Which term best describes this behavior? a. Avoidant b. Dependent c. Histrionic d. Paranoid ANS: A Patients with avoidant personality disorder are timid, socially uncomfortable, and withdrawn and avoid situations in which they might fail. They believe themselves to be inferior and unappealing. Individuals with dependent personality disorder are clinging, needy, and submissive. Individuals with histrionic personality disorder are seductive, flamboyant, shallow, and attention seeking. Individuals with paranoid personality disorder are suspicious and hostile and project blame. DIF: Cognitive Level: Comprehension (Understanding) REF: Page: 220 21. What is the priority intervention for a nurse beginning a therapeutic relationship with a patient diagnosed with a schizotypal personality disorder? a. Respect the patient’s need for periods of social isolation. b. Prevent the patient from violating the nurse’s rights. c. Engage the patient in many community activities. d. Teach the patient how to match clothing. ANS: A Patients diagnosed with schizotypal personality disorder are eccentric and often display perceptual and cognitive distortions. They are suspicious of others and have considerable difficulty trusting. They become highly anxious and frightened in social situations, thus the need to respect their desire for social isolation. Teaching the patient to match clothing is not the priority intervention. Patients diagnosed with schizotypal personality disorder rarely engage in behaviors that violate the nurse’s rights or exploit the nurse. DIF: Cognitive Level: Application (Applying) REF: Page: 216 | Page: 224 22. A patient diagnosed with borderline personality disorder self-inflicted wrist lacerations after gaining new privileges on the unit. The cause of the self-mutilation is probably related to: a. inherited disorder that manifests itself as an incapacity to tolerate stress. b. use of projective identification and splitting to bring anxiety to manageable levels. c. constitutional inability to regulate affect, predisposing to psychic disorganization. d. fear of abandonment associated with progress toward autonomy and independence. ANS: D Fear of abandonment is a central theme for most patients diagnosed with borderline personality disorder. This fear is often exacerbated when patients diagnosed with borderline personality disorder experience success or growth. The incorrect options are not associated with self-mutilation. REF: Pages: 217-218 | Pages: 226-227 TOP: Nursing Process: Evaluation 23. A patient diagnosed with borderline personality disorder has self-inflicted wrist lacerations. The health care provider prescribes daily dressing changes. The nurse performing this care should: a. encourage the patient to express anger. b. provide care in a matter-of-fact manner. c. be very kind, sympathetic, and concerned. d. offer to listen to the patient’s feelings about cutting. ANS: B A matter-of-fact approach does not provide the patient with positive reinforcement for self-mutilation. The goal of providing emotional consistency is supported by this approach. The incorrect options provide positive reinforcement of the behavior. REF: Pages: 217-218 | Pages: 226-227 TOP: Nursing Process: Implementation 24. A nurse set limits for a patient diagnosed with a borderline personality disorder. The patient tells the nurse, “You used to care about me. I thought you were wonderful. Now I can see I was mistaken. You’re terrible.” This outburst can be assessed as: a. denial. b. splitting. c. reaction formation. d. separation-individuation strategies. ANS: B Splitting involves loving a person and then hating the person; the patient is unable to recognize that an individual can have both positive and negative qualities. Denial is an unconscious motivated refusal to believe something. Reaction formation involves unconsciously doing the opposite of a forbidden impulse. Separation-individuation strategies refer to childhood behaviors related to developing independence from the caregiver. REF: Pages: 217-218 | Pages: 226-227 TOP: Nursing Process: Assessment 25. Which characteristic of individuals diagnosed with personality disorders makes it most necessary for staff to schedule frequent meetings? a. Ability to achieve true intimacy b. Flexibility and adaptability to stress c. Ability to evoke interpersonal conflict d. Inability to develop trusting relationships ANS: C Frequent team meetings are held to counteract the effects of the patient’s attempts to split staff and set them against one another, causing interpersonal conflict. Patients with personality disorders are inflexible and demonstrate maladaptive responses to stress. They are usually unable to develop true intimacy with others and are unable to develop trusting relationships. Although problems with trust may exist, it is not the characteristic that requires frequent staff meetings. DIF: Cognitive Level: Application (Applying) REF: Page: 214 | Page: 224 26. Which common assessment finding would be most applicable to a patient diagnosed with any personality disorder? The patient: a. demonstrates behaviors that cause distress to self rather than to others. b. has self-esteem issues, despite his or her outward presentation. c. usually becomes psychotic when exposed to stress. d. does not experience real distress from symptoms. ANS: B Self-esteem issues are present, despite patterns of withdrawal, grandiosity, suspiciousness, or unconcern. They seem to relate to early life experiences and are reinforced through unsuccessful experiences in loving and working. Personality disorders involve lifelong, inflexible, dysfunctional, and deviant patterns of behavior that cause distress to others and, in some cases, to self. Patients with personality disorders may experience very real anxiety and distress when stress levels rise. Some individuals with personality disorders, but not all, may decompensate and show psychotic behaviors under stress. DIF: Cognitive Level: Application (Applying) REF: Pages: 214-215 MULTIPLE RESPONSE 1. A nurse plans the care for an individual diagnosed with antisocial personality disorder. Which characteristic behaviors will the nurse expect? Select all that apply. a. Reclusive behavior b. Callous attitude c. Perfectionism d. Aggression e. Clinginess f. Anxiety ANS: B, D Individuals diagnosed with antisocial personality disorders characteristically demonstrate manipulative, exploitative, aggressive, callous, and guilt-instilling behaviors. Individuals diagnosed with antisocial personality disorders are more extroverted than reclusive, rarely show anxiety, and rarely demonstrate clinging or dependent behaviors. Individuals diagnosed with antisocial personality disorders are more likely to be impulsive than to be perfectionists. DIF: Cognitive Level: Application (Applying) REF: Page: 217 2. For which patients diagnosed with personality disorders would a family history of similar problems be most likely? Select all that apply. a. Obsessive-compulsive b. Antisocial c. Dependent d. Schizotypal e. Narcissistic ANS: A, B, D Some personality disorders have evidence of genetic links; therefore the family history would show other family members with similar traits. Heredity plays a role in schizotypal and antisocial problems, as well as obsessive-compulsive personality disorder. DIF: Cognitive Level: Application (Applying) REF: Page: 215 Questions from the back of the chapter 1. A person who shoplift merchandise from a community cancer thrift shop. When confronted, the thief replies “all this stuff is donated, so I can take it.” this comment suggests features of which personality disorder? Answer- antisocial 2 After a power outage, a facility must serve dinner of sandwiches and fruit to patients. Which comment is ost likely from a patient diagnosed with a narcissistic personality disorder? answer= “you should have ordered a to-go meal from a local restaurant for me. “ 3. A nurse plans care for a patient diagnosed with borderline personality disorder. Which nursing diagnosis is most likely to apply to this patient? Answer- ineffective relationships related to frequent splitting 4. the nurse assesses a new patient suspected of having a schizotypal personality disorder. Which assessment question is this patient most likely to answer affirmatively? Answer= “Is anyone in your family diagnosed with a mental illness?” 5. a mental health nurse assesses a patient diagnosed with an antisocial personality disorder. Which comorbid problem is most important for the nurse to include in the assessment? answer=alcohol use and abuse Lesly Chapter 14 Chapter 14: Eating Disorders Chapter 14: Eating Disorders Varcarolis: Essentials to Psychiatric Mental Health Nursing, 2nd Edition – Revised Reprint MULTIPLE CHOICE 1. Over the past year, a woman has cooked gourmet meals for her family but eats only tiny servings. She wears layered, loose clothing and now has amenorrhea. Her current weight is 95 pounds, a loss of 35 pounds. Which medical diagnosis is most likely? a. Binge eating disorder b. Anorexia nervosa c. Bulimia nervosa d. Pica ANS: B Overly controlled eating behaviors, extreme weight loss, amenorrhea, preoccupation with food, and wearing several layers of loose clothing to appear larger are part of the clinical picture of an individual with anorexia nervosa. The individual with bulimia usually is near normal weight. The binge eater is often overweight. Pica refers to eating nonfood items. DIF: Cognitive Level: Application (Applying) REF: Page: 234 TOP: Nursing Process: Assessment MSC: NCLEX: Physiological Integrity 2. Disturbed body image is the nursing diagnosis for a patient diagnosed with an eating disorder. Which outcome indicator is most appropriate to monitor? a. Weight, muscle, and fat are congruent with height, frame, age, and sex. b. Calorie intake is within the required parameters of the treatment plan. c. Weight reaches the established normal range for the patient. d. Patient expresses satisfaction with body appearance. ANS: D Body image disturbances are considered improved or resolved when the patient is consistently satisfied with his or her own appearance and body function. This consideration is subjective. The other indicators are more objective but less related to the nursing diagnosis. DIF: Cognitive Level: Application (Applying) REF: Page: 236 TOP: Nursing Process: Outcomes Identification MSC: NCLEX: Psychosocial Integrity 3. A patient who is referred to the eating disorders clinic has lost 35 pounds in the past 3 months. To assess the patient’s oral intake, the nurse should ask: a. “Do you often feel fat?” b. “Who plans the family meals?” c. “What do you eat in a typical day?” d. “What do you think about your present weight?” ANS: C Although all the questions might be appropriate to ask, only “What do you eat in a typical day?” focuses on the eating patterns. Asking if the patient often feels fat focuses on distortions in body image. Questions about family meal planning are unrelated to eating patterns. Asking for the patient’s thoughts on present weight explores the patient’s feelings about weight. DIF: Cognitive Level: Application (Applying) REF: Pages: 234-235 TOP: Nursing Process: Assessment MSC: NCLEX: Psychosocial Integrity 4. A patient diagnosed with anorexia nervosa virtually stopped eating 5 months ago and has lost 25% of body weight. A nurse asks, “Describe what you think about your present weight and how you look.” Which response by the patient is most consistent with the diagnosis? a. “I am fat and ugly.” b. “What I think about myself is my business.” c. “I am grossly underweight, but that’s what I want.” d. “I am a few pounds overweight, but I can live with it.” ANS: A Patients diagnosed with anorexia nervosa do not recognize their thinness. They perceive themselves to be overweight and unattractive. The patient with anorexia will usually disclose perceptions about self to others. The patient with anorexia will persist in trying to lose more weight. DIF: Cognitive Level: Analysis (Analyzing) REF: Page: 231 | Pages: 233-235 TOP: Nursing Process: Assessment MSC: NCLEX: Psychosocial Integrity 5. A patient was diagnosed with anorexia nervosa. The history shows the patient virtually stopped eating 5 months ago and has lost 25% of body weight. The patient’s current serum potassium is 2.7 mg/dl. Which nursing diagnosis applies? a. Adult failure to thrive, related to abuse of laxatives as evidenced by electrolyte imbalances and weight loss b. Disturbed energy field, related to physical exertion in excess of energy produced through caloric intake as evidenced by weight loss and hyperkalemia c. Ineffective health maintenance, related to self-induced vomiting as evidenced by swollen parotid glands and hyperkalemia d. Imbalanced nutrition: less than body requirements, related to malnutrition as evidenced by loss of 25% of body weight and hypokalemia ANS: D The patient’s history and laboratory results support the correct nursing diagnosis. Available data do not confirm that the patient uses laxatives, induces vomiting, or exercises excessively. The patient has hypokalemia rather than hyperkalemia. DIF: Cognitive Level: Analysis (Analyzing) REF: Pages: 236-237 TOP: Nursing Process: Diagnosis MSC: NCLEX: Physiological Integrity 6. Outpatient treatment is planned for a patient diagnosed with anorexia nervosa. Select the most important outcome related to the nursing diagnosis: Imbalanced nutrition: less than body requirements. Within 1 week, the patient will: a. weigh self accurately using balanced scales. b. limit exercise to less than 2 hours daily. c. select clothing that fits properly. d. gain 1 to 2 pounds. ANS: D Only the outcome of a gain of 1 to 2 pounds can be accomplished within 1 week when the patient is an outpatient. The focus of an outcome is not on the patient weighing self. Limiting exercise and selecting proper clothing are important, but weight gain takes priority. DIF: Cognitive Level: Application (Applying) REF: Pages: 236-237 TOP: Nursing Process: Outcomes Identification MSC: NCLEX: Physiological Integrity 7. Which nursing intervention has priority as a patient diagnosed with anorexia nervosa begins to gain weight? a. Assess for depression and anxiety. b. Observe for adverse effects of re-feeding. c. Communicate empathy for the patient’s feelings. d. Help the patient balance energy expenditure and caloric intake. ANS: B The nursing intervention of observing for adverse effects of re-feeding most directly relates to weight gain and is a priority. Assessing for depression and anxiety and communicating empathy relate to coping. Helping the patient balance energy expenditure and caloric intake is an inappropriate intervention. DIF: Cognitive Level: Application (Applying) REF: Page: 236 TOP: Nursing Process: Implementation MSC: NCLEX: Physiological Integrity 8. A patient diagnosed with anorexia nervosa is resistant to weight gain. What is the rationale for establishing a contract with the patient to participate in measures designed to produce a specified weekly weight gain? a. Because severe anxiety concerning eating is expected, objective and subjective data must be routinely collected. b. Patient involvement in decision-making increases a sense of control and promotes compliance with the treatment. c. A team approach to planning the diet ensures that physical and emotional needs of the patient are met. d. Because of increased risk for physical problems with re-feeding, obtaining patient permission is required. ANS: B A sense of control for the patient is vital to the success of therapy. A diet that controls weight gain can allay patient fears of a too-rapid weight gain. Data collection is not the reason for contracting. A team approach is wise but is not a guarantee that the patient’s needs will be met. Permission for treatment is a separate issue. The contract for weight gain is an additional aspect of treatment. DIF: Cognitive Level: Application (Applying) REF: Pages: 236-237 TOP: Nursing Process: Planning MSC: NCLEX: Psychosocial Integrity 9. The nursing care plan for a patient diagnosed with anorexia nervosa includes the intervention “Monitor for complications of re-feeding.” Which body system should a nurse closely monitor for dysfunction? a. Renal b. Endocrine c. Central nervous d. Cardiovascular ANS: D Re-feeding resulting in a too-rapid weight gain can overwhelm the heart, resulting in cardiovascular collapse. Focused assessment becomes a necessity to ensure patient physiologic integrity. The other body systems are not initially involved in the re-feeding syndrome. DIF: Cognitive Level: Application (Applying) REF: Page: 236 TOP: Nursing Process: Assessment MSC: NCLEX: Physiological Integrity 10. A psychiatric clinical nurse specialist uses cognitive therapy techniques with a patient diagnosed with anorexia nervosa. Which statement by the staff nurse supports this type of therapy? a. “What are your feelings about not eating the food that you prepare?” b. “You seem to feel much better about yourself when you eat something.” c. “It must be difficult to talk about private matters to someone you just met.” d. “Being thin does not seem to solve your problems. You are thin now but still unhappy.” ANS: D The correct response is the only strategy that attempts to question the patient’s distorted thinking. DIF: Cognitive Level: Application (Applying) REF: Pages: 238-239 TOP: Nursing Process: Implementation MSC: NCLEX: Psychosocial Integrity 11. An appropriate intervention for a patient diagnosed with bulimia nervosa who binges and purges is to teach the patient to: a. eat a small meal after purging. b. avoid skipping meals or restricting food. c. concentrate oral intake after 4 PM daily. d. understand the value of reading journal entries aloud to others. ANS: B One goal of health teaching is the normalization of eating habits. Food restriction and skipping meals lead to rebound bingeing. Teaching the patient to eat a small meal after purging will probably perpetuate the need to induce vomiting. Teaching the patient to concentrate intake after 4 PM will lead to late-day bingeing. Journal entries are private. DIF: Cognitive Level: Application (Applying) REF: Pages: 241-242 TOP: Nursing Process: Implementation MSC: NCLEX: Physiological Integrity 12. What behavior by a nurse caring for a patient diagnosed with an eating disorder indicates the nurse needs supervision? a. The nurse’s comments are nonjudgmental. b. The nurse uses an authoritarian manner when interacting with the patient. c. The nurse teaches the patient to recognize signs of increasing anxiety and ways to intervene. d. The nurse refers the patient to a self-help group for individuals with eating disorders. ANS: B In the effort to motivate the patient and take advantage of the decision to seek help and be healthier, the nurse must take care not to cross the line toward authoritarianism and assume the role of a parent. The helpful nurse uses a problem-solving approach and focuses on the patient’s feelings of shame and low self-esteem. Referral to a self-help group is an appropriate intervention. DIF: Cognitive Level: Application (Applying) REF: Page: 233 TOP: Nursing Process: Evaluation MSC: NCLEX: Psychosocial Integrity 13. A nursing diagnosis for a patient diagnosed with bulimia nervosa is: Ineffective coping, related to feelings of loneliness as evidenced by overeating to comfort self, followed by self-induced vomiting. The best outcome related to this diagnosis is, “Within 2 weeks the patient will: a. appropriately express angry feelings.” b. verbalize two positive things about self.” c. verbalize the importance of eating a balanced diet.” d. identify two alternative methods of coping with loneliness.” ANS: D The outcome of identifying alternative coping strategies is most directly related to the diagnosis of Ineffective coping. Verbalizing positive characteristics of self and verbalizing the importance of eating a balanced diet are outcomes that might be used for other nursing diagnoses. Appropriately expressing angry feelings is not measurable. DIF: Cognitive Level: Application (Applying) REF: Pages: 240-242 TOP: Nursing Process: Outcomes Identification MSC: NCLEX: Psychosocial Integrity 14. Which nursing intervention has the highest priority for a patient diagnosed with bulimia nervosa? a. Assist the patient to identify triggers to binge eating. b. Provide corrective consequences for weight loss. c. Explore patient needs for health teaching. d. Assess for signs of impulsive eating. ANS: A For most patients with bulimia nervosa, certain situations trigger the urge to binge; purging then follows. The triggers are often anxiety-producing situations. Identifying these triggers makes it possible to break the binge-purge cycle. Because binge eating and purging directly affect physical status, the need to promote physical safety assumes the highest priority. The question calls for an intervention rather than an assessment. DIF: Cognitive Level: Analysis (Analyzing) REF: Pages: 241-242 TOP: Nursing Process: Planning MSC: NCLEX: Psychosocial Integrity 15. One bed is available on the inpatient eating disorders unit. Which patient should be admitted? The patient whose weight dropped from: a. 150 to 100 pounds over a 4-month period. Vital signs: temperature, 35.9° C; pulse, 38 beats/min; blood pressure, 60/40 mm Hg b. 120 to 90 pounds over a 3-month period. Vital signs: temperature, 36° C; pulse, 50 beats/min; blood pressure, 70/50 mm Hg c. 110 to 70 pounds over a 4-month period. Vital signs: temperature, 36.5° C; pulse, 60 beats/min; blood pressure, 80/66 mm Hg d. 90 to 78 pounds over a 5-month period. Vital signs: temperature, 36.7° C; pulse, 62 beats/min; blood pressure, 74/48 mm Hg ANS: A Physical criteria for hospitalization include weight loss of more than 30% of body weight within 6 months, temperature below 36° C (hypothermia), heart rate less than 40 beats/min, and systolic blood pressure less than 70 mm Hg. DIF: Cognitive Level: Analysis (Analyzing) REF: Pages: 235-236 TOP: Nursing Process: Assessment MSC: NCLEX: Safe, Effective Care Environment 16. While providing health teaching for a patient diagnosed with bulimia nervosa, a nurse should emphasize information about: a. self-monitoring of daily food and fluid intake. b. establishing the desired daily weight gain. c. recognizing symptoms of hypokalemia. d. self-esteem maintenance. ANS: C Hypokalemia results from potassium loss associated with vomiting. Physiologic integrity can be maintained if the patient can self-diagnose potassium deficiency and adjust the diet or seek medical assistance. Self-monitoring of daily food and fluid intake is not useful if the patient purges. Daily weight gain may not be desirable for a patient with bulimia nervosa. Self-esteem is an identifiable problem but is of lesser priority than the risk for hypokalemia. DIF: Cognitive Level: Application (Applying) REF: Page: 231 | Page: 234 TOP: Nursing Process: Implementation MSC: NCLEX: Physiological Integrity 17. As a patient admitted to the eating disorders unit undresses, a nurse observes that the patient’s body is covered by fine, downy hair. The patient weighs 70 pounds and is 5 feet, 4 inches tall. Which condition should be documented? a. Amenorrhea b. Alopecia c. Lanugo d. Stupor ANS: C The fine, downy hair noted by the nurse is called lanugo. It is frequently seen in patients with anorexia nervosa. None of the other conditions can be supported by the data the nurse has gathered. DIF: Cognitive Level: Comprehension (Understanding) REF: Page: 235 TOP: Nursing Process: Assessment MSC: NCLEX: Physiological Integrity 18. A patient being admitted to the eating disorders unit has a yellow cast to the skin and fine, downy hair covering the body. The patient weighs 70 pounds; height is 5 feet, 4 inches. The patient is quiet and says only, “I won’t eat until I look thin.” What is the priority initial nursing diagnosis? a. Anxiety, related to fear of weight gain b. Disturbed body image, related to weight loss c. Ineffective coping, related to lack of conflict resolution skills d. Imbalanced nutrition: less than body requirements, related to self-starvation ANS: D The physical assessment shows cachexia, which indicates imbalanced nutrition. Addressing the patient’s self-starvation is the priority above the incorrect responses. DIF: Cognitive Level: Analysis (Analyzing) REF: Page: 236 TOP: Nursing Process: Diagnosis| Nursing Process: Analysis MSC: NCLEX: Physiological Integrity 19. A nurse conducting group therapy on the eating disorders unit schedules the sessions immediately after meals for the primary purpose of: a. maintaining patients’ concentration and attention. b. shifting the patients’ focus from food to psychotherapy. c. focusing on weight control mechanisms and food preparation. d. processing the heightened anxiety associated with eating. ANS: D Eating produces high anxiety for patients with eating disorders. Anxiety levels must be lowered if the patient is to be successful in attaining therapeutic goals. Shifting the patients’ focus from food to psychotherapy and focusing on weight control mechanisms and food preparation are not desirable. Maintaining patients’ concentration and attention is important, but not the primary purpose of the schedule. DIF: Cognitive Level: Application (Applying) REF: Pages: 237-240 TOP: Nursing Process: Planning MSC: NCLEX: Psychosocial Integrity 20. Physical assessment of a patient diagnosed with bulimia nervosa often reveals: a. prominent parotid glands. b. peripheral edema. c. thin, brittle hair. d. amenorrhea. ANS: A Prominent parotid glands are associated with repeated vomiting. The other options are signs of anorexia nervosa and are not usually observed in bulimia. DIF: Cognitive Level: Comprehension (Understanding) REF: Page: 234 TOP: Nursing Process: Assessment MSC: NCLEX: Physiological Integrity 21. Which personality characteristic is a nurse most likely to assess in a patient diagnosed with anorexia nervosa? a. Carefree flexibility b. Rigidity, perfectionism c. Open displays of emotion d. High spirits and optimism ANS: B Rigid thinking, inability to demonstrate flexibility, and difficulty changing cognitions are characteristic of patients diagnosed with eating disorders. The incorrect options are rare in a patient with anorexia nervosa. Inflexibility, controlled emotions, and pessimism are more the norm. DIF: Cognitive Level: Comprehension (Understanding) REF: Pages: 233-237 TOP: Nursing Process: Assessment MSC: NCLEX: Psychosocial Integrity 22. Which assessment finding for a patient diagnosed with an eating disorder meets a criterion for hospitalization? a. Urine output: 40 ml/hr b. Pulse rate: 58 beats/min c. Serum potassium: 3.4 mEq/L d. Systolic blood pressure: 62 mm Hg ANS: D Systolic blood pressure less than 70 mm Hg is an indicator for inpatient care. Many people without eating disorders have bradycardia (pulse less than 60 beats/min). Urine output should be more than 30 ml/hr. A potassium level of 3.4 mEq/L is within the normal range. DIF: Cognitive Level: Analysis (Analyzing) REF: Page: 235 TOP: Nursing Process: Assessment MSC: NCLEX: Physiological Integrity 23. Which statement is a nurse most likely to hear from a patient diagnosed with anorexia nervosa? a. “I would be happy if I could lose 20 more pounds.” b. “My parents don’t pay much attention to me.” c. “I’m thin for my height.” d. “I have nice eyes.” ANS: A Patients with eating disorders have distorted body images and cognitive distortions. They see themselves as overweight even when their weight is subnormal. “I’m thin for my height” is therefore unlikely to be heard from a patient with anorexia nervosa. Poor self-image precludes making positive statements about self, such as “I have nice eyes.” Many patients with eating disorders see supportive others as intrusive and out of tune with their needs. DIF: Cognitive Level: Application (Applying) REF: Pages: 233-237 TOP: Nursing Process: Assessment MSC: NCLEX: Psychosocial Integrity 24. Which nursing diagnosis is more applicable for a patient diagnosed with anorexia nervosa who restricts intake and is 20% below normal weight than for a 130-pound patient diagnosed with bulimia nervosa who purges? a. Powerlessness b. Ineffective coping c. Disturbed body image d. Imbalanced nutrition: less than body requirements ANS: D The patient with bulimia nervosa usually maintains a close to normal weight, whereas the patient with anorexia nervosa may approach starvation. The incorrect options may be appropriate for patients with either anorexia nervosa or bulimia nervosa. DIF: Cognitive Level: Analysis (Analyzing) REF: Page: 236 | Page: 241 TOP: Nursing Process: Diagnosis| Nursing Process: Analysis MSC: NCLEX: Physiological Integrity 25. An outpatient diagnosed with anorexia nervosa has begun re-feeding. Between the first and second appointments, the patient gained 8 pounds. The nurse should: a. assess lung sounds and extremities. b. suggest the use of an aerobic exercise program. c. positively reinforce the patient for the weight gain. d. establish a higher goal for weight gain the next week. ANS: A Weight gain of more than 2 to 5 pounds weekly may overwhelm the heart’s capacity to pump, leading to cardiac failure. The nurse must assess for signs of pulmonary edema and congestive heart failure. The incorrect options are undesirable because they increase the risk for cardiac complications. DIF: Cognitive Level: Application (Applying) REF: Pages: 236-237 TOP: Nursing Process: Implementation MSC: NCLEX: Physiological Integrity 26. When a nurse finds a patient diagnosed with anorexia nervosa vigorously exercising before gaining the agreed-upon weekly weight, the nurse should state: a. “You and I will have to sit down and discuss this problem.” b. “It bothers me to see you exercising. You’ll lose more weight.” c. “Let’s discuss the relationship between exercise and weight loss and how that affects your body.” d. “According to our agreement, no exercising is permitted until you have gained a specific amount of weight.” ANS: D A matter-of-fact statement that the nurse’s perceptions are different helps avoid a power struggle. Treatment plans have specific goals for weight restoration. Exercise is limited to promote weight gain. Patients must be held accountable for required behaviors. DIF: Cognitive Level: Application (Applying) REF: Pages: 236-239 TOP: Nursing Process: Implementation MSC: NCLEX: Physiological Integrity 27. A patient diagnosed with anorexia nervosa has a body mass index (BMI) of 14.8 kg/m2. Which assessment finding is most likely to accompany this value? a. Cachexia b. Leukocytosis c. Hyperthermia d. Hypertension ANS: A The BMI value indicates extreme malnutrition. Cachexia is a hallmark of this problem. The patient would be expected to have leukopenia rather than leukocytosis. Hypothermia and hypotension are likely assessment findings. DIF: Cognitive Level: Analysis (Analyzing) REF: Page: 235 TOP: Nursing Process: Assessment MSC: NCLEX: Physiological Integrity MULTIPLE RESPONSE 1. A patient referred to the eating disorders clinic has lost 35 pounds in 3 months and has developed amenorrhea. For which physical manifestations of anorexia nervosa should a nurse assess? Select all that apply. a. Peripheral edema b. Parotid swelling c. Constipation d. Hypotension e. Dental caries f. Lanugo ANS: A, C, D, F Peripheral edema is often present because of hypoalbuminemia. Constipation related to starvation is often present. Hypotension is often present because of dehydration. Lanugo is often present and is related to starvation. Parotid swelling is associated with bulimia. Dental caries are associated with bulimia. DIF: Cognitive Level: Application (Applying) REF: Pages: 233-235 TOP: Nursing Process: Assessment MSC: NCLEX: Physiological Integrity 2. A patient diagnosed with anorexia nervosa is hospitalized for treatment. What features should the milieu provide? Select all that apply. a. Flexible mealtimes b. Unscheduled weight checks c. Adherence to a selected menu d. Observation during and after meals e. Monitoring during bathroom trips f. Privileges correlated with emotional expression ANS: C, D, E Priority milieu interventions support the restoration of weight and a normalization of eating patterns. These goals require close supervision of the patient’s eating habits and the prevention of exercise, purging, and other activities. Menus are strictly adhered to. Patients are observed during and after meals to prevent them from throwing away food or purging. All trips to the bathroom are monitored. Mealtimes are structured, not flexible. Weighing is performed on a regular schedule. Privileges are correlated with weight gain and treatment plan compliance. DIF: Cognitive Level: Application (Applying) REF: Pages: 236-240 TOP: Nursing Process: Planning MSC: NCLEX: Safe, Effective Care Environment Chapter 15 Chapter 15: Mood Disorders: Depression MULTIPLE CHOICE 1. A patient became severely depressed when the last of six children moved out of the home 4 months ago. The patient repeatedly says, “No one cares about me. I’m not worth anything.” Which response by the nurse would be the most helpful? a. “Things will look brighter soon. Everyone feels down once in a while.” b. “The staff here cares about you and wants to try to help you get better.” c. “It is difficult for others to care about you when you repeatedly say negative things about yourself.” d. “I’ll sit with you for 10 minutes now and return for 10 minutes at lunchtime and again at 2:30 this afternoon.” ANS: D Spending time with the patient at intervals throughout the day shows acceptance by the nurse and helps the patient establish a relationship with the nurse. The therapeutic technique is called offering self. Setting definite times for the therapeutic contacts and keeping the appointments show predictability on the part of the nurse, an element that fosters the building of trust. The incorrect responses would be difficult for a person with profound depression to believe, provide trite reassurance, and are counterproductive. The patient is unable to say positive things at this point. DIF: Cognitive Level: Application (Applying) 2. A patient became depressed after the last of six children moved out of the home 4 months ago. The patient has been self-neglectful, slept poorly, lost weight, and repeatedly says, “No one cares about me anymore. I’m not worth anything.” Select an appropriate initial outcome for the nursing diagnosis: Situational low self-esteem, related to feelings of abandonment. The patient will: a. verbalize realistic positive characteristics about self by (date) . b. consent to take antidepressant medication regularly by (date) . c. initiate social interaction with another person daily by (date) . d. identify two personal behaviors that alienate others by (date) . ANS: A Low self-esteem is reflected by making consistently negative statements about self and self-worth. Replacing negative cognitions with more realistic appraisals of self is an appropriate intermediate outcome. The incorrect options are not as clearly related to the nursing diagnosis. Outcomes are best when framed positively; identifying two personal behaviors that might alienate others is a negative concept. DIF: Cognitive Level: Application (Applying) REF: Page: 247 | Page: 259 Quiz 3. A nurse wants to reinforce positive self-esteem for a patient diagnosed with major depressive disorder. Today, the patient is wearing a new shirt and has neat, clean hair. Which remark is most appropriate? a. “You look nice this morning.” b. “You are wearing a new shirt.” c. “I like the shirt you’re wearing.” d. “You must be feeling better today.” ANS: B Patients with depression usually see the negative side of things. The meaning of compliments may be altered to “I didn’t look nice yesterday” or “They didn’t like my other shirt.” Neutral comments such as an observation avoid negative interpretations. Saying “You look nice” or “I like your shirt” gives approval (nontherapeutic techniques). Saying “You must be feeling better today” is an assumption, which is nontherapeutic. DIF: Cognitive Level: Application (Applying) REF: Pages: 259-261 | Pages: 263-264 Quiz 4. An adult diagnosed with major depressive disorder was treated with medication and cognitive behavioral therapy. The patient now recognizes how passivity contributed to the depression. Which intervention should the nurse suggest? a. Social skills training b. Relaxation training classes c. Use of complementary therapy d. Learning desensitization techniques ANS: A Social skills training is helpful in treating and preventing the recurrence of depression. Training focuses on assertiveness and coping skills that lead to positive reinforcement from others and the development of a patient’s support system. The use of complementary therapy refers to adjunctive therapies such as herbals. Assertiveness would be of greater value than relaxation training because passivity is a concern. Desensitization is used in the treatment of phobias. DIF: Cognitive Level: Analysis (Analyzing) REF: Pages: 259-261 Quiz 5. A priority nursing intervention for a patient diagnosed with major depressive disorder is: a. distracting the patient from self-absorption. b. carefully and inconspicuously observing the patient around the clock. c. allowing the patient to spend long periods alone in self-reflection. d. offering opportunities for the patient to assume a leadership role in the therapeutic milieu. ANS: B Approximately two thirds of people with depression contemplate suicide. Patients with depression who exhibit feelings of worthlessness are at higher risk. Regularly planned observations of the patient with depression may prevent a suicide attempt on the unit. DIF: Cognitive Level: Analysis (Analyzing) REF: Page: 255 | Pages: 259-260 Quiz 6. When counseling patients diagnosed with major depressive disorder, an advanced practice nurse will address the negative thought patterns by using: a. psychoanalytic therapy. b. desensitization therapy. c. cognitive behavioral therapy. d. alternative and complementary therapies. ANS: C Cognitive behavioral therapy attempts to alter the patient’s dysfunctional beliefs by focusing on positive outcomes rather than negative attributions. The patient is also taught the connection between thoughts and resultant feelings. Research shows that cognitive behavioral therapy involves the formation of new connections among nerve cells in the brain and that it is at least as effective as medication. Evidence does not support superior outcomes for the other psychotherapeutic modalities mentioned. DIF: Cognitive Level: Application (Applying) REF: Pages: 261-262 7. A patient says to the nurse, “My life does not have any happiness in it anymore. I once enjoyed holidays, but now they’re just another day.” How would the nurse document the complaint? a. Vegetative symptom b. Anhedonia c. Euphoria d. Anergia ANS: B Anhedonia is a common finding in many types of depression and refers to feelings of a loss of pleasure in formerly pleasurable activities. Vegetative symptoms refer to somatic changes associated with depression. Euphoria refers to an elated mood. Anergia means without energy. DIF: Cognitive Level: Comprehension (Understanding) REF: Page: 255 Quiz 8. A patient diagnosed with major depressive disorder is taking a tricyclic antidepressant. The patient says, “I don’t think I can keep taking these pills. They make me so dizzy, especially when I stand up.” The nurse should: a. explain how to manage postural hypotension, and educate the patient that side effects go away after several weeks. b. tell the patient that the side effects are a minor inconvenience compared with the feelings of depression. c. withhold the drug, force oral fluids, and notify the health care provider to examine the patient. d. teach the patient how to use pursed-lip breathing. ANS: A Drowsiness, dizziness, and postural hypotension usually subside after the first few weeks of therapy with tricyclic antidepressants. Postural hypotension can be managed by teaching the patient to stay well hydrated and rise slowly. Knowing these facts may be enough to convince the patient to remain medication compliant. The minor inconvenience of side effects as compared with feelings of depression is a convincing reason to remain on the medication. Withholding the drug, forcing oral fluids, and having the health care provider examine the patient are unnecessary steps. Independent nursing action is appropriate. Pursed-lip breathing is irrelevant. DIF: Cognitive Level: Application (Applying) REF: Pages: 265-267 Quiz 9. A patient diagnosed with major depressive disorder is receiving imipramine (Tofranil) 200 mg every night at bedtime. Which assessment finding would prompt the nurse to collaborate with the health care provider regarding potentially hazardous side effects of this drug? a. Dry mouth b. Blurred vision c. Nasal congestion d. Urinary retention ANS: D All the side effects mentioned are the result of the anticholinergic effects of the drug. Only urinary retention and severe constipation warrant immediate medical attention. Dry mouth, blurred vision, and nasal congestion may be less troublesome as therapy continues. DIF: Cognitive Level: Application (Applying) REF: Page: 266 Quiz 10. A patient diagnosed with major depressive disorder tells the nurse, “Bad things that happen are always my fault.” To assist the patient in reframing this overgeneralization, the nurse should respond: a. “I really doubt that one person can be blamed for all the bad things that happen.” b. “Let’s look at one bad thing that happened to see if another explanation exists.” c. “You are being exceptionally hard on yourself when you say those things.” d. “How does your belief in fate relate to your cultural heritage?” ANS: B By questioning a faulty assumption, the nurse can help the patient look at the premise more objectively and reframe it as a more accurate representation of fact. The incorrect responses are judgmental, irrelevant to an overgeneralization, and cast doubt without requiring the patient to evaluate the statement. DIF: Cognitive Level: Application (Applying) REF: Page: 261 | Pages: 263-264 Quiz 11. A nurse worked with a patient diagnosed with major depressive disorder who was severely withdrawn and dependent on others. After 3 weeks, the patient did not improve. The nurse is at risk for feelings of: a. overinvolvement. b. guilt and despair. c. interest and pleasure. d. ineffectiveness and frustration. ANS: D Nurses may have expectations for self and patients that are not wholly realistic, especially regarding the patient’s progress toward health. Unmet expectations result in feelings of ineffectiveness, anger, or frustration. Guilt and despair might be observed when the nurse experiences feelings about patients because of sympathy. Interest is possible but not the most likely result. The correct response is more global than overinvolvement. DIF: Cognitive Level: Comprehension (Understanding) REF: Page: 258 TOP: Nursing Process: Evaluation MSC: NCLEX: Psychosocial Integrity Quiz 12. A patient diagnosed with major depressive disorder begins selective serotonin reuptake inhibitor (SSRI) antidepressant therapy. Priority information given to the patient and family should include a directive to: a. avoid exposure to bright sunlight. b. report increased suicidal thoughts. c. restrict sodium intake to 1 g daily. d. maintain a tyramine-free diet. ANS: B Some evidence indicates that suicidal ideation may worsen at the beginning of antidepressant therapy; thus close monitoring is necessary. Avoiding exposure to bright sunlight and restricting sodium intake are unnecessary. Tyramine restriction is associated with monoamine oxidase inhibitor (MAOI) therapy. DIF: Cognitive Level: Application (Applying) REF: Page: 265 | Page: 270 13. A nurse teaching a patient about a tyramine-restricted diet would approve which meal? a. Mashed potatoes, ground beef patty, corn, green beans, apple pie b. Avocado salad, ham, creamed potatoes, asparagus, chocolate cake c. Macaroni and cheese, hot dogs, banana bread, caffeinated coffee d. Noodles with cheddar cheese sauce, smoked sausage, lettuce salad, yeast rolls ANS: A The correct answer describes a meal that contains little tyramine. Vegetables and fruits contain little or no tyramine, and fresh ground beef and apple pie should be safe. The other meals contain various amounts of tyramine-rich foods or foods that contain vasopressors: avocados, ripe bananas (banana bread), sausages and hot dogs, smoked meat (ham), cheddar cheese, yeast, caffeine drinks, and chocolate. DIF: Cognitive Level: Application (Applying) REF: Page: 272 14. What is the focus of priority nursing interventions for the period immediately after electroconvulsive therapy treatment? a. Supporting physiologic stability b. Reducing disorientation and confusion c. Monitoring pupillary responses d. Assisting the patient to identify and test negative thoughts ANS: A During the immediate post-treatment period, the patient is recovering from general anesthesia, hence the need to establish and support physiologic stability. Monitoring pupillary responses is not a priority. Reducing disorientation and confusion is an acceptable intervention but not the priority. Assisting the patient in identifying and testing negative thoughts is inappropriate in the immediate post-treatment period because the patient may be confused. DIF: Cognitive Level: Application (Applying) REF: Pages: 271-273 15. A nurse provided medication education for a patient who takes phenelzine (Nardil) for depression. Which behavior indicates effective learning? The patient: a. monitors sodium intake and weight daily. b. wears support stockings and elevates the legs when sitting. c. consults the pharmacist when selecting over-the-counter medications. d. can identify foods with high selenium content, which should be avoided. ANS: C Over-the-counter medicines may contain vasopressor agents or tyramine, a substance that must be avoided when the patient takes MAOI antidepressants. Medications for colds, allergies, or congestion or any preparation that contains ephedrine or phenylpropanolamine may precipitate a hypertensive crisis. MAOI antidepressant therapy is unrelated to the need for sodium limitation, support stockings, or leg elevation. MAOIs interact with tyramine-containing foods, not selenium, to produce dangerously high blood pressure. DIF: Cognitive Level: Analysis (Analyzing) REF: Pages: 270-271 | Page: 273 Quiz 16. A patient’s employment is terminated and major depressive disorder results. The patient says to the nurse, “I’m not worth the time you spend with me. I’m the most useless person in the world.” Which nursing diagnosis applies? a. Powerlessness b. Defensive coping c. Situational low self-esteem d. Disturbed personal identity ANS: C The patient’s statements express feelings of worthlessness and most clearly relate to the nursing diagnosis of Situational low self-esteem. Insufficient information exists to justify the other diagnoses. DIF: Cognitive Level: Application (Applying) REF: Page: 259 17. A patient diagnosed with major depressive disorder does not interact with others except when addressed and then only in monosyllables. The nurse wants to show nonjudgmental acceptance and support for the patient. Select the nurse’s most effective approach to communication. a. Make observations. b. Ask the patient direct questions. c. Phrase questions to require “yes” or “no” answers. d. Frequently reassure the patient to reduce guilt feelings. ANS: A Making observations about neutral topics such as the environment draws the patient into the reality around him or her but places no burdensome expectations on the patient for answers. Acceptance and support are shown by the nurse’s presence. Direct questions may make the patient feel that the encounter is an interrogation. Open-ended questions are preferable if the patient is able to participate in dialog. Platitudes are never acceptable; they minimize patient feelings and can increase feelings of worthlessness. DIF: Cognitive Level: Application (Applying) REF: Pages: 260-261 Quiz 18. A patient being treated for major depressive disorder has taken 300 mg amitriptyline (Elavil) daily for a year. The patient calls the case manager at the clinic and says, “I stopped taking my antidepressant 2 days ago. Now I am having cold sweats, nausea, a rapid heartbeat, and nightmares.” The nurse should advise the patient: a. “Go to the nearest emergency department immediately.” b. “Do not to be alarmed. Take two aspirin and drink plenty of fluids.” c. “Take one dose of the antidepressant. Come to the clinic to see the health care provider.” d. “Resume taking the antidepressant for 2 more weeks, and then discontinue it again.” ANS: C The patient has symptoms associated with abrupt withdrawal of the tricyclic antidepressant. Taking a dose of the drug will ameliorate the symptoms. Seeing the health care provider will allow the patient to discuss the advisability of going off the medication and to be given a gradual withdrawal schedule if discontinuation is the decision. This situation is not a medical emergency, although it calls for medical advice. Resuming taking the antidepressant for 2 more weeks and then discontinuing again would produce the same symptoms the patient is experiencing. DIF: Cognitive Level: Application (Applying) REF: Pages: 265-267 19. Which documentation indicates the treatment plan of a patient diagnosed with major depressive disorder was effective? a. Slept 6 hours uninterrupted. Sang with activity group. Anticipates seeing grandchild. b. Slept 10 hours uninterrupted. Attended craft group; stated “project was a failure, just like me.” c. Slept 5 hours with brief interruptions. Personal hygiene adequate with assistance. Weight loss of 1 pound. d. Slept 7 hours uninterrupted. Preoccupied with perceived inadequacies. States, “I feel tired all the time.” ANS: A Sleeping 6 hours, participating in a group activity, and anticipating an event are all positive happenings. All the other options show at least one negative finding. DIF: Cognitive Level: Analysis (Analyzing) REF: Page: 275 20. A woman gave birth to a healthy newborn 1 month ago. The patient now reports she cannot cope and is unable to sleep or eat. She says, “I feel like a failure. This baby is the root of my problems.” The priority nursing diagnosis is: a. Insomnia b. Ineffective coping c. Situational low self-esteem d. Risk for other-directed violence ANS: D When a new mother develops depression with a postpartum onset, ruminations or delusional thoughts about the infant often occur. The risk for harming the infant is increased; thus, it becomes the priority diagnosis. The other diagnoses are relevant but are of lower priority. DIF: Cognitive Level: Application (Applying) REF: Page: 253 21. A patient diagnosed with major depressive disorder repeatedly tells staff members, “I have cancer. It’s my punishment for being a bad person.” Diagnostic tests reveal no cancer. Select the priority nursing diagnosis. a. Powerlessness b. Risk for suicide c. Stress overload d. Spiritual distress ANS: B A patient with depression who feels so worthless as to believe cancer is deserved is at risk for suicide. Safety concerns take priority over the other diagnoses listed. DIF: Cognitive Level: Analysis (Analyzing) REF: Page: 255 | Page: 259 22. Which beverage should the nurse offer to a patient diagnosed with major depressive disorder who refuses solid food? a. Tomato juice b. Orange juice c. Hot tea d. Milk ANS: D Milk is the only beverage listed that provides protein, fat, and carbohydrates. In addition, milk is fortified with vitamins. DIF: Cognitive Level: Application (Applying) REF: Page: 262 23. During a psychiatric assessment, the nurse observes a patient’s facial expressions that are without emotion. The patient says, “Life feels so hopeless to me. I’ve been feeling sad for several months.” How should the nurse document the patient’s affect and mood? a. Affect depressed; mood flat b. Affect flat; mood depressed c. Affect labile; mood euphoric d. Affect and mood are incongruent ANS: B Mood is a person’s self-reported emotional feeling state. Affect is the emotional feeling state that is outwardly observable by others. DIF: Cognitive Level: Application (Applying) REF: Pages: 255-257 24. A disheveled patient with severe depression and psychomotor retardation has not bathed for several days. The nurse should: a. avoid forcing the issue. b. bring up the issue at the community meeting. c. calmly tell the patient, “You must bathe daily.” d. firmly and neutrally assist the patient with showering. ANS: D When patients are unable to perform self-care activities, staff members must assist them rather than ignore the issue. Better grooming increases self-esteem. Calmly telling the patient to bathe daily and bringing up the issue at a community meeting are punitive. DIF: Cognitive Level: Application (Applying) REF: Pages: 260-262 25. A patient was started on escitalopram (Lexapro) 5 days ago and now says, “This medicine isn’t working.” The nurse’s best intervention would be to: a. discuss with the health care provider the need to change medications. b. reassure the patient that the medication will be effective soon. c. explain the time lag before antidepressants relieve symptoms. d. critically assess the patient for symptom relief. ANS: C Escitalopram is an SSRI antidepressant. Between 1 and 3 weeks of treatment are usually necessary before a relief of symptoms occurs. This information is important to share with patients. DIF: Cognitive Level: Analysis (Analyzing) REF: Pages: 267-270 26. A nurse is caring for a patient with low self-esteem. Which nonverbal communication should the nurse anticipate? a. Arms crossed b. Staring at the nurse c. Smiling inappropriately d. Eyes pointed downward ANS: D Nonverbal communication is usually considered more powerful than verbal communication. Downward-casted eyes suggest feelings of worthlessness or hopelessness. DIF: Cognitive Level: Application (Applying) REF: Page: 260 27. A patient diagnosed with major depressive disorder was hospitalized for 8 days. Treatment included six electroconvulsive therapy sessions and aggressive dose adjustments of antidepressant medications. The patient owns a small business and was counseled not to make major decisions for a month. Select the correct rationale for this counseling. a. Temporary memory impairments and confusion can be associated with electroconvulsive therapy. b. Antidepressant medications alter catecholamine levels, which impair decision-making abilities. c. Antidepressant medications may cause confusion related to a limitation of tyramine in the diet. d. The patient needs time to reorient himself or herself to a pressured work schedule. ANS: A Recent memory impairment or confusion or both are often present during and for a short time after electroconvulsive therapy. An inappropriate business decision might be made because of forgotten and important details. The incorrect responses contain rationales that are untrue. The patient needing time to reorient himself or herself to a pressured work schedule is less relevant than the correct rationale. DIF: Cognitive Level: Application (Applying) REF: Pages: 271-273 28. A nurse instructs a patient taking a drug that inhibits the action of monoamine oxidase (MAO) to avoid certain foods and drugs because of the risk of: a. hypotensive shock. b. hypertensive crisis. c. cardiac dysrhythmia. d. cardiogenic shock. ANS: B Patients taking MAOIs must be on a tyramine-free diet to prevent hypertensive crisis. In the presence of MAOIs, tyramine is not destroyed by the liver and, in high levels, produces intense vasoconstriction, resulting in elevated blood pressure. DIF: Cognitive Level: Comprehension (Understanding) REF: Page: 271 MULTIPLE RESPONSE 1. The admission note indicates a patient diagnosed with major depressive disorder has anergia and anhedonia. For which measures should the nurse plan? Select all that apply. a. Channeling excessive energy b. Reducing guilty ruminations c. Instilling a sense of hopefulness d. Assisting with self-care activities e. Accommodating psychomotor retardation ANS: C, D, E Anhedonia refers to the inability to find pleasure or meaning in life; thus planning should include measures to accommodate psychomotor retardation, assist with activities of daily living, and instill hopefulness. Anergia is the lack of energy, not excessive energy. Anhedonia does not necessarily imply the presence of guilty ruminations. DIF: Cognitive Level: Application (Applying) REF: Pages: 255-264 2. A student nurse caring for a patient diagnosed with major depressive disorder reads in the patient’s medical record, “This patient shows vegetative signs of depression.” Which nursing diagnoses most clearly relate to the vegetative signs? Select all that apply. a. Imbalanced nutrition: less than body requirements b. Chronic low self-esteem c. Sexual dysfunction d. Self-care deficit e. Powerlessness f. Insomnia ANS: A, C, D, F Vegetative signs of depression are alterations in the body processes necessary to support life and growth, such as eating, sleeping, elimination, and sexual activity. These diagnoses are more closely related to vegetative signs than to diagnoses associated with feelings about self. DIF: Cognitive Level: Application (Applying) REF: Pages: 258-259 3. A patient diagnosed with major depressive disorder will begin electroconvulsive therapy tomorrow. Which interventions are routinely implemented before the treatment? Select all that apply. a. Administer pretreatment medication 30 to 45 minutes before treatment. b. Withhold food and fluids for a minimum of 6 hours before treatment. c. Remove dentures, glasses, contact lenses, and hearing aids. d. Restrain the patient in bed with padded limb restraints. e. Assist the patient to prepare an advance directive. ANS: A, B, C The correct interventions reflect routine electroconvulsive therapy preparation, which is similar to preoperative preparation: sedation and anticholinergic medication before anesthesia, maintaining nothing-by-mouth status to prevent aspiration during and after treatment, airway maintenance, and general safety by removing prosthetic devices. Restraint is not part of the pretreatment protocol. An advance directive is prepared independent of this treatment. DIF: Cognitive Level: Application (Applying) REF: Pages: 271-273 4. A patient diagnosed with major depressive disorder shows vegetative signs of depression. Which nursing actions should be implemented? Select all that apply. a. Offer laxatives, if needed. b. Monitor food and fluid intake. c. Provide a quiet sleep environment. d. Eliminate all daily caffeine intake. e. Restrict the intake of processed foods. ANS: A, B, C The correct options promote a normal elimination pattern. Although excessive intake of stimulants such as caffeine may make the patient feel jittery and anxious, small amounts may provide useful stimulation. No indication exists that processed foods should be restricted. DIF: Cognitive Level: Application (Applying) REF: Pages: 258-259 | Page: 262 5. A patient being treated with paroxetine (Paxil) 50 mg/day orally for major depressive disorder reports to the clinic nurse, “I took a few extra tablets earlier in the day and now I feel bad.” Which aspects of the nursing assessment are most critical? Select all that apply. a. Vital signs b. Urinary frequency c. Increased suicidal ideation d. Presence of abdominal pain and diarrhea e. Hyperactivity or feelings of restlessness ANS: A, D, E The patient is taking the maximum dose of this SSRI and has ingested an additional unknown amount of the drug. Central serotonin syndrome must be considered. Symptoms include abdominal pain, diarrhea, tachycardia, elevated blood pressure, hyperpyrexia, increased motor activity, and muscle spasms. Central serotonin syndrome may progress to a full medical emergency if not treated early. Although assessing for suicidal ideation is never inappropriate, in this situation physiologic symptoms should be the initial focus. The patient may have urinary retention, but frequency would not be expected. DIF: Cognitive Level: Analysis (Analyzing) REF: Page: 267 | Page: 270 Chapter 16 Chapter 16: Bipolar Spectrum Disorders MULTIPLE CHOICE 1. A person is directing traffic on a busy street while shouting and making obscene gestures at passing cars. The person has not slept or eaten for 3 days. What features of mania are evident? a. Increased muscle tension and anxiety b. Vegetative signs and poor grooming c. Poor judgment and hyperactivity d. Cognitive deficit and sad mood ANS: C Hyperactivity (directing traffic) and poor judgment (putting self in a dangerous position) are characteristic of manic episodes. The distractors do not specifically apply to mania. DIF: Cognitive Level: Comprehension (Understanding) REF: Pages: 282-287 2. A patient diagnosed with bipolar disorder is dressed in a red leotard and brightly colored scarves. The patient says, “I’ll punch you, munch you, crunch you,” while twirling and shadowboxing. Then the patient says gaily, “Do you like my scarves? Here…they are my gift to you.” How should the nurse document the patient’s mood? a. Labile and euphoric b. Irritable and belligerent c. Highly suspicious and arrogant d. Excessively happy and confident ANS: A The patient has demonstrated angry behavior and pleasant, happy behavior within seconds of each other. Excessive happiness indicates euphoria. Mood swings are often rapid and seemingly without understandable reason in patients who are manic. These swings are documented as labile. Irritability, belligerence, excessive happiness, and confidence are not entirely correct terms for the patient’s mood. A high level of suspicion is not evident. DIF: Cognitive Level: Application (Applying) REF: Pages: 282-284 3. A patient experiencing mania has not eaten or slept for 3 days. Which nursing diagnosis has priority? a. Risk for injury b. Ineffective coping c. Impaired social interaction d. Ineffective therapeutic regimen management ANS: A Although each of the nursing diagnoses listed is appropriate for a patient having a manic episode, the priority lies with the patient’s physiologic safety. Hyperactivity and poor judgment place the patient at risk for injury. DIF: Cognitive Level: Application (Applying) REF: Pages: 288-289 4. A patient diagnosed with bipolar disorder is hyperactive and manic after discontinuing lithium. The patient threatens to hit another patient. Which comment by the nurse is appropriate? a. “Stop that! No one did anything to provoke an attack by you.” b. “If you do that one more time, you will be secluded immediately.” c. “Do not hit anyone. If you are unable to control yourself, we will help you.” d. “You know we will not let you hit anyone. Why do you continue this behavior?” ANS: C When the patient is unable to control his or her behavior and violates or threatens to violate the rights of others, limits must be set in an effort to de-escalate the situation. Limits should be set in simple, concrete terms. The incorrect responses do not offer appropriate assistance to the patient and threaten the patient with seclusion as punishment. Asking “why” does not provide for environmental safety. DIF: Cognitive Level: Application (Applying) REF: Pages: 290-293 5. This nursing diagnosis applies to a patient experiencing mania: Imbalanced nutrition: less than body requirements, related to insufficient caloric intake and hyperactivity as evidenced by 5-pound weight loss in 4 days. Select the most appropriate outcome. The patient will: a. ask staff for assistance with feeding within 4 days. b. drink six servings of a high-calorie, high-protein drink each day. c. consistently sit with others for at least 30 minutes at mealtime within 1 week. d. consistently wear appropriate attire for age and sex within 1 week while in the psychiatric unit. ANS: B High-calorie, high-protein food supplements will provide the additional calories needed to offset the patient’s extreme hyperactivity. Sitting with others or asking for assistance does not mean the patient will eat or drink. Appropriate attire is unrelated to the nursing diagnosis. DIF: Cognitive Level: Application (Applying) REF: Page: 291 6. A patient develops mania after discontinuing lithium. New prescriptions are written to resume lithium twice daily and begin olanzapine (Zyprexa). The addition of olanzapine to the medication regimen will: a. minimize the side effects of lithium. b. bring hyperactivity under rapid control. c. enhance the antimanic actions of lithium. d. provide long-term control of hyperactivity. ANS: B Manic symptoms are controlled by lithium only after a therapeutic serum level is attained. Because this takes several days to accomplish, a drug with rapid onset is necessary to reduce the hyperactivity initially. Antipsychotic drugs neither enhance lithium’s antimanic activity nor minimize the side effects. Lithium is used for long-term control. DIF: Cognitive Level: Application (Applying) REF: Page: 294 7. A patient diagnosed with bipolar disorder has rapid cycles. The health care provider prescribes an anticonvulsant medication. To prepare teaching materials, which drug should the nurse anticipate will be prescribed? a. phenytoin (Dilantin) b. clonidine (Catapres) c. carbamazepine (Tegretol) d. chlorpromazine (Thorazine) ANS: C Some patients with bipolar disorder, especially those who have only short periods between episodes, have a favorable response to the anticonvulsants carbamazepine and valproate. Phenytoin is also an anticonvulsant but is not used for mood stabilization. Carbamazepine seems to work better in patients with rapid cycling and in severely paranoid, angry patients with manic episodes. DIF: Cognitive Level: Application (Applying) REF: Pages: 295-297 8. The cause of bipolar disorder has not been determined, but: a. several factors, including genetics, are implicated. b. brain structures were altered by stresses early in life. c. excess norepinephrine is probably a major factor. d. excess sensitivity in dopamine receptors may exist. ANS: A At this time, the interplay of complex independent variables is most likely the best explanation of the cause for bipolar disorder. Various theories implicate genetics, endocrine imbalance, early stress, and neurotransmitter imbalances. DIF: Cognitive Level: Comprehension (Understanding) REF: Pages: 281-282 9. The spouse of a patient diagnosed with bipolar disorder asks what evidence supports the possibility of genetic transmission of bipolar disorders. Select the nurse’s best response. a. “A high proportion of patients diagnosed with bipolar disorders are found among creative writers.” b. “A higher rate of relatives diagnosed with bipolar disorder is found among patients with bipolar disorder.” c. “Patients diagnosed with bipolar disorder have higher rates of relatives who respond in an exaggerated way to daily stresses.” d. “More individuals diagnosed with bipolar disorder come from high socioeconomic and educational backgrounds.” ANS: B Evidence of genetic transmission is supported when twins or relatives of patients with a particular disorder also show an incidence of the disorder that is higher than the incidence in the general public. The incorrect options do not support the theory of genetic transmission of bipolar disorder. DIF: Cognitive Level: Application (Applying) REF: Pages: 281-282 10. A patient diagnosed with bipolar disorder commands other patients, “Get me a book. Take this stuff out of here,” and other similar demands. The nurse wants to interrupt this behavior without entering into a power struggle. Select the best initial approach by the nurse. a. Distraction: “Let’s go to the dining room for a snack.” b. Humor: “How much are you paying servants these days?” c. Limit setting: “You must stop ordering other patients around.” d. Honest feedback: “Your controlling behavior is annoying others.” ANS: A The distractibility characteristic of manic episodes can assist the nurse to direct the patient toward more appropriate, constructive activities without entering into a power struggle. Humor usually backfires by either encouraging the patient or inciting anger. Limit setting and honest feedback may seem heavy-handed to a labile patient and may incite anger. DIF: Cognitive Level: Application (Applying) REF: Page: 290 | Pages: 292-293 11. A nurse receives this laboratory result for a patient diagnosed with bipolar disorder: lithium level 1 mEq/L. This result is: a. within therapeutic limits b. below therapeutic limits c. above therapeutic limits d. incorrect because of inaccurate testing ANS: A The normal range for a blood sample taken 8 to 12 hours after the last dose of lithium is 0.4 to 1 mEq/L. DIF: Cognitive Level: Comprehension (Understanding) REF: Pages: 294-295 12. Consider these three drugs: divalproex (Depakote), carbamazepine (Tegretol), and gabapentin (Neurontin). Which drug also belongs to this group? a. clonazepam (Klonopin) b. risperidone (Risperdal) c. lamotrigine (Lamictal) d. aripiprazole (Abilify) ANS: C The three drugs in the stem of this question are all anticonvulsants. Lamotrigine is also an anticonvulsant. Clonazepam is an anxiolytic; aripiprazole and risperidone are antipsychotic drugs. DIF: Cognitive Level: Comprehension (Understanding) REF: Page: 297 13. When a hyperactive patient experiencing acute mania is hospitalized, what initial nursing intervention is a priority? a. Allow the patient to act out his or her feelings. b. Set limits on the patient’s behavior as necessary. c. Provide verbal instructions to the patient to remain calm. d. Restrain the patient to reduce hyperactivity and aggression. ANS: B This intervention provides support through the nurse’s presence and provides structure as necessary while the patient’s control is tenuous. Acting out may lead to the loss of behavioral control. The patient will probably be unable to focus on instructions and comply. Restraint is used only after other interventions have proved ineffective. DIF: Cognitive Level: Application (Applying) REF: Pages: 290-291 14. At a unit meeting, staff members discuss the decor for a special room for patients experiencing mania. Select the best option. a. Extra-large window with a view of the street b. Neutral walls with pale, simple accessories c. Brightly colored walls and print drapes d. Deep colors for walls and upholstery ANS: B The environment for a patient experiencing mania should be as simple and as nonstimulating as possible. Patients experiencing mania are highly sensitive to environmental distractions and stimulation. Draperies present a risk for injury. DIF: Cognitive Level: Application (Applying) REF: Pages: 289-291 15. A patient experiencing acute mania has exhausted the staff members by noon. The patient has joked, manipulated, insulted, and fought all morning. Staff members are feeling defensive and fatigued. Which is the best action? a. Confer with the health care provider regarding use of seclusion for this patient. b. Hold a staff meeting to discuss consistency and limit-setting approaches. c. Conduct a meeting with all patients to discuss the behavior. d. Explain to the patient that the behavior is unacceptable. ANS: B When staff members are overwhelmed, the patient has succeeded in keeping the environment unsettled and avoided outside controls on behavior. Staff meetings can help minimize staff splitting and feelings of anger, helplessness, confusion, and frustration. Criteria for seclusion have not been met. DIF: Cognitive Level: Application (Applying) REF: Pages: 289-290 16. A patient experiencing acute mania undresses in the group room and dances. The nurse’s first intervention would be to: a. quietly ask the patient, “Why don’t you put on your clothes?” b. firmly tell the patient, “Stop dancing, and put on your clothing.” c. put a blanket around the patient, and walk with the patient to a quiet room. d. allow the patient stay in the group room. Move the other patients to a different area. ANS: C Patients must be protected from the embarrassing consequences of their poor judgment whenever possible. Protecting the patient from public exposure by matter-of-factly covering the patient and removing him or her from the area with a sufficient number of staff members to avoid argument and provide control is an effective approach. DIF: Cognitive Level: Application (Applying) REF: Page: 285 | Pages: 290-291 17. A patient experiencing acute mania waves a newspaper and says, “I must have my credit card and use the computer right now. A store is having a big sale and I need to order 10 dresses and four pairs of shoes.” Select the nurse’s most appropriate intervention. a. Suggest to the patient to ask a friend do the shopping and bring purchases to the unit. b. Invite the patient to sit with the nurse and look at new fashion magazines. c. Tell the patient that computer use is not allowed until self-control improves. d. Ask whether the patient has enough money to pay for the purchases. ANS: B Situations such as this offer an opportunity to use the patient’s distractibility to the staff’s advantage. Patients become frustrated when staff members deny requests that the patient sees as entirely reasonable. Distracting the patient can avoid power struggles. Suggesting that a friend do the shopping would not satisfy the patient’s need for immediacy and would ultimately result in the extravagant expenditure. Asking whether the patient has enough money would likely precipitate an angry response. DIF: Cognitive Level: Analysis (Analyzing) REF: Page: 284 | Page: 289 18. A patient diagnosed with bipolar disorder is being treated on an outpatient basis with lithium carbonate 300 mg three times daily. The patient complains of nausea. To reduce the nausea, the nurse can suggest that the lithium be taken with: a. meals. b. an antacid. c. a large glass of juice. d. an antiemetic medication. ANS: A Some patients find that taking lithium with meals diminishes nausea. The incorrect options are less helpful. DIF: Cognitive Level: Application (Applying) REF: Page: 296 19. A health teaching plan for a patient taking lithium should include instructions to: a. maintain normal salt and fluids in the diet. b. drink twice the usual daily amount of fluids. c. double the lithium dose if diarrhea or vomiting occurs. d. avoid eating aged cheese, processed meats, and red wine. ANS: A Sodium depletion and dehydration increase the chance for developing lithium toxicity. The incorrect options offer inappropriate information. DIF: Cognitive Level: Application (Applying) REF: Page: 296 20. Which nursing diagnosis would most likely apply to both a patient diagnosed with major depressive disorder (MDD) as well as one experiencing acute mania? a. Deficient diversional activity b. Disturbed sleep pattern c. Fluid volume excess d. Defensive coping ANS: B Patients diagnosed with mood disorders, both depression and mania, experience sleep pattern disturbances. Assessment data should be routinely gathered about this possible problem. Deficient diversional activity is more relevant for patients diagnosed with MDD. Defensive coping is more relevant for patients experiencing mania. Fluid volume excess is less relevant for patients diagnosed with mood disorders than is deficient fluid volume. DIF: Cognitive Level: Analysis (Analyzing) REF: Page: 288 21. Which dinner menu is best suited for the patient diagnosed with bipolar disorder experiencing acute mania? a. Spaghetti and meatballs, salad, a banana b. Beef and vegetable stew, a roll, chocolate pudding c. Broiled chicken breast on a roll, an ear of corn, apple d. Chicken casserole, green beans, flavored gelatin with whipped cream ANS: C The correct foods provide adequate nutrition but, more importantly, are finger foods that the hyperactive patient could “eat on the run.” The foods in the incorrect options cannot be eaten without utensils. DIF: Cognitive Level: Application (Applying) REF: Page: 291 22. Outcome identification for the treatment plan of a patient with grandiose thinking associated with acute mania focuses on: a. maintaining an interest in the environment. b. developing an optimistic outlook. c. self-control of distorted thinking. d. stabilizing the sleep pattern. ANS: C The desired outcome is that the patient will be able to control the grandiose thinking associated with acute mania as evidenced by making realistic comments about self, abilities, and plans. Patients with acute mania are already unduly optimistic as a result of their use of denial, and they are overly interested in their environment. Sleep stability is a desired outcome but is not related to distorted thought processes. DIF: Cognitive Level: Application (Applying) REF: Pages: 288-289 23. Which documentation indicates that the treatment plan for a patient experiencing acute mania has been effective? a. “Converses without interrupting; clothing matches; participates in activities.” b. “Irritable, suggestible, distractible; napped for 10 minutes in afternoon.” c. “Attention span short; writing copious notes; intrudes in conversations.” d. “Heavy makeup; seductive toward staff; pressured speech.” ANS: A The descriptors given indicate the patient is functioning at an optimal level, using appropriate behavior, and thinking without becoming overstimulated by unit activities. The incorrect options reflect manic behavior. DIF: Cognitive Level: Application (Applying) REF: Pages: 299-300 24. A patient experiencing mania dances around the unit, seldom sits, monopolizes conversations, interrupts, and intrudes. Which nursing intervention will best assist the patient with energy conservation? a. Monitor physiologic functioning b. Provide a subdued environment c. Supervise personal hygiene d. Observe for mood changes ANS: B All the options are reasonable interventions with a patient with acute mania, but providing a subdued environment directly relates to the outcome of energy conservation by decreasing stimulation and helping balance activity and rest. DIF: Cognitive Level: Application (Applying) REF: Pages: 290-293 25. A patient diagnosed with bipolar disorder has been hospitalized for 7 days and has taken lithium 600 mg three times daily. Staff members observe increased agitation, pressured speech, poor personal hygiene, hyperactivity, and bizarre clothing. What is the nurse’s best intervention? a. Educate the patient about the proper ways to perform personal hygiene and coordinate clothing. b. Continue to monitor and document the patient’s speech patterns and motor activity. c. Ask the health care provider to prescribe an increased dose and frequency of lithium. d. Consider the need to check the lithium level. The patient may not be swallowing medications. ANS: D The patient is continuing to exhibit manic symptoms. The lithium level may be low as a result of “cheeking” the medication. The prescribed dose is high, so one would not expect a need for the dose to be increased. DIF: Cognitive Level: Analysis (Analyzing) REF: Pages: 294-295 26. A patient experiencing acute mania has disrobed in the hall three times in 2 hours. The nurse should: a. direct the patient to wear clothes at all times. b. ask if the patient finds clothes bothersome. c. tell the patient that others feel embarrassed. d. arrange for one-on-one supervision. ANS: D A patient who repeatedly disrobes, despite verbal limit setting, needs more structure. One-on-one supervision may provide the necessary structure. Directing the patient to wear clothes at all times has not proved successful, considering the behavior has continued. Asking whether the patient is bothered by clothing serves no purpose. Telling the patient that others are embarrassed will not make a difference to the patient whose grasp of social behaviors is impaired by the illness. DIF: Cognitive Level: Application (Applying) REF: Pages: 290-293 27. A patient experiencing acute mania is dancing atop the pool table in the recreation room. The patient waves a cue in one hand and says, “I’ll throw the pool balls if anyone comes near me.” The nurse’s first intervention is to: a. tell the patient, “You need to be secluded.” b. help the patient down from the table. c. clear the room of all other patients. d. assemble a show of force. ANS: C Safety is of primary importance. Once other patients are out of the room, a plan for managing this patient can be implemented. A show of force is likely to frighten the patient and increase this risk for violence. DIF: Cognitive Level: Analysis (Analyzing) REF: Pages: 288-291 28. After hospital discharge, what is the priority intervention for a patient diagnosed with bipolar disorder who is taking antimanic medication, as well as for the patient’s family? a. Decreasing physical activity b. Increasing food and fluids c. Meeting self-care needs d. Psychoeducation ANS: D During the continuation phase of treatment for bipolar disorder, the physical needs of the patient are not as important an issue as they were during the acute episode. After hospital discharge, the treatment focuses on maintaining medication compliance and preventing a relapse, both of which are fostered by ongoing psychoeducation. DIF: Cognitive Level: Application (Applying) REF: Page: 296 | Page: 298 29. A patient receiving lithium should be assessed for which evidence of complications? a. Pharyngitis, mydriasis, and dystonia b. Alopecia, purpura, and drowsiness c. Diaphoresis, weakness, and nausea d. Ascites, dyspnea, and edema ANS: C Diaphoresis, weakness, and nausea are early signs of lithium toxicity. Problems mentioned in the incorrect options are unrelated to lithium therapy. DIF: Cognitive Level: Application (Applying) REF: Pages: 294-295 30. A patient diagnosed with bipolar disorder is in the maintenance phase of treatment. The patient asks, “Do I have to keep taking this lithium even though my mood is stable now?” Select the nurse’s most appropriate response. a. “You will be able to stop the medication in approximately 1 month.” b. “Taking the medication every day helps prevent relapses and recurrences.” c. “Usually patients take this medication for approximately 6 months after discharge.” d. “It’s unusual that the health care provider has not already stopped your medication.” ANS: B Patients diagnosed with bipolar disorder may be indefinitely maintained on lithium to prevent recurrences. Helping the patient understand this need promotes medication compliance. The incorrect options offer incorrect or misleading information. DIF: Cognitive Level: Application (Applying) REF: Page: 295 31. A patient diagnosed with bipolar disorder and who takes lithium telephones the nurse at the clinic to say, “I’ve had severe diarrhea 4 days. I feel very weak and unsteady when I walk. My usual hand tremor has gotten worse. What should I do?” The nurse should advise the patient: a. “Restrict oral fluids for 24 hours and stay in bed.” b. “Have someone bring you to the clinic immediately.” c. “Drink a large glass of water with 1 teaspoon of salt added.” d. “Take an over-the-counter antidiarrheal medication hourly until the diarrhea subsides.” ANS: B The symptoms described suggest lithium toxicity. The patient should have a lithium level drawn and may require further treatment. Because neurologic symptoms are present, the patient should not drive and should be accompanied by another person. The incorrect options will not address the patient’s symptoms. Restricting oral fluids will make the situation worse. DIF: Cognitive Level: Application (Applying) REF: Pages: 294-295 32. Lithium is prescribed for a new patient. Which information from the patient’s history indicates that monitoring serum concentrations of the drug will be especially challenging and critical? a. Arthritis b. Epilepsy c. Psoriasis d. Congestive heart failure ANS: D The patient with congestive heart failure will likely need diuretic drugs, which will complicate the maintenance of the fluid balance necessary to avoid lithium toxicity. Arthritis, epilepsy, and psoriasis do not directly involve fluid balance and kidney function. DIF: Cognitive Level: Analysis (Analyzing) REF: Pages: 294-295 MULTIPLE RESPONSE 1. A patient diagnosed with bipolar disorder is being treated as an outpatient during a hypomanic episode. Which suggestions should the nurse provide to the family? Select all that apply. a. Provide structure b. Limit credit card access c. Encourage group social interaction d. Limit work to half days e. Monitor the patient’s sleep patterns ANS: A, B, E A patient with hypomania is expansive, grandiose, and labile; uses poor judgment; spends inappropriately; and is overstimulated by a busy environment. Providing structure helps the patient maintain appropriate behavior. Financial irresponsibility may be avoided by limiting access to cash and credit cards. Continued decline in sleep patterns may indicate the condition has evolved to full mania. Group socialization should be kept to a minimum to reduce stimulation. A full leave of absence from work is necessary to limit stimuli and to prevent problems associated with poor judgment and the inappropriate decision making that accompany hypomania. DIF: Cognitive Level: Application (Applying) REF: Pages: 290-291 | Page: 298 2. A nurse prepares the plan of care for a patient having a manic episode. Which nursing diagnoses are most likely? Select all that apply. a. Imbalanced nutrition: more than body requirements b. Disturbed thought processes c. Sleep deprivation d. Chronic confusion e. Social isolation ANS: B, C People with mania are hyperactive and often do not take the time to eat and drink properly. Their high levels of activity consume calories; therefore deficits in nutrition may occur. Sleep is reduced. Their socialization is impaired but not isolated. Confusion may be acute but not chronic. DIF: Cognitive Level: Application (Applying) REF: Page: 288 3. A patient tells the nurse, “I am so ashamed of being bipolar. When I’m manic, my behavior embarrasses my family. Even if I take my medication, there’s no guarantee I won’t have a relapse. I am such a burden to my family.” These statements support which nursing diagnoses? Select all that apply. a. Powerlessness b. Defensive coping c. Chronic low self-esteem d. Impaired social interaction e. Risk-prone health behavior ANS: A, C Chronic low self-esteem and powerlessness are interwoven in the patient’s statements. No data support the other diagnoses. DIF: Cognitive Level: Analysis (Analyzing) REF: Page: 288 TOP: Nursing Process: Diagnosis| Nursing Process: Analysis MSC: NCLEX: Psychosocial Integrity Chapter 17 Chapter 17: Schizophrenia Spectrum Disorders MULTIPLE CHOICE 1. A person diagnosed with schizophrenia has had difficulty keeping a job because of arguing with co-workers and accusing them of conspiracy. Today the person shouts, “They’re all plotting to destroy me.” Select the nurse’s most therapeutic response. a. “Everyone here is trying to help you. No one wants to harm you.” b. “Feeling that people want to destroy you must be very frightening.” c. “That is not true. People here are trying to help if you will let them.” d. “Staff members are health care professionals who are qualified to help you.” ANS: B Resist focusing on content; instead, focus on the feelings the patient is expressing. This strategy prevents arguing about the reality of delusional beliefs. Such arguments increase patient anxiety and the tenacity with which the patient holds to the delusion. The other options focus on content and provide opportunity for argument. DIF: Cognitive Level: Application (Applying) REF: Pages: 320-323 2. A newly admitted patient diagnosed with schizophrenia is hypervigilant and constantly scans the environment. The patient states, “I saw two doctors talking in the hall. They were plotting to kill me.” The nurse may correctly assess this behavior as: a. echolalia. b. an idea of reference. c. a delusion of infidelity. d. an auditory hallucination. ANS: B Ideas of reference are misinterpretations of the verbalizations or actions of others that give special personal meanings to these behaviors; for example, when seeing two people talking, the individual assumes they are talking about him or her. The other terms do not correspond with the scenario. DIF: Cognitive Level: Comprehension (Understanding) REF: Page: 315 3. A patient diagnosed with schizophrenia says, “My co-workers are out to get me. I also saw two doctors plotting to overdose me.” How does this patient perceive the environment? a. Disorganized b. Unpredictable c. Dangerous d. Bizarre ANS: C The patient sees the world as hostile and dangerous. This assessment is important because the nurse can be more effective by using empathy to respond to the patient. Data are not present to support any of the other options. DIF: Cognitive Level: Comprehension (Understanding) REF: Page: 315 | Page: 317 4. When a patient diagnosed with schizophrenia was discharged 6 months ago, haloperidol (Haldol) was prescribed. The patient now says, “I stopped taking those pills. They made me feel like a robot.” What common side effects should the nurse validate with the patient? a. Sedation and muscle stiffness b. Sweating, nausea, and diarrhea c. Mild fever, sore throat, and skin rash d. Headache, watery eyes, and runny nose ANS: A Typical antipsychotic drugs often produce sedation and extrapyramidal side effects such as stiffness and gait disturbance, effects the patient might describe as making him or her feel like a “robot.” The side effects mentioned in the other options are usually not associated with typical antipsychotic therapy or would not have the effect described by the patient. DIF: Cognitive Level: Application (Applying) REF: Page: 326 | Page: 330 5. A nurse works with a patient diagnosed with schizophrenia regarding the importance of medication management. The patient repeatedly says, “I don’t like taking pills.” Which treatment strategy should the nurse discuss with the health care provider? a. Use of a long-acting antipsychotic injections b. Addition of a benzodiazepine, such as lorazepam (Ativan) c. Adjunctive use of an antidepressant, such as amitriptyline (Elavil) d. Inpatient hospitalization because of the high risk for exacerbation of symptoms ANS: A Medications such as fluphenazine decanoate and haloperidol decanoate are long-acting forms of antipsychotic medications. They are administered by depot injection every 2 to 4 weeks, thus reducing daily opportunities for nonadherence. The incorrect options do not address the patient’s dislike of taking pills. DIF: Cognitive Level: Application (Applying) REF: Page: 326 6. A patient’s care plan includes monitoring for auditory hallucinations. Which assessment findings suggest the patient may be hallucinating? a. Aloofness, haughtiness, suspicion b. Darting eyes, tilted head, mumbling to self c. Elevated mood, hyperactivity, distractibility d. Performing rituals, avoiding open places ANS: B Clues to hallucinations include looking around the room as though to find the speaker; tilting the head to one side as though intently listening; and grimacing, mumbling, or talking aloud as though responding conversationally to someone. DIF: Cognitive Level: Application (Applying) REF: Page: 312 | Page: 320 7. A health care provider considers which antipsychotic medication to prescribe for a patient diagnosed with schizophrenia who has auditory hallucinations and poor social functioning. The patient is also overweight and has hypertension. Which drug should the nurse advocate? a. clozapine (Clozaril) b. ziprasidone (Geodon) c. olanzapine (Zyprexa) d. aripiprazole (Abilify) ANS: D Aripiprazole is an atypical antipsychotic medication that is effective against both positive and negative symptoms of schizophrenia. It causes little or no weight gain and no increase in glucose, high- or low-density lipoprotein cholesterol levels, or triglycerides, making it a reasonable choice for a patient with obesity or heart disease. Clozapine may produce agranulocytosis, making it a poor choice as a first-line agent. Ziprasidone may prolong the QT interval, making it a poor choice for a patient with cardiac disease. Olanzapine fosters weight gain. DIF: Cognitive Level: Analysis (Analyzing) REF: Pages: 325-329 8. A patient diagnosed with schizophrenia tells the nurse, “I eat skiller. Tend to end. Easter. It blows away. Get it?” Select the nurse’s best response. a. “Nothing you are saying is clear.” b. “Your thoughts are very disconnected.” c. “Try to organize your thoughts, and then tell me again.” d. “I am having difficulty understanding what you are saying.” ANS: D When a patient’s speech is loosely associated, confused, and disorganized, pretending to understand is useless. The nurse should tell the patient that he or she is having difficulty understanding what the patient is saying. If a theme is discernible, ask the patient to talk about the theme. The incorrect options tend to place blame for the poor communication with the patient. The correct response places the difficulty with the nurse rather than being accusatory. DIF: Cognitive Level: Application (Applying) REF: Pages: 318-323 Quiz 5 9. A patient diagnosed with schizophrenia has catatonia. The patient has little spontaneous movement and waxy flexibility. Which patient needs are of priority importance? a. Psychosocial b. Physiologic c. Self-actualization d. Safety and security ANS: B Physiologic needs must be met to preserve life. A patient who is catatonic may need to be fed by hand or tube, toileted, and given range-of-motion exercises to preserve physiologic integrity. The assessment findings do not suggest safety concerns. Higher level needs (psychosocial and self-actualization) are of lesser concern. DIF: Cognitive Level: Application (Applying) REF: Page: 314 | Page: 317 Quiz 5 10. A patient diagnosed with schizophrenia has catatonia. The patient is stuporous, demonstrates little spontaneous movement, and has waxy flexibility. The patient’s activities of daily living are severely compromised. An appropriate outcome is that the patient will: a. demonstrate increased interest in the environment by the end of week 1. b. perform self-care activities with coaching by the end of day 3. c. gradually take the initiative for self-care by the end of week 2. d. voluntarily accept tube feeding by day 2. ANS: B Outcomes related to self-care deficit nursing diagnoses should deal with increasing the patient’s ability to perform self-care tasks independently, such as feeding, bathing, dressing, and toileting. Performing the tasks with coaching by the nursing staff denotes improvement over the complete inability to perform the tasks. The incorrect options are not directly related to self-care activities; they are difficult to measure and are unrelated to maintaining nutrition. DIF: Cognitive Level: Application (Applying) REF: Page: 314 | Page: 316 11. A nurse observes a patient who is diagnosed with schizophrenia. The patient is standing immobile, facing the wall with one arm extended in a salute. The patient remains immobile in this position for 15 minutes, moving only when the nurse gently lowers the arm. What is the name of this phenomenon? a. Echolalia b. Waxy flexibility c. Depersonalization d. Thought withdrawal ANS: B Waxy flexibility is the ability to hold distorted postures for extended periods, as though the patient were molded in wax. Echolalia is a speech pattern. Depersonalization refers to a feeling state. Thought withdrawal refers to an alteration in thinking. DIF: Cognitive Level: Comprehension (Understanding) REF: Page: 313 12. Which patient diagnosed with schizophrenia would be expected to have the lowest level of overall functioning? a. 39 years old; paranoid ideation since age 35 years b. 32 years old; isolated episodes of catatonia since age 24 years; stable for 3 years c. 19 years old; diagnosed with schizophreniform disorder 6 months ago d. 40 years old; frequent relapses since age 18; often does not take medication as prescribed ANS: D The 40-year-old patient who has been diagnosed with schizophrenia since 18 years of age could logically be expected to have the lowest overall level of functioning secondary to deterioration associated with frequent relapses. The 39-year-old patient who has had paranoid ideation since 35 years of age could be expected to have a higher level because schizophrenia of short duration may be less impairing than other types. The patient who has had episodes of catatonia since the age of 24 years has been stable for more than 3 years, suggesting a higher functional ability. The 19-year-old patient diagnosed with schizophreniform disorder has been ill for only 6 months, and disability is likely to be minimal. DIF: Cognitive Level: Analysis (Analyzing) REF: Page: 308 13. A patient with delusions of persecution about being poisoned has refused all hospital meals for 3 days. Which intervention is most likely to be acceptable to the patient? a. Allow the patient to have supervised access to food vending machines b. Allow the patient to telephone a local restaurant to deliver meals c. Offer to taste each portion on the tray for the patient d. Begin tube feedings or total parenteral nutrition ANS: A The patient who is delusional about food being poisoned is likely to believe restaurant food might still be poisoned and to say that the staff member tasting the food has taken an antidote to the poison before tasting. Attempts to tube feed or give nutrition intravenously are considered aggressive and usually promote violence. Patients often perceive foods in sealed containers, packages, or natural shells as being safe. DIF: Cognitive Level: Analysis (Analyzing) REF: Pages: 318-321 14. A community mental health nurse wants to establish a relationship with a very withdrawn patient diagnosed with schizophrenia. The patient lives at home with a supportive family. Select the nurse’s best plan. a. Visit daily for 4 days, then visit every other day for 1 week; stay with the patient for 20 minutes; accept silence; state when the nurse will return. b. Arrange to spend 1 hour each day with the patient; focus on asking questions about what the patient is thinking or experiencing; avoid silences. c. Visit twice daily; sit beside the patient with a hand on the patient’s arm; leave if the patient does not respond within 10 minutes. d. Visit every other day; remind the patient of the nurse’s identity; encourage the patient to talk while the nurse works on reports. ANS: A Severe constraints on the community mental health nurse’s time will probably not allow more time than what is mentioned in the correct option, yet important principles can be used. A severely withdrawn patient should be met “at the patient’s own level,” with silence accepted. Short periods of contact are helpful to minimize both the patient’s and the nurse’s anxiety. Predictability in returning as stated will help build trust. An hour may be too long to sustain a home visit with a withdrawn patient, especially if the nurse persists in leveling a barrage of questions at the patient. Twice-daily visits are probably not possible, and leaving after 10 minutes would be premature. Touch may be threatening. Working on reports suggests the nurse is not interested in the patient. REF: Pages: 318-319 | Pages: 323-325 15. Patients diagnosed with schizophrenia who are suspicious and withdrawn: a. universally fear sexual involvement with therapists. b. are socially disabled by the positive symptoms of schizophrenia. c. exhibit a high degree of hostility as evidenced by rejecting behavior. d. avoid relationships because they become anxious with emotional closeness. ANS: D When an individual is suspicious and distrustful and perceives the world and the people in it as potentially dangerous, withdrawal into an inner world can be a defense against uncomfortable levels of anxiety. When someone attempts to establish a relationship with such a patient, the patient’s anxiety rises until trust is established. No evidence suggests that withdrawn patients with schizophrenia universally fear sexual involvement with therapists. In most cases, it is not considered true that withdrawn patients with schizophrenia are socially disabled by the positive symptoms of schizophrenia or exhibit a high degree of hostility by demonstrating rejecting behavior. DIF: Cognitive Level: Comprehension (Understanding) REF: Page: 309 16. A newly admitted patient diagnosed with schizophrenia says, “The voices are bothering me. They yell and tell me I’m bad. I have got to get away from them.” Select the nurse’s most helpful reply. a. “Do you hear the voices often?” b. “Do you have a plan for getting away from the voices?” c. “I will stay with you. Focus on what we are talking about, not the voices.” d. “Forget the voices. Ask some other patients to sit and talk with you.” ANS: C Staying with a distraught patient who is hearing voices serves several purposes: ongoing observation, the opportunity to provide reality orientation, a means of helping dismiss the voices, the opportunity of forestalling an action that would result in self-injury, and general support to reduce anxiety. Asking if the patient hears voices is not particularly relevant at this point. Asking if the patient plans to “get away from the voices” is relevant for assessment purposes but is less helpful than offering to stay with the patient while encouraging a focus on their discussion. Asking other patients to talk incorrectly shifts responsibility for intervention from the nurse to other patients. DIF: Cognitive Level: Application (Applying) REF: Page: 320 17. A patient diagnosed with schizophrenia has taken fluphenazine (Prolixin) 5 mg orally twice daily for 3 weeks. The nurse now assesses a shuffling, propulsive gait; a masklike face; and drooling. Which term applies to these symptoms? a. Neuroleptic malignant syndrome b. Hepatocellular effects c. Pseudoparkinsonism d. Akathisia ANS: C Pseudoparkinsonism induced by antipsychotic medication mimics the symptoms of Parkinson disease. It frequently appears within the first month of treatment. Hepatocellular effects would produce abnormal liver test results. Neuroleptic malignant syndrome is characterized by autonomic instability. Akathisia produces motor restlessness. DIF: Cognitive Level: Comprehension (Understanding) REF: Page: 326 18. A patient diagnosed with schizophrenia is acutely disturbed and violent. After several doses of haloperidol (Haldol), the patient is calm. Two hours later the nurse sees the patient’s head rotated to one side in a stiff position; the lower jaw is thrust forward, and the patient is drooling. Which problem is most likely? a. Acute dystonic reaction b. Tardive dyskinesia c. Waxy flexibility d. Akathisia ANS: A Acute dystonic reactions involve painful contractions of the tongue, face, neck, and back; opisthotonos and oculogyric crisis may be observed. Dystonic reactions are considered emergencies that require immediate intervention. Tardive dyskinesia involves involuntary spasmodic muscular contractions that involve the tongue, fingers, toes, neck, trunk, or pelvis; it appears after prolonged treatment. Waxy flexibility is a symptom observed in catatonic schizophrenia. Akathisia is evidenced by internal and external restlessness, pacing, and fidgeting. DIF: Cognitive Level: Application (Applying) REF: Page: 331 19. An acutely violent patient diagnosed with schizophrenia receives several doses of haloperidol (Haldol). Two hours later the nurse notices the patient’s head rotated to one side in a stiffly fixed position; the lower jaw is thrust forward, and the patient is drooling. Which intervention by the nurse is indicated? a. Administer diphenhydramine (Benadryl) 50 mg IM from the PRN medication administration record. b. Reassure the patient that the symptoms will subside. Practice relaxation exercises with the patient. c. Give trihexyphenidyl (Artane) 5 mg orally at the next regularly scheduled medication administration time. d. Administer atropine sulfate 2 mg subcutaneously from the PRN medication administration record. ANS: A Diphenhydramine, trihexyphenidyl, benztropine, and other anticholinergic medications may be used to treat dystonias. Swallowing will be difficult or impossible; therefore, oral medication is not an option. Medication should be administered immediately; therefore, the intramuscular route is best. In this case, the best option given is diphenhydramine. DIF: Cognitive Level: Analysis (Analyzing) REF: Page: 331 20. A patient has taken trifluoperazine (Stelazine) 30 mg/day orally for 3 years. The clinic nurse notes that the patient grimaces and constantly smacks both lips. The patient’s neck and shoulders twist in a slow, snakelike motion. Which problem would the nurse suspect? a. Agranulocytosis b. Tardive dyskinesia c. Tourette syndrome d. Anticholinergic effects ANS: B Tardive dyskinesia is a neuroleptic-induced condition involving the face, trunk, and limbs. Involuntary movements such as tongue thrusting; licking; blowing; irregular movements of the arms, neck, and shoulders; rocking; hip jerks; and pelvic thrusts are observed. These symptoms are frequently not reversible, even when the drug is discontinued. The scenario does not present evidence consistent with the other disorders mentioned. Agranulocytosis is a blood disorder. Tourette syndrome is a condition in which tics are present. Anticholinergic effects include dry mouth, blurred vision, flushing, constipation, and dry eyes. DIF: Cognitive Level: Application (Applying) REF: Page: 332 21. A nurse sits with a patient diagnosed with schizophrenia. The patient starts to laugh uncontrollably, although the nurse has not said anything funny. Select the nurse’s best response. a. “Why are you laughing?” b. “Please share the joke with me.” c. “I don’t think I said anything funny.” d. “You are laughing. Tell me what’s happening.” ANS: D The patient is likely laughing in response to inner stimuli such as hallucinations or fantasy. Focusing on the hallucinatory clue (i.e., the patient’s laughter) and then eliciting the patient’s observation is best. The incorrect options are less useful in eliciting a response; no joke may be involved, “Why” questions are difficult to answer, and the patient is probably not focusing on what the nurse has said in the first place. DIF: Cognitive Level: Application (Applying) REF: Page: 320 | Pages: 322-323 22. Which symptoms are expected for a patient diagnosed with schizophrenia who has disorganization? a. Extremes of motor activity, from excitement to stupor b. Social withdrawal and ineffective communication c. Severe anxiety with ritualistic behavior d. Highly suspicious, delusional behavior ANS: B Patients with disorganization demonstrate the most regressed and socially impaired behaviors. Communication is often incoherent, with silly giggling and loose associations predominating. Highly suspicious, delusional behavior relates more to paranoia. Extremes of motor activity, from excitement to stupor, relate to catatonia. Severe anxiety and ritualistic behaviors relate to obsessive-compulsive disorder. DIF: Cognitive Level: Comprehension (Understanding) REF: Page: 315 23. What assessment findings mark the prodromal stage of schizophrenia? a. Withdrawal, misinterpreting, poor concentration, and preoccupation with religion b. Auditory hallucinations, ideas of reference, thought insertion, and broadcasting c. Stereotyped behavior, echopraxia, echolalia, and waxy flexibility d. Loose associations, concrete thinking, and echolalia neologisms ANS: A Withdrawal, misinterpreting, poor concentration, and preoccupation with religion are prodromal symptoms, which are the symptoms present before the development of florid symptoms. The incorrect options each list the positive symptoms of schizophrenia that are more likely to be apparent during the acute stage of the illness. DIF: Cognitive Level: Comprehension (Understanding) REF: Page: 308 Quiz 5 24. A patient diagnosed with schizophrenia says, “Everyone has skin lice that jump on you and contaminate your blood.” Which problem is evident? a. Poverty of content b. Concrete thinking c. Neologisms d. Paranoia ANS: D The patient’s unrealistic fear of contamination indicates paranoia. Neologisms are invented words. Concrete thinking involves literal interpretation. Poverty of content refers to an inadequate fund of information. DIF: Cognitive Level: Application (Applying) REF: Page: 315 Quiz 5 25. A patient diagnosed with schizophrenia has paranoid thinking. The patient angrily tells a nurse, “You are mean and nasty. No one trusts you or wants to be around you.” Select the most likely analysis. The patient: a. is trying to manipulate the nurse by using negative comments. b. is likely to experience disorganization and catatonia in the near future. c. is jealous of the nurse’s position of power in the relationship. d. may be identifying another person’s shortcomings in order to preserve his or her own self-esteem. ANS: D Patients with paranoid ideation often use disparaging comments to preserve one’s own self-esteem. There is no evidence the patient is trying to manipulate the nurse or is jealous. This behavior is not predictive of catatonia or disorganization. DIF: Cognitive Level: Analysis (Analyzing) REF: Page: 315 | Page: 320 26. A patient diagnosed with schizophrenia says, “High heat. Last time here. Did you get a coat?” What type of verbalization is evident? a. Neologism b. Idea of reference c. Thought broadcasting d. Associative looseness ANS: D Looseness of association refers to jumbled thoughts incoherently expressed to the listener. Neologisms are newly coined words. Ideas of reference are a type of delusion. Thought broadcasting is the belief that others can hear one’s thoughts. DIF: Cognitive Level: Comprehension (Understanding) REF: Page: 311 27. A patient diagnosed with schizophrenia has taken a conventional antipsychotic medication for a year. Hallucinations are less intrusive but the patient continues to have apathy, poverty of thought, and social isolation. The nurse expects a change to which medication? a. haloperidol (Haldol) b. olanzapine (Zyprexa) c. chlorpromazine (Thorazine)F d. diphenhydramine (Benadryl) ANS: B Olanzapine is an atypical antipsychotic medication that targets both positive and negative symptoms of schizophrenia. Haloperidol and chlorpromazine are conventional antipsychotic agents that target only positive symptoms. Diphenhydramine is an antihistamine. DIF: Cognitive Level: Analysis (Analyzing) REF: Pages: 325-326 28. The family of a patient diagnosed with schizophrenia is unfamiliar with the illness and the family’s role in recovery. Which type of therapy should the nurse recommend? a. Psychoeducational b. Psychoanalytic c. Transactional d. Family ANS: A A psychoeducational group explores the causes of schizophrenia, the role of medications, the significance of medication compliance, and the importance of support for the ill member of the family, and also provides recommendations for living with a person with schizophrenia. Such a group can be of practical assistance to the family members. The other types of therapy do not focus on psychoeducation. DIF: Cognitive Level: Application (Applying) REF: Pages: 324-325 29. A patient diagnosed with schizophrenia has been stable for a year; however, the family now reports the patient is tense, sleeps 3 to 4 hours per night, and has difficulty concentrating. The patient says, “Demons are in the basement and they can come through the floor.” The nurse can correctly assess this information as an indication of: a. need for psychoeducation b. medication noncompliance c. chronic deterioration d. relapse ANS: D Signs of potential relapse include feeling tense, difficulty concentrating, trouble sleeping, increased withdrawal, and increased bizarre or magical thinking. Medication noncompliance may not be implicated. Relapse can occur even when the patient is regularly taking his or her medication. Psychoeducation is more effective when the patient’s symptoms are stable. Chronic deterioration is not the best explanation. DIF: Cognitive Level: Analysis (Analyzing) REF: Pages: 316-318 30. A patient diagnosed with schizophrenia begins to talks about “cracklomers” in the local shopping mall. The term “cracklomers” should be documented as: a. neologism. b. concrete thinking. c. thought insertion. d. an idea of reference. ANS: A A neologism is a newly coined word having special meaning to the patient. “Cracklomers” is not a known word. Concrete thinking refers to the inability to think abstractly. Thought insertion refers to thoughts of others that are implanted in one’s mind. An idea of reference is a type of delusion in which trivial events are given personal significance. DIF: Cognitive Level: Comprehension (Understanding) REF: Page: 312 31. A patient diagnosed with schizophrenia anxiously says, “I can see the left side of my body merging with the wall, then my face appears and disappears in the mirror.” While listening, the nurse should: a. sit close to the patient. b. place an arm protectively around the patient’s shoulders. c. place a hand on the patient’s arm and exert light pressure. d. maintain a normal social interaction distance from the patient. ANS: D The patient is describing phenomena that indicate personal boundary difficulties. The nurse should maintain an appropriate social distance and not touch the patient, because the patient is anxious about the inability to maintain ego boundaries and merging with or being swallowed by the environment. Physical closeness or touch could precipitate panic. DIF: Cognitive Level: Application (Applying) REF: Page: 313 | Pages: 318-321 32. A patient diagnosed with schizophrenia has auditory hallucinations. The patient anxiously tells the nurse, “The voice is telling me to do things.” Select the nurse’s priority assessment question. a. “How long has the voice been directing your behavior?” b. “Do the messages from the voice frighten you?” c. “Do you recognize the voice speaking to you?” d. “What is the voice telling you to do?” ANS: D Learning what a command hallucination is telling the patient to do is important; the command often places the patient or others at risk for harm. Command hallucinations can be terrifying and may pose a psychiatric emergency. The incorrect questions are of lesser importance than identifying the command. DIF: Cognitive Level: Application (Applying) REF: Page: 317 | Page: 320 33. A patient receiving risperidone (Risperdal) reports severe muscle stiffness at 10:30 AM. By noon, the patient is diaphoretic, drooling, and has difficulty swallowing. By 4:00 PM, vital signs are body temperature, 102.8° F; pulse, 110 beats per minute; respirations, 26 breaths per minute; and blood pressure, 150/90 mm Hg. Select the nurse’s best analysis and action. a. Agranulocytosis. Institute reverse isolation. b. Tardive dyskinesia. Withhold the next dose of medication. c. Cholestatic jaundice. Begin a high-protein, low fat diet. d. Neuroleptic malignant syndrome. Immediately notify the health care provider. ANS: D Taking an antipsychotic medication coupled with the presence of extrapyramidal symptoms, such as severe muscle stiffness and difficulty swallowing, hyperpyrexia, and autonomic symptoms (pulse elevation), suggest neuroleptic malignant syndrome, a medical emergency. The symptoms given in this scenario are not consistent with the medical problems listed in the incorrect options. DIF: Cognitive Level: Analysis (Analyzing) REF: Pages: 331-332 34. A patient diagnosed with schizophrenia begins a new prescription for lurasidone HCl (Latuda). The patient is 5?2’6?3? tall and currently weighs 204 pounds. Which topic is most important for the nurse to include in the teaching plan related to this medication? a. How to recognize tardive dyskinesia b. Weight management strategies c. Ways to manage constipation d. Sleep hygiene measures ANS: B Lurasidone HCl (Latuda) is an atypical antipsychotic medication. The incidence of weight gain, diabetes, and high cholesterol is high with this medication. The patient is overweight now, so weight management is especially important. The incidence of tardive dyskinesia is low with atypical antipsychotic medications. Constipation may occur, but it is less important than weight management. This drug usually produces drowsiness. DIF: Cognitive Level: Analysis (Analyzing) REF: Pages: 328-329 35. A patient diagnosed with schizophrenia has auditory hallucinations, delusions of grandeur, poor personal hygiene, and motor agitation. Which assessment finding would the nurse regard as a negative symptom of schizophrenia? a. Auditory hallucinations b. Delusions of grandeur c. Poor personal hygiene d. Motor agitation ANS: C Negative symptoms include apathy, anhedonia, poor social functioning, and poverty of thought. Poor personal hygiene is an example of poor social functioning. The distractors are positive symptoms of schizophrenia. DIF: Cognitive Level: Analysis (Analyzing) REF: Pages: 313-314 MULTIPLE RESPONSE 1. The family members of a patient newly diagnosed with schizophrenia state that they do not understand what has caused the illness. The nurse’s response should be based on which models? Select all that apply. a. Neurobiological b. Environmental c. Family theory d. Genetic e. Stress ANS: A, D Compelling evidence exists that schizophrenia is a neurologic disorder probably related to neurochemical abnormalities, neuroanatomical disruption of brain circuits, and genetic vulnerability. Stress and family disruption may contribute but are not considered etiologic factors. Environmental factors are not recognized as causative variables in schizophrenia. DIF: Cognitive Level: Comprehension (Understanding) REF: Pages: 306-307 2. A nurse at the mental health clinic plans a series of psychoeducational groups for persons diagnosed with schizophrenia. Which two topics would take priority? a. How to complete an application for employment b. The importance of correctly taking your medication c. How to dress when attending community events d. How to give and receive compliments e. Ways to quit smoking ANS: B, E Stabilization is maximized by the adherence to the antipsychotic medication regimen. Because so many patients with schizophrenia smoke cigarettes, this topic relates directly to the patients’ physiologic well-being. The other topics are also important but are not priority topics. DIF: Cognitive Level: Application (Applying) REF: Page: 305 | Pages: 317-325 3. A patient diagnosed with schizophrenia is hospitalized after arguing with co-workers and threatening to harm them. The patient is aloof and suspicious and says, “Two staff members I saw talking were plotting to assault me.” Based on data gathered at this point, which nursing diagnoses relate? Select all that apply. a. Risk for other-directed violence b. Disturbed thought processes c. Risk for loneliness d. Spiritual distress e. Social isolation ANS: A, B Delusions of persecution and ideas of reference support the nursing diagnosis of Disturbed thought processes. Risk for other-directed violence is substantiated by the patient’s paranoia and feeling endangered by persecutors. Fearful individuals may strike out at perceived persecutors or attempt self-harm to get away from persecutors. Data are not present to support the other diagnoses. Chapter 18 Chapter 18: Neurocognitive Disorders MULTIPLE CHOICE Quiz 5 1. An older adult takes digoxin and hydrochlorothiazide daily, as well as lorazepam (Ativan) as needed for anxiety. Over 2 days, this adult developed confusion, slurred speech, an unsteady gait, and fluctuating levels of orientation. These findings are most characteristic of: a. delirium. b. dementia. c. amnestic syndrome. d. Alzheimer disease. ANS: A Delirium is characterized by an abrupt onset of fluctuating levels of awareness, clouded consciousness, perceptual disturbances, and disturbed memory and orientation. The onset of dementia or Alzheimer disease, a type of dementia, is more insidious. Amnestic syndrome involves memory impairment without other cognitive problems. DIF: Cognitive Level: Application (Applying) REF: Pages: 338-340 2. A patient experiencing fluctuating levels of awareness, confusion, and disturbed orientation shouts, “Bugs are crawling on my legs! Get them off!” Which problem is the patient experiencing? a. Aphasia b. Dystonia c. Tactile hallucinations d. Mnemonic disturbance ANS: C The patient feels bugs crawling on both legs, although no sensory stimulus is actually present. This description coincides with the definition of a hallucination, a false sensory perception. Tactile hallucinations may be part of the symptom constellation of delirium. Aphasia refers to a speech disorder. Dystonia refers to excessive muscle tonus. Mnemonic disturbance is associated with dementia rather than delirium. Comprehension (Understanding) REF: Pages: 340-343 3. A patient experiencing fluctuating levels of consciousness, disturbed orientation, and perceptual alteration begs, “Someone get these bugs off me.” What is the nurse’s best response? a. “There are no bugs on your legs. Your imagination is playing tricks on you.” b. “Try to relax. The crawling sensation will go away sooner if you can relax.” c. “Don’t worry. I will have someone stay here and brush off the bugs for you.” d. “I don’t see any bugs, but I know you are frightened so I will stay with you.” ANS: D When hallucinations are present, the nurse should acknowledge the patient’s feelings and state the nurse’s perception of reality, but not argue. Staying with the patient increases feelings of security, reduces anxiety, offers the opportunity for reinforcing reality, and provides a measure of physical safety. Denying the patient’s perception without offering help does not emotionally support the patient. Telling the patient to relax makes the patient responsible for self-soothing. Telling the patient that someone will brush the bugs away supports the perceptual distortions. REF: Page: 340 | Pages: 342-344 | Page: 352 4. What is the priority nursing diagnosis for a patient experiencing fluctuating levels of consciousness, disturbed orientation, and visual and tactile hallucinations? a. Bathing/hygiene self-care deficit, related to altered cerebral function, as evidenced by confusion and inability to perform personal hygiene tasks b. Risk for injury, related to altered cerebral function, misperception of the environment, and unsteady gait c. Disturbed thought processes, related to medication intoxication, as evidenced by confusion, disorientation, and hallucinations d. Fear, related to sensory perceptual alterations, as evidenced by hiding from imagined ferocious dogs ANS: B The physical safety of the patient is the highest priority among the diagnoses given. Many opportunities for injury exist when a patient misperceives the environment as distorted, threatening, or harmful; when the patient exercises poor judgment; and when the patient’s sensorium is clouded. The other diagnoses may be concerns but are lower priorities. DIF: Cognitive Level: Application (Applying) REF: Pages: 342-343 Quiz 5 5. What is the priority intervention for a patient diagnosed with delirium who has fluctuating levels of consciousness, disturbed orientation, and perceptual alterations? a. Avoidance of physical contact b. High level of sensory stimulation c. Careful observation and supervision d. Application of wrist and ankle restraints ANS: C Careful observation and supervision are of ultimate importance because an appropriate outcome would be that the patient remains safe and free from injury while hospitalized. Physical contact during care cannot be avoided. Restraint is a last resort, and sensory stimulation should be reduced. DIF: Cognitive Level: Application (Applying) REF: Pages: 342-343 Quiz 5 6. Which environmental adjustment should the nurse make for a patient experiencing delirium with perceptual alterations? a. Keep the patient by the nurse’s desk while the patient is awake. Provide rest periods in a room with a television on. b. Light the room brightly, day and night. Awaken the patient hourly to assess mental status. c. Maintain soft lighting day and night. Keep a radio on low volume continuously. d. Provide a well-lit room without glare or shadows. Limit noise and stimulation. ANS: D A quiet, shadow-free room offers an environment that produces the fewest sensory perceptual distortions for a patient experiencing cognitive impairment associated with delirium. The other options have the potential to produce increased perceptual alterations. DIF: Cognitive Level: Application (Applying) REF: Page: 342 | Page: 344 7. Which description best applies to a hallucination? A patient: a. looks at shadows on a wall and says, “I see scary faces.” b. states, “I feel bugs crawling on my legs and biting me.” c. becomes anxious when the nurse leaves his or her bedside. d. tries to hit the nurse when vital signs are taken. ANS: B hallucination is a false sensory perception occurring without a corresponding sensory stimulus. Feeling bugs on the body when none are present is a tactile hallucination. Misinterpreting shadows as faces is an illusion. An illusion is a misinterpreted sensory perception. The incorrect options are examples of behaviors that sometimes occur during delirium and are related to fluctuating levels of awareness and misinterpreted stimuli. DIF: Cognitive Level: Comprehension (Understanding) REF: Page: 340 8. Consider these health problems: Lewy body disease, Pick disease, and Korsakoff syndrome. Which term unifies these problems? a. Intoxication b. Dementia c. Delirium d. Amnesia ANS: B The listed health problems are all forms of dementia. DIF: Cognitive Level: Comprehension (Understanding) REF: Page: 338 9. When used for treatment of patients diagnosed with Alzheimer disease, which medication would be expected to antagonize N-methyl-D-aspartate (NMDA) channels rather than cholinesterase? a. donepezil (Aricept) b. rivastigmine (Exelon) c. memantine (Namenda) d. galantamine (Razadyne) ANS: C Memantine blocks the NMDA channels and is used in moderate-to-late stages of the disease. Donepezil, rivastigmine, and galantamine are all cholinesterase inhibitors. These drugs increase the availability of acetylcholine and are most often used to treat mild-to-moderate Alzheimer disease. DIF: Cognitive Level: Application (Applying) REF: Page: 356 10. An older adult was stopped by police for driving through a red light. When asked for a driver’s license, the adult hands the police officer a pair of sunglasses. What sign of dementia is evident? a. Aphasia b. Apraxia c. Agnosia d. Memory impairment ANS: C Agnosia refers to the loss of sensory ability to recognize objects. Aphasia refers to the loss of language ability. Apraxia refers to the loss of purposeful movement. No evidence of memory loss is revealed in this scenario. DIF: Cognitive Level: Comprehension (Understanding) REF: Page: 346 11. An older adult drove to a nearby store but was unable to remember how to get home or state an address. When police took the person home, the spouse reported frequent wandering into neighbors’ homes. Alzheimer disease was subsequently diagnosed. Which stage of Alzheimer disease is evident? a. 1 (mild) b. 2 (moderate) c. 3 (moderate to severe) d. 4 (late) ANS: B In stage 2 (moderate), deterioration is evident. Memory loss may include the inability to remember addresses or the date. Activities such as driving may become hazardous, and frustration by the increasing difficulty of performing ordinary tasks may be experienced. Hygiene may begin to deteriorate. Stage 3 (moderate to severe) finds the individual unable to identify familiar objects or people and needing direction for the simplest of tasks. In stage 4 (late), the ability to talk and walk are eventually lost, and stupor evolves. DIF: Cognitive Level: Analysis (Analyzing) REF: Pages: 346-348 12. Consider these problems: apolipoprotein E (apoE) malfunction, neuritic plaques, neurofibrillary tangles, granulovascular degeneration, and brain atrophy. Which condition corresponds to this group? a. Alzheimer disease b. Wernicke encephalopathy c. Central anticholinergic syndrome d. Acquired immunodeficiency syndrome (AIDS)–related dementia ANS: A The problems are all aspects of the pathophysiologic characteristics of Alzheimer disease. DIF: Cognitive Level: Analysis (Analyzing) REF: Pages: 345-346 13. A patient diagnosed with stage 1 Alzheimer disease tires easily and prefers to stay home rather than attend social activities. The spouse does the grocery shopping because the patient cannot remember what to buy. Which nursing diagnosis applies at this time? a. Risk for injury b. Impaired memory c. Self-care deficit d. Caregiver role strain ANS: B Memory impairment is present and expected in stage 1 Alzheimer disease. Patients diagnosed with early Alzheimer disease often have difficulty remembering names, so socialization is minimized. Data are not present to support the other diagnoses. DIF: Cognitive Level: Application (Applying) REF: Pages: 346-351 14. A patient has progressive memory deficit associated with dementia. Which nursing intervention would best help the individual function in the environment? a. Assist the patient to perform simple tasks by giving step-by-step directions. b. Reduce frustration by performing activities of daily living for the patient. c. Stimulate intellectual function by discussing new topics with the patient. d. Promote the use of the patient’s sense of humor by telling jokes. ANS: A Patients with a cognitive impairment should perform all tasks of which they are capable. When simple directions are given in a systematic fashion, the patient is better able to process information and perform simple tasks. Stimulating intellectual functioning by discussing new topics is likely to prove frustrating for the patient. Patients with cognitive deficits may lose their sense of humor and find jokes meaningless. DIF: Cognitive Level: Application (Applying) REF: Pages: 351-356 15. Two patients in a residential care facility are diagnosed with dementia. One shouts to the other, “Move along, you’re blocking the road.” The other patient turns, shakes a fist, and shouts, “I know what you’re up to; you’re trying to steal my car.” What is the nurse’s best action? a. Administer one dose of an antipsychotic medication to both patients. b. Reinforce reality. Say to the patients, “Walk along in the hall. This is not a traffic intersection.” c. Separate and distract the patients. Take one to the day room and the other to an activities area. d. Step between the two patients and say, “Please quiet down. We do not allow violence here.” ANS: C 16. An older adult patient in the intensive care unit has visual and auditory illusions. Which intervention will be most helpful? a. Place large clocks and calendars on the wall. b. Place personally meaningful objects in view. c. Use the patient’s glasses and hearing aids. d. Keep the room brightly lit at all times. ANS: C 17. A patient diagnosed with stage 2 Alzheimer disease calls the police saying, “An intruder is in my home.” Police investigate and discover the patient misinterpreted a reflection in the mirror as an intruder. This phenomenon can be assessed as: a. hyperorality. b. aphasia. c. apraxia. d. agnosia. ANS: D 18. During morning care, a nursing assistant asks a patient diagnosed with dementia, “How was your night?” The patient replies, “It was lovely. I went out to dinner and a movie with my friend.” Which term applies to the patient’s response? a. Sundown syndrome b. Confabulation c. Perseveration d. Delirium ANS: B 19. A patient diagnosed with Alzheimer disease wanders at night. Which action should the nurse recommend for a family to use in the home to enhance safety? a. Place throw rugs on tile or wooden floors. b. Place locks at the tops of doors. c. Encourage daytime napping. d. Obtain a bed with side rails. ANS: B Quiz 5 20. Goals and outcomes for an older adult patient experiencing delirium caused by fever and dehydration will focus on: a. returning to premorbid levels of function. b. identifying stressors negatively affecting self. c. demonstrating motor responses to noxious stimuli. d. exerting control over responses to perceptual distortions. ANS: A 21. An older adult diagnosed with moderate-stage dementia forgets where the bathroom is and has episodes of incontinence. Which intervention should the nurse suggest to the patient’s family? a. Label the bathroom door. b. Take the older adult to the bathroom hourly. c. Place the older adult in disposable adult diapers. d. Make sure the older adult does not eat nonfood items. ANS: A 22. A patient diagnosed with dementia no longer recognizes family members. The family asks how long it will be before their family member recognizes them when they visit. What is the nurse’s best reply? a. “Your family member will never again be able to identify you.” b. “I think that is a question the health care provider should answer.” c. “One never knows. Consciousness fluctuates in persons with dementia.” d. “It is disappointing when someone you love no longer recognizes you.” ANS: D 23. A patient diagnosed with severe dementia no longer recognizes family members and becomes anxious and agitated when they attempt reorientation. Which alternative could the nurse suggest to the family members? a. Wear large name tags. b. Focus interaction on familiar topics. c. Frequently repeat the reorientation strategies. d. Strategically place large clocks and calendars. ANS: B 24. What is the priority need for a patient diagnosed with late-stage dementia? a. Promotion of self-care activities b. Meaningful verbal communication c. Maintenance of nutrition and hydration d. Prevention of the patient from wandering ANS: C Quiz 5 25. Which intervention is appropriate to use for patients diagnosed with either delirium or dementia? a. Speak in a loud, firm voice. b. Touch the patient before speaking. c. Reintroduce the health care worker at each contact. d. When the patient becomes aggressive, use physical restraint instead of medication. ANS: C Quiz 5 26. A hospitalized patient experiencing delirium misinterprets reality, and a patient diagnosed with dementia wanders about the home. Which outcome is the priority in both scenarios? Each patient will: a. remain safe in the environment. b. participate actively in self-care. c. communicate verbally. d. acknowledge reality. ANS: A MULTIPLE RESPONSE 1. A patient diagnosed with Alzheimer disease has a dressing and grooming self-care deficit. Designate the appropriate interventions to include in the patient’s plan of care. Select all that apply. a. Provide clothing with elastic and hook-and-loop closures. b. Label clothing with the patient’s name and name of the item. c. Administer antianxiety medication before bathing and dressing. d. Provide necessary items, and direct the patient to proceed independently. e. If the patient resists, use distraction and then try again after a short interval. ANS: A, B, E Quiz 5 2. Which assessment findings would the nurse expect in a patient experiencing delirium? Select all that apply. a. Impaired level of consciousness b. Disorientation to place and time c. Wandering attention d. Apathy e. Agnosia ANS: A, B, C TOP: Nursing Process: Assessment MSC: NCLEX: Psychosocial Integrity 3. A nurse should anticipate that which symptoms of Alzheimer disease will become apparent as the disease progresses from moderate to severe to late stage? Select all that apply. a. Agraphia b. Hyperorality c. Fine motor tremors d. Hypermetamorphosis e. Improvement of memory ANS: A, B, D Chapter 21-24 Chapter 21: Child, Partner, and Elder Violence Varcarolis: Essentials to Psychiatric Mental Health Nursing, 2nd Edition – Revised Reprint MULTIPLE CHOICE 1. A nurse visits the home of an 11-year-old child and finds the child caring for three younger siblings. Both parents are at work. The child says, “I want to go to school, but we can’t afford a babysitter. It doesn’t matter; I’m too dumb to learn.” What preliminary assessment is evident? a. Insufficient data are present to make an assessment. b. Child and siblings are experiencing neglect. c. Children are at high risk for sexual abuse. d. Children are experiencing physical abuse. ANS: B 2. An 11-year-old child is absent from school to care for siblings while the parents work. The family cannot afford a babysitter. When asked about the parents, the child reluctantly says, “My parents don’t like me. They call me stupid and say I never do anything right.” Which type of abuse is likely? a. Sexual b. Physical c. Emotional d. Economic ANS: C 3. What feelings are most commonly experienced by nurses working with abusive families? a. Outrage toward the victim and sympathy for the abuser b. Sympathy for the victim and anger toward the abuser c. Unconcern for the victim and dislike for the abuser d. Vulnerability for self and empathy with the abuser ANS: B derstanding) REF: Pages: 416-417 4. Which rationale best explains why a nurse should be aware of personal feelings while working with a family experiencing family violence? a. Self-awareness protects one’s own mental health. b. Strong negative feelings interfere with assessment and judgment. c. Strong positive feelings lead to underinvolvement with the victim. d. Positive feelings promote the development of sympathy for patients. ANS: B 5. A clinic nurse interviews an adult patient who reports fatigue, back pain, headaches, and sleep disturbances. The patient seems tense and then becomes reluctant to provide more information and hurries to leave. How can the nurse best serve the patient? a. Explore the possibility of patient social isolation. b. Have the patient complete an abuse assessment screen. c. Ask whether the patient has ever had psychiatric counseling. d. Ask the patient to disrobe; then assess for signs of physical abuse. ANS: B 6. A patient at the emergency department is diagnosed with a concussion. The patient is accompanied by a spouse who insists on staying in the room and answering all questions. The patient avoids eye contact and has a sad affect and slumped shoulders. Assessment of which additional problem has priority? a. Risk of intimate partner violence b. Phobia of crowded places c. Migraine headaches d. Major depression ANS: A 7. What is a nurse’s legal responsibility if child abuse or neglect is suspected? a. Discuss the findings with the child’s teacher, principal, and school psychologist. b. Report the suspected abuse or neglect according to state regulations. c. Document the observations and speculations in the medical record. d. Continue the assessment. ANS: B 8. Several children are seen in the emergency department for treatment of illnesses and injuries. Which finding would create a high index of suspicion for child abuse? The child who has: a. repeated middle ear infections. b. severe colic. c. bite marks. d. croup. ANS: C 9. An 11-year-old child says, “My parents don’t like me. They call me stupid and say I never do anything right, but it doesn’t matter. I’m too dumb to learn.” Which nursing diagnosis applies to this child? a. Chronic low self-esteem, related to negative feedback from parents b. Deficient knowledge, related to interpersonal skills with parents c. Disturbed personal identity, related to negative self-evaluation d. Complicated grieving, related to poor academic performance ANS: A 10. An adult has recently been absent from work for 3-day periods on several occasions. Each time, the individual returns to work wearing dark glasses. Facial and body bruises are apparent. What is the occupational health nurse’s priority assessment? a. Interpersonal relationships b. Work responsibilities c. Socialization skills d. Physical injuries ANS: D 11. An adult has recently been absent from work for 3-day periods on several occasions. Each time, this person returns to work wearing dark glasses. Facial and body bruises are apparent. What is the occupational health nurse’s priority question? a. “Do you drink excessively?” b. “Did your partner beat you?” c. “How did this happen to you?” d. “What did you do to deserve this?” ANS: C 12. An employee has recently been absent from work on several occasions. Each time, this employee returns to work wearing dark glasses. Facial and body bruises are apparent. During the occupational health nurse’s interview, the employee says, “My partner beat me, but it was because there are problems at work.” What should the nurse’s next action be? a. Call the police. b. Arrange for hospitalization. c. Call the adult protective agency. d. Document injuries with a body map. ANS: D 13. A patient tells the nurse, “My husband is abusive most often when he drinks too much. His family was like that when he was growing up. He always apologizes and regrets hurting me.” What risk factor was most predictive for the husband to become abusive? a. History of family violence b. Loss of employment c. Abuse of alcohol d. Poverty ANS: A 14. An adult tells the nurse, “My partner abuses me most often when drinking. The drinking has increased lately, but I always get an apology afterward and a box of candy. I’ve considered leaving but haven’t been able to bring myself to actually do it.” Which phase in the cycle of violence prevents the patient from leaving? a. Tension building b. Acute battering c. Honeymoon d. Recovery ANS: C 15. After treatment for a detached retina, a victim of intimate partner violence says, “My partner only abuses me when intoxicated. I’ve considered leaving, but I was brought up to believe you stay together, no matter what happens. I always get an apology, and I can tell my partner feels bad after hitting me.” Which nursing diagnosis applies? a. Social isolation, related to lack of community support system b. Risk for injury, related to partner’s physical abuse when intoxicated c. Deficient knowledge, related to resources for escape from the abusive relationship d. Disabled family coping, related to uneven distribution of power within a relationship ANS: B 16. A victim of physical abuse by an intimate partner is treated for a broken wrist. The patient has considered leaving but says, “You stay together, no matter what happens.” Which outcome should be met before the patient leaves the emergency department? The patient will: a. limit contact with the abuser by obtaining a restraining order. b. name two community resources that can be contacted. c. demonstrate insight into the abusive relationship. d. facilitate counseling for the abuser. ANS: B 17. An older adult diagnosed with dementia lives with family and attends a day care center. A nurse at the day care center notices the adult has a disheveled appearance, a strong odor of urine, and bruises on the limbs and back. What type of abuse might be occurring? a. Psychological b. Financial c. Physical d. Sexual ANS: C 18. An older adult diagnosed with Alzheimer disease lives with family. During the week, the person attends a day care center while the family is at work. In the evenings, members of the family provide care. Which factor makes this patient most vulnerable to abuse? a. Dementia b. Living in a rural area c. Being part of a busy family d. Being home only in the evening ANS: A 19. An older adult diagnosed with Alzheimer disease lives with family. After observing multiple bruises, the home health nurse talks with the older adult’s daughter, who becomes defensive and says, “My mother often wanders at night. Last night she fell down the stairs.” Which nursing diagnosis has priority? a. Risk for injury, related to poor judgment, cognitive impairment, and lack of caregiver supervision b. Noncompliance, related to confusion and disorientation as evidenced by lack of cooperation c. Impaired verbal communication, related to brain impairment as evidenced by the confusion d. Insomnia, related to cognitive impairment as evidenced by wandering at night ANS: A 20. An older adult diagnosed with dementia lives with family and attends day care. After observing poor hygiene, the nurse at the center talks with the patient’s adult child. This caregiver becomes defensive and says, “It takes all my time and energy to care for my mother. She’s awake all night. I never get any sleep.” Which nursing intervention has priority? a. Teach the caregiver more about the effects of dementia. b. Secure additional resources for the mother’s evening and night care. c. Support the caregiver to grieve the loss of the mother’s ability to function. d. Teach the family how to give physical care more effectively and efficiently. ANS: B 21. A patient has a history of physical violence against family members when frustrated and then experiences periods of remorse after each outburst. Which finding indicates success in the plan of care? The patient: a. expresses frustration verbally instead of physically. b. explains the rationale for behaviors to the victim. c. identifies three personal strengths. d. agrees to seek counseling. ANS: A 22. Which referral is most appropriate for a woman who is severely beaten by her husband, has no relatives or friends in the community, is afraid to return home, and has limited financial resources? a. Support group b. Law enforcement c. Women’s shelter d. Vocational counseling ANS: C 23. Which family scenario presents the greatest risk for family violence? a. An unemployed husband with low self-esteem, a wife who loses her job, and a developmentally delayed 3-year-old child b. A husband who finds employment 2 weeks after losing his previous job, a wife with stable employment, and a child doing well in school c. A single mother with an executive position, a talented child, and a widowed grandmother living in the home to provide child care d. A single homosexual male parent, an adolescent son who has just begun dating girls, and the father’s unmarried sister who has come to visit for 2 weeks ANS: A MULTIPLE RESPONSE 1. A 10-year-old child cares for siblings while the parents work because the family cannot afford a babysitter. This child says, “My father doesn’t like me. He calls me stupid all the time.” The mother says the father is easily frustrated and has trouble disciplining the children. The community health nurse should consider which resources to stabilize the home situation? Select all that apply. a. Parental sessions to teach childrearing practices b. Anger management counseling for the father c. Continuing home visits to provide support d. Safety plan for the wife and children e. Placement of the children in foster care ANS: A, B, C 2. A nurse assists a victim of intimate partner violence to create a plan for escape if it becomes necessary. The plan should include which components? Select all that apply. a. Keep a cell phone fully charged. b. Hide money with which to buy new clothes. c. Have the telephone number for the nearest shelter. d. Take enough toys to amuse the children for 2 days. e. Secure a supply of current medications for self and children. f. Determine a code word to signal children that it is time to leave. g. Assemble birth certificates, Social Security cards, and licenses. ANS: A, C, E, F, G The victim must prepare for a quick exit and so should assemble necessary items. Keeping a cell phone fully charged will help with access to support persons or agencies. The individual should be advised to hide a small suitcase containing a change of clothing for self and for each child. Taking a large supply of toys would be cumbersome and might compromise the plan. People are advised to take one favorite small toy or security object for each child, but most shelters have toys to further engage the children. Accumulating enough money to purchase clothing may be difficult. DIF: Cognitive Level: Application (Applying) REF: Page: 421 | Pages: 424-426 3. A community health nurse visits a family with four children. The father behaves angrily, finds fault with a child, and asks twice, “Why are you such a stupid kid?” The wife says, “I have difficulty disciplining the children. It’s so frustrating.” Which comments by the nurse will facilitate the interview with these parents? Select all that apply. a. “Tell me how you punish your children.” b. “How do you stop your baby from crying?” c. “Caring for four small children must be difficult.” d. “Do you or your husband ever beat the children?” e. “Calling children ‘stupid’ injures their self-esteem.” ANS: A, B, C An interview with possible abusing individuals should be built on concern and carried out in a nonthreatening, nonjudgmental way. Empathic remarks are helpful in creating rapport. Questions requiring a descriptive response are less threatening and elicit more relevant information than questions that can be answered by “yes” or “no.” DIF: Cognitive Level: Analysis (Analyzing) REF: Pages: 416-417 Chapter 21 chapter review 1. An emergency department nurse assess a woman suspected of being abused by an intimate partner. Which assessment finding most clearly confirms the suspicion? B: Injuries in a bikini pattern 2. An emergency department nurse assesses a child with a fractured ulna. The nurse also observes yellow and purple bruises across the child's back and shoulders. Which comment by the parents should prompt the nurse to consider making a report to child protective services? B “this child is always creating problems for the family” 3. A women in a relationship characterized by a long history of battering and abuse tells the nurse “we’ve had a rough time lately . I admit it :he beat me last night but then he said he was sorry.” which event would the nurse expect to occur next in the relationship? B love, gifts, and praise from the abusive partner 4. The nurse assessed an elderly person who was abused by the caregiver. Afterward which internal dialogue should prompt the nurse to seek guidance? D “ i hope the abuser gets victimized so they know how what it feels like.” 5. A university football coach invites the campus nurse to talk to the team about healthy relationships in the community. Which topic has priority for the nurse to include ? A. Appropriate behavior with intimate partners . Chapter 22 book questions 1. An elderly widow tells the nurse “ Since my sister-in-law’s death, her husband has been making advances at me. He tried to come in my home with a bottle of wine. Even though he’s family, I’m afraid of what might happen if I let him in.” Which action should the nurse first take? C. Positively reinforce the widow for addressing the problem with a caring professional 2. An emergency department nurse talks with a newly admitted victim of reported rape. Which communication should the nurse offer to comfort this patient? A. “ you are safe now. I will stay with you in this private room. “ 3. A patient tells the nurse, “ I was raped 8 years ago but never told anyone. Nevertheless, the memories haunt me everyday. I should be over it by now.”Which communication should the nurse offer next? A. “ it sounds like your judging yourself for continuing to struggle with your reaction.” 4. An emergency department nurse prepares to discharge a victim of reported rape . Which comment by the victim indicates that the nurses’ teaching was effective ? C. “ It's important for me to follow up with counseling” 5. A victim of reported sexual assault tells the nurse.” This was entirely my fault . I should never have gone to that party alone.” Which response by the nurse is most therapeutic? D. “It sounds like your blaming yourself for the assailants behavior.” Chapter 22: Sexual Violence Varcarolis: Essentials to Psychiatric Mental Health Nursing, 2nd Edition – Revised Reprint MULTIPLE CHOICE 1. A nurse works with a person who was raped four years ago. This person says, “It took a long time for me to recover from that horrible experience.” Which term should the nurse use when referring to this person? a. Victim b. Survivor c. Plaintiff d. Perpetrator ANS: B 2. A person was abducted and raped at gunpoint. The nurse observes this person is confused, talks rapidly in disconnected phrases, and is unable to concentrate or make simple decisions. What is the person’s level of anxiety? a. Weak b. Mild c. Moderate d. Severe ANS: D 3. A person was abducted and raped at gunpoint by an unknown assailant. Which assessment finding best indicates the person is in the acute phase of rape trauma syndrome? a. Confusion and disbelief b. Decreased motor activity c. Flashbacks and dreams d. Fears and phobias ANS: A Shock, emotional numbness, confusion, disbelief, restlessness, and agitated motor activity depict the acute phase of rape trauma syndrome. Flashbacks, dreams, fears, and phobias occur in the long-term reorganization phase of rape trauma syndrome. Decreased motor activity, by itself, is not indicative of any particular phase. DIF: Cognitive Level: Application (Applying) REF: Page: 438 4. A nurse interviews a person abducted and raped at gunpoint by an unknown assailant. The person says, “I can’t talk about it. Nothing happened. I have to forget!” What is the person’s present coping strategy? a. Somatic reaction b. Repression c. Projection d. Denial ANS: D Disbelief is a common finding during the acute stage following sexual assault. Denial is evidence of the disbelief. This mechanism may be unconsciously used to protect the person from the emotionally overwhelming reality of rape. The patient’s statements do not reflect somatic symptoms, repression, or projection. DIF: Cognitive Level: Application (Applying) REF: Page: 438 5. A child was abducted and raped. Which personal reaction by the nurse could interfere with the child’s care? a. Anger b. Concern c. Empathy d. Compassion ANS: A Feelings of empathy, concern, and compassion are helpful. Anger, on the other hand, may make objectivity impossible. DIF: Cognitive Level: Application (Applying) REF: Page: 439 | Pages: 441-442 6. A nurse working in the county jail interviews a man who recently committed a violent sexual assault against a woman. Which comment from this perpetrator is most likely? a. “She was very beautiful.” b. “I gave her what she wanted.” c. “I have issues with my mother.” d. “I’ve been depressed for a long time.” ANS: B - 7. A rape victim asks an emergency department nurse, “Maybe I did something to cause this attack. Was it my fault?” Which response by the nurse is the most therapeutic? a. Pose questions about the rape, helping the patient explore why it happened. b. Reassure the victim that the outcome of the situation will be positive. c. Make decisions for the victim because of the temporary confusion. d. Support the victim to separate issues of vulnerability from blame. ANS: D Although the victim may have made choices that increased vulnerability, the victim is not to blame for the rape. The incorrect options either suggest the use of a nontherapeutic communication technique or do not permit the victim to restore control. No confusion is evident. DIF: Cognitive Level: Application (Applying) REF: Page: 437 | Pages: 441-442 8. A rape victim tells the nurse, “I should not have been out on the street alone.” Which is the nurse’s most therapeutic response? a. “Rape can happen anywhere.” b. “Blaming yourself only increases your anxiety and discomfort.” c. “You believe this would not have happened if you had not been alone?” d. “You are right. You should not have been alone on the street at night.” ANS: C A reflective communication technique is helpful. Looking at one’s role in the event serves to explain events that the victim would otherwise find incomprehensible. The incorrect options discount the victim’s perceived role and interfere with further discussion. DIF: Cognitive Level: Application (Applying) REF: Page: 437 | Pages: 441-442 9. The nursing diagnosis rape trauma syndrome applies to a rape victim in the emergency department. Which outcome should occur before the patient’s discharge? a. Patient states, “I feel safe and entirely relaxed.” b. Memory of the rape is less vivid and frightening. c. Physical symptoms of pain and discomfort are no longer present. d. Patient agrees to keep a follow-up appointment with the rape crisis center. ANS: D Agreeing to keep a follow-up appointment is a realistic short-term outcome. The incorrect options are unlikely to occur during the limited time the victim is in the emergency department. DIF: Cognitive Level: Application (Applying) REF: Pages: 439-440 10. The nurse cares for a victim of a violent sexual assault. What is the most therapeutic intervention? a. Use accepting, nurturing, and empathetic communication techniques. b. Educate the victim about strategies to avoid attacks in the future. c. Discourage the expression of feelings until the victim stabilizes. d. Maintain a matter-of-fact manner and objectivity. ANS: A Victims require the nurse to provide unconditional acceptance of them as individuals, because they often feel guilty and engage in self-blame. The nurse must be nurturing if the victim’s needs are to be met and must be empathetic to convey understanding and to promote an establishment of trust. DIF: Cognitive Level: Application (Applying) REF: Page: 437 | Pages: 441-442 11. What is the primary motivator for most rapists? a. Anxiety b. Need for humiliation c. Overwhelming sexual desires d. Desire to humiliate or control others ANS: D Rape is not a crime of sex; rather, it is a crime of power, control, and humiliation. The perpetrator wishes to subjugate the victim. The dynamics listed in the other options are not the major motivating factors for rape. DIF: Cognitive Level: Comprehension (Understanding) REF: Pages: 436-437 12. A nurse working a rape telephone hotline should focus communication with callers to: a. arrange long-term counseling. b. serve as a sympathetic listener. c. obtain information to relay to the local police. d. explain immediate steps that a victim of rape should take. ANS: D The telephone counselor establishes where the victim is and what has happened and provides the necessary information to enable the victim to decide what steps to take immediately. Long-term aftercare is not the focus until immediate problems are resolved. The victim remains anonymous. The incorrect options are inappropriate or incorrect because counselors should be empathic rather than sympathetic. DIF: Cognitive Level: Application (Applying) REF: Page: 437 13. A rape victim tells the emergency department nurse, “I feel so dirty. Please let me take a shower before the doctor examines me.” The nurse should: a. arrange for the patient to shower. b. explain that washing would destroy evidence. c. give the patient a basin of hot water and towels. d. instruct the victim to wash above the waist only. ANS: B No matter how uncomfortable, the patient should not bathe until the forensic examination is completed. The collection of evidence is critical if the patient is to be successful in court. The incorrect options would result in the destruction of evidence or are untrue. DIF: Cognitive Level: Application (Applying) REF: Pages: 437-439 14. Which situation constitutes consensual sex rather than rape? a. After coming home intoxicated from a party, a person forces the spouse to have sex. The spouse objects. b. A person’s lover pleads to have oral sex. The person gives in but then regrets the decision. c. A person is beaten, robbed, and forcibly subjected to anal penetration by an assailant. d. A physician gives anesthesia for a procedure and has intercourse with an unconscious patient. ANS: B Only the correct answer describes a scenario in which the sexual contact is consensual. Consensual sex is not considered rape if the participants are, at least, the age of majority. DIF: Cognitive Level: Application (Applying) REF: Pages: 432-433 15. When a victim of sexual assault is discharged from the emergency department, the nurse should: a. arrange support from the victim’s family. b. provide referral information verbally and in writing. c. advise the victim to try not to think about the assault. d. offer to stay with the victim until stability is regained. ANS: B Immediately after the assault, rape victims are often disorganized and unable to think well or remember what they have been told. Written information acknowledges this fact and provides a solution. The incorrect options violate the patient’s right to privacy, evidence a rescue fantasy, and offer a platitude that is neither therapeutic nor effective. DIF: Cognitive Level: Application (Applying) REF: Pages: 437-440 16. A victim of a sexual assault that occurred approximately 1 hour earlier sits in the emergency department rocking back and forth and repeatedly saying, “I can’t believe I’ve been raped.” This behavior is characteristic of which phase of the rape trauma syndrome? a. Anger phase b. Acute phase c. Outward adjustment phase d. Long-term reorganization phase ANS: B The victim’s response is typical of the acute phase and evidences cognitive, affective, and behavioral disruptions. The response is immediate and does not include a display of behaviors suggestive of the outward adjustment, long-term reorganization, or anger phases. DIF: Cognitive Level: Application (Applying) REF: Page: 438 17. A survivor in the long-term reorganization phase of the rape trauma syndrome has experienced intrusive thoughts of the rape and developed a fear of being alone. Which finding demonstrates this survivor has made improvement? The survivor: a. temporarily withdraws from social situations. b. plans coping strategies for fearful situations. c. uses increased activity to reduce fear. d. expresses a desire to be with others. ANS: B The correct response shows a willingness and ability to take personal action to reduce the disabling fear. The incorrect responses demonstrate continued ineffective coping. DIF: Cognitive Level: Application (Applying) REF: Pages: 438-439 18. A patient comes to the hospital for treatment of injuries sustained during a rape. The patient abruptly decides to decline treatment and return home. Before the patient leaves, the nurse should: a. tell the patient, “You may not leave until you receive prophylactic treatment for sexually transmitted diseases.” b. provide written information concerning the physical and emotional reactions that may be experienced. c. explain the need and importance of human immunodeficiency virus (HIV) testing. d. offer verbal information about legal resources. ANS: B All information given to a patient before he or she leaves the emergency department should be in writing. Patients who are anxious are unable to concentrate and therefore cannot retain much of what is verbally imparted. Written information can be read and referred to at later times. Patients cannot be kept against their will or coerced into receiving medication as a condition of being allowed to leave. This constitutes false imprisonment. DIF: Cognitive Level: Application (Applying) REF: Pages: 439-440 19. An unconscious person is brought to the emergency department by a friend. The friend found the person in a bedroom at a college fraternity party. Semen is observed on the person’s underclothes. The priority actions of staff members should focus on: a. maintaining the airway. b. preserving rape evidence. c. obtaining a description of the rape. d. determining what drug was ingested. ANS: A Because the patient is unconscious, the risk for airway obstruction is present. The incorrect options are of lower priority than preserving physiologic functioning. DIF: Cognitive Level: Application (Applying) REF: Pages: 433-434 | Page: 438 20. A victim of a violent rape has been in the emergency department for 3 hours. Evidence collection is complete. As discharge counseling begins, the victim says softly, “I will never be the same again. I can’t face my friends. There is no sense of trying to go on.” Select the nurse’s most important response. a. “Are you thinking of suicide?” b. “It will take time, but you will feel the same as before.” c. “Your friends will understand when you tell them.” d. “You will be able to find meaning in this experience as time goes on.” ANS: A The victim’s words suggest hopelessness. Whenever hopelessness is present, so is the risk for suicide. The nurse should directly address the possibility of suicidal ideation with the victim. The other options attempt to offer reassurance before making an assessment. DIF: Cognitive Level: Application (Applying) REF: Pages: 438-442 21. A nurse cares for a rape victim who was given flunitrazepam (Rohypnol) by the assailant. Which intervention has priority? Monitoring for: a. coma. b. seizures. c. hypotonia. d. respiratory depression. ANS: D Monitoring for respiratory depression takes priority over hypotonia, seizures, or coma in this situation. DIF: Cognitive Level: Application (Applying) REF: Pages: 433-434 | Page: 438 22. When working with rape victims, immediate care focuses first on: a. collecting evidence. b. notifying law enforcement. c. helping the victim feel safe. d. documenting the victim’s comments. ANS: C The first focus of care is helping the victim feel safe. An already vulnerable individual may view assessment questions and the physical procedures as intrusive violations of privacy and even physically threatening. The patient might decline to have evidence collected or to involve law enforcement. DIF: Cognitive Level: Application (Applying) REF: Pages: 439-440 MULTIPLE RESPONSE 1. When an emergency department nurse teaches a victim of the rape about reactions that may occur during the long-term reorganization phase, which symptoms should be included? Select all that apply. a. Development of fears and phobias b. Decreased motor activity c. Feelings of numbness d. Flashbacks, dreams e. Syncopal episodes ANS: A, C, D 2. A person was abducted and raped at gunpoint by an unknown assailant. Which interventions should the nurse use while caring for this person in the emergency department? Select all that apply. a. Allow the person to talk at a comfortable pace. b. Pose questions in nonjudgmental, empathic ways. c. Place the person in a private room with a caregiver. d. Reassure the person that a family member will arrive as soon as possible. e. Invite family members to the examination room and involve them in taking the history. f. Put an arm around the person to offer reassurance that the nurse is caring and compassionate. ANS: A, B, C Neutral, nonjudgmental care and emotional support are critical to crisis management for the victim of rape. The rape victim should have privacy but not be left alone. Some rape victims prefer not to have family members involved. The patient’s privacy may be compromised by the presence of family. The rape victim’s anxiety may escalate when he or she is touched by a stranger, even when the stranger is a nurse. DIF: Cognitive Level: Application (Applying) REF: Pages: 439-440 TOP: Nursing Process: Implementation MSC: NCLEX: Psychosocial Integrity 3. Which activities are in the scope of practice of a sexual assault nurse examiner? Select all that apply. a. Requiring HIV testing of a victim b. Collecting and preserving evidence c. Providing long-term counseling for rape victims d. Obtaining signed consents for photographs and examinations e. Providing pregnancy and sexually transmitted disease prophylaxis ANS: B, D, E HIV testing is not mandatory for a victim of sexual assault. Long-term counseling would be provided by other members of the team. The other activities would be included within this practice role. DIF: Cognitive Level: Comprehension (Understanding) REF: Page: 437 TOP: Nursing Process: Implementation MSC: NCLEX: Physiological Integrity 4. After assessing a victim of sexual assault, which terms could the nurse use in the documentation? Select all that apply. a. Alleged b. Reported c. Penetration d. Intercourse e. Refused f. Declined ANS: B, C, F The nurse should refrain from using pejorative language when documenting assessments of victims of sexual assault. “Reported” should be used instead of “alleged.” “Penetration” should be used instead of “intercourse.” “Declined” should be used instead of “refused.” DIF: Cognitive Level: Application (Applying) REF: Pages: 437-440 1. a parent tells the nurse about the death of a child 2 years ago. Which comment by this parent warrants the nurse's priority attention? B. “ A parent should never live longer than their child” 2. A patient diagnosed with major depressive disorder was hospitalized for 2 weeks on an acute psychiatric unit. One day after discharge the patient completed suicide. Recognizing likely reactions among staff, which action should the nursing supervisor implement first? B Provide a private setting for staff members to talk about feelings associated with the event 3. On the sixth anniversary of her spouse's death a widow says, “ sometimes life does not seem worth living anymore. I wish i could go to sleep and never wake up.” Which response by the nurse has priority? A “Are you considering suicide?” 4. A patient who had a stroke 3 days ago tearfully tell the nurse.” what’s the use in living? I'm no good to anybody like this.” which action should the nurse employ first when caring for a patient demonstrating hopelessness? A Implement the institutional protocol for suicide risk. 5. A single adult says to the nurse, “both of my parents died several years ago and my only sibling committed suicide 2 weeks ago. I feel so alone.” After determining that the adult has no suicidal ideation. The nurse should; D Refer the adult to self-help group for suicide survivors. Chapter 23: Suicidal Thoughts and Behavior Varcarolis: Essentials to Psychiatric Mental Health Nursing, 2nd Edition – Revised Reprint MULTIPLE CHOICE 1. Which changes in brain biochemical function is most associated with suicidal behavior? a. Dopamine excess b. Serotonin deficiency c. Acetylcholine excess d. Gamma-aminobutyric acid deficiency ANS: B Research suggests that low levels of serotonin may play a role in the decision to commit suicide. The other neurotransmitter alterations have not been implicated in suicidal crises. DIF: Cognitive Level: Comprehension (Understanding) REF: Page: 449 2. A college student failed two tests. Afterward, the student cried for hours and then tried to telephone a parent but got no answer. The student then gave several expensive sweaters to a roommate. Which behavior provides the strongest clue of an impending suicide attempt? a. Calling parents b. Excessive crying c. Giving away sweaters d. Staying alone in a dorm room ANS: C Giving away prized possessions may signal that the individual thinks he or she will have no further need for the items, such as when a suicide plan has been formulated. Calling parents and crying do not provide clues to suicide, in and of themselves. Remaining in the dormitory would be an expected behavior because the student has nowhere else to go. DIF: Cognitive Level: Application (Applying) REF: Page: 449 | Page: 451 3. A nurse uses the SAD PERSONS scale to interview a patient. This tool provides data relevant to: a. current stress level. b. mood disturbance. c. suicide potential. d. level of anxiety. ANS: C The SAD PERSONS tool evaluates 10 major risk factors in suicide potential: sex, age, depression, previous attempt, ethanol use, rational thinking loss, social supports lacking, organized plan, no spouse, and sickness. The tool does not have appropriate categories to provide information on the other options listed. DIF: Cognitive Level: Comprehension (Understanding) REF: Pages: 451-452 4. A person intentionally overdoses on antidepressant drugs. Which nursing diagnosis has the highest priority? a. Powerlessness b. Social isolation c. Risk for suicide d. Ineffective management of the therapeutic regimen ANS: C This diagnosis is the only one with life-or-death ramifications and is therefore higher in priority than the other options. DIF: Cognitive Level: Application (Applying) REF: Page: 452 5. A person attempts suicide by overdose, is treated in the emergency department, and then hospitalized. What is the best initial outcome? The patient will: a. verbalize a will to live by the end of the second hospital day. b. describe two new coping mechanisms by the end of the third hospital day. c. accurately delineate personal strengths by the end of first week of hospitalization. d. exercise suicide self-restraint by refraining from gestures or attempts to kill self for 24 hours. ANS: D Suicide self-restraint relates most directly to the priority problem of risk for self-directed violence. The other outcomes are related to hope, coping, and self-esteem. DIF: Cognitive Level: Application (Applying) REF: Pages: 452-453 6. A college student who attempted suicide by overdose is hospitalized. When the parents are contacted, they respond, “There must be a mistake. This could not have happened. We’ve given our child everything.” The parents’ reaction reflects: a. denial. b. anger. c. anxiety. d. rescue feelings. ANS: A The parents’ statements indicate denial. Denial or minimization of suicidal ideation or attempts is a defense against uncomfortable feelings. Family members are often unable to acknowledge suicidal ideation in someone close to them. The feelings suggested in the distractors are not clearly described in the scenario. DIF: Cognitive Level: Application (Applying) REF: Page: 453 | Pages: 457-458 7. An adolescent tells the school nurse, “My friend threatened to take an overdose of pills.” The nurse talks to the friend who verbalized the suicide threat. The most critical question for the nurse to ask would be: a. “Why do you want to kill yourself?” b. “Do you have access to medications?” c. “Have you been taking drugs and alcohol?” d. “Did something happen with your parents?” ANS: B The nurse must assess the patient’s access to the means to carry out the plan and, if there is access, alert the parents to remove them from the home. The other questions may be important to ask but are not the most critical. DIF: Cognitive Level: Application (Applying) REF: Pages: 451-452 8. An adult attempts suicide after declaring bankruptcy. The patient is hospitalized and takes an antidepressant medication for five days. The patient is now more talkative and shows increased energy. Select the highest priority nursing intervention. a. Supervise the patient 24 hours a day. b. Begin discharge planning for the patient. c. Refer the patient to art and music therapists. d. Consider the discontinuation of suicide precautions. ANS: A The patient now has more energy and may have decided on suicide, especially considering the history of the prior suicide attempt. The patient is still a suicide risk; therefore, continuous supervision is indicated. DIF: Cognitive Level: Analysis (Analyzing) REF: Page: 452 9. A nurse and patient construct a no-suicide contract. Select the preferable wording for the contract. a. “I will not try to harm myself during the next 24 hours.” b. “I will not make a suicide attempt while I am hospitalized.” c. “For the next 24 hours, I will not kill or harm myself in any way.” d. “I will not kill myself until I call my primary nurse or a member of the staff.” ANS: C The correct answer leaves no loopholes. The wording about not harming oneself and not making an attempt leaves loopholes or can be ignored by the patient who thinks, “I am not going to harm myself, I am going to kill myself,” or “I am not going to attempt suicide, I am going to commit suicide.” A patient may call a therapist and leave the telephone to carry out the suicidal plan. DIF: Cognitive Level: Application (Applying) REF: Page: 453 10. A tearful, anxious patient at the outpatient clinic reports, “I should be dead.” The initial task of the nurse conducting the assessment interview is to: a. assess the lethality of a suicide plan. b. encourage expression of anger. c. establish a rapport with the patient. d. determine risk factors for suicide. ANS: C Establishing rapport will allow the nurse to obtain relevant assessment data such as the presence of a suicide plan, the lethality of a suicide plan, and the presence of risk factors for suicide. DIF: Cognitive Level: Analysis (Analyzing) REF: Pages: 451-454 11. Select the most helpful response for a nurse to make when a patient being treated as an outpatient states, “I am considering suicide.” a. “I’m glad you shared this. Please do not worry. We will handle it together.” b. “I think you should admit yourself to the hospital to get help.” c. “We need to talk about the good things you have to live for.” d. “Bringing this up is a very positive action on your part.” ANS: D This response gives the patient reinforcement and validation for making a positive response rather than acting out the suicidal impulse. It gives neither advice nor false reassurance, and it does not imply stereotypes such as, “You have a lot to live for.” It uses the patient’s ambivalence and sets the stage for more realistic problem-solving strategies. DIF: Cognitive Level: Application (Applying) REF: Pages: 453-456 12. Which intervention should a nurse recommend for the distressed family and friends of someone who has committed suicide? a. Participating in reminiscence therapy b. Attending a self-help group for survivors c. Contracting for two sessions of group therapy d. Completing a psychological postmortem assessment ANS: B Survivors need outlets for their feelings about the loss and the deceased person. Self-help groups provide peer support while survivors work through feelings of loss, anger, and guilt. Psychological postmortem assessment would not provide the support necessary to work through feelings of loss associated with the suicide of a family member. Reminiscence therapy is not geared to loss resolution. Contracting for two sessions of group therapy would probably not provide sufficient time to work through the issues associated with a death by suicide. DIF: Cognitive Level: Application (Applying) REF: Page: 458 13. Which statement provides the best rationale for why a nurse should closely monitor a severely depressed patient during antidepressant medication therapy? a. As depression lifts, physical energy becomes available to carry out suicide. b. Suicide may be precipitated by a variety of internal and external events. c. Suicidal patients have difficulty using social supports. d. Suicide is an impulsive act. ANS: A Antidepressant medication has the objective of relieving depression. The risk for suicide is greater as the depression lifts, primarily because the patient has more physical energy at a time when he or she may still have suicidal ideation. The other options have little to do with nursing interventions relating to antidepressant medication therapy. DIF: Cognitive Level: Comprehension (Understanding) REF: Page: 452 14. A nurse assesses a patient who reports a 3-week history of depression and crying spells. The patient says, “My business is bankrupt, and I was served with divorce papers.” Which subsequent statement by the patient alerts the nurse to a concealed suicidal message? a. “I wish I were dead.” b. “Life is not worth living.” c. “I have a plan that will fix everything.” d. “My family will be better off without me.” ANS: C Verbal clues to suicide may be overt or covert. The incorrect options are overt references to suicide. The correct option is more veiled. It alludes to the patient’s suicide as being a way to “fix everything” but does not say it outright. DIF: Cognitive Level: Application (Applying) REF: Pages: 451-452 15. A depressed patient says, “Nothing matters anymore.” What is the most appropriate response by the nurse? a. “Are you having thoughts of suicide?” b. “I am not sure I understand what you are trying to say.” c. “Try to stay hopeful. Things have a way of working out.” d. “Tell me more about what interested you before you began feeling depressed.” ANS: A The nurse must make overt what is covert; that is, the possibility of suicide must be openly addressed. Often, patients feel relieved to be able to talk about suicidal ideation. DIF: Cognitive Level: Application (Applying) REF: Pages: 453-456 16. A nurse counsels a patient with recent suicidal ideation. Which is the nurse’s most therapeutic comment? a. “Let’s make a list of all your problems and think of solutions for each one.” b. “I’m happy you’re taking control of your problems and trying to find solutions.” c. “When you have bad feelings, try to focus on positive experiences from your life.” d. “Let’s consider which problems are most important and which are less important.” ANS: D The nurse helps the patient develop effective coping skills. He or she assists the patient to reduce the overwhelming effects of problems by prioritizing them. The incorrect options continue to present overwhelming approaches to problem solving. DIF: Cognitive Level: Application (Applying) REF: Pages: 453-456 17. When assessing a patient’s plan for suicide, what aspect has priority? a. Patient’s financial and educational status b. Patient’s insight into suicidal motivation c. Availability of means and lethality of method d. Quality and availability of patient’s social support ANS: C If a person has definite plans that include choosing a method of suicide readily available, and if the method is one that is lethal (i.e., will cause the person to die with little probability for intervention), the suicide risk is considered high. These areas provide a better indication of risk than the areas mentioned in the other options. DIF: Cognitive Level: Application (Applying) REF: Pages: 451-452 18. Which understanding about individuals who attempt suicide will help a nurse plan the care for a suicidal patient? Every suicidal person should be considered: a. mentally ill. b. intent on dying. c. cognitively impaired. d. experiencing hopelessness. ANS: D Hopelessness is the characteristic common among people who attempt suicide. The incorrect options reflect myths about suicide. Not all who attempt suicide are intent on dying. Not all are mentally ill or cognitively impaired. DIF: Cognitive Level: Comprehension (Understanding) REF: Page: 449 19. Which statement by a patient during an assessment interview should alert the nurse to the patient’s need for immediate, active intervention? a. “I am mixed up, but I know I need help.” b. “I have no one for help or support.” c. “It is worse when you are a person of color.” d. “I tried to get attention before I shot myself.” ANS: B Lack of social support and social isolation increase the suicide risk. The willingness to seek help lowers the risk. Being a person of color does not suggest a higher risk; more whites commit suicide than do individuals of other racial groups. Attention seeking is not correlated with a higher risk of suicide. DIF: Cognitive Level: Application (Applying) REF: Pages: 451-452 20. The feeling experienced by a patient that should be assessed by the nurse as most predictive of elevated suicide risk is: a. hopelessness. b. sadness. c. elation. d. anger. ANS: A Of the feelings listed, hopelessness is most closely associated with increased suicide risk. Depression, aggression, impulsivity, and shame are other feelings noted as risk factors for suicide. DIF: Cognitive Level: Application (Applying) REF: Page: 449 21. Four individuals have given information about their suicide plans. Which plan evidences the highest suicide risk? a. Jumping from a 100-foot-high railroad bridge located in a deserted area late at night b. Turning on the oven and letting gas escape into the apartment during the night c. Cutting the wrists in the bathroom while the spouse reads in the next room d. Overdosing on aspirin with codeine while the spouse is out with friends ANS: A This is a highly lethal method with little opportunity for rescue. The other options are lower lethality methods with higher rescue potential. DIF: Cognitive Level: Analysis (Analyzing) REF: Pages: 451-452 22. Which individual in the emergency department should be considered at the highest risk for completing suicide? a. An adolescent Asian-American girl with superior athletic and academic skills who has asthma b. A 38-year-old single African-American female church member with fibrocystic breast disease c. A 60-year-old married Hispanic man with 12 grandchildren who has type 2 diabetes d. A 79-year-old single white man with cancer of the prostate gland ANS: D High-risk factors include being an older adult, single, and male and having a co-occurring medical illness. Cancer is one of the somatic conditions associated with increased suicide risk. Protective factors for African-American women and Hispanic individuals include strong religious and family ties. Asian Americans have a suicide rate that increases with age. DIF: Cognitive Level: Application (Applying) REF: Pages: 449-451 23. A nurse answers a suicide crisis line. A caller says, “I live alone in a home several miles from my nearest neighbors. I have been considering suicide for 2 months. I have had several drinks and now my gun is loaded. I’m going to shoot myself in the heart.” How would the nurse assess the lethality of this plan? a. No risk b. Low level c. Moderate level d. High level ANS: D The patient has a highly detailed plan, a highly lethal method, the means to carry it out, lowered impulse control because of alcohol ingestion, and a low potential for rescue. DIF: Cognitive Level: Application (Applying) REF: Pages: 451-452 24. A staff nurse tells another nurse, “I evaluated a new patient using the SAD PERSONS scale and got a score of 10. I’m wondering if I should send the patient home.” Select the best reply by the second nurse. a. “That action would seem appropriate.” b. “A score over 8 requires immediate hospitalization.” c. “I think you should strongly consider hospitalization for this patient.” d. “Give the patient a follow-up appointment. Hospitalization may be needed soon.” ANS: B A SAD PERSONS scale score of 0 to 5 suggests home care with follow-up. A score of 6 to 8 requires psychiatric consultation. A score over 8 calls for hospitalization. DIF: Cognitive Level: Application (Applying) REF: Page: 452 25. A patient recently hospitalized for two weeks committed suicide during the night. Which initial measure will be most helpful for staff members and other patients regarding this event? a. Request the public information officer to make an announcement to the local media. b. Hold a staff meeting to express feelings and plan the care for other patients. c. Ask the patient’s roommate not to discuss the event with other patients. d. Quickly discharge as many patients as possible to prevent panic. ANS: B Interventions should be aimed at helping the staff and patients come to terms with the loss and to grow because of the incident. Then, a community meeting should be scheduled to allow other patients to express their feelings and request help. Staff members should be prepared to provide additional support and reassurance to patients and should seek opportunities for peer support. The incorrect options will not control information or may result in unsafe care. DIF: Cognitive Level: Application (Applying) REF: Page: 453 | Pages: 457-458 26. A severely depressed patient who has been on suicide precautions tells the nurse, “I am feeling a lot better, so you can stop watching me. I have taken too much of your time already.” Which is the nurse’s best response? a. “I wonder what this sudden change is all about. Please tell me more.” b. “I am glad you are feeling better. The team will consider your request.” c. “You should not try to direct your care. Leave that to the treatment team.” d. “Because we are concerned about your safety, we will continue with our plan.” ANS: D When a patient seeks to have precautions lifted by professing to feel better, the patient may be seeking greater freedom in which to attempt suicide. Changing the treatment plan requires careful evaluation of outcome indicators by the staff. The incorrect options will not cause the patient to admit to a suicidal plan, do not convey concern for the patient, or suggest that the patient is not a partner in the care process. DIF: Cognitive Level: Application (Applying) REF: Pages: 451-454 27. A new nurse says to a peer, “My newest patient is diagnosed with schizophrenia. At least I won’t have to worry about suicide risk.” Which response by the peer would be most helpful? a. “Let’s reconsider your plan. Suicide risk is high in patients diagnosed with schizophrenia.” b. “Suicide is a risk for any patient diagnosed with schizophrenia who uses alcohol or drugs.” c. “Patients diagnosed with schizophrenia are usually too disorganized to attempt suicide.” d. “Visual hallucinations often prompt suicide among patients diagnosed with schizophrenia.” ANS: A Up to 10% of patients diagnosed with schizophrenia die from suicide, usually related to depressive symptoms occurring in the early years of the illness. Depressive symptoms are related to suicide among patients diagnosed with schizophrenia. Patients diagnosed with schizophrenia usually have auditory, not visual, hallucinations. Although the use of drugs and alcohol compounds the risk for suicide, it is independent of schizophrenia. DIF: Cognitive Level: Application (Applying) REF: Pages: 448-449 | Page: 451 28. The parents of identical twins ask a nurse for advice. One twin committed suicide a month ago. Now the parents are concerned that the other twin may also have suicidal tendencies. Which comment by the nurse is accurate? a. “Genetics are associated with suicide risk. Monitoring and support are important.” b. “Apathy underlies suicide. Instilling motivation is the key to health maintenance.” c. “Your child is unlikely to act out suicide when identifying with a suicide victim.” d. “Fraternal twins are at higher risk for suicide than identical twins.” ANS: A Twin studies suggest the presence of genetic factors in suicide; however, separating genetic predisposition to suicide from predisposition to depression or alcoholism is difficult. Primary interventions can be helpful in promoting and maintaining health and possibly counteracting the genetic load. The incorrect options are untrue statements or oversimplifications. DIF: Cognitive Level: Application (Applying) REF: Page: 449 MULTIPLE RESPONSE 1. A college student failed two examinations. The student cried for hours and then tried to call a parent but got no answer. The student then suspended access to his social networking web site. Which suicide risk factors are present? Select all that apply. a. History of earlier suicide attempt b. Co-occurring medical illness c. Recent stressful life event d. Self-imposed isolation e. Shame or humiliation ANS: C, D, E Failing examinations in the academic major constitutes a recent stressful life event. Shame and humiliation related to the failure can be hypothesized. The inability to contact parents can be seen as a recent lack of social support, as can the roommate’s absence from the dormitory. Terminating access to one’s social networking site represents self-imposed isolation. This scenario does not provide data regarding a history of an earlier suicide attempt, a family history of suicide, or of co-occurring medical illness. DIF: Cognitive Level: Application (Applying) REF: Pages: 448-452 TOP: Nursing Process: Assessment MSC: NCLEX: Psychosocial Integrity 2. A patient with suicidal impulses is on the highest level of suicide precautions. Which measures should the nurse incorporate into the patient’s plan of care? Select all that apply. a. Allow no glass or metal on meal trays. b. Remove all potentially harmful objects from the patient’s possession. c. Maintain arm’s length, one-on-one nursing observation around the clock. d. Check the patient’s whereabouts every hour. Make verbal contact at least three times each shift. e. Check the patient’s whereabouts every 15 minutes, and make frequent verbal contacts. f. Keep the patient within visual range while he or she is awake. Check every 15 to 30 minutes while the patient is sleeping. ANS: A, B, C One-on-one observation is necessary for anyone who has limited control over suicidal impulses. Plastic dishes on trays and the removal of potentially harmful objects from the patient’s possession are measures included in any level of suicide precautions. The remaining options are used in less stringent levels of suicide precautions. DIF: Cognitive Level: Application (Applying) REF: Pages: 453-454 TOP: Nursing Process: Planning MSC: NCLEX: Safe, Effective Care Environment 3. A nurse assesses five newly hospitalized patients. Which patients have the highest suicide risk? Select all that apply. a. 82-year-old white man b. 17-year-old white female adolescent c. 39-year-old African-American man d. 29-year-old African-American woman e. 22-year-old man with traumatic brain injury ANS: A, B, E Whites have suicide rates almost twice those of nonwhites, and the rate is particularly high for older adult men, adolescents, and young adults. Other high risk groups include young African-American men, Native-American men, older Asian Americans, and persons with traumatic brain injury. DIF: Cognitive Level: Analysis (Analyzing) REF: Pages: 448-451 TOP: Nursing Process: Assessment MSC: NCLEX: Safe, Effective Care Environment 4. A nurse assesses the health status of soldiers returning from Afghanistan. Screening for which health problems will be a priority? Select all that apply. a. Schizophrenia b. Eating disorder c. Traumatic brain injury d. Oppositional defiant disorder e. Post-traumatic stress disorder ANS: C, E Traumatic brain injury and post-traumatic stress disorder each occur in approximately 20% of soldiers returning from Afghanistan. Some soldiers have both problems. The incidence of disorders identified in the distractors would be expected to parallel the general population. DIF: Cognitive Level: Comprehension (Understanding) REF: Pages: 447-448 1. Select the completion of this sentence that demonstrates an adult in coping in a healthy way: “ I am feeling so angry right now… C I want to talk to someone about it 2. In a hostile voice, a patient experiencing mania yells at the nurse” You will listen to me and not interrupt. I have some really important stuff to say. I'm tired of you nurses and doctors acting like you have all the answers.’ To facilitate effective communication, which initial response should the nurse provide? C. “ It's our job to help you get through this manic episode.” 3. A female nurse is appointed to a committee with seven men. At the beginning of the meeting the chairman asks the nurse to be the secretary. The nurse responds,”No you're just asking me to be secretary because I’m the only women here.” which response would have been more effective? B “I would be glad to perform another role for our committee” 4. An 8 year old tells a parent, “I like to scare kids at school by showing them pictures of clowns. Some kids are terrified. “ how should the nurse counsel the parents regarding this behavior? C. Educate both parents about bullying, including possible origins and long term effects 5.A women experienced a double mastectomy yesterday. Now she cheerfully says to the nurse, “I didnt need those things anyway. No more wet T-shirt contests for me!” How should the nurse interpret this comment? B. The comment is sarcastic, which may reflect anger. 1. While interacting with a 62-year old adult with progressive neurocognitive disorder, the nurse observes that the adult has slow responses and difficulty finding the right words. What is the nurses best initial action? a. C. Use silence to allow the adult an opportunity to compose responses. 2. An adult diagnosed with stage 2 alzheimer's disease begins a new prescription for rivastigmine(exelon). Which nursing diagnosis has the highest priority to add to the plan of care? D Risk for imbalanced nutrition, less than body requirements 3. Which newly hospitalized patients should the nurse monitor closely for development of delirium? A 48-year old who usually drinks a six-pack of beer daily 4. An 84-year old tells the nurse, “ i do four or five number puzzles everyday to keep my brain healthy and sharp .” When considering a holistic approach to maintaining mental health, the nurse should respond: B “let’s think of some other activities we can add to your daily routine” 5. A family member asks the nurse, “ I know my uncle’s alzheimer's disease has progressed but is there any medication that can help him now?” Which response by the nurse is correct? C “there are few medications that may help. Let’s discuss it with the healthcare provider.” Chapter 24: Anger, Aggression, and Violence Varcarolis: Essentials to Psychiatric Mental Health Nursing, 2nd Edition – Revised Reprint MULTIPLE CHOICE 1. Which behavior best demonstrates aggression? a. Stomping away from the nurses’ station, going to another room, and grabbing a snack from another patient. b. Bursting into tears, leaving the community meeting, and sitting on a bed hugging a pillow and sobbing. c. Telling the primary nurse, “I felt angry when you said I could not have a second helping at lunch.” d. Telling the medication nurse, “I am not going to take that or any other medication you try to give me.” ANS: A Aggression is harsh physical or verbal action that reflects rage, hostility, and the potential for physical or verbal destructiveness. Aggressive behavior violates the rights of others. The incorrect options do not feature violation of another’s rights. DIF: Cognitive Level: Comprehension (Understanding) REF: Page: 462 2. Which scenario predicts the highest risk for directing violent behavior toward others? a. Major depressive disorder with delusions of worthlessness b. Obsessive-compulsive disorder; performing many rituals c. Paranoid delusions of being followed by a military attack team d. Completion of alcohol withdrawal and beginning a rehabilitation program ANS: C The correct answer illustrates the greatest disruption of ability to perceive reality accurately. People who feel persecuted may strike out against those believed to be persecutors. The patients identified in the distractors have better reality-testing ability. DIF: Cognitive Level: Application (Applying) REF: Pages: 465-466 3. A patient is hospitalized after an arrest for breaking windows in the home of a former domestic partner. The history reveals childhood abuse by a punitive parent, torturing family pets and an arrest for disorderly conduct. Which nursing diagnosis has priority? a. Risk for injury b. Post-trauma response c. Disturbed thought processes d. Risk for other-directed violence ANS: D The defining characteristics for Risk for other-directed violence include a history of being abused as a child, having committed other violent acts, and demonstrating poor impulse control. The defining characteristics for the other diagnoses are not present in this scenario. DIF: Cognitive Level: Analysis (Analyzing) REF: Page: 466 4. A confused older adult patient in a skilled care facility is sleeping. The nurse enters the room quietly and touches the bed to see if it is wet. The patient awakens and hits the nurse in the face. Which statement best explains the patient’s action? a. Older adult patients often demonstrate exaggerations of behaviors used earlier in life. b. Crowding in skilled care facilities increases individual tendencies toward violence. c. The patient interpreted the health care worker’s behavior as potentially harmful. d. This patient learned violent behavior by watching other patients act out. ANS: C Confused patients are not always able to evaluate accurately the actions of others. This patient behaved as though provoked by the intrusive actions of the staff member. DIF: Cognitive Level: Application (Applying) REF: Pages: 465-466 | Pages: 474-475 5. A patient is pacing the hall near the nurses’ station and swearing loudly. An appropriate initial intervention for the nurse would be to address the patient by name and say: a. “Hey, what’s going on?” b. “Please quiet down immediately.” c. “I’d like to talk with you about how you’re feeling right now.” d. “You must go to your room and try to get control of yourself.” ANS: C Intervention should begin with an analysis of the patient and situation. With this response, the nurse is attempting to hear the patient’s feelings and concerns, which leads to the next step of planning an intervention. The incorrect responses are authoritarian, creating a power struggle between the patient and nurse. DIF: Cognitive Level: Application (Applying) REF: Pages: 468-471 6. A patient was responding to auditory hallucinations earlier in the morning. The patient approaches the nurse, shaking a fist and shouting, “Back off!” and then goes into the day room. As the nurse follows the patient into the day room, the nurse should: a. make sure adequate physical space exists between the nurse and the patient. b. move into a position that allows the patient to be close to the door. c. maintain one arm’s length distance from the patient. d. sit down in a chair near the patient. ANS: A Making sure space is present between the nurse and the patient avoids invading the patient’s personal space. Personal space needs increase when a patient feels anxious and threatened. Allowing the patient to block the nurse’s exit from the room is not wise. Closeness may be threatening to the patient and provoke aggression. Sitting is inadvisable until further assessment suggests the patient’s aggression is abating. One arm’s length is inadequate space. DIF: Cognitive Level: Application (Applying) REF: Pages: 467-468 7. An intramuscular dose of antipsychotic medication needs to be given to a patient who is becoming increasingly more aggressive. The patient is in the day room. The nurse should enter the day room: a. and say, “Would you like to come to your room and take some medication your doctor prescribed for you?” b. accompanied by three staff members and say, “Please come to your room so I can give you some medication that will help you feel more comfortable.” c. and place the patient in a basket-hold and then say, “I am going to take you to your room to give you an injection of medication to calm you.” d. accompanied by two security guards and tell the patient, “You can come to your room willingly so I can give you this medication, or the aide and I will take you there.” ANS: B A patient gains feelings of security if he or she sees that others are present to help with control. The nurse gives a simple direction, honestly states what is going to happen, and reassures the patient that the intervention will be helpful. This positive approach assumes that the patient can act responsibly and will maintain control. Physical control measures should be used only as a last resort. The security guards are likely to intimidate the patient and increase feelings of vulnerability. DIF: Cognitive Level: Application (Applying) REF: Pages: 465-469 8. After an assault by a patient, a nurse has difficulty sleeping, startles easily, and is preoccupied with the incident. The nurse says, “I dread facing potentially violent patients.” Which response would be the most urgent reason for this nurse to seek supervision? a. Startle reactions b. Difficulty sleeping c. A wish for revenge d. Preoccupation with the incident ANS: C The desire for revenge signals an urgent need for professional supervision to work through anger and counter the aggressive feelings. The distractors are normal in a person who has been assaulted. Nurses are usually relieved with crisis intervention and follow-up designed to give support, help the individual regain a sense of control, and make sense of the event. DIF: Cognitive Level: Analysis (Analyzing) REF: Page: 473 9. The staff development coordinator plans to teach use of physical management techniques when patients become assaultive. Which topic should be emphasized? a. Practice and teamwork b. Spontaneity and surprise c. Caution and superior size d. Diversion and physical outlets ANS: A Intervention techniques are learned behaviors that must be practiced to be used in a smooth, organized fashion. Every member of the intervention team should be assigned a specific task to carry out before beginning the intervention. The other options are useless if the staff does not know how to use physical techniques and how to apply them in an organized fashion. DIF: Cognitive Level: Application (Applying) REF: Pages: 467-469 10. An adult patient assaults another patient and is restrained. One hour later, which statement by this restrained patient necessitates the nurse’s immediate attention? a. “I hate all of you!” b. “My fingers are tingly.” c. “You wait until I tell my lawyer.” d. “It was not my fault. The other patient started it.” ANS: B The correct response indicates impaired circulation and necessitates the nurse’s immediate attention. The incorrect responses indicate that the patient has continued aggressiveness and agitation. DIF: Cognitive Level: Analysis (Analyzing) REF: Pages: 471-473 11. Which is an effective nursing intervention to assist an angry patient to learn to manage anger without violence? a. Help the patient identify a thought that increases anger, find proof for or against the belief, and substitute reality-based thinking. b. Provide negative reinforcement such as restraint or seclusion in response to angry outbursts, whether or not violence is present. c. Use aversive conditioning, such as popping a rubber band on the wrist, to help extinguish angry feelings. d. Administer an antipsychotic or antianxiety medication when the patient feels angry. ANS: A Anger has a strong cognitive component; therefore, using cognition to manage anger is logical. The incorrect options do nothing to help the patient learn anger management. DIF: Cognitive Level: Application (Applying) REF: Pages: 475-476 12. Which assessment finding presents the greatest risk for violent behavior? A patient who: a. is severely agoraphobic. b. has a history of intimate partner violence. c. demonstrates bizarre somatic delusions. d. verbalizes hopelessness and powerlessness. ANS: B A history of prior aggression or violence is the best predictor of patients who may become violent. Patients diagnosed with anxiety disorders are not particularly prone to violence unless panic occurs. Patients experiencing hopelessness and powerlessness may have co-existing anger, but violence is not often demonstrated. Patients experiencing paranoid delusions are at greater risk for violence than those with bizarre somatic delusions. DIF: Cognitive Level: Application (Applying) REF: Pages: 465-466 TOP: Nursing Process: Assessment MSC: NCLEX: Safe, Effective Care Environment 13. A patient being admitted suddenly pulls a knife from a coat pocket and threatens, “I will kill anyone who tries to get near me.” An emergency code is called. The patient is safely disarmed and placed in seclusion. Justification for the use of seclusion is that the patient: a. evidences a thought disorder, rendering rational discussion ineffective. b. presents a clear and present danger to others. c. presents a clear escape risk. d. is psychotic. ANS: B The patient’s threat to kill self or others with the knife he possesses constitutes a clear and present danger to self and others. The distractors are not sufficient reasons for seclusion. DIF: Cognitive Level: Comprehension (Understanding) REF: Pages: 467-469 | Pages: 471-473 TO 14. A patient sits in silence for 20 minutes after a therapy appointment, appearing tense and vigilant. The patient abruptly stands and paces back and forth, clenching and unclenching fists, and then stops and stares in the face of a staff member. The patient is: a. demonstrating withdrawal. b. working through angry feelings. c. attempting to use relaxation strategies. d. exhibiting clues to potential aggression. ANS: D The description of the patient’s behavior shows the classic signs of someone whose potential for aggression is increasing. DIF: Cognitive Level: Comprehension (Understanding) REF: Pages: 465-466 15. A cognitively impaired patient has been a widow for 30 years. This patient is frantically trying to leave the unit, saying, “I have to go home to cook dinner before my husband arrives from work.” To intervene with validation therapy, the nurse should first say: a. “You must come away from the door.” b. “You have been a widow for many years.” c. “You want to go home to prepare your husband’s dinner?” d. “Was your husband angry if you did not have dinner ready on time?” ANS: C Validation therapy meets the patient “where she or he is at the moment” and acknowledges the patient’s wishes. Validation does not seek to redirect, reorient, or probe. The incorrect options do not validate the patient’s feelings. DIF: Cognitive Level: Application (Applying) REF: Pages: 474-475 16. A patient with a history of anger and impulsivity is hospitalized after an accident resulting in injuries. When in pain, the patient loudly scolds the nursing staff for “not knowing enough to give me pain medicine when I need it.” Which nursing intervention would best address this problem? a. Tell the patient to notify nursing staff 30 minutes before the pain returns so the medication can be prepared. b. Urge the health care provider to change the prescription for pain medication from as needed to a regular schedule. c. Tell the patient that verbal assaults on nurses will not shorten the wait for pain medication. d. Have the clinical nurse leader request a psychiatric consultation. ANS: B Scheduling the medication at specific intervals will help the patient anticipate when the medication can be given. Receiving the medication promptly on schedule, rather than expecting nurses to anticipate the pain level, should reduce anxiety and anger. The patient cannot predict the onset of pain before it occurs. DIF: Cognitive Level: Analysis (Analyzing) REF: Pages: 466-469 TOP: Nursing Process: Implementation MSC: NCLEX: Physiological Integrity 17. A patient has a history of impulsively acting out anger by striking others. Which would be an appropriate plan for avoiding such incidents? a. Explain that restraint and seclusion will be used if violence occurs. b. Help the patient identify incidents that trigger impulsive acting out. c. Offer one-on-one supervision to help the patient maintain control. d. Administer lorazepam (Ativan) every 4 hours to reduce the patient’s anxiety. ANS: B Identifying trigger incidents allows the patient and nurse to plan interventions to reduce irritation and frustration that lead to acting out anger and to put more adaptive coping strategies eventually into practice. DIF: Cognitive Level: Application (Applying) REF: Pages: 466-468 18. A patient with severe injuries is irritable, angry, and belittles the nurses. As a nurse changes a dressing, the patient screams, “Don’t touch me! You are so stupid. You will make it worse!” Which intervention uses a cognitive technique to help this patient? a. Discontinue the dressing change without comments and leave the room. b. Stop the dressing change, saying, “Perhaps you would like to change your own dressing.” c. Continue the dressing change, saying, “Do you know this dressing change is needed so your wound will not get infected?” d. Continue the dressing change, saying, “Unfortunately, you have no choice. Your doctor ordered this dressing change.” ANS: C Anger is cognitively driven. The correct answer helps the patient test his cognitions and may help lower his anger. The incorrect options will escalate the patient’s anger by belittling or escalating the patient’s sense of powerlessness. DIF: Cognitive Level: Application (Applying) REF: Pages: 475-476 19. Which medication should a nurse administer to provide immediate intervention for a psychotic patient whose aggressive behavior continues to escalate despite verbal intervention? a. lithium (Eskalith) b. trazodone (Desyrel) c. olanzapine (Zyprexa) d. valproic acid (Depakene) ANS: C Olanzapine is a short-acting antipsychotic drug that is useful in calming angry, aggressive patients regardless of their diagnosis. The other drugs listed require long-term use to reduce anger. Lithium is for patients with bipolar disorder. Trazodone is for patients with depression, insomnia, or chronic pain. Valproic acid is for patients with bipolar disorder or borderline personality disorder. DIF: Cognitive Level: Application (Applying) REF: Page: 476 20. An emergency department nurse realizes that the spouse of a patient is becoming increasingly irritable while waiting. Which intervention should the nurse use to prevent escalation of anger? a. Explain that the patient’s condition is not life threatening. b. Periodically provide an update and progress report on the patient. c. Explain that all patients are treated in order, based on their medical needs. d. Suggest that the spouse return home until the patient’s treatment is completed. ANS: B Periodic updates reduce anxiety and defuse anger. This strategy acknowledges the spouse’s presence and concerns. The incorrect options are likely to increase anger because they imply that the anxiety is inappropriate. DIF: Cognitive Level: Application (Applying) REF: Pages: 473-474 21. Information from a patient’s record that indicates marginal coping skills and the need for careful assessment of the risk for violence is a history of: a. childhood trauma. b. family involvement. c. academic problems. d. substance abuse. ANS: D The nurse should suspect marginal coping skills in a patient with substance abuse. He or she is often anxious, may be concerned about inadequate pain relief, and may have a personality style that externalizes blame. The incorrect options do not signal as high a degree of risk as chemical dependence. DIF: Cognitive Level: Comprehension (Understanding) REF: Pages: 465-466 22. A patient diagnosed with pneumonia has been hospitalized for 4 days. Family members describe the patient as “a difficult person who finds fault with others.” The patient verbally abuses nurses for providing poor care. The most likely explanation for this behavior lies in: a. poor childrearing that did not teach respect for others. b. automatic thinking, leading to cognitive distortion. c. personality style that externalizes problems. d. delusions that others wish to deliver harm. ANS: C Patients whose personality style causes them to externalize blame see the source of their discomfort and anxiety as being outside themselves. They displace anger and are often unable to soothe themselves. The incorrect options are less likely to have a bearing on this behavior. DIF: Cognitive Level: Application (Applying) REF: Pages: 465-466 23. A patient with burn injuries has had good coping skills for several weeks. Today, a new nurse is poorly organized and does not follow the patient’s usual schedule is. By mid-afternoon, the patient is angry and loudly complains to the nurse manager. Which is the nurse manager’s best response? a. Explain the reasons for the disorganization, and take over the patient’s care for the rest of the shift. b. Acknowledge and validate the patient’s distress and ask, “What would you like to have happen?” c. Apologize and explain that the patient will have to accept the situation for the rest of the shift. d. Ask the patient to control the anger and explain that allowances must be made for new staff members. ANS: B When a patient with good coping skills is angry and overwhelmed, the goal is to reestablish a means of dealing with the situation. The nurse should solve the problem with the patient by acknowledging the patient’s feelings, validating them as understandable, apologizing if necessary, and then seeking an acceptable solution. Often patients can tell the nurse what they would like to have happen as a reasonable first step. DIF: Cognitive Level: Application (Applying) REF: Pages: 468-469 24. When a patient’s aggression quickly escalates, which principle applies to the selection of nursing interventions? a. Staff members should match the patient’s affective level and tone of voice. b. Ask the patient what intervention would be most helpful. c. Immediately use physical containment measures. d. Begin with the least restrictive measure possible. ANS: D Standards of care require that staff members use the least restrictive measure possible. This becomes the guiding principle for intervention. Physical containment is seldom the least restrictive measure. Asking the out-of-control patient what to do is rarely helpful. It may be an effective strategy during the preassaultive phase but is less effective during escalation. DIF: Cognitive Level: Comprehension (Understanding) REF: Pages: 466-467 25. A new patient immediately requires seclusion on admission. The assessment is incomplete, and no prescriptions have been written. Immediately after safely secluding the patient, which action has priority? a. Provide an opportunity for the patient to go to the bathroom. b. Notify the health care provider and obtain a seclusion order. c. Notify the hospital risk manager. d. Debrief the staff. ANS: B Emergency seclusion can be effected by a credentialed nurse but must be followed by securing a medical order within the period specified by the state and agency. The incorrect options are not immediately necessary from a legal standpoint. DIF: Cognitive Level: Application (Applying) REF: Pages: 471-473 MULTIPLE RESPONSE 1. A patient with a history of command hallucinations approaches the nurse, yelling obscenities. The patient mumbles and then walks away. The nurse follows. Which nursing actions are most likely to be effective in de-escalating this scenario? Select all that apply. a. State the expectation that the patient will stay in control. b. State that the patient cannot be understood when mumbling. c. Tell the patient, “You are behaving inappropriately.” d. Offer to provide the patient with medication to help. e. Speak in a firm but calm voice. ANS: A, D, E Stating the expectation that the patient will maintain control of behavior reinforces positive, healthy behavior and avoids challenging the patient. Offering an as-needed medication provides support for the patient trying to maintain control. A firm but calm voice will likely comfort and calm the patient. Belittling remarks may lead to aggression. Criticism will probably prompt the patient to begin shouting. DIF: Cognitive Level: Analysis (Analyzing) REF: Pages: 468-471 TOP: Nursing Process: Implementation MSC: NCLEX: Safe, Effective Care Environment 2. A nurse directs the intervention team who must take an aggressive patient to seclusion. Other patients were removed from the area. Before approaching the patient, the nurse should ensure that staff take which actions? Select all that apply. a. Remove jewelry, glasses, and harmful items from the patient and staff members. b. Appoint a person to clear a path and open, close, or lock doors. c. Quickly approach the patient, and grab the closest extremity. d. Select the person who will communicate with the patient. e. Move behind the patient to use the element of surprise. ANS: A, B, D Injury to staff members and to the patient should be prevented. Only one person should explain what will happen and direct the patient; this person might be the nurse or staff member who has a good relationship with the patient. A clear pathway is essential; those restraining a limb cannot use keys, move furniture, or open doors. The nurse is usually responsible for administering the medication once the patient is restrained. Each staff member should have an assigned limb rather than just grabbing the closest limb. This system could leave one or two limbs unrestrained. Approaching in full view of the patient reduces suspicion. DIF: Cognitive Level: Application (Applying) REF: Pages: 467-469 | Pages: 471-473 TOP: Nursing Process: Planning MSC: NCLEX: Safe, Effective Care Environment 3. Which central nervous system structures are most associated with anger and aggression? Select all that apply. a. Amygdala b. Cerebellum c. Basal ganglia d. Temporal lobe e. Parietal lobe ANS: A, D The amygdala mediates anger experiences and helps a person judge an event as either rewarding or aversive. The temporal lobe, which is part of the limbic system, also plays a role in aggressive behavior. The cerebellum manages equilibrium, muscle tone, and movement. The basal ganglia are involved in movement. The parietal lobe is involved in interpreting sensations. DIF: Cognitive Level: Comprehension (Understanding) REF: Pages: 463-464 TOP: Nursing Process: Planning MSC: NCLEX: Physiological Integrity 4. Which behaviors are most consistent with the clinical picture of a patient who is becoming increasingly aggressive? Select all that apply. a. Pacing b. Crying c. Withdrawn affect d. Rigid posture with clenched jaw e. Staring with narrowed eyes into the eyes of another ANS: A, D, E Crying and a withdrawn affect are not cited by experts as behaviors indicating that the individual has a high potential to behave violently. The other behaviors are consistent with the increased risk for other-directed violence. DIF: Cognitive Level: Application (Applying) REF: Pages: 465-466 TOP: Nursing Process: Assessment MSC: NCLEX: Safe, Effective Care Environment 5. Because an intervention is required to control a patient’s aggressive behavior, a critical incident debriefing takes place. Which topics should be the focus of the discussion? Select all that apply. a. Patient behavior associated with the incident b. Genetic factors associated with aggression c. Intervention techniques used by staff d. Effect of environmental factors e. Review of theories of aggression ANS: A, C, D The patient’s behavior, the intervention techniques used, and the environment in which the incident occurred are important to establish realistic outcomes and effective nursing interventions. Discussing the views about the theoretical origins of aggression is less effective. DIF: Cognitive Level: Analysis (Analyzing) REF: Pages: 472-473 Chapter 18: Neurocognitive Disorders MULTIPLE CHOICE Quiz 5 1. An older adult takes digoxin and hydrochlorothiazide daily, as well as lorazepam (Ativan) as needed for anxiety. Over 2 days, this adult developed confusion, slurred speech, an unsteady gait, and fluctuating levels of orientation. These findings are most characteristic of: a. delirium. b. dementia. c. amnestic syndrome. d. Alzheimer disease. ANS: A Delirium is characterized by an abrupt onset of fluctuating levels of awareness, clouded consciousness, perceptual disturbances, and disturbed memory and orientation. The onset of dementia or Alzheimer disease, a type of dementia, is more insidious. Amnestic syndrome involves memory impairment without other cognitive problems. DIF: Cognitive Level: Application (Applying) REF: Pages: 338-340 2. A patient experiencing fluctuating levels of awareness, confusion, and disturbed orientation shouts, “Bugs are crawling on my legs! Get them off!” Which problem is the patient experiencing? a. Aphasia b. Dystonia c. Tactile hallucinations d. Mnemonic disturbance ANS: C The patient feels bugs crawling on both legs, although no sensory stimulus is actually present. This description coincides with the definition of a hallucination, a false sensory perception. Tactile hallucinations may be part of the symptom constellation of delirium. Aphasia refers to a speech disorder. Dystonia refers to excessive muscle tonus. Mnemonic disturbance is associated with dementia rather than delirium. Comprehension (Understanding) REF: Pages: 340-343 3. A patient experiencing fluctuating levels of consciousness, disturbed orientation, and perceptual alteration begs, “Someone get these bugs off me.” What is the nurse’s best response? a. “There are no bugs on your legs. Your imagination is playing tricks on you.” b. “Try to relax. The crawling sensation will go away sooner if you can relax.” c. “Don’t worry. I will have someone stay here and brush off the bugs for you.” d. “I don’t see any bugs, but I know you are frightened so I will stay with you.” ANS: D When hallucinations are present, the nurse should acknowledge the patient’s feelings and state the nurse’s perception of reality, but not argue. Staying with the patient increases feelings of security, reduces anxiety, offers the opportunity for reinforcing reality, and provides a measure of physical safety. Denying the patient’s perception without offering help does not emotionally support the patient. Telling the patient to relax makes the patient responsible for self-soothing. Telling the patient that someone will brush the bugs away supports the perceptual distortions. REF: Page: 340 | Pages: 342-344 | Page: 352 4. What is the priority nursing diagnosis for a patient experiencing fluctuating levels of consciousness, disturbed orientation, and visual and tactile hallucinations? a. Bathing/hygiene self-care deficit, related to altered cerebral function, as evidenced by confusion and inability to perform personal hygiene tasks b. Risk for injury, related to altered cerebral function, misperception of the environment, and unsteady gait c. Disturbed thought processes, related to medication intoxication, as evidenced by confusion, disorientation, and hallucinations d. Fear, related to sensory perceptual alterations, as evidenced by hiding from imagined ferocious dogs ANS: B The physical safety of the patient is the highest priority among the diagnoses given. Many opportunities for injury exist when a patient misperceives the environment as distorted, threatening, or harmful; when the patient exercises poor judgment; and when the patient’s sensorium is clouded. The other diagnoses may be concerns but are lower priorities. DIF: Cognitive Level: Application (Applying) REF: Pages: 342-343 Quiz 5 5. What is the priority intervention for a patient diagnosed with delirium who has fluctuating levels of consciousness, disturbed orientation, and perceptual alterations? a. Avoidance of physical contact b. High level of sensory stimulation c. Careful observation and supervision d. Application of wrist and ankle restraints ANS: C Careful observation and supervision are of ultimate importance because an appropriate outcome would be that the patient remains safe and free from injury while hospitalized. Physical contact during care cannot be avoided. Restraint is a last resort, and sensory stimulation should be reduced. DIF: Cognitive Level: Application (Applying) REF: Pages: 342-343 Quiz 5 6. Which environmental adjustment should the nurse make for a patient experiencing delirium with perceptual alterations? a. Keep the patient by the nurse’s desk while the patient is awake. Provide rest periods in a room with a television on. b. Light the room brightly, day and night. Awaken the patient hourly to assess mental status. c. Maintain soft lighting day and night. Keep a radio on low volume continuously. d. Provide a well-lit room without glare or shadows. Limit noise and stimulation. ANS: D A quiet, shadow-free room offers an environment that produces the fewest sensory perceptual distortions for a patient experiencing cognitive impairment associated with delirium. The other options have the potential to produce increased perceptual alterations. DIF: Cognitive Level: Application (Applying) REF: Page: 342 | Page: 344 7. Which description best applies to a hallucination? A patient: a. looks at shadows on a wall and says, “I see scary faces.” b. states, “I feel bugs crawling on my legs and biting me.” c. becomes anxious when the nurse leaves his or her bedside. d. tries to hit the nurse when vital signs are taken. ANS: B hallucination is a false sensory perception occurring without a corresponding sensory stimulus. Feeling bugs on the body when none are present is a tactile hallucination. Misinterpreting shadows as faces is an illusion. An illusion is a misinterpreted sensory perception. The incorrect options are examples of behaviors that sometimes occur during delirium and are related to fluctuating levels of awareness and misinterpreted stimuli. DIF: Cognitive Level: Comprehension (Understanding) REF: Page: 340 8. Consider these health problems: Lewy body disease, Pick disease, and Korsakoff syndrome. Which term unifies these problems? a. Intoxication b. Dementia c. Delirium d. Amnesia ANS: B The listed health problems are all forms of dementia. DIF: Cognitive Level: Comprehension (Understanding) REF: Page: 338 9. When used for treatment of patients diagnosed with Alzheimer disease, which medication would be expected to antagonize N-methyl-D-aspartate (NMDA) channels rather than cholinesterase? a. donepezil (Aricept) b. rivastigmine (Exelon) c. memantine (Namenda) d. galantamine (Razadyne) ANS: C Memantine blocks the NMDA channels and is used in moderate-to-late stages of the disease. Donepezil, rivastigmine, and galantamine are all cholinesterase inhibitors. These drugs increase the availability of acetylcholine and are most often used to treat mild-to-moderate Alzheimer disease. DIF: Cognitive Level: Application (Applying) REF: Page: 356 10. An older adult was stopped by police for driving through a red light. When asked for a driver’s license, the adult hands the police officer a pair of sunglasses. What sign of dementia is evident? a. Aphasia b. Apraxia c. Agnosia d. Memory impairment ANS: C Agnosia refers to the loss of sensory ability to recognize objects. Aphasia refers to the loss of language ability. Apraxia refers to the loss of purposeful movement. No evidence of memory loss is revealed in this scenario. DIF: Cognitive Level: Comprehension (Understanding) REF: Page: 346 11. An older adult drove to a nearby store but was unable to remember how to get home or state an address. When police took the person home, the spouse reported frequent wandering into neighbors’ homes. Alzheimer disease was subsequently diagnosed. Which stage of Alzheimer disease is evident? a. 1 (mild) b. 2 (moderate) c. 3 (moderate to severe) d. 4 (late) ANS: B In stage 2 (moderate), deterioration is evident. Memory loss may include the inability to remember addresses or the date. Activities such as driving may become hazardous, and frustration by the increasing difficulty of performing ordinary tasks may be experienced. Hygiene may begin to deteriorate. Stage 3 (moderate to severe) finds the individual unable to identify familiar objects or people and needing direction for the simplest of tasks. In stage 4 (late), the ability to talk and walk are eventually lost, and stupor evolves. DIF: Cognitive Level: Analysis (Analyzing) REF: Pages: 346-348 12. Consider these problems: apolipoprotein E (apoE) malfunction, neuritic plaques, neurofibrillary tangles, granulovascular degeneration, and brain atrophy. Which condition corresponds to this group? a. Alzheimer disease b. Wernicke encephalopathy c. Central anticholinergic syndrome d. Acquired immunodeficiency syndrome (AIDS)–related dementia ANS: A The problems are all aspects of the pathophysiologic characteristics of Alzheimer disease. DIF: Cognitive Level: Analysis (Analyzing) REF: Pages: 345-346 13. A patient diagnosed with stage 1 Alzheimer disease tires easily and prefers to stay home rather than attend social activities. The spouse does the grocery shopping because the patient cannot remember what to buy. Which nursing diagnosis applies at this time? a. Risk for injury b. Impaired memory c. Self-care deficit d. Caregiver role strain ANS: B Memory impairment is present and expected in stage 1 Alzheimer disease. Patients diagnosed with early Alzheimer disease often have difficulty remembering names, so socialization is minimized. Data are not present to support the other diagnoses. DIF: Cognitive Level: Application (Applying) REF: Pages: 346-351 14. A patient has progressive memory deficit associated with dementia. Which nursing intervention would best help the individual function in the environment? a. Assist the patient to perform simple tasks by giving step-by-step directions. b. Reduce frustration by performing activities of daily living for the patient. c. Stimulate intellectual function by discussing new topics with the patient. d. Promote the use of the patient’s sense of humor by telling jokes. ANS: A Patients with a cognitive impairment should perform all tasks of which they are capable. When simple directions are given in a systematic fashion, the patient is better able to process information and perform simple tasks. Stimulating intellectual functioning by discussing new topics is likely to prove frustrating for the patient. Patients with cognitive deficits may lose their sense of humor and find jokes meaningless. DIF: Cognitive Level: Application (Applying) REF: Pages: 351-356 15. Two patients in a residential care facility are diagnosed with dementia. One shouts to the other, “Move along, you’re blocking the road.” The other patient turns, shakes a fist, and shouts, “I know what you’re up to; you’re trying to steal my car.” What is the nurse’s best action? a. Administer one dose of an antipsychotic medication to both patients. b. Reinforce reality. Say to the patients, “Walk along in the hall. This is not a traffic intersection.” c. Separate and distract the patients. Take one to the day room and the other to an activities area. d. Step between the two patients and say, “Please quiet down. We do not allow violence here.” ANS: C Separating and distracting prevents escalation from verbal to physical acting out. Neither patient loses self-esteem during this intervention. Medication is probably not necessary. Stepping between two angry, threatening patients is an unsafe action, and trying to reinforce reality during an angry outburst will probably not be successful when the patients are cognitively impaired. DIF: Cognitive Level: Application (Applying) REF: Pages: 351-356 16. An older adult patient in the intensive care unit has visual and auditory illusions. Which intervention will be most helpful? a. Place large clocks and calendars on the wall. b. Place personally meaningful objects in view. c. Use the patient’s glasses and hearing aids. d. Keep the room brightly lit at all times. ANS: C Illusions are sensory misperceptions. Glasses and hearing aids help clarify sensory perceptions. Without glasses, clocks, calendars, and personal objects are meaningless. Round-the-clock lighting promotes sensory overload and sensory perceptual alterations. DIF: Cognitive Level: Analysis (Analyzing) REF: Pages: 343-344 17. A patient diagnosed with stage 2 Alzheimer disease calls the police saying, “An intruder is in my home.” Police investigate and discover the patient misinterpreted a reflection in the mirror as an intruder. This phenomenon can be assessed as: a. hyperorality. b. aphasia. c. apraxia. d. agnosia. ANS: D Agnosia is the inability to recognize familiar objects, parts of one’s body, or one’s own reflection in a mirror. Hyperorality refers to placing objects in the mouth. Aphasia refers to the loss of language ability. Apraxia refers to the loss of purposeful movements, such as being unable to dress. DIF: Cognitive Level: Comprehension (Understanding) REF: Page: 346 TOP: Nursing Process: Assessment MSC: NCLEX: Psychosocial Integrity 18. During morning care, a nursing assistant asks a patient diagnosed with dementia, “How was your night?” The patient replies, “It was lovely. I went out to dinner and a movie with my friend.” Which term applies to the patient’s response? a. Sundown syndrome b. Confabulation c. Perseveration d. Delirium ANS: B Confabulation is the making up of stories or answers to questions by a person who does not remember. It is a defensive tactic to protect self-esteem and prevent others from noticing memory loss. The patient’s response was not sundown syndrome. Perseveration refers to repeating a word or phrase over and over. Delirium is not present in this scenario. DIF: Cognitive Level: Comprehension (Understanding) REF: Page: 346 19. A patient diagnosed with Alzheimer disease wanders at night. Which action should the nurse recommend for a family to use in the home to enhance safety? a. Place throw rugs on tile or wooden floors. b. Place locks at the tops of doors. c. Encourage daytime napping. d. Obtain a bed with side rails. ANS: B Placing door locks at the top of the door makes it more difficult for the patient with dementia to unlock the door because the ability to look up and reach upward is diminished. All throw rugs should be removed to prevent falls. The patient will try to climb over side rails, increasing the risk for injury and falls. Day napping should be discouraged with the hope that the patient will sleep during the night. DIF: Cognitive Level: Application (Applying) REF: Page: 355 Quiz 5 20. Goals and outcomes for an older adult patient experiencing delirium caused by fever and dehydration will focus on: a. returning to premorbid levels of function. b. identifying stressors negatively affecting self. c. demonstrating motor responses to noxious stimuli. d. exerting control over responses to perceptual distortions. ANS: A The desired overall goal is that the patient with delirium will return to the level of functioning held before the development of delirium. Demonstrating motor responses to noxious stimuli is an appropriate indicator for a patient whose arousal is compromised. Identifying stressors that negatively affect the self is too nonspecific to be useful for a patient experiencing delirium. Exerting control over responses to perceptual distortions is an unrealistic indicator for the patient with sensorium problems related to delirium. DIF: Cognitive Level: Application (Applying) REF: Page: 342 21. An older adult diagnosed with moderate-stage dementia forgets where the bathroom is and has episodes of incontinence. Which intervention should the nurse suggest to the patient’s family? a. Label the bathroom door. b. Take the older adult to the bathroom hourly. c. Place the older adult in disposable adult diapers. d. Make sure the older adult does not eat nonfood items. ANS: A patient with moderate Alzheimer disease has memory loss that begins to interfere with activities. This patient may be able to use environmental cues such as labels on doors to compensate for memory loss. Regular toileting may be helpful, but a 2-hour schedule is often more reasonable. Placing the patient in disposable diapers is more appropriate as a later stage intervention. Making sure the patient does not eat nonfood items will be more relevant when the patient demonstrates hyperorality. DIF: Cognitive Level: Application (Applying) REF: Pages: 346-347 | Pages: 351-356 22. A patient diagnosed with dementia no longer recognizes family members. The family asks how long it will be before their family member recognizes them when they visit. What is the nurse’s best reply? a. “Your family member will never again be able to identify you.” b. “I think that is a question the health care provider should answer.” c. “One never knows. Consciousness fluctuates in persons with dementia.” d. “It is disappointing when someone you love no longer recognizes you.” ANS: D Therapeutic communication techniques can assist family members to come to terms with the losses and irreversibility dementia imposes on both the loved one and themselves. Two of the incorrect responses close communication. The nurse should take the opportunity to foster communication. Consciousness does not fluctuate in patients with dementia. DIF: Cognitive Level: Application (Applying) REF: Pages: 347-348 | Page: 350 | Page: 354 23. A patient diagnosed with severe dementia no longer recognizes family members and becomes anxious and agitated when they attempt reorientation. Which alternative could the nurse suggest to the family members? a. Wear large name tags. b. Focus interaction on familiar topics. c. Frequently repeat the reorientation strategies. d. Strategically place large clocks and calendars. ANS: B Reorientation may seem like arguing to a patient experiencing cognitive deficits and increases the patient’s anxiety. Validating, talking with the patient about familiar, meaningful things, and reminiscing give meaning to existence both for the patient and family members. The option that suggests using validating techniques when communicating is the only option that addresses an interactional strategy. Wearing large name tags and strategically placing large clocks and calendars are reorientation strategies. Frequently repeating the reorientation strategies is inadvisable; patients with dementia sometimes become more agitated with reorientation. DIF: Cognitive Level: Application (Applying) REF: Pages: 351-354 24. What is the priority need for a patient diagnosed with late-stage dementia? a. Promotion of self-care activities b. Meaningful verbal communication c. Maintenance of nutrition and hydration d. Prevention of the patient from wandering ANS: C In late-stage dementia, the patient often seems to have forgotten how to eat, chew, and swallow. Nutrition and hydration needs must be met if the patient is to live. The patient is incapable of self-care, ambulation, or verbal communication. DIF: Cognitive Level: Comprehension (Understanding) REF: Pages: 347-348 Quiz 5 25. Which intervention is appropriate to use for patients diagnosed with either delirium or dementia? a. Speak in a loud, firm voice. b. Touch the patient before speaking. c. Reintroduce the health care worker at each contact. d. When the patient becomes aggressive, use physical restraint instead of medication. ANS: C Short-term memory is often impaired in patients with delirium and dementia. Reorientation to staff is often necessary with each contact to minimize misperceptions, reduce anxiety level, and secure cooperation. Loud voices may be frightening or sound angry. Speaking before touching prevents the patient from feeling threatened. Physical restraint is not appropriate; the least restrictive measure should be used. DIF: Cognitive Level: Comprehension (Understanding) REF: Page: 340 | Page: 344 | Page: 352 Quiz 5 26. A hospitalized patient experiencing delirium misinterprets reality, and a patient diagnosed with dementia wanders about the home. Which outcome is the priority in both scenarios? Each patient will: a. remain safe in the environment. b. participate actively in self-care. c. communicate verbally. d. acknowledge reality. ANS: A Risk for injury is the nurse’s priority concern in both scenarios. Safety maintenance is the desired outcome. The other outcomes may not be realistic. DIF: Cognitive Level: Application (Applying) REF: Pages: 342-343 | Pages: 349-350 MULTIPLE RESPONSE 1. A patient diagnosed with Alzheimer disease has a dressing and grooming self-care deficit. Designate the appropriate interventions to include in the patient’s plan of care. Select all that apply. a. Provide clothing with elastic and hook-and-loop closures. b. Label clothing with the patient’s name and name of the item. c. Administer antianxiety medication before bathing and dressing. d. Provide necessary items, and direct the patient to proceed independently. e. If the patient resists, use distraction and then try again after a short interval. ANS: A, B, E Providing clothing with elastic and hook-and-loop closures facilitates patient independence. Labeling clothing with the patient’s name and the name of the item maintains patient identity and dignity (and provides information if the patient has agnosia). When a patient resists, using distraction and trying again after a short interval are appropriate because patient moods are often labile; the patient may be willing to cooperate during a later opportunity. Providing the necessary items for grooming and directing the patient to proceed independently are inappropriate. Staff members are prepared to coach by giving step-by-step directions for each task as it occurs. Administering anxiolytic medication before bathing and dressing is inappropriate. This measure would result in unnecessary overmedication. DIF: Cognitive Level: Application (Applying) REF: Pages: 351-356 Quiz 5 2. Which assessment findings would the nurse expect in a patient experiencing delirium? Select all that apply. a. Impaired level of consciousness b. Disorientation to place and time c. Wandering attention d. Apathy e. Agnosia ANS: A, B, C Disorientation to place and time is an expected finding. Orientation to person (self) usually remains intact. Attention span is short, and difficulty focusing or shifting attention as directed is often noted. Patients with delirium commonly experience illusions and hallucinations. Fluctuating levels of consciousness are expected. Agnosia occurs with dementia. Apathy is associated with depression. DIF: Cognitive Level: Application (Applying) REF: Pages: 338-342 TOP: Nursing Process: Assessment MSC: NCLEX: Psychosocial Integrity 3. A nurse should anticipate that which symptoms of Alzheimer disease will become apparent as the disease progresses from moderate to severe to late stage? Select all that apply. a. Agraphia b. Hyperorality c. Fine motor tremors d. Hypermetamorphosis e. Improvement of memory ANS: A, B, D The memories of patients with Alzheimer disease continue to deteriorate. These patients demonstrate the inability to read or write (agraphia), the need to put everything into the mouth (hyperorality), and the need to touch everything (hypermetamorphosis). Fine motor tremors are associated with alcohol withdrawal delirium, not dementia. Memory does not improve. DIF: Cognitive Level: Application (Applying) REF: Pages: 346-348 Pharm Varcarolis Chapter 26 1. A 5-year-old child moves and talks constantly, is easily distracted, and does not listen to the parents. The child awakens before the parents every morning. The child attended kindergarten, but the teacher could not handle the behavior. What is this child's most likely problem? a. Mental retardation b. Oppositional defiant disorder (ODD) c. Pervasive developmental disorder d. Attention deficit hyperactivity disorder (ADHD) ANS: D The excessive motion, distractibility, and excessive talkativeness suggest ADHD. Developmental delays would be observed if either a pervasive developmental disorder or mental retardation were present. ODD includes serious violations of the rights of others. 2. A child with attention deficit hyperactivity disorder (ADHD) has hyperactivity, distractibility, and impaired play. The health care provider prescribed methylphenidate (Concerta). The desired behavior for which the nurse should monitor is: a. increased expressiveness in communicating with others b. improved ability to participate in play with other children c. ability to identify anxiety and implement self-control strategies d. improved socialization skills with other children and authority figures ANS: B The goal is improvement in the child's hyperactivity, distractibility, and play. The remaining options are more relevant for a child with a pervasive developmental or anxiety disorder. 3. A 5-year-old child with attention deficit hyperactivity disorder (ADHD) bounces out of a chair in the waiting room, runs across the room, and begins to slap another child. What is the nurse's best action? a. Call for emergency assistance from another staff member. b. Instruct the parents to take the child home immediately. c. Direct this child to stop, and then comfort the other child. d. Take the child into another room with toys to act out feelings. ANS: D The use of play to express feelings is appropriate; the cognitive and language abilities of the child may require the acting out of feelings if verbal expression is limited. The remaining options provide no outlet for feelings or opportunity to develop coping skills. 4. A 16-year-old adolescent with a conduct disorder (CD) has been in a residential program for 3 months. Which outcome should occur before discharge? a. The teen and parents create and consent to a behavioral contract with rules, rewards, and consequences. b. The teen completes an application to enter a military academy for continued structure and discipline. c. The teen is temporarily placed with a foster family until the parents complete a parenting skills class. d. The teen has an absence of anger and frustration for 1 week. ANS: A The patient and the parents must agree on a behavioral contract that clearly outlines rules, expected behaviors, and consequences for misbehavior. It must also include rewards for following the rules. The patient will continue to experience anger and frustration. The patient and parents must continue with family therapy to work on boundary and communication issues. Separating the patient from the family to work on these issues is not necessary, and separation is detrimental to the healing process. 5. A child with attention deficit hyperactivity disorder (ADHD) is going to begin medication therapy. The nurse should plan to teach the family about which classification of medications? a. Central nervous system stimulants b. Monoamine oxidase inhibitors (MAOIs) c. Antipsychotic medications d. Anxiolytic medications ANS: A Central nervous system stimulants increase blood flow to the brain and have proven helpful in reducing hyperactivity in children and adolescents with ADHD. The other medication categories listed would not be appropriate. 6. Shortly after an adolescent's parents announce a plan to divorce, the teen stops participating in sports, sits alone at lunch, and avoids former friends. The adolescent says, "If my parents loved me, then they would work out their problems." What nursing diagnosis is most applicable? a. Ineffective coping b. Decisional conflict c. Chronic low self-esteem d. Disturbed personal identity ANS: A Ineffective coping is evident in the adolescent's response to family stress and discord. Adolescents value peer interactions, and yet this child has eliminated that source of support. The distracters are not supported by the data in this scenario. 7. Shortly after a 15-year-old's parents announce a plan to divorce, the adolescent stops participating in sports, sits alone at lunch, and avoids former friends. The adolescent says, "All the other kids have families. If my parents loved me, then they would stay together." Which nursing intervention is most appropriate? a. Develop a plan for activities of daily living. b. Communicate disbelief relative to the adolescent's feelings. c. Assist the adolescent to differentiate reality from perceptions. d. Assess and document the adolescent's level of depression daily. ANS: C The patient's perceptions that all the other kids are from two-parent households and that he or she is different are not based in reality. Assisting the patient to test the accuracy of the perceptions is helpful. 8. When group therapy is to be used as a treatment modality, the nurse should suggest placing a 9-year-old in a group that uses: a. play activities exclusively. b. group discussion exclusively. c. talk focused on a specific issue. d. play and talk about the play activity. ANS: D Group therapy for young children takes the form of play. For elementary school children, therapy combines play and talk about the activity. For adolescents, group therapy involves more talking. 9. When assessing a 2-year-old toddler with suspected autistic disorder, a nurse expects: a. hyperactivity and attention deficits. b. failure to develop interpersonal skills. c. history of disobedience and destructive acts. d. high levels of anxiety when separated from a parent. ANS: B Autistic disorder involves distortions in the development of social skills and language that include perception, motor movement, attention, and reality testing. Caretakers frequently mention the child's failure to develop interpersonal skills. The distracters are more relevant to ADHD, separation anxiety, and CD. 10. A 4-year-old child cries and screams from the time the parents leave the child at preschool until the child is picked up 4 hours later. The child is calm and relaxed when the parents are present. The parents ask, "What should we do?" What is the nurse's best recommendation? a. "Send a picture of yourself to school to keep with the child." b. "Arrange with the teacher to let the child call home at playtime." c. "Talk with the school about withdrawing the child until maturity increases." d. "Talk with your health care provider about a referral to a mental health clinic." ANS: D Separation anxiety disorder becomes apparent when the child is separated from the attachment figure. Often, the first time separation occurs is when the child goes to kindergarten or nursery school. Separation anxiety may be based on the child's fear that something will happen to the attachment figure. The child needs professional help. 11. A 15-year-old adolescent has run away from home six times. After the adolescent was arrested for prostitution, the parents told the court, "We can't manage our teenager." The adolescent is physically abusive to the mother and defiant with the father. The adolescent's problem is most consistent with criteria for: a. Attention deficit hyperactivity disorder ADHD. b. Childhood depression. c. Conduct disorder CD. d. autism spectrum disorder ASD ANS: C CDs are manifested by a persistent pattern of behavior in which the rights of others and age-appropriate societal norms are violated. The Diagnostic and Statistical Manual of Mental Disorders (fourth edition, text revision) (DSM-IV-TR) identifies CDs as: serious violations of rules. The patient's clinical manifestations do not coincide with the other disorders listed. 12. A 15-year-old adolescent is referred to a residential program after an arrest for theft and running away from home. At the program, the adolescent refuses to participate in scheduled activities and pushes a staff member, causing a fall. Which approach by the nursing staff would be most therapeutic? a. Neutrally permit refusals b. Coax to gain compliance c. Offer rewards in advance d. Establish firm limits ANS: D Firm limits are necessary to ensure physical safety and emotional security. Limit setting will also protect other patients from the teenager's thoughtless or aggressive behavior. Permitting refusals to participate in the treatment plan, coaxing, and bargaining are strategies that do not help the patient learn to abide by rules or structure. 13. An adolescent was arrested for prostitution and assault on a parent. The adolescent says, "I hate my parents. They focus all their attention on my brother, who's perfect in their eyes." Which type of therapy might promote the greatest change in this adolescent's behavior? a. Bibliotherapy b. Play therapy c. Family therapy d. Art therapy ANS: C Family therapy focuses on problematic family relationships and interactions. The patient has already identified problems within the family. 14. An adolescent is arrested for prostitution and assault on a parent. The adolescent says, "I hate my parents. They focus all their attention on my brother, who's perfect in their eyes." Which nursing diagnosis is most applicable? a. Ineffective coping, related to seeking parental attention as evidenced by acting out b. Disturbed personal identity, related to acting out as evidenced by prostitution c. Impaired parenting, related to showing preference for one child over another d. Hopelessness, related to feeling unloved by parents ANS: A The patient demonstrates an inability to problem solve by using adaptive behaviors to meet life's demands and roles. The defining characteristics are not present for the other nursing diagnoses. The patient has never mentioned hopelessness, low self-esteem, or disturbed personal identity. 15. Which assessment finding would cause the nurse to consider an 8-year-old child to be most at risk for the development of a psychiatric disorder? a. Being raised by a parent with chronic major depression b. Moving to three new homes over a 2-year period c. Not being promoted to the next grade d. Having an imaginary friend ANS: A Statistics tell us that children raised by a depressed parent have a 30% to 50% chance of developing an emotional disorder. The chronicity of the parent's depression means it has been a consistent stressor. The other factors do not create ongoing stress. 16. Which child shows behaviors indicative of mental illness? a. 4-year-old who stuttered for 3 weeks after the birth of a sibling b. 9-month-old who does not eat vegetables and likes to be rocked c. 3-month-old who cries after feeding until burped and sucks a thumb d. 3-year-old who is mute, passive toward adults, and twirls while walking ANS: D Symptoms consistent with an autistic spectrum disorder (ASD) are evident in the correct answer. The behaviors of the other children are within normal ranges. 17. The child most likely to receive propranolol (Inderal) to control aggression, deliberate self-injury, and temper tantrums is one diagnosed with: a. Attention deficit hyperactivity disorder ADHD. b. Posttraumatic stress disorder. c. Separation anxiety. d. Autistic disorder ASD. ANS: D Propranolol is useful for controlling aggression, deliberate self-injury, and temper tantrums of some autistic children. It is not indicated in any of the other disorders. 18. A 12-year-old child has been the neighborhood bully for several years. The parents say, "We can't believe anything our child says." Recently, the child shot a dog with a pellet gun and set fire to a trash bin outside a store. The child's behaviors are most consistent with: a. conduct disorder CD. b. attention deficit ADHD. c. defiance of authority. d. anxiety over separation from a parent. ANS: A The behaviors mentioned are most consistent with the DSM-IV-TR criteria for CD: aggression against people and animals; destruction of property; deceitfulness; rule violations; and impairment in social, academic, or occupational functioning. The behaviors are not consistent with ADHD and separation anxiety and are more pervasive than defiance of authority. 19. The parent of a child with Tourette's disorder says to the nurse, "I think my child is faking the tics because they come and go." Which response by the nurse is accurate? a. "Perhaps your child was misdiagnosed." b. "Your observation indicates the medication is effective." c. "Tics often change frequency or severity. That doesn't mean they aren't real." d. "This finding is unexpected. How have you been administering your child's medication?" ANS: C Tics are sudden, rapid, involuntary, repetitive movements or vocalizations characteristic of Tourette's disorder. They often fluctuate in frequency and severity and are reduced or absent during sleep. 20. An 11-year-old child, who has been diagnosed with oppositional defiant disorder ODD, becomes angry over the rules at a residential treatment program and begins shouting at the nurse. Select the best method to defuse the situation. a. Suggest that the child go to the gym and shoot baskets. b. Administer an antipsychotic medication. c. Place the child in a basket-hold. d. Call a staff member to seclude the child. ANS: A Redirecting the expression of feelings into nondestructive age-appropriate behaviors such as a physical activity helps the child learn how to modulate the expression of feelings and exert self-control. This method is the least restrictive alternative of those listed and should be tried before resorting to more restrictive measures. 21. An adolescent acts out in disruptive ways. When this adolescent threatens to throw a pool ball at another adolescent, which comment by the nurse would set limits? a. "Attention, everyone. We are all going to the craft room." b. "You will be taken to seclusion if you throw that ball." c. "Do not throw the ball. Put it back on the pool table." d. "Please do not lose control of your emotions." ANS: C Setting limits uses clear, sharp statements about the prohibited behavior and provides guidance for performing a behavior that is expected. The incorrect options represent a threat, use restructuring (which would be inappropriate in this instance), and make a direct appeal to the child's developing self-control that may be ineffective. 22. When a 5-year-old child is disruptive, the nurse says, "You must take a time-out." The expectation is that the child will: a. go to a quiet room until called for the next meal. b. slowly count to 20 before returning to the group activity. c. sit on the edge of the activity until able to regain self-control. d. sit quietly on the lap of a staff member until able to apologize for the behavior. ANS: C Time-out is designed so that staff can be consistent in their interventions. Time-out may require having the child sit on the periphery of an activity until he or she gains self-control and reviews the episode with a staff member. Time-out may not require having the child go to a designated room and does not involve special attention such as holding. Having the child count to 10 or 20 is not sufficient. 23. A child blurts out answers to questions before the questions are complete, demonstrates an inability to take turns, and persistently interrupts and intrudes in the conversations of others. Assessment data show these behaviors relate primarily to: a. hyperactivity b. impulsivity c. inattention d. defiance ANS: B These behaviors demonstrate impulsivity. Hyperactive behaviors are more physical in nature, such as running, pushing, and the inability to sit. Inattention is a failure to listen. Defiance is willfully doing what an authority figure has said not to do. 24. A parent with schizophrenia and her 13-year-old child live in a homeless shelter. The child has formed a trusting relationship with a shelter volunteer. The child says, "My three friends and I got an A on our school science project." The nurse can assess that the child: a. displays resiliency. b. has a difficult temperament. c. is at risk for posttraumatic stress disorder. d. uses intellectualization to deal with problems. ANS: A Resiliency enables a child to handle the stresses of a difficult childhood. Resilient children can adapt to changes in the environment, take advantage of nurturing relationships with adults other than parents, distance themselves from emotional chaos occurring within the family, learn, and use problem-solving skills. 25. A parent with schizophrenia and 13-year-old child live in a homeless shelter. The child has formed a trusting relationship with a volunteer. The teen says, "I have three good friends at school. We talk and sit together at lunch." What is the nurse's best suggestion to the treatment team? a. Suggest foster home placement. b. Seek assistance from the domestic violence program. c. Make referrals for existing and emerging developmental problems. d. Foster healthy characteristics and existing environmental supports. ANS: D Because the teenager shows no evidence of poor mental health, the best action would be to foster existing healthy characteristics and environmental supports. No other option is necessary or appropriate under the current circumstances. 26. Which behavior indicates that the treatment plan for a child with autism spectrum disorder was effective? The child: a. plays with one toy for 30 minutes. b. repeats words spoken by a parent. c. holds the parent's hand while walking. d. spins around and claps hands while walking. ANS: C Holding the hand of another person suggests relatedness. Usually, a child with autism would resist holding someone's hand and stand or walk alone, perhaps flapping arms or moving in a stereotypical pattern. The other options reflect behaviors that are consistent with autistic disorder. 1. What are the primary distinguishing factors between the behavior of children with oppositional defiant disorder (ODD) and those with conduct disorder (CD)? The child with: (select all that apply). a. ODD relives traumatic events by acting them out. b. ODD tests limits and disobeys authority figures. c. ODD has difficulty separating from the parents. d. CD uses stereotypical or repetitive language. e. CD often violates the rights of others. ANS: B, E Children with ODD are negativistic, disobedient, and defiant toward authority figures without seriously violating the basic rights of others, whereas children with CD frequently behave in ways that violate the rights of others and age-appropriate societal norms. Reliving traumatic events occurs with posttraumatic stress disorder. Stereotypical language behaviors are observed in autistic children. Separation problems with resultant anxiety occur with separation anxiety disorder. 2. A nurse prepares the plan of care for a 15-year-old adolescent with moderate intellectual developmental disorder. What are the highest outcomes that are realistic for this patient? Within 5 years, the patient will: select all that apply. a. Live unaided in an apartment. b. Complete high school or earn a general equivalency diploma (GED). c. Independently perform his or her own personal hygiene. d. Obtain employment in a local sheltered workshop. e. Correctly use public buses to travel in the community. ANS: C, D, E Individuals with moderate intellectual developmental disorder progress academically to about a second grade level. These people can learn to travel in familiar areas and perform unskilled or semiskilled work. With supervision, they can function in the community, but independent living is not likely. Textbook Questions 1. The nurse interviews the parent of a 7 yr old child diagnosed with moderate autism spectrum disorder. Which comment from the parent best describes autistic behavior? - “my child swings for hours on our backyard gym set.” 2. A nurse plans to lead a group in a residential facility for kindergarten age, abused children. Which strategy should the nurse incorporate? - Telling a story about a child who felt sad. 3. Which scenario present the highest risk of a pregnancy resulting in offspring with an intellectual developmental disability (IDD)? -32 yr old woman diagnosed with anorexia nervosa. 4. A community mental health nurse talks with a 6 yr old child whose divorced parents have shared custody. Which initial question will best help the nurse explore the child’s perception of home life? - “What are some of the good and bad things about living in two places?” 5. The parent of an adolescent recently diagnosed with schizophrenia says to the nurse, “This is entirely my fault. I should have spent more time with my child when he was a toddler.” Which response by the nurse if correct? -Schizophrenia is genetically transmitted, so it was not in your control.” Chapter review questions Chapter 13 varcolis 1.A person shoplifts merchandise from a community cancer thrift shop. When confronted the thief replies “all this stuff was donated, so I can take it.” this comment suggests features of which personality disorder? -Antisocial 2.After a power outage, a facility must serve a dinner of sandwiches and fruit to clients. Which comment is most likely from a client diagnosed with a narcissistic personality disorder? -"You should have ordered a to-go meal from a local restaurant for me." 3.A nurse plans care for a patient diagnosed with borderline personality disorder. Which nursing diagnosis is most likely to apply to this patient? -ineffective relationships related to frequent splitting 4.The nurse assesses a new patient suspected of having a schizotypal personality disorder. Which assessment question is this patient most likely to answer affirmatively? -”is anyone in your family diagnosed with a mental illness?” 5.A mental health nurse assesses a client diagnosed with antisocial personality disorder. Which co-morbid problem is most important for the nurse to include in the assessment? -Alcohol use and abuse Chapter 14 varcolis 1.The school nurse assesses four adolescents, all of whom outward appear healthy. Which adolescent meets one criterion for anorexia nervosa with mild severity? -5’7” tall; weight 110 lbs 2.A nurse assesses four adolescents diagnosed with various eating disorders. Which comment would the nurse most expect from the adolescent diagnosed with anorexia nervosa ? -"I have lost 60 lb but I'm still a size 2. I want to be a size 0." 3.While weighing clients on an eating disorders unit, a psychiatric technician says to the client, "I wish I had an eating disorder; maybe I'd lose a little weight." A nurse overhears the comment. What is the nurse's best action? -Privately discuss the importance of sensitivity with the psychiatric technician. 4.shortly after hopitalization, an adolescent diagnosed with anorexia nervosa says to the nurse, “being fat is the worst thing in the world. I hope it never happens to me.” which response by the nurse is appropriate? -”tell me how your world would be different if you were fat.” 5.client is hospitalized with a diagnosis of anorexia nervosa. The nurse reviews the client's laboratory results. Which action should the nurse take next? -Auscultate the client's heart rate, rhythm, and sounds Chapter 15 varcolis 1.A 28 year old second-grade teacher is diagnosed with major depressive disorder. She grew up in Texas but moved to Alaska 10 years ago to separate from an abusive mother. Her father died by suicide when she was 12 years old. Which combination of factors in this scenario best demonstrates the stress-diathesis model? -Family history of mental illness coupled with history of abuse 2.A patient tells the nurse , “no matter what i do, i feel like there’s always a dark cloud following me.”select the nurse's initial action. -say to the patient, “tell me more about what you mean by a dark cloud” 3.A patient experiencing depression says to the nurse, "my health provider said i need 'talk' therapy but i think i need a prescription for an antidepressant medication. what should i do? Select the nurses best response. -“let’s consider some ways to address your concerns with your healthcare provider.” 4.the nurse cares for a hospitalized adolescent diagnosed with major depressive disorder. The health care provider prescribes a low does antidepressant. In consideration of published warnings about use of antidepressant medications in younger patients, which action should the nurse employ? -monitor the adolescent closely for evidence of adverse effects, particularly suicidal thinking or behavior 5.over the past 2 months a patient made eight suicide attempts with increasing lethality. The health care provider informs the patient and family the ECT is needed. The family whispers to the nurse, "isn't this a dangerous treatment?" How should the nurse reply? -ECT is very effective when urgent help is needed. Your family member was carefully evaluated for possible risks Chapter 16 varcolis 1. 2. which comment by a patient diagnosed with bipolar disorder best indicates the patient is experiencing mania? -”yesterday I made 487 posts on my social network page.” 3. A community mental health nurse counsels a group of patient about the upcoming flu season. What instruction does the nurse provide for patients who are prescribed lithium? -”stop taking your medication and contact me if you have nausea, vomiting, and or/diarrhea.” 4. A patient was diagnosed with bipolar disorder many years ago. The patient tells the nurse, “when I have a manic episode, there’s always a feeling of gloom behind it and I know I will soon be totally depressed.” what is the nurse’s best response? -”your comment indicates you have an understanding and insight about your disorder.” 5. A patient diagnosed with bipolar disorder lives in the community Review Q’s: 1. A pt has long hx of bipolar disorder with frequent episodes of mania secondary to stopping prescribed meds. The pt says “I will use my whole check next month to buy lottery tickets. Winning will solve my money probs” Select nurse’s best action. –confer with the treatment team about appointing a legal guardian for the pt 2. Which comment by a pt diagnosed with bipolar disorder best indicates the pt is experiencing mania? –“Yesterday I made 487 posts on my social network page” 3. A community mental health nurse counsels a group of pts about the upcoming flu season. What instruction does the nurse provide for pts who are prescribed lithium? –“Stop taking your medicine and contact me if you have nausea, vomit and/or diarrhea” 4. A pt was dx’d with bipolar disorder many yrs ago. The pt tells nurse “When I have a manic episode, there’s always a feeling of gloom behind it and I know I will soon be totally depressed.” What’s nurse’s best response? –“Your comment indicates you have an understanding and insight about your disorder” 5. A pt dx’d with bipolar disorder lives in the community and is showing early signs of mania. The pt says “I need to go visit my daughter but she lives across the country. I put some requests on the internet to get a ride. I’m sure someone will take me” what is nurse’s most therapeutic response? –“I’m concerned about your safety when meeting or riding with strangers” [Show More]

Last updated: 1 year ago

Preview 1 out of 266 pages

Reviews( 0 )

$30.00

Add to cart

Instant download

Can't find what you want? Try our AI powered Search

OR

GET ASSIGNMENT HELP
172
0

Document information


Connected school, study & course


About the document


Uploaded On

Mar 02, 2021

Number of pages

266

Written in

Seller


seller-icon
succeeded

Member since 3 years

224 Documents Sold


Additional information

This document has been written for:

Uploaded

Mar 02, 2021

Downloads

 0

Views

 172

Document Keyword Tags

Recommended For You


$30.00
What is Browsegrades

In Browsegrades, a student can earn by offering help to other student. Students can help other students with materials by upploading their notes and earn money.

We are here to help

We're available through e-mail, Twitter, Facebook, and live chat.
 FAQ
 Questions? Leave a message!

Follow us on
 Twitter

Copyright © Browsegrades · High quality services·